Sei sulla pagina 1di 138

• •


••••••••••••••••••••• •• ....----
•••••••
•••••••• ••••••••• •••••
••••••• ••••••
•••••••••••••••••
••••• •••••••••• •••••
•••
••••••••••••••••••••••••••
••••••••••••••
••• ••••••••••••
••••••••••••••••••••••••••
•••••••••••••••••••••
•••••••••••••••••••••
••••
•••••• ••••••••••••• •••
••••••••••••••••
••••••• ••••••••
••••••••••••••••••
••••••
•••• ••• •••••••••••••
•••••••• ••••••••
•• •••••••••••••••••••
••••••••••• •• •••••••
•••••••• ••••••••••••••
•••••••••••••••••
•••••••
••••• ••••
••••• •• •••••••
• • • • • • • • • • • • • • . r---:==:

••• ••••••••••••••••••
•••••• •••••••••••••
•••••••••••• ••••••••
••• •••••••••••••••••
••• • • • • • • • • • • • • • • • • •
•• •• •• •• •• •• •• •• •• •• •• •• •• •• •• •• •• • • •

• •

••••••• • • • • • • • • • • •
•••• • • • • • • • • • • • •
•••• • • • • • • • • • • • • • •
••• • • • • • •• • • • •
•••• • • • • • • • • • • •
••••• • • • • • •
•••• • • • • • • • • • •
••• • • •
1 • • • • •
• • •
• • •

• •
1 • • • • • • • • •

1 • • • • • • • •

••• • • • • •
••• • • • • •
• • • • •
• • • •
Manual CTO
de Medicina y Cirugía

2.a edición

• Neumología
• Cirugía torácica

Grupo CTO
• • Editorial
Manual CTO
de Medicina y Cirugía

2.a edición

Neumología

Revisores
Josué Daniel Cadeza Aguilar
Jesús lván García Rivera
David Calleja Crespo

Autores
Jorge Castelao Naval
Jesús Fernández Francés

Grupo CTO
Editorial
NOTA

La medicina es una ciencia sometida a un cambio constante. A medida que la investigación y la experiencia
clínica amplían nuestros conocimientos, son necesarios cambios en los tratamientos y la farmacoterapia.
Los editores de esta obra han contrastado sus resultados con fuentes consideradas de confianza,
en un esfuerzo por proporcionar información completa y general, de acuerdo con los criterios aceptados
en el momento de la publicación. Sin embargo, debido a la posibilidad de que existan errores humanos
o se produzcan cambios en las ciencias médicas, ni los editores ni cualquier otra fuente implicada
en la preparación o la publicación de esta obra garantizan que la información contenida en la misma sea
exacta y completa en todos los aspectos, ni son responsables de los errores u omisiones ni de los resultados
derivados del empleo de dicha información. Por ello, se recomienda a los lectores que contrasten dicha
información con otras fuentes. Por ejemplo, y en particular, se aconseja revisar el prospecto informativo
que acompaña a cada medicamento que deseen administrar, para asegurarse de que la información
contenida en este libro es correcta y de que no se han producido modificaciones en la dosis recomendada
o en las contraindicaciones para la administración. Esta recomendación resulta de particular importancia
en relación con fármacos nuevos o de uso poco frecuente. Los lectores también deben consultar
a su propio laboratorio para conocer los valores normales.

No está permitida la reproducción total o parcial de este libro, su tratamiento informático, la transmisión
de ningún otro formato o por cualquier medio, ya sea electrónico, mecánico, por fotocopia, por registro
y otros medios, sin el permiso previo de los titulares del copyright.

© CTO EDITORIAL, S.L. 2016

Diseño y maquetación: CTO Editorial

C/ Francisco Silvela, 106; 28002 Madrid


Tfno.: (0034) 91 782 43 30- Fax: (0034) 91 782 43 43
E-mail: ctoeditorial@cto medicina.com
Página Web: www.grupocto.e s

ISBN Neumología: 978-84-16527-26-7


ISBN Obra completa: 978-84-16527-11-3
Depósito legal: M-26880-2015
Neumología

Manual CTO
de Medicina y Cirugía

2.a edición

Grupo CTO
Editorial
Q)
(.)
·-
-e _
,_e
... C't:S
c::::n 05. Asma ....................... ........ . ........................................... . 29
o 5.1 . Definición . . ........................ .... .... . . 29
o 5.2. Epidemiología ...................................... . ................................................................. 29
E 5.3. Patogenia .. ................................................................................................................. 29
::S
Q) 5.4. Factores implicados en el desarrollo
2 y expresión del asma....... ........................... 30
5.5. Fisiopatología ........................ . 32
5.6. Clínica..................................... . ................................................................. 32
5.7. Diagnóstico ..................................................................................................... 32
01. Recuerdo anatómico . . 1 5.8. Tratamiento ................................................................................................................... 34

1.1. División de la vía aérea ..... .


1.2. Estructura íntima
de las vías respiratorias inferiores .....
06. Bronquiect asias ... .. .......... . 39
2
1.3. Vasos pulmonares 3 6.1. Concepto ........ .. . . .................................................. ............................................ 39
6.2. Anatomía patológica... ... . ........................... 39
6.3. Patogenia ........................ .. . . . . . 39
02. Malformaci ones .................................... 4 6.4. Etiología ........................................................................................................................ 39
6.5. Clínica y diagnóstico ... ............................................................................... 40
2.1. Malformaciones pulmonares .......... 4
6.6. Tratamiento ........................ . .................................... ················ 40
2.2. Malformaciones de la pared torácica ..... .. .............. s

03. Fisiología y fisiopatolog ía . ............ 07. Enfermeda des pulmonare s


7
intersticiales ... 42
3.1 . Ventilación pulmonar ........ . .................................................................... 7
3.2. Circulación pulmonar .................................................................................. 12 7.1. Generalidades ......................................................................................... ............. 42
3.3. Intercambio gaseoso ..................................................................................... 13 7.2. Neumonías intersticiales idiopáticas . 42
7.3. Enfermedad intersticial
asociada a las enfermedades del colágeno ........ 45
04. Enfermeda d pulmonar 7.4. Histiocitosis X pulmonar ....................................................................... 45
7.5. Linfangioleiomi omatosis ..................................................... 46
obstructiva crónica (EPOC) 21
7.6. Proteinosis alveolar ............................................................ .................. 47
4.1. Concepto ...... 21 7.7. Granulomatosis broncocéntrica .............. 48
4.2. Epidemiología .... . ............................................................................ 2 1 7.8 . Afectación pulmonar en la amiloidosis ............................ 48
4.3. Etiología ..................................................................................................................................... 21 7.9. Afectación pulmonar por fármacos ........................................ 48
4.4. Patogenia y anatomía patológica ................................................ 23
4.5. Fisiopatología .... . ......................................................................................... 23
4.6. Clínica ..... ..................... 24
4.7. Diagnóstico ..... . ..................................................................................... 25
4.8. Tratamiento .................................................................... ..................................... 25
4.9. Pronóstico ...... . . .................................... 27
,
Neumología 1 1n d i Ce

08. Enfermedades por inhalación 13. Tromboembolismo pulmonar 71


de polvos 51 13.1. Concepto .............. .. ................................... 71

8.1. Neumonitis por hipersensibilidad ..... ............................. 51 13.2. Factores de riesgo . . .................................................. . .................................................. 71

8.2. Bisinosis.......... .............. ..................................................... 52 13.3. Fisi opatología .. . .......................................................................................... 71

8.3. Silicosis ..................................................................... ............................................................... 53 13.4. Di ag nóstico .............................................. .. ... .. .... 72


8.4. Neumoconiosis de los mineros del carbón 13.5. Tratamiento ......................................... ............................................................................... 74
(NMC) .. .... ... ................................. 54
8.5. Exposición al asbesto . ... . ...... ...... .. ... 54
8.6. Beriliosis .. . .. ............... .... ........... .......... ................................................... 55 14. Trastornos de la ventilación . 77
14. 1. Regulación de la ventilación ....................... ........................................ 77
14.2. Síndromes de hipoventilación ............................................................. 77
09. Eosinofilias pulmonares ... 57 14.3. Síndromes de hiperventilación .......................................................... 80
9.1. Aspergilosis broncopulmonar alérgica
(ABPA) .... ................................ 57
9.2. Otras eosinofilias pulmonares 15. Síndrome de la apnea del sueño.... 82
de etiología conocida ..... . ..... .............. 58
15.1. Apnea obstructiva del sueño ..... ... .............. .. ........................ . 82
9.3. Eosinofilias pulmonares
15.2. Apnea central del sueño ..... . . . ... .. . . .. 84
de etiología desconocida ............................................................ 58

16. Síndrome de distrés


10. Síndromes de hemorragia
respiratorio agudo ..... 85
alveolar difusa . . . . .. . ... . . 60
16.1. Etiología .... ... 85
10.1. Generalidades ..... . . ... 60
16.2. Fisiopatología .. ..... ... ....... ................. ............... ... 85
10.2. Hemosiderosis pulmonar idiopática ....................... ................ 60
16.3. Clínica. .. . .................. ................... . .................................................................... 86
16.4. Tratamiento ...... ............................................................................................................... 86

11. Sarcoidosis . . 62
11 .1. Epidemiología .... ............. ..................................... ........... 62 17. Ventilación mecánica.. 87
11 .2. lnmunopatogenia .... .... ............. 62
17.1. Ventilación mecánica no invasiva .. . .................... 87
11.3. Anatomía patológica ........................................................................................... 62
17.2. Ventilación mecánica invasiva ....... . ....... . ............................................ 88
11.4. Clínica..... ..... .... ............ ............ ......... . .... ............. ......... 62
11.5. Diagnóstico ..... . .. ..... .. . .. .......... ...... ....................... .............. 65
11.6. Tratamiento ......................................................................................... 65
Bibliografía ... 92

12. Hipertensión pulmonar .......... ............ 67


12.1 . Generalidades ...................................... . .............. . ........ . .................... . . ............................... 67
12.2. Hipertensión pulmonar idiopática ....... 67
12.3. Hipertensión pulmonar
tromboembólica crónica . .................... 69
Neumolog. ía._
. . . . _ _ _ _---·

Recuerdo anatómico

ORIENTACIÓN Este tema es poco importante.


Con las Ideas Clave
ENARM se sabe lo fundamental.
1

1.1. División de la vía aérea

La func ión principal del apa rato resp iratorio es el intercambio gaseoso,
y para el lo son necesa ri as unas estructu ras anat óm icas dispuestas de ta l
forma que sean ca paces de poner en ínt imo contacto aire y sa ng re, para
que sea posible la difusión de gases.

El apara to res piratorio se d ivide, a nive l de l ca rtílago cricoides, en dos por-


ciones: t racto respiratorio superior e inferior.
Tracto respirat orio superior. Compren de: fosas nasa les, fa ringe y la-
ringe.
La lari nge es rica en glándulas linfáti cas. Está constitu ida funda-
menta lmente por ca rtílagos y posee una g ran im portancia, ya q ue
cont iene las cu erd as voca les. La apertura inst antánea de las cuerdas
voca les y de la gloti s por un aum ento de la presión int ratorácica pro-
d uce la tos, q ue es uno de los m eca nismos de defensa más im por-
tantes del aparato resp iratorio.
Tracto respi ratorio inferior. Comienza en la t ráq uea y term ina en los
sacos alveola res. Se pueden d istin g uir varias zonas: zona de con-
d ucción, zona de t ransición y zo na respiratoria propiamente d icha
(Figura 1).

Zona de conducción

Incluye la t ráquea, q ue a nivel de la carina se divide en los bronquios


principa les derecho e izquierdo, los bronq uios loba res, segmentarios, los
bronq uiolos, los bronquio los lobulil lares (se o rigina n a partir del primer
bronquiolo t ras unas cuatro d ivisiones y ventilan los lobulillos secunda-
rios) y los terminales (de cada bronq uio lo lobu lillar se o rig inan unos cua- Figura 1. División de la vía aérea infe ri or
tro o cinco bro nq uiolos term inales) .
Zona de transición
Todas estas divisiones constituyen las 16 primeras generaciones. A esta
zona también se la denom ina espacio m uerto anat ómico y t iene un vo- La constituyen los bronq uiolos resp iratorios (generaciones 17, 18 y 19),
lumen aproximado de 150 mi. en cuyas paredes apa recen ya algunos alvéolos.
Manual CTO de Medicina y Cirugía, 2.8 edición

Tráquea y g randes bronquios


Zona respiratoria propiamente dicha Moco

Cél. caliciforme
La forman los cond uctos y sacos alveolares y alvéo los (generación 20 a mu cosa
23). Una vez descritas las divisiones de la vía aérea inferior, se puede dis-
Cél. caliciforme
tingu ir varias un idades: acino, lobul illo primario y lobu lillo secunda rio. El serosa
acino es la unidad anatómica situada d istal al bronqu iolo term ina l.
Membrana
ba sa l
El lobu lillo primario lo forman todos los conductos alveolares d istales a
un bronquiolo respiratorio, y ellobulil lo secundario es la m ínima porción
del parénquima pulmona r rodeada de tabiq ues de tejido conjuntivo e in-
depend iente de los lobu lillos vecinos. Todas estas d icotomías se rea lizan Cél. cili ada Cél. basa l Cél. ind iferenciada
en los primeros años de vida.

Bronquiolos
La circu lación del aire a través de los cond uctos se determina por las d ife-
rencias de presión que se generan entre el exterior y los alvéolos durante
los movimientos respiratorios. En la inspiración, la presión intrapulmonar
es menor que la atmosférica, permitiendo la entrada de aire, y en la espi-
ración ocurre lo contra rio.

El flujo de aire es t urbulento en las vías de mayor ca li bre (tráq uea y bro n-
qu ios principa les), lam inar en las vías más fi nas, y m ixto en los cond uctos
Cél.de Clara
intermedios.
Figura 2. Epitelio de vías aéreas

1 .2. Estructura íntima Fi brocartíla go. Se encuentra en la parte externa de la submucosa,


en la tráquea y los bronquios principales. Tiene una disposición en
de las vías respiratorias inferiores forma de arcos de herradura y el músculo une los dos extremos pos-
teriores de los cartílagos.
A partir de los bronq uios lobares, los cartíl agos no forman ani llos
Tráquea y bronquios continuos, sino placas aisladas unidas por fib ras muscu lares, y a me-
dida que los bronqu ios se ram ifican, las placas son más escasas.
Tienen estructuras histológicas aná logas, pero su d isposición varía desde
la tráquea hasta los bronquios más periféricos. La pared consta de tres Bronquiolos
componentes principa les: mucosa, submucosa y fibrocartílago.
Mucosa. Está formada por el epitelio, la membrana basal y el corion. A pa rt ir de los bronquiolos respiratorios, el epitelio es cuboideo, y estos
El epit el io es pseudoestratificado y consta de célu las ciliadas, célu- bronqu iolos no poseen célu las cal iciformes. Los bronqu iolos no tienen
las con microve llosidades sin cilios, células ca liciformes mucosas y ca rt íl ago ni g lándu las mucosas y poseen unas células denominadas de
serosas, cé lulas basa les, células ind iferenciadas y alg unas células de Clara, de forma columnar baja y que hacen prominencia en la luz bron-
Ku lchitsky. quiolar.
Todas estas célu las se apoyan en la membrana basal (Figura 2). Las
células basa les no afloran a la superficie, y de ellas derivan todos los Estas célu las producen el componente fluido del moco bronquial y el
demás t ipos. Las célu las ca liciformes, productoras de una m ínima llamado surfactante bronquiolar (puede encontrarse una pequeña canti-
cantidad de moco bronquial, dism inuyen progresivamente hacia la dad en los grandes bronquios).
periferia y no existen en los bronquiolos respiratorios.
Las célu las ciliadas se caracterizan por t ener su superficie recu- Las células ci liadas aún se encuentran en los bronquiolos respiratorios,
bierta de cilios. La mucosa epitelia l se recub re de fo rma d iscon- pero desaparecen antes de los alvéolos.
tinu a po r el moco b ro nq uial, q ue const a de dos capas: un a capa
superficia l más densa, en forma de ge l, donde se deposita n las Alvéolos
partículas extrañas inha ladas y los detritus celulares, y una capa
más p rofunda y fl uida, en forma de so l, donde baten de modo La pared alveolar se compone de: líqu ido de revestimiento epitelial, epi-
sincrónico los ci lios. telio alveolar, membrana basa l alveolar, intersticio, membrana basal capi-
Por debajo de la membrana basal se encuentra el corion, que cons- lar y endotel io capilar.
tituye un laxo entramado de colágeno, fibras elásticas y reticu lares,
vasos, nervios y distintos t ipos ce lulares (neutrófil os, linfocitos, eosi- El epitelio alveolar es escamoso, y está formado por varios tipos de célu-
nófi los, macrófagos, mastocitos y células plasmáticas). las. La mayoría son los neumocitos t ipo 1(95%), y entre estos se encuen-
Submucosa. Contiene las g lánd ulas productoras de moco (consti- tran los neumocitos t ipo 11 o g ranu losos, que son célu las cuboideas.
tuidas por células mucosas, serosas, un ca nal colector y uno excretor
ciliada que se abre a la luz bronqu ial), que producen la mayor pa rte Se cree que los neumocitos t ipo 1 no son capaces de reproducirse. Los
del moco bronquia l. Las glándulas disminuyen progresivamente ha- de tipo 11 son células secretoras, cuya misión principa l es la producción
cia la periferia (igual que las célu las caliciformes). de surfactante (que impide el colapso alveolar en la espiración). Sumer-

01 · Recuerdo anatómico
Neumología 01
gidos en el surfactante están los macrófagos, con capacidad fagoc itaría . 1.3. Vasos pulmonares
Los alvéolos se comu nican entre sí por los poros de Koh n. El surfactan-
te está compuesto por fosfol ípidos (el pri ncipa l es la dipa lmitoil lecitina),
apoproteínas su rfacta ntes e iones ca lcio. Los fosfolípidos producen una El pul món recibe sangre no oxigenada de las arterias pul monares y oxi-
disminución en la tensión superfi cia l, por lo que se d ice q ue el surfactan- genada de la circulación bro nquiaL Las arterias bronquia les proceden de
te es un tensioactivo. Estos fosfo lípidos están dispuestos como una lámi- la aorta y de las intercosta les e irriga n la t ráq uea inferior y los bronquios
na en la superficie alveolar, pero para que se exti endan adecuadamente, hasta los bronquiolos respiratorios.
es necesaria la presencia del ca lcio y las apoprot eínas. Por todo esto, el
surfacta nte aumenta la dist ensi bil idad pu lmona r e impide el cola pso al- La art eria pulmonar se divide debajo del cayado aórtico en dos ramas,
veola r. Su síntesis es detectable en líquido amn iót ico desde la sema na 34 derecha e izquierda, que acompañan a los bronq uios y se rami fi can de
de gestación. modo sim ilar hasta los bronqu iolos term inales, y a partir de aquí las arte-
rias se distri buyen pa ra irriga r el lecho ca pilar alveolar.
Si existe una altera ción del epitelio alveolar (de los neumocitos tipo 11),
un defecto de perfu sión o la hipoxemia mantenida, dismi nuye su pro- El diámetro de las arterias pu lmo nares es similar al de los bronquios
ducción, meca nismo im plica do en los cu adros de di strés respiratori o del acompaña ntes.
lactante y de l ad ulto.
La sa ngre oxige nada de los ca pilares se recoge por las vénulas pul mona-
La membrana basa l est á form ada por colágeno tipo IV y lipo proteínas res, que co nfl uyen entre sí, t ranscurriendo entre los lobulillos y lóbu los
(fi bronectina) y hay dos, la del endot elio vascular y la del epitelio alveolar. pul mo nares, fo rm ando las cuatro venas pulmonares (dos derechas y dos
Entre ambas membra nas hay un espacio, que es el tejido intersticial, con izquierd as) que desem boca n en la aurícu la izq uierda.
fib ras colágenas de tipo 1y 111 (re lación 2:1 para el t ipo 1), fib ras elásticas y
células (como fibrob lastos). Las venas bronq uiales drenan direct amente en las pulmonares (es una
causa norma l de cortocircuito arteriovenoso).
Por último, en el interior de los alvéo los está el líqu ido de revest imie n-
to epitelia l, que contiene susta ncia t ensioactiva, fi bronectina e inmu- Las arterias pulmona res y sus ramas son mucho más ext ensib les que las
noglobul inas. Un id os de forma laxa a las cé lulas del epi telio alveolar sistém icas, y su paredes son m uy finas (contienen escasas fib ras de mús-
o li bres en la luz, est án los macrófagos alveo lares, los li nfocitos y un culo liso). Las arteriolas tienen una capa muscular única, y son los vasos
pequeño número de pol imorfonucleares. Estas capas (epite lio alveo- que más contribuyen a la resistencia vascular.
lar, endotelio vascular, las membranas basa les, intersticio y el líqu ido
de revestim iento) co nst ituyen la membrana de int ercambio, de una Dentro de las paredes alveolares no existen linfáticos, pero sí en el tej ido
delgadez extraord inaria. conj unti vo pleural, pa redes de venas, arterias y vías aéreas.

" El surfactante se produce por los neumocitos tipo 11, y la hipoxe-


Ideas clave 1!6 mia o la ausencia de perfusión impiden su síntesis.

" La vía aérea inferior se divide en zon a de conducción (desde la " Las arterias bronquiales proceden de la aorta y de las intercosta-
tráquea hasta el bronquiolo terminal), zona de transición (bron- les, e irrigan la zona de conducción .
quiolos respiratorios) y zona resp iratoria (conductos alveolares
y sacos alveolares). " Las venas bronqu iales con sangre venosa drenan en las venas
pu lmonares y causan el shunt fisiológico.
" El volumen de la zona de conducción se denomina espacio
muerto anatóm ico, y ocupa 150 mi. " Las arterias pulmonares se dividen de la misma forma que los
bronquios e irrigan la zona respiratoria (alvéolos).
" El acino pu lmonar es la zona de parénquima dista 1a un bron-
quiolo terminal.

1) The ABG has an acute resp iratory acidos is.


Case Study 2) ABG demonstrates a m ixed acidosis.
3) In t his case, th ere is chro nic res piratory fa ilu re w it h hypoxem ia.
A COPO patient presents arterial blood gas values: pH 7.4, Pa0 2 4) The 0 2 grad ient (Aa) is probably 10 mmH g.
58 mmHg, PaC0 2 46 mmHg. Before a visit to the emergency
room, infectious decompensation and blood gas is obtained Correct answer: 4
which shows: pH 7.36; Pa0 2 50 mmHg; PaC0 2 60 mmHg; bicar-
bonate 35 mEq/L. One of the following is correct:
Neumología

Malformaciones

fanc ia, pero en caso de no existir lesiones asociadas, el pacien te puede vi -


Centrarse en las Ideas Clave.
ORIENTACIÓN vir asi ntomático hasta la edad adu lta, con un pu lmón único funcionante.
El resto sólo merece
ENARM una lectura comprensiva
sin detenerse en detalles. Radiológ icamente, se observan signos de atelectasia masiva. La fib ro-
broncoscopia, la TC torácica y la arteriografía da n el d iag nóstico.

No se debe co nfund ir estos cuadros con la d isplasia bro ncopulmonar,


2.1. Malformaciones pulmonares (Figura3l que no es una ma lformación, sino una alteración en relació n con ciertos
proced im ientos terapéut icos, como la oxigenoterapia en neonatos en al-
tas concent rac io nes.
Algunas de las malformaciones pu lmonares se exponen a cont inuación .
Quistes broncógenos. Resultan de la ramificac ión anormal del árbol
Agenesia. Es la ausencia completa de un pu lmón o de ambos (no existe traqueobronq uial dura nte su desa rro llo. Pueden local izarse en el parén-
parénqu ima, bronqu ios ni vasos). quima pu lmonar (genera lmente tercio medio) o en el mediastino (pa-
raesofágicos, paratraquea les, parahil iares o subcarinales; esta última es la
Aplasia . Consiste en la existencia de un bronqu io rudimenta rio que ter- localización más frecuente). Generalmente, no tienen comunicación con
mina en fondo de saco (no hay vasos ni parénquima). el árbol bronqu ial y son un hallazgo casua l en la radiografía de tórax. El
paciente se ma ntiene asint omático hasta que el q uiste se infecta, con lo
Hipoplasia. Existe un bronquio completamente formado, au nque de ta- que surge comunicación con el árbol bronquia l y aparecen hemoptisis y
maño reduc ido, q ue termi na en una estructura parenq ui matosa no fun- expectoración puru lenta.
cionante. Vasos hi poplásicos.
Secuestro broncopulmonar. Consiste en la presencia de una masa de
tejido pulmo nar separada del parénq uima sano, sin co municación con
la vía aérea. El aport e sang uíneo proviene de una arteria sist émica (más
frecue ntemente de la aorta torácica o abdom ina l, pero en ocasiones de
una intercosta l). A menudo, el pu lmón secuestrado contiene ai re, pese
a no estar com unicado con la vía aérea, que llega hasta aquí a través de
m inúsculas conexiones alveolares.

La característica que define al secuestro es el aporte


sanguíneo desde la ci rc ulac ió n sistém ica.

Agenesia Aplasia

Existen dos tipos de secuestro:


Sec uestro int ralobar. Carece de pleura propia (está dentro de la pleu-
ra visceral de un lóbulo norma l).
Secuest ro extraloba r. Difie re del ante ri or fu ndamenta lmente en tres
Figura 3. Algunas malformaciones pulmonares aspectos:
Tiene pleura propia.
Estas tres enfermedades se asocian, en un 60% de los casos, a anoma lías El d renaje venoso es anormal, dirigiéndose a la circulación sisté -
ca rdiológicas o de grandes vasos que provocan la muerte durante la in- m ica, creando así un cortocircuito izquierda-derecha.
Neumología \ 02
Anatómicamente está en relación con el hemidiafragma izquier- quias. La presencia o no de clínica se ha relacionado con el tamaño
do en más del 90o/o de las ocasiones. Puede estar en el espesor de la fístula, que condiciona el volumen de sangre que recorre el cor-
de este músculo o en comunicación con el esófago. toc ircu ito derecha-izquierda. Así, cuando este volumen sanguíneo es
significativo, el paciente prese nta hipoxemia por shunt, que no seco-
En referencia a la clínica de los secuestros, suelen ser asintomáticos hasta rrige admin istra ndo oxíge no al 1OOo/o. Son caracte rísticas la platipnea
que se infecta n, momento en el que aparecerán hem opti sis y expecto- (d isnea que empeora con el ortostat ismo y m ej ora con el decúbito) y
ración purulenta. En caso de existir relación con el esófago, el paciente la ortodesoxia (desatu rac ión con el ortost atis m o, al aumentar el aflujo
puede presentar disfagia y/o hematemesis. Radiológicamente, se suelen sanguíneo de la fístu la por la gravedad). Radiológ icamente, la imagen
manifestar como una masa en el ángu lo costofrénico posterior izqu ierdo clásica es una masa homogénea, redondeada, algo lobulada, más fre-
(Figura 4). cuentemente en lóbulos inferiores en la que, en ocasiones, se pueden
identificar los vasos aferente y eferente. El diagnóstico se confirma
med iante arteriografía.

Fístulas traqueoesofágicas. Se exponen en la Sección de Pediatría. En


la mayoría, el segmento proxima l del esófago termina en saco ciego y el
distal se comun ica con la tráquea (Figura 5).

Tipol ll Tipol Tipo IV


87o/o So/o 4o/o

Figura 4. Rx de tóra x y arteriografía de un secuestro broncopulmonar


tr:
[
El t ratamiento es la extirpación qu irúrgica, aun cuando se trate de un ha-
llazgo casual, dada la posibilidad de complicaciones. El estudio preope-
ratorio incluye la realización de una arteriografía (para localizar el vaso
\\Y,¡
de irrigación anormal) y un esofagograma (para desca rtar com unicación
con el esófago).
~[
Tipoll Tipo V
< 1o/o < 1o/o
Malformación adenoide quística. Radiológicamente, es típica la ima-
gen en "queso de Gruyére'; originada por la existencia de grandes quistes.
En ocasiones, los quistes atrapan aire de forma va lvular, ocasionando in- Figura 5. Clasificación de las fístulas traqueoesofá gicas
suficiencia respiratoria aguda y la muerte del rec ién nacido.

Drenaje venoso pulmonar anómalo (DVPA). Puede ser parcial o tota l. 2.2. Malformaciones
En esta enfermedad, la sangre que proviene de l pa rénqu ima pulmonar
drena en la aurícula derecha o en algu na de las venas cavas. Esta patolo-
de la pared torácica
gía se detalla en la Sección de Cardiología y cirugía cardiovascular en el
Capítulo de Cardiopatías congénitas. Costilla cervical. Es la causa más frecuente de compresión del estrecho
torácico superior (este cuadro se expone detenidamente en la Sección
Síndrome de la cimitarra (pulmón hipogenético) . Consiste en una de Traumatología). Este espacio está situado entre la primera costilla y los
hipoplasia pa renquimatosa y de la arteria pulmonar, junto a un drenaje escalenos anterior y medio. Por él discurren la arteria subclavia y el plexo
venoso anómalo, a través de una vena pu lmonar d ilatada que finaliza braquial, lo que explica que la clínica compresiva consista en ausencia de
en la cava inferior. Típicamente, en la rad iografía de tórax se visualiza la pu lso y parestesias en el brazo afectado (Figura 6).
vena anóma la como u na image n cu rvi línea pa raca rd íaca, sim il ar a una
cim itarra . Pectus excavatum (tórax en embudo). Depresión de la porción infe-
rior del estern ón y cartíl agos adyacentes. En ocasiones coexiste con otras
Fístulas arteriovenosas. La mit ad de los pac ientes presen tan tam- malformaciones, como pie equino-varo, sínd rome de Marfan o síndrome
bién fístulas en otros órga nos (t elang iectasia hemo rrág ica hereditaria de Klippei-Feil. Genera lmente asintomático, aunque en ocasiones puede
o enfermedad de Rendu -Osler-Weber). En ocasiones son asintomá- orig inar arritmias benignas y un soplo funcional. En el electrocardiograma
ticas, otras veces se manifiestan con cianosis, poliglobulia o acropa- se puede apreciar desviación del eje a la derecha. La intervención qui-
Manual CTO de Medicina y Cirugía, 2. a edición

rúrgica suele tener ind icación estética, ofreciendo los mejores result ados
cuando se practi ca entre los siet e años y la adolescencia .

Pectus carinatum (tórax en qu illa). Su incidencia es d iez veces meno r


que el anterior. También su ele ser asinto mát ico, interviniéndose por mo-
t ivos estéti cos.

Esternón bífido. Se prod uce por un fallo en la fu sió n de las ba ndas es-
terna les, creá ndose un defect o en la pared t orácica anterior. En ocasio nes
se asocia a ectopia cordi s. La intervención consiste en la aproximación de
los segmentos estern ales. En caso de no ser posib le, se recu rre a la colo-
cación de una prót esis o de auto inj ertos.

Síndrome de Poland. Ausencia co ngénita del músculo pectoral mayor


asociada a sindactilia homo lateral. A veces t ambién existe hipoplasia de
otros músculos torácicos, ca rt íl agos costa les, costillas o mami la. Se cree Arteria su bclavia
que es debido a un esca so aporte sa ng uíneo en el territorio de la subcla-
via durante el embarazo. Figura 6. Costilla cervica l

" Predomina en lób ulos inferiores.


Ideas clave R)
" El diagnóstico se establece por arteriografía.
" El secuestro pul monar se caracteriza porque recibe aporte san-
guíneo exclusivamente de la circulación sistémica, casi siempre " El tratamiento es q ui rú rgico.
de la aorta.

" Puede ser intra lobar, si carece de p leura visceral propia, o extra-
lobar, cuan do sí t iene pleura vi sceral propia.

1) Horner syndrome.
Case Study : · 2) Pancoast tumor.
3) Shoulder d islocation.
A 35-year-old woman presents to her physician with paresthesia 4) Thorac ic o utlet o bstruction.
of the left shoulder and arm. She has also noticed a hard, bony
structure on the left side of her neck above the clavicle. Which of Correct answer: 4
the following diagnoses best accounts for her symptoms?

02 · Malformaciones
,
Neuma lo

Fisiología y fisiopatología

ORIENJACIÓN Conocer el funcionamiento y la fisiopatología respiratoria permite abordar de forma adecuada


el estudio de la asignatura. Se deben conocer bien los trastornos ventilatorios, los mecanismos
ENARM de hipoxemia y el tratamiento de la insuficiencia respiratoria.

El aparato respiratorio está formado por el sistema nervioso centra l y pe- están interrelacionadas. Existe un nivel de volumen pulmonar en el que
riférico (que coordinan el funcionam iento del resto de estructuras), los la tendencia de los pulmones a contraerse y la tendencia opuesta de la
pulmones y las vías aéreas, la vascularización pulmonar y la caja torácica pared torácica a expandirse son iguales, denominándose capacidad fun-
(tanto la parte muscular como la osteocartilaginosa). Si se produce una cional residual (CFR), que es, por así decirlo, la posición de reposo del
alteración en cua lquiera de estos elementos o en la relación entre ellos, aparato respiratorio (Figura 7.C).
acontecen alteraciones en la función resp iratoria. En este capítu lo se van
a estud iar las alteraciones de la func ión ventilatoria, las de la circulación Para conseguir un volumen pu lmonar d iferente del de reposo (CRF), hay
pulmonar y las del intercambio gaseoso. que modificar las presiones a las que están sometidos los pulmones y la
caja torácica mediante la contracción activa de los músculos inspiratorios
y/o espiratorios.
3.1. Ventilación pulmonar
Simplificando, durante la inspiración, la fuerza muscular vence la tenden-
cia a la retracción del pulmón y la caja torácica, pero a med ida que los
Fisiología pulmones se llenan de aire, como si de un resorte se tratara, esta fuerza
elástica es mayor, por lo que llega un punto en q ue se igua la a la fuerza
Los pulmones son unas estructuras elásticas, puesto que contienen com- muscu lar, no pudiendo incorporar más volumen al espacio aéreo. Ésa es
ponentes fibrilares que le confieren resistencia a la expansión de volu- la capacidad pulmonar tota l (CPT).
men. Por ello, en condiciones norma les, el pulmón contiene aire (en su
interior) gracias a la existencia de una presión positiva en su interior, en el La espiración desde la CPT hasta la CFR es, pues, un proceso pasivo indu-
espacio aéreo, y una presión negativa externa, en el espacio pleural. cido por esa fue rza elástica que hace vo lver al pulmón a su posición de
reposo. Para seguir expulsando aire hasta un volumen inferior a la CFR,
Se denomina presión transpulmonar (PTP) a la diferencia resultante de la es necesaria la contracción de los múscu los espiratorios, pero también
presión interna menos la presión externa mencionadas. La relación en- aparece una fuerza elástica que tiende a expandir los pulmones (y, por
tre la presión de distensión y el volumen de aire contenido en los pul- tanto, a evitar su colapso) y la caja torácica, fuerza que es mayor a medida
mones se suele representar med iante una curva de presión-volumen que se aleja de la CFR (como un resorte), hasta que llega un punto en
(Figura 7.A). Como más adelante se comenta, se denomina comp liance que iguala la fuerza muscular, no pud iendo vaciar más contenido aéreo
o distensibilidad al cambio de volumen en relac ión con el cambio de pre- (volumen residual, VR).
sión.
El desplazamiento del aire desde la atmósfera a los alvéolos tiene que
La pared torácica es también una estructu ra elástica, que una presión de vencer una resistencia doble:
distensión positiva expande la pared, y que una presión de d istensión Resistencia aérea (raw, del inglés airway resistance), se rige por las
negativa la comprime, pudiendo representarse este hecho en una curva leyes de la tlu idodinámica. Según la ecuación de Poiseuille, el princi-
similar (Figu ra 7.8). pal determinante es el rad io de la sección transversal del conducto.
El SOo/o de esta resistencia corresponde a las vías aéreas superiores. El
En circunstancias norma les, los pu lmones se ajusta n a la pared torácica, resto se divide entre el 80o/o que generan la trá quea y las ocho prime-
de modo que las fue rzas y presiones que actúa n sobre estas estructuras ras generaciones bronquia les, y el 20o/o q ue origina la vía aérea distal.
Manual CTO de Medicina y Cirugía, 2.a edición

Volumen Volumen 1 pulmonar total (CPT), capacidad vital


CPT 1 CPT
................................ ... ....~:·~~~:·:·~ -~~ :: ;·:·;·;·; .,.•.. '
.....................................................¡. ...................
1
(CV), capacidad inspiratoria (CI) y ca-
pacidad funcional residua l (CFR).
1
1
..········· 1 CFR Las abreviaturas ing lesas de estos vo-

-~;..;~/' _ _ CFR
/
/
1 lúmenes y capacidades son respecti-
vamente: RV, VT, ERV, IRV, TLC, VC, IC y
/// FRC.
VR
,;
............................................................................. - < . . ......... . . . .............. ......... . . . . . . ~~... . La CPT es el volumen de gas que con-
tienen los pulmones en la posición de
o 10 20 30 40 -40 -30 -20 -10 o 10 20 máxima inspiración (aproximadamen-
Presión (cm H20) Presión (cm H20) te 5.800 mi). La CV es el volumen de gas

7.A. Curva presión-volumen


espi rado máximo t ras una inspiración
7.8. Curva presión-volumen
pulmona r pared t orácica máxima (aproximadamente 4.600 mi).
El VR es el volumen que contienen los
pulmones después de una espiración
máxima (aproximadamente 1.200 mi).
Volumen
.......................................................................... .........¡.1................................................................
CPT
.
El VC es el volumen que moviliza un
1 1 .. ········
ind ividuo respirando en reposo (apro-
1 .. ····
....····· ximadamente 500 m i). El VRE es el volu-
men que se puede espirar después de
/ / CFR
···································································/ ······· ....................••~ ............................................................. .
....·
una espiración normal (aproximada-
/
/ mente 1.1 00 m i). El VRI es el volumen
/ :
;
//
:
/ que se puede inspirar después de una

~" : inspiración norma l (aproximadamente


~- ¡ ~
3.000 m i). La Cl es el volumen máximo
······································································· ··•····································································
inspirado (aproximadamente 3.500 mi).
Como ya se comentó, la CFR es el volu-
-40 -30 -20 -1 0 o 10 20 30 40
men de gas que contienen los pulmo-
Presión (cm H20 )
nes después de una espiración normal
Figura 7. Curvas de presión-volumen (aproximadamente 2.300 mi). Algunos
volúmenes pu lmonares estáticos se
Estas resistencias se determ inan mediante oscilometría . pueden ca lcu lar med iante espirometría, pero para medi r el VR, y por tanto,
Resistencia elástica, de la que ya se ha hablado, po r la oposición a la la CFR, y la CPT se hace necesario emplear la pletismografía corpora l (más
deform idad inspiratoria que ofrecen las estructu ras elásticas del pul- precisa) o la técnica de d ilución de helio.
món y la pared torácica. Se expresa como el incremento de volumen
en relación al incremento de presión. Ese cociente volumen/p resión
se denomina distensibilidad, o compliance, es decir, que a menor 8 Vo lumen (litros)
distensibi lidad mayor resistencia a la entrada de aire. Característi- 7
camente, la distensibi lidad disminuye en los procesos intersticiales
CPT 6
con formación de tej ido fibroso y aumenta en los que se produce
destrucción del tejido elástico, como es el enfi sema. La elastancia 5
Cl
representa la fuerza de retroceso elástico del pulmón.
cv 4

Parámetros que evalúan


la función ventilatoria 2

CFR
Se estudian dos tipos de volúmenes pulmonares: los estáticos y los diná-
micos.
Figura 8. Volúmenes pulmonares estáticos
Volúmenes pu lmonares estáticos
Además de los mencionados volúmenes pu lmonares estáticos, en un ci-
Miden el volumen de gas q ue contiene el pul món en distintas posiciones clo resp irato ri o norma l conviene recorda r estos cuatro conceptos:
de la caja torácica (Figura 8). Espacio m uerto anatómico: consta de unos 150 mi de aire conte-
nidos en la parte de la vía aérea que no participa en el intercambio
Se habla de cuatro volúmenes: volumen residua l (VR), volumen corriente gaseoso, es decir, de la nariz a los bronquiolos terminal es.
(VC), volumen de reserva espiratorio (VRE), volumen de reserva inspirato- Espacio muerto alveolar: es el aire contenido en alvéolos no perfun-
rio (VRI), y cuatro capacidades, que son suma de los anteriores: capacidad didos, que no intervienen por tanto en el intercambio de gases. En

03 · Fisiología y fisiopatología
Neumología 1 03
personas sanas es despreciable, pues todos los alvéolos son funcio- Los valores de volúmenes estáticos y dinámicos que se han mencionado
na les, pero aumenta en ciertas enfermedades como el TEP, enferme- son los normales para un individuo sano y joven, pero deben ajustarse
dades intersticiales, etcétera. según edad, sexo y ta lla de la persona. Se considera normal si el va lor
Espacio muerto fisiológico: es la suma de los dos anteriores. encontrado de cua lquiera de los parámetros se encuentra entre el 80 y
Ventilación alveolar: es el volumen qu e participa en el interca mbio el1 20% del esperad o para el paciente, seg ún sus datos antropométricos.
gaseoso por unidad de tiempo.
Conviene reseñar el concepto de flujo espiratorio máximo o indepen-
Volúmenes pulmonares dinámicos (Figura 9) dencia del esfuerzo del flujo espiratorio fo rzado. Durante la espiración
forzada, inicialmente los flujos aumentan a medida que aumenta la fuer-
Introducen en su defin ición el factor tiempo, por lo que se estudian ade- za muscular hasta alcanzar un máximo (el pico de flujo espiratorio, peak
más flujos (volumen/tiempo). Para su medida, se utiliza el espirómetro. expiratory flow, PEF). A partir de ese momento, por mucho que se incre-
El individuo llena de aire sus pu lmones hasta la CPT y luego realiza una mente la fuerza muscular espiratoria, el flujo de aire no puede aumentar
espiración forzada durante un mínimo de 6 segundos. Los volúmenes más. Esto ocurre porque el esfuerzo muscular espiratorio crea una eleva-
pulmonares dinámicos principa les son : ción de presión de la caja torácica sobre los pulmones que los "exprime"
La capacidad vita l forzada (CVF), que representa el vol umen t ota l y hace que se vacíe n, pero esa presión también se transmite sobre los
que el paciente espira mediante una espiración forzada máxima. bronquiolos, comprimiénd olos. Se ha demostrado que a partir de ese
El volumen de gas espirado en el primer segundo de la espiración mome nto los mecanismos que determinan el fluj o espiratorio son la re-
forzada (VEF,, FEVJ tracción elástica del pulmón, la resistencia de la vía aérea entre el alvéolo
El flujo de aire en la parte media de la espiración, es decir, entre el y el lugar donde se produce la compresión de la vía aérea (punto de igual
25% y el 75% de la CVF (F EF25%-75% o MMEF, velocidad máxima del presión) y la capacidad de distensión de la vía aérea en ese punto. Así,
flujo mesoespiratorio), se mide en litros/s. El FEF25%-75% es la me- aunque aumente la presión sobre el pulmón, no se consigue incrementar
dida más sensible de la obstrucción precoz de las vías respiratoria s, el flujo espiratorio.
sobre todo, de las de pequeño calibre, por lo que suele ser la pri-
mera alteración detectada en fumadores. Otra prueba para detectar Este flujo espiratorio máximo es mayor cuando los pulmones están llenos
obstrucción precozmente es la determ inación del volumen de cierre de aire que cuando est án vacíos, pues si el volu men pulmonar es menor,
pulmona r med iante el lava do de N2. la retracción elástica, que t iende a mantener abierta la vía aérea, se hace
La relación VEF/CVF, que se conoce como índ ice de Tiffeneau (valor menor, siendo más fác il que se colapse. Esto explica por qué los pacientes
patológico menor de 0,7). afectados de un trastorno obstru ctivo tienen una CVF menor que la CV.
debido al colapso precoz de la vía aérea en la espiración forzada en el
punto de igual presión que impide al aire salir y provoca atrapamiento
Volumen aéreo. El flujo de aire espirado se puede representar en relación al vo-
CPT
lumen pulmonar, obten iendo así la denom inada cu rva flujo-vol umen
(Figuras 1 O y 11 ).

Flujo
Normal
{1/ min)

e:
·O
·¡:¡
~
·c.
"'
UJ

a:
> Vol umen
Tiempo
Segundos
CPT VR

Volumen

Figura 1 O. Curva flujo-volumen normal

Si se representa n también los fluj os insp irato rios, se obtienen las asas
de flujo-vo lu men. Cuando el paciente t iene los pulmones llenos de
aire (CPT) y empieza la espiración forzada, el flujo de aire aumenta
Tiempo rápidamente hasta su va lor máximo (u nos 400 1/min), y luego des-
Figura 9. Volúmenes pulmonares dinámicos ciende de forma progresiva y li neal hasta que deja de sa lir ai re (VR). Sin
Manual CTO de Medicina y Cirugía, 2. a edición

embargo, la inspiración forzada desde el VR consigue el pico de Aujo A medida que avanza la enfermedad, se observan disminu ción pro-
inspi ratorio en la parte med ia de la inspiración, por lo que la curva gresiva del FEF 25 %·?S%' del índice de Tiffeneau, aumento del VR con
t iene forma de U. CPT normal o aumentada, así como aumento de la relación VR/CPT y
descenso de la CV por aumentar el VR. Se acepta que, en adu ltos, un
descenso de l índ ice de Tiffe neau por debaj o de 0.7 define el trastorno
Flujo
{1/ min ) Normal obstructivo.
Restrictivo
parenquimatoso
Obstrucción de la vía aérea superior. La forma de la curva Aujo-volu-
men es el método más sensible para detectar una obstrucción de la
vía aérea superior (Figura 13). Se describen tres patrones: la obstru c-
Obstructivo
ción fija, que afecta por igua l a la rama inspiratoria y a la espiratoria, la
obstrucción variable intratorácica, que afecta fundamentalmente a la

.···. .. rama espiratoria, y la obstrucción variable extratorácica, que reduce los

. ··...
/. Aujos inspi ratorios.

f · · .....
·······
La obstrucción se define por un coeficiente VEF/CVF
Figura 11. Curvas Aujo-volumen. Patrones respiratorios patológ icos que está dism inu ido.

La presión inspiratoria máxima (PIM) y la presión espiratoria máxima


(PEM) son parámetros que valoran la fuerza muscular desarrollada en una El diferente comportamiento en estas dos últimas se debe a la inAuencia
inspiración o espiración forzada contra una vía aérea ocluida. Tienen inte- de las presiones pleurales sobre la vía aérea: durante la espiración forzada
rés en las alteraciones restrictivas. se ve comprimida la vía aérea intratorácica, por lo que afecta más a la
espirac ión, mientras que durante la in spiración la porción intratorácica
Patrones de función anormal t iende a dilatarse por efecto de la presión negativa pleu ral y la vía aérea

Guiándose por las alteraciones en


los vo lúmenes pulmonares estáticos
y dinámicos, las alteraciones ventil a- Bronquiolitis
Obstructivas 1- - - _ . . t CPT, t VR, IT<0,7
torias se clas ifican en obstructivas y Bronquiecta sias
Linfangioleiomiomatosis
restrictivas (Tabla 1 y Figura 12).
Histiocitosis X

Restrictivas
Alteraciones obstructivas ib rosis pu lmonar idiopática
Otras enfermedad es intersticiales
Parenquimatosas _ _ _....._ ..1- VR, IT <:: 0,8 Sa rcoidosis
Se caracte ri za n por una dific ultad
- - - {Neumoconiosis
para el vaciamiento pu lmonar, aun-
Extraparenquimatosas
que la entrada del aire sea norma l o
casi normal, que se traduce en una
lnspiratorias lnspiratorias-espiratorias
disminución en la velocidad del Aujo
IT=O,B; Í VR IT variable; t VR
espi ratorio para cua lquier volumen
· Guillain-Barré
pulmonar y un aumento del volu- Enfermedades
Pará lisis diafragma · Distrofias musculares
neuromusculares
men residua l. Inicialmente d isminu- · Miastenia gravis
ye el FEF 25%_ 75% y se altera la fase fina l Alteraciones · Cifoescoliosis
de la curva Aujo-vol umen espiratori a, de la caj a torácica Espondilitis anquilosante
·Obesidad
que tiende a hacerse cóncava por la
reducc ión en el Auj o de sa lida de aire. Figura 12. Enfermedades respiratori as frecuentes por categorías diagnósti cas

TIFFENEAU
Obstruct iva Not Not No . J. ..j.. ..J..(<70%) N N
Restrictiva parenquimatosa ..j.. ..j.. ..j.. No . J. Not N N
(<::80%)
Restrictiva extraparenquimatosa ..j.. . J. o N ..j.. No . J. Not No . J. N
inspiratoria (<::80%)
Restrictiva extraparenquimatosa ..j.. t ..j.. No . J. Variable
inspiratoria y espiratoria
*Disminuidas en las causas de origen neuromuscular
Tabla 1. Alteraciones de la función ventilatoria

03 · Fisiología y fisiopatología
Neumología 1 03
Flujo
ejemp lo, por debilidad o parálisis del
Espi ración diafragma, músculo excl usiva mente
inspiratorio; no así los intercostales,
que intervienen en la inspiración y es-
piración forzadas. Sin embargo, el VR y
el flujo durante la espiración no suelen
afectarse.

Volumen En los casos de restricción extrapa-


renquimatosa con disfunción de la
inspiración y la espiración, al pulmón
le cuesta tanto llenarse de aire como
vacia rse, por lo que el VR suele au-
mentar. El índice de Tiffeneau puede
In spi ración disminuir por la dificultad del vaciado,
Obstrucción fija Obstrucción variable Obstrucción varia ble pero si hay integ ri dad muscular espi-
int ratorácica extratorácica
ratoria y el defecto reside en la pared
Figura 13. Alteraciones obstructivas de la vía aérea superior torácica, puede incluso aumentar.

extratorácica se ve afectada por la presión negativa en la vía aérea, que


tiende al colapso.
Restricción parenqu imatosa, VR bajo. Restricción ex-
traparenquimatosa inspiratoria y espiratoria, VR nor-
Alteraciones restrictivas mal o alto.

Se caracterizan por dificultad para el llenado de aire pulmonar, que


origina una disminución en los volúmenes pu lmonares, especialmen- Regulación nerviosa de la ventilación (Figura 15)
te la CPT y la CV. El diagnóstico de alteración restrictiva se establece
en presencia de una CPT < 80% del va lor teórico. Según dónde se Existen dos sistemas de control, uno voluntario y otro involuntario.
localice la restricción al llenado, se clasifican en parenquimatosas (en
los pulmones) y extraparenqu imatosas (en la pared torácica o el sis- El sistema voluntario se localiza en las neuronas de la corteza cerebral y es
tema neuromuscular). Estas últimas, a su vez, se d ividen en dos gru- responsab le de la capacidad de estim ular o inh ibir el impulso respiratorio
pos: unas en las que predomina la restr icción durante la inspiración, de forma consciente.
y otras en las que se afectan tanto la inspiración como la espiración
(Figura 14). El control automático o involuntario se ubica principalmente en un
centro bulbar, que es el más importante por ser origen de los estímulos
inspiratorios regulares, que se ve influenciado por diversos factores que
estimulan dicho impulso. Así, el incremento de la PaC0 2, el descenso
En las restricciones, la CPT es siempre baja. Este pará-
de la Pa0 2, el descenso del pH y el aumento de temperatura del líquido
metro no se puede calcu lar con la esp iro metría sim-
ple. cefalorraqu ídeo son estimulantes de la venti lación, siendo la hipercap-
nia el má s importante de todos ellos en cond iciones normales. Esto se
debe a que el principal estimu lante directo del centro bulbar es el ion
En las parenquimatosas, como la fibrosis pulmonar idiopática, al pulmón W, que se forma in situ en el LCR por formarse ácido carbónico (H 2CO)
le cuesta llenarse de aire por la rigidez que presenta el parénquima, y el de la unión C0 2+Hp, que se d isocia en anión bicarbonato (HC03·) y H+.
VR suele disminuir con un flujo espiratorio normal o casi normal. Cuan- Este último atraviesa mal la barrera hematoencefálica, por lo que los
do la enfermedad es extraparenquimatosa por disfunción inspiratoria, cambios en el pH sanguíneo no afectan tanto al impulso venti latorio
también predom ina la dificultad para llena r de aire los pulmones, por como a los cambios bruscos en la PaC0 2, que sí d ifunde fác il mente.

- Patrón obstructivo

Capacidad
pulmonar Alteración parenquimatosa
total (CPT) pulmonar
No ------ o extraparenquimat osa
inspiratoria *

Si ______ Alteración extrapa renq uimatosa


• Estos patrones pueden ser indistinguibles con estas pruebas, aunque el VR tiende a estar más disminuido en el parenquimatoso inspiratoria y espiratoria

Figura 14. Esquema de patrones espirométricos


Manual CTO de Medicina y Cirugía, 2. a edición

do la inspiración, siendo el principa l


Centro
pneu motáxico
determinante de la duración de la
(protuberancia) m isma. Así, el estímu lo intenso des-
Control voluntari o
de este núcleo hará las inspiracio-
(córtex)
nes más cortas e incrementará, por
tanto, la frecuencia resp iratoria. Es
tema de discus ión la existencia de
un núcleo protuberancia! apnéusti-
co cuya func ión es inversa a la del
pneumotáxico.

Quimiorreceptores centrales
Aumento PCO,IT.'
Quimiorreceptores 3.2. Circulación
periféricos
Dism inución pH Disminución p0 2 pulmonar

El sistema vascu lar pu lmonar está


M ecanorreceptores formado por una red de vasos di-
Pulm ón
ferentes de los de la circulación
sistém ica. Las paredes arteriales y
arteriolares son mucho más finas, y
en consecuencia, la res istencia que
oponen al flujo sanguíneo es mu-
cho menor, por lo q ue las presiones
medidas en territorio pu lmonar son
mucho más bajas q ue sus equiva len-
tes en la circu lac ión sistémica . Así, la
Figura 15. Control de la respiración presión media de la arteria pulmo-
na r ronda los 15 mmHg, frente a
los 90-100 m m Hg que existen en la aorta. Por el lo, en condiciones
La hipercapn ia es el principal estímulo respiratorio, de norma lidad, la masa muscu lar del ventrículo derecho es mucho
excepto en la EPOC, que es la hipoxem ia. menor que la del ventrículo izqu ierdo, pues debe vencer una menor
resistencia al fluj o.

Pero en pacientes con retención crónica de C0 2, como en la EPOC el Otra diferencia capita l es la respuesta a la hipoxemia. En las arterias
principal estímu lo pasa a ser la hipoxem ia, pues el centro bulbar en sistém icas, si la sangre lleva un contenido bajo de oxígeno, se produce
uno o dos días se "acostumbra" a trabajar con elevadas concentraciones vasod ilatación para aumentar en lo posible el aporte de oxígeno a los
de co2 y se hace "insensible" a su incremento, dado que el ajuste renal tej idos. Por el contrario, las arterias pulmonares responden a la hipoxia
en respuesta al aumento de PaC0 2 tiende a la retención de HCQ3·, que alveolar con una vasoconstricción, que impide perfund ir unidades al-
pasa al LCR, se une al H+ y baja su concentración. Por el lo, no se deben veolares ma l venti ladas. Con esto se logra mantener el equil ibrio ven-
emplear altas fracc iones inspiratorias de 0 2 (Fi0 2) en estos pacientes, t ilación/perfusión. Este reflejo de vasoconstricción hipóxica pulmonar
para no inhibir el estímu lo derivado de la hipoxemia, que pasa a ser el es un eficaz mecanismo para compensar la alteración sobre la Pa0 2
más importante. que producen las enfermedades pulmonares, pero si se mantiene, pro-
voca cambios proliferativos en la pared arteri al que causan hiperten-
En el control automático intervienen, además, receptores periféricos que sión pulmonar irreversible.
llevan información al centro bu lbar, como son los del seno carotídeo (a
través del glosofaríngeo) o del cuerpo aórtico (a través del vago), muy La perfusión no es igual en todas las pa rtes del pulmón, pues en bipedes-
sensibles a los descensos de la Pa0 2 (más incluso que el núcleo bu lbar), tación la presión hidrostática es mayor en las bases que en los vértices, lo
y mecanorreceptores pu lmonares, algunos loca lizados en bronqu ios y que hace que las bases estén mejor perfundidas.
bronqu iolos, que responden al estiram iento del parénquima pu lmonar
enviando seña les inhibitorias a través del nervio vago que tienden a ha- Clásicamente, se hab la de la existencia de t res zonas, debido al jue-
cer cesar la inspiración, haciéndola más corta y aumentando, así, la fre- go entre las pres io nes arterial, venosa y alveo lar (se debe recordar
cuencia resp iratoria (reflejo de Hering-Breuer), receptores de irritación de que los vasos está n eng lobados por alvéo los llenos de aire). En la
las vías respiratorias (que también orig inan la tos y el estornudo) y otros zona 1, no hay f luj o de sangre de arteria a ve na, pues la presión
receptores "J"yuxtacapilares que se estimulan al aumenta r el volumen de alveo lar es mayor q ue ambas a lo largo de todo el ciclo cardíaco.
los vasos capilares pu lmonares, como ocu rre en el edema pu lmona r car- En la zona 2, la pres ió n art erial sistó li ca supera la alveola r, pero ésta
diogénico. es mayo r que la venosa, por lo q ue el flujo es intermitente (durante
el pico sistó li co). En la zona 3, la pres ió n alveo lar es me nor que las
En la protubera ncia alta, existe además un centro pneumotáx ico otras dos, por lo q ue hay fluj o de forma contin ua, independ iente de
que envía seña les inh ibitorias al centro bu lbar cuando se ha inicia- la m isma.

03 · Fisiología y fisiopatología
Neumología 1 03
En condiciones normales, lo más aceptado es que, en bipedestación, mecanismos que intervienen en dicho proceso, es decir, la ventil ación
existe zona 2 en la parte superior y zona 3 en la inferior de los pul- adecuada de los espacios aéreos, la difusión de los gases a través de la
mones, y en decúbito sólo zo na 3. La zona 1 únicamente aparece en membrana alveolocapilar, y la adecuada perfusión de las unidades alveo-
condiciones de enfermedad (hipovolemia, hipoten sión pu lmonar... ) o lares de intercambio.
venti lación mecánica con presiones alveolares continuamente eleva-
das, como la aplicación de PEEP (presión positiva en la espiración). Ventilación

Si se produce un aumento del gasto cardíaco y, por tanto, del flujo pu l- Del volumen de aire que se moviliza en la respiración normal, no todo in-
monar, por ejemplo, en el ejercicio físico, se ponen en marcha unos terviene en el intercambio de gases (básicamente, captación de 0 2 y elimi-
mecan ismos para consegu ir que el aumento de la presión de la arteria nación de C0 2). Se denom ina ventilación total o volumen minuto al volu-
pulmonar sea muy pequeño, que son el fenómeno de reclutami ento de men total de aire movilizado en un minuto, es decir, el volumen corriente
capila res "de reserva" normalmente cerra dos y la distensión de los vasos (500 mi) por el número de respiraciones en un minuto (frecuencia respi-
(de paredes "finas·: como ya se ha comentado). ratoria en reposo: 12-16 por minuto). Como ya se ha comentado, hay una
parte de aire (150 mi) que sólo alcanza la vía aéra (espacio muerto anató-
mico) y, por tanto, no llega al lugar de intercambio gaseoso (alvéolos). Así,
la venti lación alveolar resulta de mu ltiplicar (volumen corriente- volumen
En la circulación pulmonar, la hipoxia provoca vaso- del espacio muerto): 350 mi por la frecuencia respiratoria. Ese es el volu-
constricción. men real de aire que interviene en el intercambio gaseoso en un minuto.

El parámetro fundamenta l para determinar el estado de la ventilación


El aumento de las res istencias vasculares pulmonares puede deberse a: en un individuo es la presión parcial de C0 2 en sangre arterial (PaCOJ
El reflejo de la vasoconstricción por la hipoxia alveolar (usualmente, Además, ya se comentó que la PaC0 2 es el principal mecanismo de regu-
el mecanismo más importante). lación a nivel bulbar de la ventilación. La PaC0 2 se puede estimar con la
El aumento del grosor y de la resistencia de las paredes vasculares siguiente fórmula:
por proliferación muscular y endotelial.
La presencia de trombos en el lecho capilar que disminuyen la sec- PaC0 2 = 0,8 x VC0 2 1 VA
ción transversal total del mismo.
La desestructuración de la histoa rquitectura capilar por fenómenos Donde VC0 2 representa la cantidad total de C0 2 que se produce por mi-
de fibrosis y cicatrización. Cuando esto ocurre, la presión en la arteria nuto, resultado del metabolismo celular, y VA es la ventilación alveolar
pulmonar debe elevarse para mantener el gasto cardíaco y vencer por minuto, siendo 0,863 la constante de proporcionalidad. Fácilmente se
ese aumento de resisten cia que presenta el lecho vascula r. deduce de esta fórmula que si disminuye la ventilación alveolar, aumenta
la PaC0 2.
Las arterias bronquiales (ramas de la circu lación sistémica) llevan el 1-2%
del gasto cardíaco izquierdo, irrigan las estructuras de sostén (tabiques Difusión
conjuntivos, bronquios y bronquiolos) y drenan a las venas pulmonares,
por lo que el gasto del ventrículo derecho es algo menor que el del iz- La membrana alveolocapi lar debe permitir el intercambio de los gases
quierdo. C0 2 y 0 2, que difunden por gradiente de presiones parciales desde la
sangre al alvéolo (oxígeno) y viceversa (dióxido de carbono). Conviene
Para cuantificar los parámetros de la hemodinámica pu lmonar (presión recordar que la capacidad de difusión del C0 2 es unas 20 veces mayor
arteria l pulmonar sistólica, diastólica y media, presión de enclavamien- que la del 0 , por lo que, en general, en la insuficiencia respiratoria, la
2
to pulmonar, resistencia vascu lar pulmonar... ), se emplean el catéter de disminución de la Pa0 2 suele preceder al aumento de PaC0 2•
Swan-Ganz y procedimientos matemáticos indirectos. Además, actual-
mente, la ecocardiografía permite la estimación de alguno de estos pa-
rámetros. Hay que recordar que la presión de enclavamiento pulmonar
es tan sólo unos 2 mmHg superior a la de la aurícu la izquierda y que se Hipoventilación es sinónimo de hipercapnia.
eleva si ésta aumenta, por ejemplo, en la insuficiencia cardíaca. Asimismo,
el pulmón tiene una amplia red de capila res linfáticos que se originan en
el tejido intersticial y desembocan en los ganglios hiliares, encargados de
drenar líquido, proteínas y partículas que llegan al espacio alveolar desde En la Figura 16, se representan las presiones parciales de los gases en los
el intersticio, por presentar esos capi lares presiones negativas en su inte- distintos puntos del aparato respiratorio.
rior (no hay linfáticos alveolares).
En condiciones normales, basta el tercio inicial de recorrido del capilar
junto al alvéolo (tiempo de tránsito de los hematíes a través del lecho ca-
3.3. Intercambio gaseoso pilar) para que se igualen las presiones, es decir, para que el intercambio
gaseoso tenga lugar. En los restantes dos tercios de recorrido no hay di-
fusión de gases, pues ya no existe gradiente de presiones. Por eso es raro
Recuerdo fisiológico que una alteración aislada en la difusión llegue a producir hipoxemia en
reposo, ya que queda todo este espacio de"reserva" por si hubiese alguna
Para que el aparato respiratorio rea lice de forma adecuada su función (el alteración en la membrana alveolocapilar que la engrosase o disminuye-
intercambio de gases en el alvéolo), es necesaria la integridad de los tres se su superfici e de intercambio.
Manual CTO de Medicina y Cirugía, 2 .a edición

Adecuación ventilación/perfusión
Aire ambiental Aire espirado
La adecuada rel ació n entre la ventilación y la perfusión de las unidades
P0 2= 159 mmHg P0 2= 120 mmHg
PC0 2= 0,3 mmHg PC0 2= 27 mmHg alveolares de interca m bio es necesa ria pa ra aseg ura r un correcto inter-
Espacio
PH 2 0= 3,7 mmHg muerto PH 20= 47 mmHg cam bio de gases. Es decir, q ue los alvéolos bien venti lados deben estar,
anatómico además, bien perfundid os pa ra q ue dicha vent ilació n sea útil.

Esta concorda ncia entre ventilación/perfusión (V!Q) determina la presión


parcial de 0 2 y C0 2 en la sangre que abandona cada un idad alveolocapi-
lar, y puede verse alterada, de modo que los dos extremos se comportan
de la sigu iente manera (Figura 17):
Si una un idad está poco ventilada (la re lación tiende a cero, pues el
numerador así lo hace), se comporta como un cortocircu ito (shunt)
de sa ngre venosa no oxigenada (pues no ha sufrido intercambio
gaseoso alguno) que se mezcla con la sa ng re oxigenada por otras
unidades en las venas pulmo nares y aurícula izq uierda; la compos i-
ció n de la sa ng re que sa le de esa unidad será similar a la de la sa ngre
venosa que llegó al cap ilar pulmo nar.
Sangre venosa Sangre poscapilar Si una unidad está pobremente perfundida (la relación t iende a in-
~ pulmonar
Py02 = 40 mmHg
finito), se com porta como un espacio muerto fis iológico que no in-
PvC0 2= 46 mmHg terviene en el interca m bio, y la poca sa ng re q ue sa lga tendrá unas
presio nes de 0 2 y C0 2 simila res a las del aire alveolar.

La sit uación idea l es la concorda ncia com p let a entre la ventilació n y la


perfu sió n, con lo que la V/Q ti ende al valor de l. No obstante, en bi pe-
dest ació n existe un g radiente de venti laci ó n desde los vérti ces (peor
ventil ados po r la disposició n anat ó mica de la vía aérea) hast a las bases
(mejor ventiladas), y un g radiente de perfusió n desde los vértices (peor
Shunt fisiológ ico
(arterias y venas bronqu iales) perfu ndidos) hasta las bases (mejo r perfund idos, en parte por efecto de
la g ravedad).
Pa0 2= 95 mmHg
PaC0 2= 40 mmHg
El gradiente de perfusión es más ma rcado que el de ventilación, por lo
que, en los vérti ces, la re lación V/ Q es algo mayor (luego la sangre t iene
Figura 16. Presiones parciales de los gases en las d istintas partes una Pa0 2 mayor y una PaC0 2 meno r) q ue en las bases, con lo que queda
del apa rato respiratorio compensado y el resultado g lobal de V/Q es aproximado al va lor ideal l.

Ventilación/perfusión V.fQ

La ventilación Normalidad No se perfunden los alveolos

¡
alveolar es nula (efecto espacio muerto)
(efecto shunt)

El aire alveolar está Ha habido intercambio El aire alveolar es igual


en equilibrio gaseoso entre alveolo al aire humidificado
con la sangre venosa y sangre inspirado

Aire inspirado

(Efecto shunt)
-
Arteria
pulmonar
(Normal)
pulmonar
(Efecto espacio muerto)

Figura 17. Interca mbio gaseoso

03 · Fisiología y fisiopatología
Neumología 1 03
Evaluación del intercambio gaseoso Para hallarlo, es necesario calcular la presión parcial de 0 2 en el alvéolo
(PAO), que requiere para su cálculo conocer:
Para evaluar su idoneidad, se uti lizan la gasometría arteria l, la pulsioxime- La Fi0 2 (fracción de 0 2 en el aire inspirado, 0,21 en aire ambiental,
tría y la capacidad de difusión. pues el 0 2 supone el 21% de su composición).
La presión barométrica ambiental (PB = presión atmosférica, 1 at-
Gasometría arterial mósfera = 760 mmHg a nivel del mar).
La presión parcia l del vapor de agua en el aire (PHp = 47 mmHg, si
Se obtiene una muestra de sangre realizando una punción arterial, ge- el aire está totalmente saturado).
neralmente la radial o la humera l. El análisis suele incluir el pH, la Pa0 2, La PaCOr
la PaC0 2, el HC0 3- y/o el exceso de bases (EB) y el gradiente o d iferencia El cociente respiratorio (la relación ent re producción de C0 2 y consu-
alveoloarterial de oxígeno (D(A-a)O¡l. mo de 0 2, que en condiciones normales es 0,8).

El oxígeno se transporta en la sangre de dos form as. La mayor parte, dada PA0 2 = [Fi0 2 x (PB- PHPll - [PaC0 2 1 R]
su afinidad, va un ida a la hemoglobina (formando la oxihemoglobina, he-
moglobina saturada con 0), de tal manera que cada gramo de hemog- En ind ividuos jóvenes sin enfermedad, respirando aire ambiente, el va lor
lobina saturada transporta 1,34 mi de Or del gradiente alveoloarterial de 0 1 es menor de 15 mm Hg. A medida que
avanza la edad, el gradiente normal aumenta, de modo que, en ancianos,
El porcentaje de la hemoglobina (Hb) que se encuentra saturada con 0 2 el va lor normal puede ser de 30 mmHg o más.
(%Sat) depende de la Pa0 2, sig uiendo la rel ación una curva sigmoidea
conocida como curva de d isociación de la hemoglobina (Figura 18). El transporte del C01 por la sangre difiere del oxígeno. En general, se
transportan unos 4 mi de C01 por decilitro de sang re venosa. Aproxima-
damente el 7% va disuelto en el plasma .

(%) Saturación Sangre Sangre


hemoglobina venosa arterial Un 70% es transportado en forma de anión bica rbonato. Los hematíes
son ricos en anh idrasa ca rbónica, enzima que acelera enormemente la
reacción natural del C01 con el Hp para formar ácido carbón ico, H1C0 3,
80 que se disocia en HC03- (q ue pasa al plasma) y H+ que es neutralizado rá-
pidamente por tampones intracelula res, principalmente la hemoglobina.

60 El resta nte 20-30% va unido a la hemoglobina formando la carbamino-


hemoglobina. Existe una curva de disociación del C01 y la hemoglobina
Disminución de pH similar a la del 0 1, aunque t iene forma lineal, no sigmoidea.
40 Aumenf:o de C0 2
Aumen~o de 2,3 DPG
Aument:O de temperatura La un ión del oxígeno a la hemog lobina desplaza de su unión al C01, de-
20 nominándose este hecho "efecto Haldane'; cuantitativamente incluso
más importante para el transporte de C01 que el efecto Bbhr para el 0 2.
La mejor forma de eva luar el estado de la eliminación de C01 es la PaCOr
o 20 40 60 80 100 120
P0 2 (mmHg) Pulsioximetría

Figura 18. Curva de disociación de la hemog lobina Mediante el pulsioxímetro se puede conocer el grado de saturación de la
hemog lobina por el 0 1 (%Sat).
Una pequeña proporción del 0 2 (aproximadamente el 3%) va d isuelto en
el plasma, exacta mente 0,003 m i de 0 2 por 100 ce de sangre por mmHg Se coloca una pinza o ded il en un dedo del paciente y aparece en la pan-
de Pa0 2 tall a en todo momento el %Sat, por lo que es el método de elección para
vigilar la oxigenación en pacientes críticos o inestables. Este dispositivo
En general, la mejor forma de eva luar el estado de oxigenación es la me- mide la absorción de luz por la hemoglobina en sangre arteria l emplean-
dida de la Pa0 2, aunque en ocasiones, como cuando existe un tóxico que do dos long itudes de onda, para la hemoglobina oxigenada y para la re-
desplaza el 0 2 de su unión a la Hb, como el monóxido de carbono (CO), ducida respectivamente.
el resultado puede ser normal, siendo necesario conocer el %Sat rea l para
evaluarlo. Tiene la ventaja de la rapidez y carácter incruento de la determinación,
por lo que, en los servicios de urgencias, es generalmente el método em-
pleado para realizar la primera aproximación respecto al estado de oxige-
nación del paciente que acude con comprom iso respiratori o importante.
La gra n mayoría del oxígeno se transporta unido a la Tambi én es muy útil en los estudios del sueño para evidenciar eventuales
hemoglobina . desaturaciones nocturnas.

Los principales inconven ientes de la técnica son que si disminuye la


Otro parámetro de interés que aporta la gasometría es el gradiente o di- perfus ión o la temperatura cutánea, si hay arritmias graves o temblo-
ferencia alveoloarterial de oxígeno (D(A-a)O). res importantes, la seña l del oxímetro es menos fiable, al igua l que
Manual CTO de Medicina y Cirugía, 2 .a edición

cuando existen variantes de la hemog lob ina (carboxihemog lobina y Como la DLCO mide una difusión de un gas en un líquido, su va lor sigue
metahem oglob ina), no reco nocidas po r este aparato. Sin embargo, las leyes fís icas que reg ul an este hecho. Así: "la ve locidad de difusión de
en la gasomet ría se emplea el co-oxímetro, que emp lea cuatro lon- un gas en un líqu ido (D) es directamente proporcional al g radiente de
gitudes de onda y no dos, de forma que reconoce las hemoglobinas presiones (DP), a la superficie de contacto (A) y a su solubilidad (S), e in-
oxigenada, reducida, carbox ihemoglob ina y meta hemoglobina. Ade- versamente proporcional a la distancia de difusión (d) y la raíz cuadrada
más, la oximetría es poco sensible a los camb ios de Pa0 2 que aconte- de su peso molecular (Pm)':
cen por encima de 60 mmHg, si bien esto no suele tener re levanc ia
clínica. El coeficiente de difusión en el agua es constante para cada gas; consi-
derando el va lor 1 para el 0 2, al C0 2 le correspondería 20,3; y al CO 0,81.
Cuando se uti liza el oxímetro, es fun damenta l conocer con det alle la cur- Dadas las d ificu ltades técnicas de rea liza r el cá lculo de la capacidad de
va de disociación de la oxihemog lobina (véase la Figura 18). difusión del 0 2, que es la que rea lmente inte resa, se hace una estimación
ind irecta med iante la DLCO.
Esta curva tiene forma sigmoidea, por lo que se pueden d iferenciar tres
partes. Inicialmente, con las presiones de 0 2 más bajas, la pendiente de la Según ésto, existen cinco factores fundamentales que determinan el va-
curva es pequeña, pero de menor interés, pues estos valores de Pa0 2 son lor de la DLCO:
prácticamente incompatibles con la vida. En la parte media, la pendien- La superficie de intercambio (la superficie alveolocapilar total). La
te es m uy grande, hecho fundamenta l, pues pequeñas variaciones en la causa más frecuente de d isminución en la DLCO es la pérdida de
Pa0 2 producirá n grandes camb ios en la saturación de la hemog lobina. En dicha superficie por destrucción del pa rénqu ima (enfisema, fibrosis
la parte final, la pend iente vuelve a ser pequeña (fase de "meseta"), por lo pulmonar...), hecho más importante que el propio aumento de gro-
que cambios grandes de Pa0 2 casi no afectan al %Sa t, pues ya con una sor de la membrana alveolocapilar.
Pa0 2 de 60 mmHg, el %Sat es aproximadamente del 90%, valor suficiente Concentración de Hb en la sangre, pues la Hb es la "encargada" de fijar
para asegurar una adecuada oxigenación tisu lar en la mayoría de las oca- el CO, y si existe anemia, ésta puede dar un va lor de DLCO fa lsamente
siones. La PSO es la Pa0 2 para la que la Hb se encuentra saturada al 50% bajo, ya que el CO difunde bien, pero no hay Hb que lo fije. Por este
(25 -27 mmHg). motivo, hay que corregi r el va lor de DLCO con la Hb del paciente.
Vo lumen de sang re en los capila res pu lmonares que intervienen en
Es de capital importancia conocer ta m bién los factores que mod ifican el el intercambio, por el mismo motivo (más vo lumen de sangre, más
grado de afin idad de la Hb por el oxígeno, o lo que es equivalente, que hemoglobina).
desplazan la curva de disociación a la derecha (con lo que la afin idad es El grado de discordancia entre la ventilación y la perfusión pulmo-
menor y la Hb requiere Pa0 2 mayores para captar el 0 2) o a la izquierda nares.
(Tabla 2): Espesor de la membrana alveolocapilar (distancia de difusión).

Med iante la DLCO se hace una estimación del estado funcional de la


Desplazamiento a la derecha Desplazamiento a la izquierda membrana alveolocap ilar.
Descenso del pH (efecto Aumento del pH
Bóhr), aumento Descenso de la PaC0 2 La DLCO disminuye típicamente en el enfisema (la destrucción de las paredes
de hidrogeniones de alvéolos disminuye la superficie total de intercambio gaseoso), las enfer-
Descenso del 2,3 difosfogl icerato
Aumento de la PaC0 2 (DPG) medades intersticiales (la fibrosis intersticial produce destrucción de unidades
Aumento del 2,3 Descenso de la temperatura de intercambio y del vol umen total de sangre en los capilares pulmonares), el
difosfog licerato (DPG) TEP recurrente y la hipertensión pulmonar (en las que disminuye la superficie
Aumento de la temperatura total capi lar pulmonar y el volumen de sangre capilar pulmonar).
Tabla 2. Factores que modifican la afin idad de la hemoglobina
por el oxígeno

Capacidad de difusión La DLCO disminuye siempre que la superficie total ---~


de intercambio gaseoso se reduzca.

Se estima mediante la determ inación de la capacidad de difusión del mo-


nóxido de carbono (DLCO). Se inspira una pequeña cantidad conocida
de CO mezclada con aire, se mantiene en los pu lmones durante unos 1O La DLCO aumenta en dos situaciones:
segundos y se mide la cantidad que q ueda en el aire espirado. El COque En las fases in iciales de la insuficiencia card íaca congest iva y en las
"falta" genera lmente ha d ifundido a través de la membrana alveo locapilar insuficiencias cardíacas de alto gasto (por ejemplo, la que sucede
y se ha unido a la Hb de los hematíes que pasan por los capilares alveo- en ocasiones en el hipertiroidismo), ya que aumenta el contenido
lares, si no hay fugas aéreas. La cantidad de CO absorbida por minuto y de sangre en los cap ilares pulmonares por congestión, de ahí que,
por mmHg de gradiente de presión entre el alvéo lo y la sangre capilar es al haber más hemoglobina, "secuestre" más CO. Pero si sigue avan -
la DLCO. zando la enfermedad, el edema alveo lar y el del intersticio pulmonar
d ificultan la d ifusión, y la DLCO puede ser norma l o incluso baja.
La KCO resulta de dividir la DLCO entre el valor del volumen alveolar. Am- En la hemorragia alveolar (enfermedad de Goodpasture, hemosi-
bos va lores están tabulados para la edad, sexo y ta lla de l paciente, pero, derosis pu lmona r id iopática, LES, leg ione losis, PAN microscópica,
para recordar una cifra, habitualmente el va lor de la DLCO ronda los 20 Wegener...) pues la hemog lobina de los hematíes vertidos al alvéolo
ml/min. Son necesarios volúmenes de ventilación no excesivamente pe- también captan CO, que disminuye en el aire espirado, por lo que el
queños para que el resultado obtenido sea válido. va lor de la DLCO se eleva.

03 · Fisiología y fisiopatología
Neumología 1 03
Mecanismos de hipoxemia activa que acontece. La D(A-a)0 2 se eleva. La administración de 0 2
no co nsigue correg ir la hi poxemia, si bien en la clínica se emplea
Se considera que existe hipoxem ia cuando la Pa0 2 es menor de 80 para que ayude a elevar la Pa0 2 ta n pronto se vaya resolviendo la
mmHg. Conviene hacer una distinción t erm inológ ica entre los térmi- situación que origina el shunt (por ejemplo, en una neumonía, a me-
nos hipoxemia (descenso del 0 2 en la sangre) e hi poxia (déficit de la dida que se vaya reabsorb iendo el contenido purulento, la oxigeno-
oxigenación y aprovechamient o del 0 2 en los tej idos). El aporte de terapia será más eficaz).
oxígeno a los tej idos es el producto del conten ido de 0 2 (0 2 disuelto
más unido a la hemoglobina) por el gasto cardíaco. Según el factor
afectado de esta ecuación se deduce n las causas de hipoxia: hipoxé-
mica (sus causas son las de la hipoxem ia -véase más adela nte-), ané- El shunt es el ún ico mecanismo de hipoxemia que no _._;:::...;;....
se corrige con 0 2 .
mica (falta hemoglobina, el principa l transportador sa nguíneo del 0 2,
o intoxicación por CO), o circulatoria (descenso del gasto cardíaco
o alteraciones arteria les locales que provocan isquem ia); una cuarta
causa es la hipoxia citotóxica o disóxica, que se produce cuando las Alteraciones de la relación V/Q. Es el más frecue nte de los meca-
mitocondrias no pueden utilizar el 0 2 (intoxicación por cianuro, sepsis nismos. Acontece en enfermedades de la vía aérea (asma, EPOC...),
por gramnegativos). enfermedades, intersticia les, enfermedades vasculares pulmona-
res (como el TEP, en el que se produce un aumento del espacio
Existen varios mecanismos causantes de hipoxemia, que pueden di- muerto fisiológico), etc. Ya se ha comentado que las regiones con
ferenciarse según el va lor de la PaC0 2, de la D(A-a)0 2 y la respuesta al una relación V/Q disminu ida proporcionan sangre con una P0 2
tratamiento con oxígeno suplementario. baja, y que la poca sangre que sale de áreas con una relación VIQ
Disminución de la P0 2 en el aire inspirado. tste es el mecanis- aumentada esta rá saturada con 0 2, pero no puede captar mucho
mo que origina la hipoxemia en individuos que ascienden a gra n más 0 2 del que transporta la sangre saturada normal (ya que habi-
altitud y en los que respiran ai re con concentraciones de 0 2 in- tualmente lo está por encima del 95%), por lo que no es capaz de
feriores a la habitual (21 %), por ejemplo, en el co nfinami ento en compensar el déficit de 0 2 que inducen las zonas con V/Q baja, lo
espacios reducidos y herméticos. La D(A-a)0 2 es norma l, y el trata- que lleva a la hipoxemia. En est os casos, la PaC0 2 suele ser normal
miento con 0 2 corrige la hipoxemia, pues la Pa0 2 dism inuye por- (aunq ue depende de la enfermedad subyacente. Por ejemplo, en
que lo hace la PA0 2. la EPOC tipo bronqu it is crónica suele aumentar, y en el TEP dismi-
Hipoventilación. Muchas situaciones (a lteraciones del centro res- nuir). La D(A-a)0 2 aumenta, y la Pa0 2 mejora con oxigenoterapia
piratorio, neuromuscu la res, de la pared torácica, de las vías aéreas suplementaria.
superiores o de los propios pulmones) conllevan hipoventilación al- Alteraciones de la difusión. Pa ra algunos autores, no es una cau-
veolar. En estos casos, es muy ca racterístico el aumento de la PaC0 2. sa verdadera de hipoxemia, pues sólo es capaz de producirla con el
La D(A-a)0 2 permanece inalterada, y si au menta, hay que pensar en ejercicio físico, no en reposo. Típicamente, aparece en enfermedades
la existencia de un mecanismo causante de hi poxemia acompaña n- intersticiales y en el enfisema, y si un paciente afectado por estas
te, como el shunt o la discordancia V/Q. El tratamie nto con 0 2 con- enfermedades presenta hipoxemia en reposo, hay que considerar la
sigue correg ir la hipoxemia, pues la Pa0 2 desciende porque la PA0 2 concu rrenc ia de alteración en la relación V/Q como causante de la
disminuye a expensas del aumento de la PAC0 2• misma, y no la alteración en la d ifus ión aislada . La PaC0 2 está normal
Cortocircuito o efecto shunt. Hace refere ncia a la situación en que o dism in uida por hiperventilación, la D(A-a)0 2 está aumentada, y la
existen alvéo los perfund idos que no so n ventilados, como ocurre en: oxigenoterapia consigue mejorar la Pa0 2.
Colapso alveolar (at electasia).
Ocupación del espacio aéreo: por hemorragia alveolar (Goo- En la Figura 19 se muestra el algoritmo diag nóstico de la hipoxem ia.
dpastu re), edema pu lmona r
ca rdiogénico o no (a lgunos
tóxicos capaces de inducir
Hipoxemia
la aparición de edema pu l-
monar no cardiogén ico son
los sal icilatos, los opiáceos,
el monóxido de carbono o el
cianuro), y material purulen-
to (neumonías).
Cortoci rcuitos vascu lares N
intrapu lmonares heredita- - ,J. Fi0 2
rios (Rendu-Osler) o adqu i-
ridos (cirrosis, q ue a veces
induce a la aparición de
0(A-a)02 ¿Pa02 corrige con 0 2 ?
malformaciones vasculares
pulmonares) o extrapu lmo- N t No Sí
nares (CIA) . f
Extrapulmonar
+
Pulmonar
f
Shunt
+
Alt.V/Q
(lntox. opiáceos) (EPOC) Alt. difusión
La PaC0 2 es normal o incluso des-
ciende por la hipervent ilación re- Figura 19. Al goritmo diagnóstico de la hipoxemia
Manual CTO de Medicina y Cirugía, 2.a edición

0 2 en los tej idos, bien por d ism inución del aporte de sangre a ese tejido.
Si la causa de hi poxem ia es la hipovent il ación, la
PCO, estará elevada y el grad iente alvéo lo-arteria l de Clínicamente, la d ife rencia ent re ellas es que la central suele ser más in-
02 será norma l. tensa y generalizada y las extremidades están cal ientes, m ientras que en
la peri fé ri ca están frías.

Hipercapnia La hipoxemia crónica puede presentar estos signos y síntomas de una


forma más larvada, así como otros derivados de los mecan ismos de com-
Como norma general, hay que recordar que siempre que se detecte hi- pensación (cefalea por vasod ilatación y por la hipercapnia, hiperviscosi-
percapnia, se debe pensar que existe una inadecuada venti lación alveolar dad por la polig lobu lia, card iopatía, etc.).
que es incapaz de "evacuar" la cantidad de c o2 producida por el metabo-
lismo celular. Los pacientes con insufic iencia respiratoria crónica pueden agudizarse
(por ejemplo, la EPOC avanzada), situación que se debe sospechar ante la
Esto ocurre en las enfermedades en las que hay dism inución del proceso presencia de cambios clín icos (por ej., cambios en el esputo, aumento de
ventilatorio, como en los t rastornos del centro respi ratorio (hipoventi- la tos ... ), o ante un descenso del pH, aumento de la PaC0 2 o descenso de
lación alveolar central, etc.), de la bomba respirato ria (trastornos neuro- la Pa0 2 respecto a los va lores habituales del ind ividuo.
muscula res, cifoescoliosis, obesidad-hipoventilación ... ), de las vías resp i-
ratorias (EPOC avanzada, etc.), así como en casos de grave trastorno del El tratamiento de la insuficiencia respiratoria descansa sobre dos pi-
intercambio gaseoso (enfisema avanzado, etc.). la res básicos:
Intentar correg ir la causa desencadenante mediante su tratamiento
Insuficiencia respiratoria específico (por ejemplo, en una crisis asmática, el t ratam iento bron-
codi latador).
Se define con un dato gasométrico. Se dice que un paciente presenta insufi- La oxigenoterapia, para intentar mantener la Pa0 2 por encima de 60
ciencia respiratoria si, respi rando aire ambiente (Fi0 2 = 21 %) al nivel del mar, mmHg, y conseguir una saturación de la hemoglobina en torno al
presenta una Pa0 2 menor de 60 mm Hg. Si añade una PaC0 2 >SO mmHg, se 90%, que asegu re un aporte de 0 2 suficiente a los tejidos para man-
habla de insuficiencia respiratoria global, y si no, de insuficiencia respiratoria tener una adecuada actividad metabólica.
hipoxémica pura.
Conviene recordar, como ya se ha comentado, que en los pacientes con
Según el t iempo en que se desarrolla, la insuficiencia respiratoria puede retención crónica de C0 2, es pelig roso el tratam iento con 0 2 a Fi0 2 altas,
ser aguda o crónica, hecho importante, pues la hipoxia activa una serie pues en estos individuos, el principal estímu lo ventilatorio es la hipoxe-
de mecan ismos compensadores que serán tanto más eficaces cuanto m ia, que no conviene corregir en exceso (Fi0 2 = 24-28%). Sin embargo,
más lenta y progresiva sea la instauración de esta situación . hay que tener presente que si con Fi0 2 bajas no se consigue aumentar
la Pa0 2, es necesario incrementarla, pues lo que potencialmente es más
Dichos mecanismos son: peligroso para el paciente es la hipoxem ia (por hipoxia tisular). Se deben
Aumento del gasto card íaco. vigilar estrechamente el nivel de consciencia y el estado de la ventilación
Aumento de la eritropoyesis por estimulación de la secreción de eri- (clínicamente y con la PaC0 2), ya que si éstos d isminuyen estaría indicada
tropoyetina. Ambos mecan ismos consiguen incrementar el aporte la ventilación mecán ica.
de 0 2 a los tejidos.
Aumento de la ventilación (si están indemnes los mecan ismos en- Así pues, en los casos en que la oxigenoterapia suplementaria aislada no
cargados de la regulac ión de la respiración) por estímulo hipoxém i- es suficiente para tratar la insuficiencia respiratoria, se emplea la ventila-
co de receptores carotídeos y aórticos, que induce alca losis respi- ción mecánica, con el objetivo general de mantener una Pa0 2 por enci-
ratoria. Es ese caso, hay una tendencia a perder HC03- a nivel rena l ma de 60 mm Hg y un pH normal.
para compensar ese t rastorno.
Aumento de la capacidad de difusión, fundamentalmente hística, Intoxicación por CO
por aumento del volumen de sa ngre en los capilares pulmonares.
Aumento del 2,3-difosfog licerato, para desviar la cu rva de disocia- El CO se produce en la combustión incompleta de un hidrocarburo.
ción de la hemoglobina a la derecha, con lo que se cede el 0 2 a los Es capaz de unirse con alta afinidad a los grupos hemo (hemoglobina,
tejidos más fácilmente. m iog lobina, citocromos), cuya intoxicación produce un descenso del
Vasodi latación loca l, pa ra aumenta r el aporte sanguíneo a esca la ti- t ransporte de oxígeno con Pa0 2 y sat0 2 (medida por pulsioximetría)
sular. norma les, y alteración de la respi ración celular. Según los niveles de car-
boxihemog lobina (en gasometría arterial, fundamentales para el d iag-
La hipoxemia aguda muestra una serie de signos y síntomas de presen- nóstico), puede producir cefa lea, náuseas, somnolencia (de 1O a 30%,
tación, como son: somnolencia, fatiga muscu lar, torpeza motora y men- intoxicación moderada), debi lidad muscular y resp iratoria y confusión
tal, cefalea, cianosis (que apa rece cuando la Hb reducida supera los S g/di (de 30 a 40%, grave, va lora r ingreso), acidos is metabólica y color rojo
en sang re capi lar), náuseas, vómitos o sensación de euforia. Si progresa, cereza de piel y mucosas (más del 4S%) e incluso edema pulmonar y
puede lleva r al coma, provocar convulsiones y muerte. La cianosis puede coma (más del 60%).
ser central, cuando la hemog lobina red uci da aumenta en sangre arterial,
o periférica, cuando el au mento de Hb red ucida se produce en sangre ve- El tratamiento con 0 2 a Fi0 2 elevadas (1 00%) reduce la vida media de la
nosa (la concent ración de hemog lobina en sa ng re capilar es la med ia de ca rboxihemoglobina de cinco horas a 90 m inutos, y es precisa la monito-
ambas). Las causas de cianosis central son las de la hi poxem ia, m ientras ri zación gasométrica si la intoxicación es, al menos, moderada; el alta es
que la cianosis periférica se produce bien por aumento del consumo de posible con niveles menores del 5% y ausencia de síntomas.

03 · Fisiología y fisiopatología
Neumología 1 03
" La hipoxemia debida a hipoventilación cursa con hipercapnia y
Ideas clave Ji6 gradiente alvéolo-arterial de 0 2 normal.

" El t rastorno ventilatorio obstructivo se define po r un cociente " La hipoxemia debida a shunt es la única causa de hipoxemi a,
VEF,fCVF disminuido (por debajo del 70%) con capacidad pul- que no se corrige con 0 2 •
monar total conservada.
" La alteraci ón ventilación-perfusión es la causa más frecuente de
" El trastorno ventil atorio restrictivo cursa con capacidad pulmo- hipoxemia y se caracteriza por gradiente alvéolo-arterial de 0 2
nar total disminuida. elevado y mejora con 0 2•

Un hombre de 35 años acude a un servicio de Urgencias de Ali-


CaS OS C1Í ni C O S ·_-- cante por disnea. En la gasometría arterial basal, tiene un pH
de 7,48. Pa0 2 de 59 mmHg. PaC0 2 de 26 mmHg y un HC0 3 de
Un día en que la presión atmosférica es de 705 mmHg, un pa- 26 mEq/1. Tras administrarle oxígeno al 31%, la Pa0 2 asciende a
ciente de 40 años se presenta en la sala de Urgencias con una 75 mmHg. ¿Cuál de los siguientes diagnósticos es el más proba-
presión arterial de oxígeno de 37 mmHg, una presión de anhí- ble?
drido carbónico de 82 mmHg y un pH de 7,22. Calculamos la
presión alveolar de oxígeno, que resulta ser de 39 mm Hg. Entre 1) Intoxicación por monóxido de carbono.
las causas de insuficiencia respiratoria mencionadas más abajo, 2) Crisis asmática .
¿cuál será la más probable? 3) Atelectasia del lóbulo inferior derecho por cuerpo extraño in-
trabronqu ial.
1) Una embolia de pulmón. 4) Síndrome de distrés resp irat orio de l ad ulto.
2) Es una insuficiencia resp iratoria crónica reagud izada en un pa-
ciente con enfermedad pu lmonar obstructiva crón ica (EPOC) . RC: 2
3) Tiene una crisis asmát ica grave.
4) Una sobredosis de morfina .

RC: 4

es headache, fatigue, dizziness, and nausea. Which point on the


graph above best corresponds to the relationship between plas-
ma bicarbonate, pH, and PC0 2 in this patient?
A patient presents to his physician with a ch ronic cough. He
notes occasional streaks of blood in his sput um. A chest X-ray 1) A.
film reveals multi-nodular, cavitating lesions in the apical pos- 2) B.
terior segments of both lungs, with evident satellite lesions. The 3) C.
condition reported is likely to occur in the apices of the lungs, 4) D.
rather than in the base, beca use the apices:
Correct answer: 2
1) Are better perfused.
2) Are more acid ic. A 26-year-old man receives a concussion from a car accident.
3) Have a higher P0 2 • The brain edema that follows causes compression of t he
4) Vent ilate better. cerebral arteries to such an extent that he needs to be placed on
mechanical ventilation to control his breathing. His respiratory
Correct answer: 3 drive is diminished mainly because of decreased:

A 65-year-old man presents with a productive cough and dif- 1) Arterial PC0 2 acting t hrough centra l chemoreceptors.
ficulty breathing. His sputum cu lture is positive fo r encapsu- 2) Arterial PC0 2 acting th roug h peri pheral chemoreceptors.
lated gram-positive cocci, which are often seen in pairs. The 3) Arterial pH acting th ro ug h ce nt ral chemorecepto rs.
patient's dyspnea is primarily due to which of the following 4) Arterial pH acting throug h peripheral ch emoreceptors.
mechan isms?
Correct answer: 1
1) lnadequate perfusion.
2) lnadequate ventilation. A medica! student volunteers to have his lung volumes and
3) lncreased airway resista nce. capacities measured for his organ physiology laboratory class.
4) Poor oxygen diffusion . He is connected at the end of a normal expiration toa spirometer
containing a known concentration of helium. He is instructed to
Correct an swer: 4 breathe severa! times until the helium has equilibrated between
the spirometer and his lungs. Calculations are made to deter-
A 31-year-old stockbroker drives to a high-altitude mountain mine the amount of air in his lungs when he was connected to
resort to do some rock climbing. Later that day, he experienc- the spirometer, which is called the:
Manual CTO de Medicina y Cirugía, 2.a edición

1) Expiratory reserve vol u me. 1) Residual vol u me.


2) Tidal volume. 2) Tidal volu me.
3) Functional residual capacity. 3) Expiratory reserve volume.
4) Vital capacity. 4) lnspiratory reserve vol u me.

Correct answer: 3 Correct answer: 4

The volume expelled by an active expiratory effort after passive The amount of air remaining in the lungs after maximal expira-
expiration is: tion is called:

1) Vital capacity. 1) Tidal vol u me.


2) lnspiratory reserve volume. 2) Residual volume.
3) Expiratory reserve vol u me. 3) Vital capacity.
4) Residual volume. 4) Totallung capacity.

Correct answer: 3 Correct answer: 2

The maximal amount of inspired a ir after a passive expiration is: The amount of air inspired (or expired) with each normal
breath is:
1) Expiratory reserve vo l u me.
2) lnspiratory capacity. 1) Tidal vol u me.
3) Residual volume. 2) lnspiratory reserve vol u me.
4) Tidal volume. 3) Expiratory reserve volume.
4) Vital capacity.
Correct answer: 2
Correct answer: 1
The amount of inspired a ir with a maximal inspiratory effort over
and above the tidal vol u me is:

03 · Fisiología y fisiopatología
Neumolog...__ía_________

Enfermedad pulmonar
obstructiva crónica (EPOC)

ORIENTACIÓN Algunos aspectos es obligado conocerlos muy bien: el concepto de EPOC, las diferencias entre
bronquitis crónica y enfisema, la clasificación GOLD de la gravedad y el tratamiento de la fase
ENARM estable y de las exacerbaciones.

4.1. Concepto al flujo aéreo en la morbimorta lidad de la EPOC; la tos y la expectoración


crónicas pueden preceder al desa rro llo de limitación al flujo aéreo pero,
de igual forma, algunos pacientes desarrollan limitación sig nificativa al
La EPOC es una enfermedad preven ible y tratable que se caracteriza por flujo aéreo sin haber presentado previamente tos y expectoración cró-
una limitación al flujo aéreo no totalmente reversible y generalmente pro- nicas.
gresiva, causada por una reacción inflamatoria anormal de los pulmones
ante partícu las nocivas y gases, fundamentalmente el humo del tabaco.
4.2. Epidemiología

Considerando todas las edades, un 14% de hombres ad ultos y un 8% de


mujeres ad ultas ti enen bronqu itis crónica, EPOC o ambas. Esa diferen-
cia de prevalencias puede estar relacionada con un mayor consumo de
cigarrillos en varones y una mayor exposición a agentes nocivos indus-
La limitación al flujo aéreo está prod ucida por una mezcla de enferme- triales. Actua lmente existe un aumento de consumo de cigarrillos en las
dad de las pequeñas vías aéreas (bronquiolitis obstructiva, puesta de ma- mujeres jóvenes, lo que puede influir en un futuro los datos de morbili-
nifiesto por una reducción del FEF 25 _75%) y destrucción parenquimatosa dad de la EPOC. Esta enfermedad a menudo se diagnostica tarde, ya que
(enfisema), con una g ran variabilidad en cuanto a la cont ri bución relativa los pacientes pueden no tener síntomas. Las espirometrías de rutina en
de cada una de ellas. La inflamación crónica produce cambios estru ctu- poblaciones determinadas podrían contri buir a detectarla en fases más
rales y estrechamiento de las peque ñas vías aéreas. La destrucción del tempranas.
parénq ui ma pulmona r, también de origen inflamatorio, conduce a la ru p-
tura de las uniones entre los alvéolos y las pequeñas vías aéreas, y a una La mortalidad varía ampliamente entre los distintos pa íses, siendo una
disminución de la retracción elást ica del pul món. Estos cambios afectan de las principa les causas de muerte (es la cuarta causa de mortalidad en
a la capacidad de las vías aéreas para permanecer abiertas durante la es- el mundo). Tanto la morbil idad como la mortalidad por esta enfermedad
piración. tienden a aumentar debido al consumo de cigarrillos y a la mayor expec-
tativa de vida en la población.
Durante mucho t iempo se ha hecho mención en la definición a los tér-
minos "bronquiti s crón ica" y "enfisema': El enfisema consiste en la d ilata-
ción de l acino (vía aérea dista l a un bro nq uiolo termina l) y destrucción de 4.3. Etiología
la pared alveolar; es un concepto anatomopato lógico que a menudo es
usado incorrectamente de forma clínica, y describe tan sólo una de las
varias alteraciones estructurales que se pueden encontrar en los pacien- Tabaco
tes con EPOC. La bronquitis crón ica (defin ida por la existencia de tos y
expectoración durante al menos tres meses al año, d urante al menos dos El consumo de cigarrillos es, sin ninguna duda, el principal factor de ries-
años consecutivos) es un concepto clínico, útil desde el punto de vista go de EPOC. Los fumadores de cigarril los tienen una tasa de disminución
epidemiológico, pero que no refleja la gra n importancia de la limitación anual del FEV, mayor y una preva lencia más alta de EPOC que la población
Manual CTO de Medicina y Cirugía, 2. 8 edición

general. Este efecto es algo menor en los fumadores de pipa o ciga rros. El A pesa r de las dificu ltades, dada la complejidad de la verificación de las
riesgo de desarrollar EPOC es dosis-depend iente, aumenta a medida que infecciones, parece que hay relac ión entre infecciones infanti les, espe-
aumentan los paquetes/año (n.O de paquetes al día x no años fumando). cialmente por el virus resp iratorio sincit ial y los síntomas respiratorios y
Sin embargo, no todos los fumadores desa rrollan EPOC. lo que sugiere la la alteración func ional pu lmonar en la edad adu lta. Los rinovirus han sido
existencia de factores genéticos q ue pueda n mod ifica r el ri esgo ind ividua l. los gérmenes más frecuentemente implica dos en las exacerbaciones de
la EPOC. aunque en los últimos estudios, las bacterias los superan en las
Son factores predictivos de morta lidad por EPOC la edad de com ienzo agud izaciones graves como causa de reagudización.
del hábito de fuma r, el número de paquetes-a ño y la sit uación actua l en
cuanto al consumo de cigarri llos. Los hijos de padres fumadores t ienen Factores genéticos
una mayor prevalencia de síntomas y enfermedades respiratorias, e inclu-
so de ciertas deficiencias en los test funciona les, que puede presagiar una El déficit de a 1-antitripsina (AAT) o a 1-proteasa inh ibidor es la única ano-
futura hiperreactividad bronquia l, aunque no está cla ra su contribución al malía genética conocida que conduce a EPOC y justifica, al menos, el1 o/o
futuro desarrollo de EPOC. de los casos. Es una glucoproteína sérica que se produce en el hígado y
que se encuentra normalmente en los pulmones, cuyo papel principa l es
El abandono del tabaco no lleva consigo una recuperación de la func ión la inhi bició n de la elastasa de los neutrófil os. Es cod ificada por un único
pu lmonar perdida, pero se prod uce una lent ificación de la ca ída anua l gen en el cromosoma 14.
del FEV,.
El fenotipo de la AAT se determ ina genéticamente por la expresión in-
El consumo de tabaco altera la motilidad cil iar, inhibe la función de los dependiente de los dos alelos paternos. Es un gen pleomórfico con más
macrófagos y produce hiperplasia e hipertrofia g landular, así como un de 70 alelos conocidos (M, S, Z, etc.), 20 de los cuales son capaces de
incremento agudo de la resistencia de la vía aérea por constricción mus- producir enfermedad de distinto grado de gravedad. La mayoría de la po-
cular lisa de origen vaga l. blación norma l tiene dos genes M, y su fe notipo se designa como PiMM.
Los va lores normales de AAT sérica son 150-350 mg/d l. El nivel protector
Contaminación ambiental de la enzima es 80 mg/dl (35% de lo normal).

Los contami na ntes am b ientales urba nos son perju d iciales pa ra las per- Más del 95% de las personas con deficiencia g rave son PiZZ, la mayoría
sonas con enfermedades card iopu lmonares, pero su papel en la etiolo- caucásicos, cuyos niveles sé ricos son del 16% de lo normal. Los homoci-
gía de la EPOC no está claro. La incidencia y mortalidad de la bronquitis gotos Pi SS tienen va lores del52o/o, por lo que no padecen enfisema. Existe
crónica y del enfisema es mayor en las áreas industrializadas. Las exacer- controversia sobre si el estado de heterocigoto se asocia con alteración
baciones de la bronquitis crónica sí que están en clara re lación con la de la función pu lmona r. Este estado t iene una incidencia del 5-14%. Los
excesiva contaminación por dióxido de azufre. heterocigotos PiMZ y PiSZ tienen niveles entre 50 y 250 mg/dl.

Profesión
El déficit de a l -antitri psina es la úni ca alteración
Se ha demostrado una interacción entre el tabaco y ciertas exposiciones genética conoc ida que pred ispone al desarrol lo de
labora les (trabajadores de plásticos expuestos a d iisocianato de tolueno, EPOC. Provoca enfisema precoz (por debajo de 45
años) en fu mado res y es del tipo pa nacinar.
algodón, minería y g rano).

Alcohol La deficiencia grave de AAT conduce a enfisema prematuro, a menudo


con bronquitis crón ica y, en ocasiones, con bronquiectasias. El comienzo
Aunque son conocidos los efectos del alcohol sobre la función de los de la enfermedad pu lmonar se acelera con el tabaco. La disnea comien-
macrófagos y las células cil iadas, no hay evidencias que demuestren que za genera lmente a los 40 años en fumadores, y alrededor de los 50 en
altere la función pu lmonar de modo independiente al tabaco. no fumadores. Acontece en esta entidad la pérdida del normal equilibrio
en tre proteólisis (ind ucida, sobre todo, por la elast asa del neut rófi lo, y de
Hiperreactividad bronquial inespecífica modo menos re levante por la acción de macrófagos, plaquetas, fibrob las-
tos y múscu lo liso) y antiproteólisis (fundamenta lmente la AAT. y menos
No está cla ro su papel en el desarrollo de EPOC. pero hay datos que la re- importante, el in hibidor bronqu ial de bajo peso molecula r de las células
lacionan con una caída acelerada de la función pu lmonar en fumadores. de Clara), predom inando, pues, la primera y dando lugar a la destrucción
proteolítica de la vía aérea. El enfisema es panacinar y comienza en las
Sexo, raza y nivel socioeconómico bases.

Incluso considerando el tabaco, hay una mayor preva lencia en hombres. El déficit de AAT también produce enfermedad hepática. El 20% de los
La tasa de mortalidad en EPOC es más alta en caucásicos. La morbimorta- niños PiZZ padecen hepatit is neonata l con marcada colestasis, necrosis
lidad se re laciona de modo inverso con el estatus socioeconómico. hepática y pro liferación de duetos biliares, con gránu los PAS+ en los he-
patocitos (corres po nden a AAT) . El daño suele ser reversible, pero un 1Oo/o
Infecciones de los casos evoluciona prog resivamente a cirros is en la segunda déca-
da de la vida. En adultos, se ha descrito cirrosis asociada a los fenotipos
Varios estudios han documentado la asociación entre una historia de en- PiZZ, PiMZ y PiMS. Existe un mayor riesgo de hepatocarcinoma, vascu litis
fermedad de vías resp iratorias bajas (infecciones recurrentes, tos crón ica y carcinoma de cérvix en estos pacientes. Se puede sospechar con un
y sibilancias) y una disminución de la función pulmonar. descenso en las a 1-globulinas en el proteinograma.

04 · Enfermedad pulmonar
obstructiva crónica (EPOC)
Neumología 1 04
El único tratamiento eficaz de la en fermedad hepáti ca es el trasplante.
Pa ra la afectación pu lmonar se puede intenta r trat am iento sustitutivo,
junto con las medidas generales de trata miento de la EPOC, incluyendo
el trasplante, si fuera necesario. Es importante el consejo genético en es-
tos pacientes.

4.4. Patogenia
y anatomía patológica
Alveolo
El proceso inflamatorio q ue oc urre en la EPOC pa rece ser una ampl ifi-
cación de la resp uesta infl amatoria que tiene lugar en el aparato res-
~
piratorio no rm al ante la ag resión de irritantes crónicos como el humo Enfisema centrolobulillar
del taba co. Los mecanismos que determinan esta amplificación no Enfisema panacinar

son bien conoc idos, pero podrían estar genéticamente determina- Figura 20. Ti pos de enfisema
dos.

El patrón inflamatorio de la EPOC incluye a neutrófil os, macrófagos


Bronquitis crónica
y li nfoc itos (fundamentalmente CD8) . Estas célu las liberan mediado-
(azul abotargado o b/ue b/oater)
res inflamatorios que atraen cé lul as inflamatorias desde la circula-
ción (factores q ui miotácticos, como elleucot rien o-64 o la interleuci-
na-8), ampl ifican el proceso inflamatorio (cit ocinas proinflamatorias,
como el TNF-a, IL-1 ~, IL-6) e inducen cambios estructurales (factores
de crecimiento, como el factor transformador del crecim iento TGF-~,
que puede inducir fibros is en las vías aéreas pequeñas). Este proce-
so inflamatorio es ampl ifi cado todavía aún más por estrés oxidati-
vo y un exceso de proteinasas en el pu lm ón. Estos meca ni smos, en
conjunto, conducen a los camb ios patológ icos caract erísticos de la Enfisema
(soplador rosado
EPOC. o pink puffer)

Los cambios estructurales que ocurren en la EPOC son muy variados, y se Figura 21. Fenotipos de EPOC
observan en las siguientes estructuras:
Vías aéreas proximales(> 2 mm de diámetro): aumento de las
células caliciformes, hipertrofia de las g lándulas de la submucosa 4 .5. Fisiopatología
y metaplasia escamosa.
Vías aéreas distales(< 2 mm de diámetro): engrosamiento de
la pared, fibrosis peribronquiolar, exudado inflamatorio endolu- Limitación al flujo aéreo y atrapamiento
minal y disminución de calibre de las vías aéreas (bronqu iolitis aéreo
obstructiva).
Parénquima pulmona r (bronquiolos respiratorios y alvéo-
los): d_estruc;c;;i.ónA e la pared alveolar y apoptosi s de células La inflamación, fibrosis y exudados endolu minales en las pequeñas vías
ep itelial~-e~teUa l es. Se reconoce el enfisema centroacina r aéreas causan la red ucción del FEV, y de la re lación FEV,f FVC. Producen
o centrolob ulillar (Cl il atac ión y destrucción de los bronqu iolos además un progresivo atrapamiento aéreo durante la espiración, lo que
resp iratorios), que se obse rva en
fumadores y predomina en los
campos pulmona res supe ri ores, Localización Localización Causas o factores
y el enfisema panacinar o panlo- en el acino en el pulmón relacionados
buli/lar (destrucción de los sacos Centroacinar CENTRAL Campos Tabaco, edad avanzada
alveolares y de los bronqu iolos (bronquiolo respiratorio) superiores
respiratorios), que se observa
Panacinar UNIFORME Campos Difuso en el déficit de AAT
en el défi cit de al -ant itripsina y
inferiores A veces foca l en ancianos y fumadores,
predom ina en los campos pu l-
asociado a centroacinar en lóbulos
monares inferio res (Figuras 20 superiores
y 21 y Tab la 3).
Vasos sanguíneos pulmona - Acinoso distal DISTAL Subpleural Jóvenes
res: eng rosamiento de la íntima, o paraseptal (tabiques alveolares, en campos Neumotórax espontá neo por rotura
duetos alveolares superiores de bullas apica les. El flujo aéreo suele
disfunción endote lia l y aumento
y alvéolos) estar conservado
de la capa muscular, lo que co n-
duce a hipertens ión pulmonar. Tabla 3. Variantes de enfi sema
Manual CTO de Medicina y Cirugía, 2.a edición

conduce a hiperinsuflación pulmonar. Predominio de enfisema Predominio de bronquitis


La hiperinsuflación reduce la capacidad Hábito exterior Asténico Pícnico
inspiratoria, de forma que aumenta la ca-
Edad en el momento ± 60 ±50
pacidad residual funcional, sobre todo, del diagnóstico
durante el ejercicio físico (hiperinsufla-
Disnea Grave Leve
ción dinámica). Actualmente, se cree
que la hipersinsuflación se desarrolla al Adquisición de la tos Después de la disnea Antes de la disnea

inicio de la enfermedad y es la principal Esputo Escaso, mucoso Abundante, purulento


responsable de la disnea de esfuerzo. Infecciones bronquiales Poco frecuentes Más frecuentes
Episodios de insuficiencia A menudo terminales Repetidos
Alteraciones respiratoria
en el intercambio PaC0 2 crónica 35-40 mmHg 50-60 mmHg
de gases Pa0 2 crónica 65-75 mmHg 45-60 mmHg
Poliglobulia Rara Frecuente
Estas alteraciones son secundarias a HTP (reposo) Normal o ligera Moderada o intensa
diferentes circunstancias: enfi sema, al-
HTP (ejercicio) Moderada Empeora
terac iones en la relación VIQ por obs-
trucción de las vías aéreas periféricas, Cor pulmona/e Raro, sa lvo en fase terminal Frecuente
alteración de la función muscular. Son Retracción elástica Disminución grave Normal
causa de hipoxem ia e hipercapnia. Resistencia a la vía aérea Normal o ligeramente aumentada Aumentada
Capacidad de difusión Disminuida Normal o ligeramente dism inuida
Hipertensión pulmonar
Esfuerzo respiratorio Intenso Moderado
La hipertensión pulmonar leve o mo- Auscultación -i murmul lo vesicular Roncus y sibi lancias que cambian
derada es una complicación tardía en con la tos
la EPOC. Su causa es la vasoconstricción Radiografía de tórax Hiperinsuflación: No hay patrón característico
pulmonar hipóxica de las arterias pu lmo- Aplanamiento diafragmático Engrosamiento de las paredes
nares de pequeño calibre, a lo que pue- Hiperclarida retroesternal bronquiales si se asocian
de sumarse hiperplasia de la íntima e hi- y retrocardíaca bronquiectasias (sombras "en raíl
pertrofia de la muscular y la pérdida del de tranvía")
Patrón de deficiencia arterial
lecho capilar pulmonar que se observa (los vasos no llegan a la periferia) i trama broncovascular
en el enfisema. En la pared de los vasos Si lueta cardíaca alargada Cardiomegalia
sanguíneos se puede observar una reac- A veces bullas (áreas radiolucentes
ción inflamatoria similar a la producida > 1 cm rodeadas por una pared muy
fina, como "dibujada con compás")
en las vías aéreas, junto con evidencia de
disfunción endotelial. Puede acabar con- Tabla 4. Diferen cias clínicas de los tipos de EPOC
duciendo a hipertrofia ventricular dere-
cha e incluso a insuficiencia cardíaca derecha (cor pulmona/e).

4.6. Clínica

Los síntomas más frecuentes en la EPOC son la tos, la expectoración y la


disnea. Es muy común evidenciar una historia de tabaquismo de, por lo
menos, 20 ciga rrill os diarios durante al menos 20 años. La disnea suele
aparecer en la sexta década y tiene un desarrollo progresivo.

Casi siempre existe historia de tos y expectoración previa a la dis-


nea. En la bronquitis crónica, puede haber hemoptisis (de hecho, es
la causa más común en la actualidad). En las exacerbaciones infec-
ciosas, hay aumento de la tos y la disnea, con esputo puru lento e
incluso sibi lancias. En la Tabla 4 se recogen las p rin cipa les diferencias
y sim ilitudes clínicas entre el enfisema (Figuras 22 y 23) y la b ronqu i-
tis crónica. El porcentaje del FEV, en relación con el teórico se utiliza
para clas ifi car la gravedad. Muchos pacientes muestran un aumento
del FEV 1 menor del 15% tras la inhalación de un ~ -a drenérgico (test
de broncodilatación negativo), aunque sue le haber clara mejoría sin- Figura 22. Rad iografía posteroanterior de tórax de un paciente
tomática. Se aprec ia un aumento de l VR, CRF y VR/CPT. enfisematoso

04 · Enfermedad pulmonar
obstructiva crónica (EPOC)
Neumología 1 04
Estadio FEV 1 (%sobre el teórico)
1 (leve) ~80%

11 (moderada) ~50% y< 80%


11 1 (grave) ~ 30y <50%
IV (m uy grave) < 30%
~ 30 y< 50% si además existe CP o IR
CP: cor pulmonale; IR: insuficiencia respiratoria
Tabla 5. Clasificación de la gravedad de la EPOC

4.8. Tratamiento

Posibilidades terapéuticas

Tratamiento no farmacológico

Abandono del tabaco. Es la medida terapéutica más importante y


más eficaz para tratar la EPOC. Ha demostrado que aumenta la su-
pervivencia.
Rehabilitación . Mejora la tolerancia al ejercicio y la calidad de vida.
No ha demostrado impacto sobre la supervivencia. Debe inclu ir con-
sejo nutriciona l.
Oxigenoterapia crónica domiciliaria (OCD) . Ad ministra da duran-
te un mín imo de 15 horas al día (incluyendo el peri odo nocturno),
Figura 23 . Radiografía lateral de tórax de un paciente enfisematoso la OCD ha demostrado que aumenta la supervivencia. Su principal
efecto hemodinámico es en lentecer la progresión de la hipertensión
La CPT está aumentada en los pacientes con obstru cción grave. Por la pulmonar. Se debe plantear en todo paciente clínicamente estable,
existencia de alteraciones en la ventilación/pe rfus ión, la gasometría ar- que realice un t ratamiento broncod ilatador completo y correcto,
terial suele evide nciar hipoxemia y, a veces, sobre todo si predomina la siempre que cumpla alguna de las siguientes indicaciones: 1) Pa0 2
bronqu itis crónica, hipercapnia, con una D(A-a)0 2 aumentada . La correla- (en reposo) < 55 mmHg, o 2) Pa0 2 (en reposo) 55-59 mmHg junto
ción entre el FEV, y la gasometría es escasa . La DLCO disminuye cuando con pol iglobulia, hipe rtensión pulmonar, cor pulmonale, arritmias o
predomina el enfisema. alteraciones de funciones intelectua les. El objetivo que se persigue
con la OCD es mantener la Pa0 2 en reposo por encima de 60 mm Hg.
Síndrome overlap o de so lapamiento. Se denomina as í a los pa-
cientes que presentan una asociac ión de dos síndro m es resp irato-
rios. La asociació n más frecue nte es la EPOC con el síndro m e de ap-
nea obstructiva del sueño. Estos pac ien t es desarrollan hipertens ión El aba ndono del tabaco y la oxigenoterapia dom ici-
liari a prolongan la supervivencia en la EPOC.
pu lmonar, cor pulmonale, poliglobulia y alteraciones diurnas de los
gases arteria les de manera más p recoz y grave que los pacientes con
EPOC aislada. Además, es característico que estas alteraciones apa-
rezcan con un meno r grado de obstrucc ión esp irométrica . Por tanto,
en pacientes co n h ipe rcap nia o po li globulia des p roporcionada a la
alterac ión pulmonar, hay que descartar la existencia de un SAOS aso- La ox igenoterapia se debe prescri bir en fase estab le y
en pac ientes bien tratados. Prolonga la supervivencia.
ciado.
Recuerda
4.7. Diagnóstico Tratamiento quirúrgico.
Bullectomía: puede mejorar la disnea y la función pulmonar
en pacientes muy seleccionados (gran bulla que comprima el
Está basado en la clíni ca y las pruebas funcionales. Para el diagnós- parénquima vecino, tratam iento de síntomas locales como he-
tico es necesa ri o demostrar (mediante espirometría la existencia de moptisis, infección o dolor torácico).
obstrucción al flujo aé reo (lo que implica una relac ión FEV,fFVC < 0,7) Cirugía de reducción de volumen pulmonar: procedimiento qui-
no completamente revers ibl e. Debe hacerse diag nóstico diferenc ial rúrgico en el que se reseca n zonas del pulmón para reducir
principa lme nte co n el asma, la insufi cie ncia card íaca, bronquiectasias la hi peri nsuflación, m ejorando la eficiencia mecá nica de los
y, en áreas de alta prevalencia, tubercu los is. múscu los resp iratorios (fu ndamenta lmente el diafra gma). Ha
demostrado mejorar la supervivencia (frente a t ratamiento con-
La EPOC se clasifica en cuatro estadios clínicos, en función del valor del vencional) en pacientes con enfisema de predominio en lóbu-
FEV, posbroncodilatador (Tabla 5). los superiores y baja tolerancia al esfuerzo físico.
Manual CTO de Medic ina y Cirugía, 2 .a edición

Trasplante pulmonar: la EPOC constituye hoy en día la indicación Dentro de est e grupo de fárm acos se incluye n los sigu ientes:
más frecuente de trasplante pulmonar. Los criterios de indi- Agonistas {3-adrenérgicos. Prod ucen broncod ilatación por acción
cación incluyen un FEV 1 < 25o/o, Pa0 2 < 55 mmHg, PaC0 2 > 50 directa en los recepto res ~-2 del músculo liso que existe desde
mm Hg e hipertensión pulmonar secunda ri a. Mej ora los sínto- la tráquea a los bronquiolos t erminales, a t ravés del ciclo intra-
mas y la calidad de vida, sin embargo, no ha demostrado un celular adenilato-ciclasa-proteincinasa A. Revierten la bronco-
impacto posit ivo sobre la supervivencia. constri cción, sin t ener en cuenta el agente contráctil.
Poseen efectos adicionales, como son que previenen la libera-
Tratamiento farmacológico (Figura 24) ción de med iadores de las células cebadas y disminuyen la ex-
travasación vascu lar tras la exposición a med iadores (estos dos
les confi eren cierto poder antiinflamat orio en la inflamación
aguda, pero no parecen tener efecto en la inflamación crónica),
aumentan el aclaram iento mucoci liar, pero no inhiben la secre-
BD de acción corta a demanda ción glandula r, reducen el efecto broncoconstrictor co li nérgico
Síntomas intermitentes
(generalmente ~-2 agonistas) y mejoran la resistenc ia de los múscu los fatigados, pero no al-

j !
teran su fuerza.
Existen de acción corta y de acción prolongada. Los de acción
corta (d uración: 4-6 horas, comienzo de acción ráp ido) son el sal-
Síntomas continuos ~ Anticolinérgico de acción larga pautado
butamol, la terbuta lina y el fe noterol. Dentro de los de acción pro-
longada hay que distingu ir entre los de 12 horas de duración de


t
¿Control de síntomas?
acción, que incluyen el sa lmeterol (com ienzo de acción lento) y el
formoterol (com ienzo de acción rápido), y los de 24 horas de du-
ración de acción, que en la actua lidad sólo incluye el indacaterol
j NC (com ienzo de acción rápido). Aunque algunos de el los (como el
sa lbutamol) está disponi ble para su adm inistración por varias vías,
Añadir ~-2 agonistas en todos ellos es de elección la vía inha latoria, ya que es la más
acción larga, pautado eficaz y la que presenta menos efectos secundarios.
o roflumilast La adm inistración por vía sistémica prod uce más efectos cola-
tera les y no aporta beneficios. Los efectos colaterales (el más


t
¿Control de síntomas?
común es el temblor muscu lar) están en relac ión con la dosis,
debido a la estimulación de los receptores ~ extrapu lmonares,
pa ra los que existe tolerancia, sin que parezca existir para el

j NO
efecto broncod ilatador. Su uso regular y temprano no parece
alterar la progresión de la enfermedad, por lo que se ind ican
cuando la sintomatología lo requiera .

Añadir teofilina

Los broncod il atadores son un tratamiento sintomáti-


Figura 24. Manejo terapéutico farmacológico de la EPOC estable co . Sólo mej ora n la ca lidad de vida, no la supervi-
venc ia.
Vacunación antigripal. Indicada en todos los pacientes. Reduce las
complicaciones y la mortalidad.
Vacunación antineumocócica. Se recom ienda en pacientes con Anticolinérgicos. En la EPOC son tanto o más eficaces que los
EPOC mayores de 65 años o co n un FEV 1 < 40o/o. ~-adrené rgicos, por lo que se consideran de elección en esta
Mucolíticos, inmunorreguladores, antioxidantes. No existe evi- enfermedad. Compiten con la acetilcolina por los receptores
dencia que apoye la utilización regu lar de ninguno de estos medi- muscarín icos posgang lionares de las células muscu lares lisas de
camentos. las vías aéreas, produciendo broncodilatación, que parece ma-
lnhibidores de la fosfodiesterasa-4 (PDE-4). El roflumilast ha de- yor a más gravedad de la obstrucción. El principal efecto parece
mostrado que puede mejorar la fun ción pulmonar y los síntomas en ocurrir en las vías más grandes.
pacientes con EPOC moderada y grave, añad ido a anticolinérgicos o Se emplean por vía inhalada o nebulizada. Se dispone de an-
~ -agonistas . Se ad ministra una vez al día por vía oral y posee un buen ticolinérg icos de acción corta, como el bromuro de ipratropio
perfil de segu ridad, con muy pocos efectos secundarios. (in icio de acción en 15-30 minutos, duración aproximada de
Corticoides inhalados. Se recom ienda añadirlos al régimen tera- ocho horas), y de acción prolongada, como el bromuro de tio-
péutico del paciente cuando presente hiperreactividad bronqu ial tropio, con una duración de acción de 24 horas, lo que mejora
o se trate de un a EPOC grave o muy grave (estad io 111 o IV) con el cumplim iento terapéutico. Es diez veces más potente que el
agud izaciones frecuentes (más de tres en los últ imos tres años) . bromuro de ipratropio y mucho más selectivo por los recepto-
Corticoides sistémicos. No se recomienda su uso en la EPOC en res muscarín icos del árbol traqueobronquial. Mejoran la función
fase estable. pulmonar y la ca lidad de vida, aumentan la tolerancia al ejerci-
Broncodilatadores. Son el pilar fundamental. Aunq ue no han de- cio y no se produce taqu ifilaxia con su uso prolongado. En el
mostrado incidencia sobre la supervivencia, mejoran los síntomas y asma son una alternativa a los ~-agonistas en pacientes con in-
la ca lidad de vida. tolerancia a estos fármacos por sus efectos secundarios.

04 · Enfermedad pulmonar
obstructiva crónica (EPOC)
Neumología 1 04
Los efectos colatera les están limitados a leve xerostomía, cortos catarrhalis. Se inicia tratam iento antibiótico con dos de estos t res criterios
paroxismos de tos y midriasis o gla ucoma, en caso de entrar ac- (criterios de Winnipeg): aumento de disnea, aumento de expectoración,
cidenta lmente en los ojos. No afectan adversamente al aclara- purulencia del esputo. Entre los antibióticos que se emplean, están la
miento mucocil iar ni tienen efectos anticolinérgicos sistémicos, amoxicilina-ácido clavulánico y las nuevas fluoroquinolonas (levofloxaci-
aunque se utilicen en dosis altas, pues no se absorben de forma no y moxifloxacino) en ciclos de 7 a 1O días. Si se sospecha infección por
significativa. Pseudomonas aeruginosa (EPOC grave con más de cuatro ciclos de anti-
biótico en el último año), el antibiótico de elección es el ciprofloxacino.
Las nuevas generaciones de antibióticos t ienen resultados favorables con
pautas más cortas.
Los anticolinérgicos son los broncodi latadores de
elección en la EPOC cuando los síntomas son con-
tinuos. El tratamiento ambu latorio debe incluir combinación de broncodilatado-
res de acción corta; se va lorará añad ir broncod ilatador de acción larga, si
no lo tomaba; un ciclo corto de corticoides sistémicos y antibioterapia si
Teofllina. Ha sido muy utilizada durante muchos años, pero en la está indicada.
actualidad se sabe que su efecto broncodilatador es menor que
el de los fármacos anteriores. Su mecanismo de acción bronco- El tratamiento hospitalario debe incluir combinación de broncodilata-
dilatadora no se conoce bien. dores de acción corta, corticoides por vía sistémica (durante 7-1 Odías),
Otros efectos son el aumento del aclaramiento mucociliar, oxigenoterapia si existe insuficiencia respiratoria, y se debe valorar
estimulación del centro respi rat o ri o, mej oría de la función de la necesidad de antibioterapia y/o venti lación mecánica no invasiva
los músculos respiratorio s, reducción de la resistencia vascu- (VMN I).
lar pulmonar, aumento de la contractilidad ca rdía ca, mejora
de la función pulmonar durante el sueño y algún efecto an- Vent ilación mecánica no invasiva
tiinflamatorio. Los efectos terapéuticos aparecen con niveles
plasmáticos de 5 a 15 11g/ml, aunque con niveles mayores Actualmente son tema de debate y estudio las indicaciones de ventila-
de 15 11g/ml ya se observan efectos secundarios. Su meta- ción mecánica no invasiva dom icilia ri a en pacientes con EPOC. Esta no
bolismo hepático se ve influido por multitud de factores. ha demostrado aún su eficacia en los pacientes con EPOC estable. Sí ha
Los efectos colaterales más comunes son cefalea, náuseas, demostrado, sin embargo, su utilidad en la ag udización moderada o gra-
vómitos, molestias abdominales e inquietud. Si los niveles ve cuando cursa con insuficiencia respiratoria hipercápnica, en acidosis
son mayores de 30 11g/ml, aparece n arritmias, convulsiones respiratoria moderada (pH > 7,20/7,25) o con excesiva taquipn ea pese a
e hipotensión. De forma ambu latoria, se administra por vía tratamiento. Ha demostrado que mejora la acidosis respiratoria, disminu-
oral en preparados de liberación prolongada, mientras que ye la frecuencia respiratoria y la disnea, acorta la estancia hospitalaria y,
existe un preparado intravenoso, la aminofilina, para empleo lo que es más importante, reduce la necesidad de intubación y la mor-
en pacientes agudizados ingresados (Tabla 6 ). talidad.

En caso de acidosis respiratoria grave (pH < 7,20/7,25) y bajo nivel de


Disminución consciencia se requiere intubación orotraq uea l y ventilación mecán ica
Aumento
del aclaramiento invasiva.
Neonatos Edad inferior a 16 años
Ancianos Tabaco
Enfermedad hepática Marihuana La VMN I está indicada en exacerbaciones que cursan
Insuficiencia cardíaca Dieta rica en proteínas con ac idos is resp iratori a leve-moderada y aumento
Fármacos: isoproterenol, del trabajo resp iratorio, siempre que el ni vel de cons-
Cor pulmona/e
fenobarbita l, rifampicina, ciencia sea aceptab le.
Enfermedades febriles
hidracidas, fenitoína
Dieta rica en hidratos de carbono
Fármacos: cimetidi na, alopurinol,
propranolol, eritromicina, lincomicina, 4.9. Pronóstico
clindamicina, esteroides

Tabla 6. Factores que modifican el metabolismo de la teofilina


La edad y el valor del FEV 1 en el momento de l diagnóstico son los
Manejo de la agudización de la EPOC mejores criterios para predecir la supervivencia de los pac ientes con
EPOC. El ritmo de disminuc ión anual del FEV 1 parece ser el índice
Debe evalua rse inicialmente la gravedad de la reagud ización (en fun- que mejor se relaciona con la morta li dad de esta enfermedad. La
ción de comorbilidad, estadio y antecedentes de reagud izaciones) mortalidad aum enta respecto a la de la población genera l cuan-
para decidir el manej o ambulatorio, hospita lario o en unidad de in- do el FEV 1 es meno r al 50o/o respecto al va lor teórico. Un índ ice de
tensivos. ma sa corporal (IMC) menor de 21 kg/m 2 se asocia a aumento de
morta lidad. Otros factores capaces de predecir la mortalidad son:
Aproximadamente un 75o/o de las agudizaciones son de causa infecciosa. la edad avanzada, la gravedad de la hipoxemia, la magnitud de la
Las bacterias son responsables del 75o/o de las agudizaciones infecciosas, hipertensión pulmonar, la hipercapnia y la existencia de comorbili-
mientras que el 25o/o son debidas a vi rus y otros agentes. Los tres principa- dad. El factor que más incide en la evolución es la persistencia en el
les patógenos son el H. influenzae (el más frecuente), S. pneumoniae y M. consumo de tabaco.
Manual CTO de Medicina y Cirugía, 2 .a edición

" En estas condiciones, está indicada cuando la Pa0 2 es < 55 mmHg.


Ideas clave RS
" Los broncodilatadores de elección, cuando los síntomas son
" La característica definitoria de la EPOC es la obstrucción crónica continuos, son los anticolinérgicos.
y no reversible al flujo aéreo.

" La indicación de oxigenoterapia debe hacerse con el paciente


en fase estable y correctamente tratado.

Un paciente de 68 años con EPOC fue dado de alta en el hos-


Casos clínicos pital con oxígeno domiciliario. Dos meses más tarde, acude a
consulta refiriendo disnea con medianos esfuerzos. Ha dejado
Varón de 38 años de edad, fumador activo, que consulta por cua- de fumar y sigue tratamiento correctamente con broncodila-
dro clínico de disnea de grandes esfuerzos, sin tos ni expectora- tadores inhalados. Presenta saturación del oxígeno de 85%,
ción, de dos años de evolución. La exploración física es normal. respirando aire ambiente y hematocrito del 49%. ¿Cuál de las
La gasometría arterial respirando aire ambiente objetiva una siguientes afirmaciones respecto a la oxigenoterapia domicilia-
Pa0 2 de 93 mmHg, una PaC0 2 de 41 mmHg y un pH de 7,38. En la ria es correcta?
exploración funcional realizada se objetiva: FVC (capacidad vital
forzada) 108%, FEV 1 (volumen espiratorio forzado en el primer 1) Debe interrumpirse por tener un hematocrito normal.
segundo) 56%, FEV,fFVC 0,52, CPT (capacidad pulmonar total) 2) Debe continuar, pero no más de 12 horas al día.
162%, VR (volumen residual) 195%, DLCO (difusión de CO) 42%. 3) Debe administrarse mediante gafas nasales con flujo de, al me-
Señale la respuesta correcta: nos, 1O 1/ minuto.
4) Debe mantenerse a largo plazo, pues se ha demostrado que
1) Hay que descartar un déficit de a 1-antitripsina. alarga la supervivencia en pacientes como éste.
2) Sólo hay que aconsejar abandono de tabaco.
3) Es una exploración funcional normal. Hay que buscar otras cau- RC: 4
sas de disnea.
4) Se trata de una EPOC tipo bronquitis crónica.

RC: 1

revealed pH 7.09 objective 0 2 , pC02 p02 70 mmHg and 75 mmHg.


Case Study - ~ The next step is:

A 60-year-old male patient, a smoker for 30 years, diagnosed with 1) Endotracheal intubation and mechanical ventilation.
COPO emphysema type, goes to the ER beca use, after an episode of 2) Remove the oxygen, beca use it is the cause of in crea sed C0 •
2
upper airway infection, there was worsening of his usual dyspnea, 3) lncreasing oxygen, since it is secondary to stupor hypoxia.
agitation and insomnia. He underwent arterial blood gases with p02 4) Commence antibiotic treatment.
SO mmHg, pC0 2 SS mmHg, pH 7.25 and bicarbonate of 31 mEq/L.
Oxygen was administered for the respiratory insufficiency, and Correct answer: 1
within hours box stupor and coma commenced. Arterial blood gases

04 · Enfermedad pulmonar
obstructiva crónica (EPOC)
MBumolng~ía__

Asma

5.3. Patogenia
Se debe conocer bien la
definición de asma (de ella
se deduce la estrategia
diagnóstica), el tratamiento en El asma es un trastorno inflamatorio crónico de las vías aéreas. La infla-
fase estable y de las crisis. mación crón ica se asocia a hi perreactividad bronq uial y a obstrucción
reve rsible, aunq ue los mecan ismos que expl ican esta asociación no están
completamente aclarados. El p roceso infl amatorio es bastante consisten-
t e entre todos los fe notipos de asma, aunq ue puede va ri ar entre pacien-
5.1. Definición tes y entre diferentes mo men tos evoluti vos de la enferm edad.

La inflamación en el asma es sim ilar al de otros procesos alérgicos. Los


El asma es una enfermedad inflamatoria crónica de la vía aérea, en factores implicados en la d iátesis inflamatoria se pueden clasificar en los
cuya patogenia intervienen diferentes células y mediadores inflamato- tres apartados sigu ientes.
ríos, condicionada parcialmente por factores genéticos, que cursa con
hiperreactividad de la vía aérea a una gran va ri edad de estímulos y con Células inflamatorias
obstrucción reversib le al flujo aéreo, bien espontáneamente, bien con
tratamiento broncodilatador. Esta inflamación causa episodios recurren- Linfocit os T
tes de sibi lancias, d isnea, o presión torácica y tos. La revers ibil idad consiste
en un aumento del FEV, de al menos un 12% sobre su va lor basa l t ras Está n aumentados en la vía aé rea, co n u n deseq uil ib ri o en la relación
aplicar un [3-adrenérgico selectivo de acción corta. La hiperreactividad Th 1!rh2 favorable a los Th2 y un aumento en las células natural killer.
consiste en la disminución del FEV, de al menos un 20% sobre su valor
basal tras la rea lización de una prueba de broncoprovocación inespecí- Mastocitos
fica (h ista mina, metacolina, ejercicio físico). El asma es un síndrome con
fenotipos diferentes que comparten unas manifestaciones clínicas simi- Aumentados tanto en la vía aérea como en el músculo liso bronquial, lo
lares, pero con etiologías probablemente distintas. que se relac iona con la hiperreactividad bronqu ial. Su activación da lugar
a la li beración de med iadores broncoconstrictores y proinflamatorios.

Eosinófilos
Para el diagnóstico de asma se req ui ere clíni ca com- _.._..,.:::..;;;..-
patib le y demostración de hiperreactividad bronquia l. Están elevados en la vía aérea de la mayoría de los asmáticos y su número se
relaciona con la gravedad del asma. Su activación libera enzimas responsa-
bles del daño epitelial y mediadores que amplifican la respuesta inflamatoria.

5.2. Epidemiología Neut rófilos

Aparecen en cantidad aumentada en la vía aérea de algunos pacientes


Prevalencia. La prevalencia global del asma varía entre el 1% y el 18% con asma grave, en exacerbaciones, en asmáticos fumadores y en algu-
según países. En edad infant il, es más frecuente en varones, se iguala en nos casos de asma profesiona l.
ambos sexos en la pubertad y predom ina en mujeres en la edad adulta.
En muchos países la preva lencia ha aumentado en las últimas décadas. Células dendríticas
Mortalidad. En todo el mundo se producen unas 250.000 muertes por
asma al año. No existe relación entre la preva lencia y la mortalidad Son células presentadoras de antígenos que en los ga ng lios li nfáticos es-
por asma. timu lan el desarrollo de li nfocitos Th2.
Manual CTO de Medicina y Cirugía, 2.8 edición

Macrófagos Fibroblastos y miofibroblastos

Contienen receptores de baja afinidad para la lgE que cuando son esti- Estimulados por mediadores inflamatorios y factores de crecimiento,
mulados por alérgenos li beran med iadores que amplifican la respuesta contribuyen med iante la producción anóma la de tejido fibroso al remo-
inflamatoria. delado de la vía aérea.

Células y elementos estructurales Terminaciones nerviosas colinérgicas


de la vía aérea (Figura 25) de las vías aéreas

Epitelio bronquial Si se activan por vía refl eja, causan broncoconstricción y secreción de
moco. Producen síntomas como la tos y opresión precordial.
El daño en el epitelio de la vía aérea es una característica del asma no
controlada, con pérdida de célu las ciliadas y secretoras. A la vez intervie- Moléculas
ne activamente en el proceso inflamatorio, liberando mediadores proin-
flamatorios en relac ión con diferentes estímu los que contribuyen a au- Quimiocinas
mentar más el daño epitelia l. El proceso de reparación de estas lesiones
es anómalo, dando lugar a un componente de obstrucción irreversible Expresadas por las célu las epiteliales, actúan reclutando células inflama-
que en ocasiones aparece en el asma, en un proceso conocido como torias en las vías aéreas.
remodelado de la vía aérea.
Cisteinil-leucotrienos

Sustancias liberadas por mastocitos y eosinófilos con fuerte efecto bron-


Mediadores
coconstrictor.
Antígeno inflamatorios lgE
Moco
Citocinas

Producidas por linfocitos y macrófagos, controlan el proceso inflamatorio


en el asma y probablemente determinan su gravedad. Las más impor-
tantes son las derivadas de los li nfocitos Th2: interleucina IL-5, favorece la
producción y activación de eosinófilos, y la IL-4 e IL-13, necesarias para la
producción de lgE por los linfocitos B.

lgE

lnmunoglobu li na responsable de los fenómenos alérgicos, se une a re-


ceptores de alta afin idad presentes en mastocitos, eosinófi los y células
dendríticas. En presencia del alérgeno específico esta unión desencade-
Aumenta
permeabilidad na la liberación de mediadores en dichas células.

Óxido nítrico (NO)

Potente vasodi latador producido en el epitelio bronquial por la acción de


la enzima NO-sintetasa. Su determinación en el aire espirado se puede
emplear como prueba diagnóstica, por ser un marcador no invasivo de
Músculo liso inflamación de la vía aérea.

Figura 25. Epitelio bronquial de un paciente asmático


5.4. Factores implicados
Músculo liso bronquial
en el desarrollo y expresión del asma
Contribuye a la obstrucción al flujo aéreo por su hipertrofia, contracción
aumentada e incluso liberación de mediadores inflamatorios igual que Los factores implicados son de dos tipos y se exponen a continuación.
las células epiteliales.
Factores del huésped
Células endoteliales
Genéticos
La expresión de moléculas de adhesión en las células del endotelio de la
circulación bronqu ial es necesaria para el reclutamiento de células infla- El asma tiene un componente hereditario complejo, en el que múltiples ge-
matorias en la vía aérea. nes están implicados e interactúan entre el los y con fenómenos ambientales.

05 · Asm a
Neumología 1 05
Atopia/hiperreactividad bronquial. Los genes implicados en la por VSR se asocia con el d iagnóstico de asma al final de la niñez. Otros
tendencia a la producción de lgE (atopia) y en el desarrollo de hi- estudios, por otro lado, ind ican que ciertas infecciones tempranas, como
perreactividad bronqu ial no están bien identificados, pero se sabe el sarampión, protegen del desarrollo posterior de asma, por lo que no es
que se localizan muy próximos unos a otros en el brazo largo del posible sacar conclusiones específicas.
cromosoma 5, por lo que t ienden a heredarse conjuntamente. Este
hecho explica la fuerte asociación entre atopia e hiperreactividad La "hipótesis de la higiene" sugiere que la exposición temprana a infeccio-
bronquial. Clásicamente se divide al asma en dos categorías en fun- nes virales protege del desarrollo de asma, modulando la maduración del
ción de la presencia o no de atopia (Tabla 7) . sistema inmunitario hacia un perfil Th 1 (no alérgico), en contra del Th2.
Esta teoría no se ha podido demostra r.

Asma extrínseca Asma intrínseca Sensibilizantes ocupacionales


Predominio en ni ños y jóvenes Predomin io en adultos
Frecuente historia personal Rara historia persona l o familiar Más de 300 sustancias se han asociado al asma profesional, que se define
o familiar de alergia de alergia como aquella que se produce como consecuencia de la exposición a un
(rinitis, urticaria, eccema ...) agente encontrado en el medio laboral. Estas sustancias incluyen molé-
Pruebas cutáneas positivas culas de bajo peso molecu lar altamente reactivas como los isocianatos,
irritantes que elevan la reactividad de la vía aérea, inmunógenos como
lgE total y específica elevadas lgE normal
sales de platino, y diferentes productos biológicos que estimulan la pro-
Hipersensibi lidad tipo 1(inmediata) A veces intolerancia a la aspirina
d ucción de lgE.
Tabla 7. Tipos de asma según su etiología
Tabaco
Respuesta al t ratamiento. Existen d iferent es alelas en el receptor
P2 con diferentes respuestas terapéuticas. Otros genes de interés Los asmáticos fumadores tienen peor función pulmonar, mayor frecuen-
modifican la respuesta a esteroides o a antileucotrienos. Esto abre cia de síntomas y peor respuesta al tratamiento. Se ha demostrado que
la posibi lidad teórica de diseñar un tratamiento específico para cada la exposición al humo del tabaco intraútero y en las primeras etapas de
paciente en función de su perfil genético. la vida (tabaquismo materno) se asocia a peor función pu lmonar y mayor
incidencia de síntomas asmáticos. No está claro si ocurre lo mismo con
Obesidad las man ifestac iones alérgicas.

Se ha demostrado que es un factor de riesgo para el asma. Algunas sus- Contaminación ambiental/dom éstica
tancias como las leptinas, implicadas en el control de la sensación de sa-
ciedad, pueden modificar la función pulmonar y aumentar el riesgo de La relación entre contaminación ambiental y crisis por asma parece cla-
desarrol lar asma. ra, pero no así su papel como agente causal de asma. La contaminación
ambiental se asocia con peor función pulmonar en niños pero no con el
Sexo diagnóstico de asma . Lo m ismo se puede decir con respecto a contami-
nantes domésticos como combustibles para ca lefacción, aire acondicio-
Ya se ha comentado q ue en niños la preva lencia de asma es mayor en nado, hongos e infestación por cucarachas.
varones, mientras que en adultos la situación se invierte.
Dieta
Factores ambientales
Está bien documentado que los niños alimentados con fórmulas a base
Algunos de los factores ambientales implicados en el desarrollo del asma de leche de vaca o proteínas de soja tienen mayor incidencia de asma y
coinciden con los implicados en las exacerbaciones, que se discuten más alergia comparado con los niños alimentados con leche materna . Algu-
adelante. nas ca racterísticas de las d ietas occidenta les, como el uso creciente de
alimentos procesados, red ucción de antioxidantes y de grasas n-6 po-
Alérgenos liinsaturadas podrían contribuir al aumento en la incidencia de asma y
atopia observada en algunos países.
Aunque los alérgenos han sido claramente implicados en las exacerbacio-
nes asmáticas, su papel como factor causante del asma no está definitiva- Factores implicados en las exacerbaciones
mente aclarado. Algunos estudios indican que la exposición a ácaros, epi-
telio de perro y gato y Aspergil/us se asocia con el d iagnóstico de asma en la Entre los agentes que provocan crisis asmáticas están los alérgenos
infancia en suj etos susceptibles (interacción herencia-ambiente); otros, sin (los más frecuentes), el ejerc icio, el aire frío, los gases irritantes, los con-
embargo, ha n cuestionado esta interpretación, llegando incluso a sugerir taminantes ambienta les, los agentes oc upac iona les, los cambios de
un papel protector. Parece que en función del alérgeno, la dosis y el tiempo temperatura y las emociones extremas. Las infecciones víricas pueden
de exposición, la edad y la genética podrían actua r en un sentido o en otro. exacerbar el asma, sobre todo, en los niños. Se han imp licado los rino-
virus (el más frecuente), el virus respiratorio sincitial y el influenzae, y en
Infecciones lactantes, el virus resp irator io sincitial y el parainfluenzae. El ejercicio es
probablemente el principal desencadenante de breves episodios. Pro-
La infección por virus sincitial respiratorio (VSR) y virus parainfluenzaecau- duce limitación al flujo aéreo en la mayoría de los niños y de los jóvenes
san un patrón de síntomas muy simi lar al fenotipo asmático, y la infección asmáticos.
Manual CTO de Medi cina y Cirugía, 2. a edición

También originan exacerbaciones ciertos conservantes y colorantes tamiento con fármacos (inhibidores de la ECA, 0-bloqueantes) . Quizás
utilizados en comidas y bebidas (sulfitos), fármacos, como los antiinfla- sea más importante que los síntomas en sí la aparición de los mismos
matorios no esteroideos en pacientes susceptibles y los 0-bloqueantes, ante situaciones características, como la exposición a pólenes, aire frío,
el estrés emocional, el dióxido su lfú rico, que no t iene efecto en las vías etc. También es muy típico del asma en los adu ltos la aparición de los
respiratorias de sujetos normales hasta alcanzar concentraciones muy síntomas de madrugada o por la mañana temprano, lo que lo diferen-
altas, los episodios de sinusitis sin tratamiento antibiótico y el reflujo cia del reflujo gastroesofágico (los síntom as sue len aparecer nada más
gastroesofágico, pues su corrección implica, en algun os casos, una me- acostarse) o la enfermedad cardíaca (donde los síntomas pueden ocu-
joría del asma. rrir en cua lquier momento). Los antecedentes familiares o personales
de atopia deben ser tenidos en cuenta.

S.S. Fisiopatología Los síntomas de asma suelen ocurrir de modo paroxístico, existiendo
temporadas donde el paciente está libre de clínica, aunque en casos gra-
ves no siempre se consigue la remisión de la enfermedad.
La disminución del calibre de la vía aérea es el evento final común que
explica los síntomas y las alteraciones funcionales. Los factores que con- En la exploración física las sibilancias, genera lmente espiratorias, son el
tribuyen a su apa rición son los siguientes: dato más característico. No son específicas del asma, y cuando la obstruc-
Contracción del músculo liso de la vía aérea. ción es grave pueden desaparecer. El uso de los músculos accesorios y el
Edema de la vía aérea, consecuencia de la inflamación bronquial. pulso paradójico se encuentra en casos graves.
Engrosamiento de la vía aérea a consecuencia de los fenómenos de
reparación definidos como remodelado de la vía aérea .
Hipersecreción de moco. S.7. Diagnóstico
La otra característica definitoria del asma es la hiperreactividad bronquia l,
que provoca obstru cció n de las vías aéreas en respuesta a estímul os que Se basa en la clínica, de donde parte la sospecha, junto con la demostra-
en sujetos sanos no evocarían respuesta alguna, y exp lica n la aparición ció n de obstrucción reversible, hiperreactividad bronqu ial o va riabilid ad
episód ica de síntomas asmáticos. Es consecuencia de la inflamación cró- de la func ión pulmona r (Figura 26).
nica de la vía aérea y se produce por los mecanismos sigu ientes:
Contractilidad aumentada del músculo liso de la vía aérea.
Desacoplamiento de la contracción de la vía aérea por la inflamación Diagnóstico
peribronquial, que impide el soporte que el parénquima pulmonar Cuadro clínico compatible
proporciona a la vía aérea, sin el que el estrechamiento de la luz se +
Hiperreactividad bronquial
exagera para un mismo nivel de contracción del músculo liso. Reversibilidad
Engrosamiento de la pared bronquial por dentro de la capa muscu- Variabilidad
lar. Para una misma contracción muscular, en una pared eng rosada
se produce una mayor reducción de la luz. p.!!l.!,.!ijfi
Terminaciones nerviosas sensitivas. Sensibi lizadas por la infl amación
provocan una broncoconstricci ón exagerada en respuesta a diferen-
tes estímulos.

Las alteraciones estructu rales, conjunto de cambios conocidos como re- Hiperreactividad
Reversibilidad Variabilid ad:
modelado de la vía aérea, incluyen todos los sigu ientes: · Niños
Fibrosis subepitelial, acúmulo de fibras colágenas por debajo de la ~ Control yseguimi~
membrana basal. Aparece antes que los síntomas asmáticos y puede FeNO > 30 ppb
ser parcia lmente revertida por el tratam iento.
Hipertrofia e hiperplasia de músculo liso bronquial. Figura 26. Algoritmo diag nóstico del asma
Proliferación de vasos sanguíneos en la pared de la vía aérea, que
contribuye a su engrosam iento. Función pulmonar. Sirve para confirmar el diagnóstico, establecer la
Hipersecreción de moco, consecuencia de la hiperplasia de células gravedad y monitorizar la respuesta al tratamiento. El diagnóstico se con-
caliciformes y de las glándulas submucosas. firma mediante espirometría, evidenciando un patrón obstructivo y una
mejoría del FEV, mayor o igual al 12% (reversibilidad) tras la prueba bron-
codi latadora, aunque su negatividad no descarta el diagnóstico. La obs-
S.6. Clínica trucción en el asma es, además de reversible, variable, como se manifiesta
con la med ición del pico de fluj o espiratorio máximo (PEF, FEM).

La tríada clásica es la d isnea, sibilancias y to s, y a veces opres ió n torá- Si la espirometría es norma l, se debe investigar la presencia de
cica. Las sibilancia s son muy frecuentes, pero no es raro encontrar a hiperreactividad bronquia l o de variabilidad de la fun ción pulmonar.
pacientes cuya única manifestación es la tos escasamente productiva,
conocida con el nombre de equ iva lente asmático. Otras causas de tos Hiperreactividad bronquial. Se d iagnostica con los test de provocación
crónica que deben ser incluid as en el d iagnóstico diferencial son el bronquial inespecífica con histamin a, metacolina o ejercicio. La disminu-
goteo posnasal (el más frecuente), el reflujo gastroesofágico y el tra- ción del FEV, de más de un 20% con respecto al va lor basa l hace que el

05 · Asma
Neumología 1 05
test sea positivo. Es muy sensible pero poco específica, pues sujetos con Test sanguíneos. La eosi nofilia es característica del asma, tanto in-
atopia o sinusitis sin asma pueden prese ntar hiperreactividad bronquia l, trínseca (en la que suele ser más m arcada) como extrín seca, aun-
e incluso algunos sujetos sanos, por lo que es m ás útil para excluir el diag- q ue su ausencia no excl uye la enfermedad. Cifras muy altas sugieren
nóstico si el resu ltado del test es negativo. otras enfermedades (Ch urg-Straus s, asperg ilosis broncopulmonar
alérg ica (AB PA), neumo nía eosinófi la crónica, etc.). La eosinofi lia pue-
Variabilidad. Se define como la fluctuación exces iva de la función pu l- de no estar presente si el paciente toma corticoides.
monar a lo largo del tiempo. Se emplean med iciones seriadas del pico de Test alérgicos. En el caso de sospechar asma alérg ica, se deben rea-
flujo espiratorio (peak f/ow, PEF, FEM) (Figura 27). La medida más usada lizar las pruebas cutáneas de hipersensibi lidad inmediata (prick-test).
es la ampl itud (diferencia entre el PEF máximo y el mín imo del día) ex- Sin embargo, hay una alta prevalencia de test cutáneos positivos en
presada como porcentaje del va lor diario del PEF y promed iado a 1-2 personas sin síntomas alérgicos, por lo que se deben correlacionar
semanas, según la siguiente fórmula : los resultados de los test cutáneos con la clínica.

(PEFmáx- PEFmín) ) Diagnóstico diferencial


L, ( (PEFmáx + PEFmín/2)
- - - - - - - - - - > 20%
N. 0 de días Se debe est ablecer con otras enfermedad es obstructivas de las vías aé-
reas (EPOC, bronquiolitis, etc.), insuficiencia cardíaca, obstrucción de las
Fracción de óxido nítrico (NO) exhalado (FeNO). Es un ma rcador no vías respiratorias superiores (por tumores o edema la ríngeo), disfunción
invasivo de inflamación eosinofílica de la vía aérea. Los pacientes asmá- laríngea funcional (cuad ro que muestra mala respuesta al tratamiento
ticos tienen valores elevados de FeNO comparados co n sujetos sanos. convencional y gasometría normal en las "crisis"), lesiones endobronqu ia-
Alcanza elevadas sensibilidad y especificidad en sujetos no fumadores les, TEP recurrentes, neumonías eosinófilas, y enfermedad por reflujo gas-
que no reciben t ratamiento antiasmático, por lo que puede tener valor t roesofágico (ERGE).
diagnóstico en pacientes con alta sospecha clínica y test funcionales no
diagnósticos. No obstan te, un va lor normal no excluye el diagnóstico de Clasificaciones del asma
asma, especialmente en personas no atópicas, y aún no ha sido eva luado
de forma prospectiva como test diagnóstico. Según la clínica, la enfermedad se puede clas ificar en cuatro grupos, de-
pendiendo del estado basal del paciente, la frecuencia de las crisis, la fun-
ción pulmonar (PEF o FEV,) y la variabilidad del PEF (Ta bla 8).

Crisis de disnea breves, menos de una vez a la semana


Menos de dos cris is nocturnas al mes
Periodo intercrítico asintomático y con func ión
normal (PEF o VEMS > 80% del teórico)
Variab ilidad < 20%
Incluye el asma por ejercicio
Más de una crisis semanal, pero menos de una diaria
Más de dos crisis nocturnas al mes
A veces hay limitación de la actividad y del sueño
Situación basal: PEF o VEMS > 80% del teórico
Variabilidad > 20- 30%

Síntomas continuos
Más de una crisis nocturna a la semana
Limitación de la actividad y del sueño
Figu ra 27. Pico de flujo espiratorio (peak-flow)
Situación basal: PEF o VEMS 60-80% del teórico
Va riabilidad> 20- 30o/o
Otras exploraciones
Síntomas co ntinuos
Exacerbaciones y crisis nocturnas frecuentes y graves
Gasometría arterial. En el asma crón ica estable es t ípicamente nor-
Ingresos hospitala rios frecuentes
mal. Durante una agu dización, suele haber hipocapnia (debida a la
hiperventilación) e hipoxemia. Generalmente, hay alcalos is respira- Limitación de la actividad y del sueño

toria. Cuando la crisis es g rave, la PaC0 2 aumenta, indicando fatiga Situación basal : PEF o VEMS < 60% del teórico
de los múscu los respiratorios, por lo que se produce acidosis respi- Variabi lidad > 30%
ratoria que en ocasiones lleva asociado un componente de acidosis Tabla 8. Clasificación clínica del asma
fáctica (acidosis m ixta).
Radiografía de tórax. El ha llazgo más frecuente es la radiografía de Esta clasificación es útil só lo en pacie ntes no t ratados. Sin embargo,
tó rax no rm al, t anto en la fase esta bl e como en las crisis. Si la cr isis es resulta más adecuado esta b lecer la gravedad teniendo en cuenta la
grave, puede o bserva rse hiperin suflación torácica. La radiografía de medicación necesaria para contro lar la enferm edad . Así, el asma grave
tórax sirve pa ra exc luir otras enfermedades y descubrir complicacio- es aquella que necesita un tratamiento de alta intensidad para conse-
nes de la agud ización asmática, como neumotórax, neumomediasti- gu ir el contro l. Los niveles de control de l asm a aparecen reflejados en
no o atelectasia por impactación de tapones m ucosos. la Tabla 9.
Manual CTO de Medicina y Cirugía, 2.a edición

Controlada Parcialmente deradas-graves. La vía intravenosa


Características No controlada
(todo lo siguiente) controlada * (hidrocortisona, prednisolona, etc.)

Síntomas diurnos Dos o menos veces Más de dos veces Al menos tres características está indicada en las exacerbaciones
por semana por semana del asma g raves. No se deben utilizar los de
acc ión pro longada, como la dexa-
Uso de medicación Dos o menos veces Más de dos veces Parcialmente controlada presentes
por semana por semana en una semana metasona.
de rescate
Síntomas nocturnos Ni nguno Alguna vez
Efectos colaterales: los esteroides
Limitación de actividad Ninguna Alguna vez inhalados en dosis altas pueden
producir supresión adrenal y otros
Función pulmonar Normal (> 80%) <80%
efectos sistémicos, por lo que se
(FEV,fPEF)
deben emp lear con la mínima dosis
Exacerbaciones Ninguna Una o más al año Una exacerbación, en cualquier semana,
que mantenga el control. Los efectos
convierte por definición el asma
locales de los esteroides inhalados
en no controlada en esa semana
más comunes son la disfonía (hasta
*Alguna de las características presente en cualquier semana
el 40%, por depósito del fármaco en
Tabla 9. Niveles de control del asma la laringe), la tos y la irritación de gar-
ganta tras la inha lación con prepara-
dos presurizados (parece deberse a los ad itivos, ya que son raras con
5.8. Tratamiento disposit ivos de polvo seco) y la candidiasis orofaríngea (hasta en un
5%). Dichos efectos d isminuyen si se emplean cámaras espaciadoras o
dispositivos en po lvo seco, y también cuando se administran sólo dos
Medidas preventivas veces al día .

Consisten en identificar y evitar alérgenos específicos, irritantes inespecí- La gran eficacia de la utilización conjunta de los ~-adrenérgicos de larga
ticos y fármacos nocivos. duración y los corticoides inhalados (mayor que altas dosis de corticoides
inha lados de fo rma aislada) ha conllevado la aparición de dispensadores
Fármacos combinados de ambos fármacos, lo que mejora la adhesión al tratamien-
to y disminuye el coste.
Existen dos grupos de fármacos que se emplean en el tratamiento del
asma: Cro mo nas ( cro moglicat o/nedocromil sódico)
Fármacos de rescate o aliviadores: usados en las agudizaciones,
alivian rápidamente los síntomas. Incluyen los p-adrenérgicos selec- Se han propuesto varios mecanismos de acción, entre ellos la estabiliza-
t ivos de acción corta, los corticoides sistém icos, los antico li nérgicos ción de las célu las cebadas, la acción sobre otras células inflamatorias y la
inha lados y las teofil inas de acción corta. Se rem ite al lector al Capí- interacción con nervios sensoria les.
tu lo de EPOC, donde se exponen ampliamente.
Fármacos controladores de la enfermedad: usados de forma re- El tratamiento prolongado reduce la hiperreactividad bronquial, ya que
gular, mantienen controlada el asma. Incluyen los corticoides inha la- también bloquea la respuesta tardía. Se emplea por vía inhalada. Son efi-
dos y sistémicos, las cromonas, las teofilinas de li beración retardada, caces en el asma inducida por esfuerzo. Son fármacos muy segu ros, por
los ~-adrenérgicos selectivos de acción prolongada, los fármacos lo que se indica en niños con asma leve, si bien su eficacia es discreta,
anti-lgE y los antagonistas de los leucotrienos. muy inferior a la de los corticoides inhalados, por lo cual tienen un escaso
papel en el tratam iento actual del asma.
Corticoides
Teofilinas
Los corticoides inha lados son los fármacos antii nflamatorios más eficaces
de los que se dispone, por lo q ue son los med icamentos controladores Son fármacos broncod ilatadores con modesta acción antiinflamatoria. Se
de elección en todos los esca lones de t ratam iento del asma (excepto en dispone de escasa evidencia acerca de su empleo como controlador de
el asma intermitente que sólo requiere ~ -agon i stas de rescate). Reducen la enfermedad, cuya eficacia es modesta. Se pueden emplear para mejo-
los síntomas, las exacerbaciones y la hiperreactividad bronquial. Los este- rar el control en pacientes que requieren altas dosis de corticoides inha-
roides suprimen la inflamación, pero no curan la enfermedad subyacen- lados a partir del s.o escalón terapéutico.
te. Además, tienen cierta capacidad protectora para evitar la pérdida en
número y función de los receptores ~-adrenérgicos. P-adre nérgicos de acción pro lo ngada

La vía inhalada ha supuesto un gran avance en el tratamiento del asma Nunca se deben emplear en monoterapia, ya que de esta forma pueden
estable, perm itiendo el control de la enfermedad sin supresión adre- empeorar el control del asma a largo plazo. Como ya se ha comentado,
na! ni efectos sistémicos en las dosis habitua les. Los preparados inha- son muy eficaces asociados a corticoides inhalados, de manera que exis-
lados son la budesonida (la que parece tener menor riesgo de efectos ten combinaciones tijas de ambos fármacos. Su efecto se mantiene unas
sistémicos), el d ipropionato de beclometasona y la fluticasona. La vía 12 horas, pero el formotero l, a diferencia del sa lmeterol, posee un rápido
oral (prednisona, predn isolona, etc.) se utiliza cuando la enfermedad inicio de acción, lo que permite usarlo también como medicación de res-
no se controla con las pautas habituales y en las agudizaciones mo- cate. Se utilizan a partir del 3• • escalón.

05 · Asma
Neumología 1 05
Anticuerpos monoclonales anti-lgE ( omalizumab) limitada y con frecuente toxicidad significativa. Están en estudio los ma-
cró lidos por sus efectos antiinflamatorios.
Se administra por vía subcutánea. Es una opción t erapéutica pa ra pacien-
tes con asma alérg ica (n ive les elevados de lgE) que no se controlan co n lnmunoterapia
dosis altas de corticoides inhala d os. La dosis debe ajustarse al nivel de lgE
sérica. Son seguros, con pocos efectos secunda rios. Se emplean en los Su papel en el tratam ient o del asma es li mitado. Los beneficios demos-
últimos esca lones debido a su alto precio. trados en ensayos clín icos son modestos y deben compararse con los
potenciales efectos secundarios, a veces graves. Recientemente existe la
Antagonistas de los receptores de leucotrienos posibilidad de admi nistra r inm unoterapia por vía sublingual, lo que au-
(montelukast, zafirlukast) menta su seguridad. Pueden tener un papel en caso de sensibilización a
un ún ico alérgeno, si las medidas de evi tación y el tratamiento estándar
no co ntro lan la enferm edad.
Además de la acción antiinfl amatoria, bloquean la respuesta aguda bron-
coconstrictora, siendo útiles en el asma inducida por el esfuerzo. Se adm i- Tratamiento del asma crónica
nistran por vía ora l. Están indicados en el asma persistente moderada-gra-
ve para reducir dosis de cortico ide inhalado, siempre que se mantenga El obj etivo es est abilizar al paciente co n la menor med icac ión posible.
el control de la enfermedad. En algunos casos se p ueden emplear en el Se conside ra que la enfermedad está contro lada si hay escasa sintoma-
asma persistente leve como alternativa a los cort icoides inhalados, aun- tología, incluida la noctu rn a, pocas exacerbaciones, poca necesidad de
que con menor efi cacia q ue éstos. Algunos pacientes co n asma inducida 13-agonistas de acción co rta, no hay li mitación en la realización de es-
por sal icilatos responden muy b ien a estos fármacos. Son muy bien tole- fuerzo físico y el va lor absoluto del PEF es normal o casi normal, y con
radas y con pocos efectos secu ndarios. una va riabilidad menor del 20o/o (véase la Tabla 9 ). El tratamiento inicial
depende de la clasificación del asma (véase la Tabla 8) y posteriormente
Otros fármacos antiasmáticos se ajusta según el nivel de control (Tabla 1 O).

Los anticolinérgicos han sido poco estudiados en el t rata miento del asma Escalón 1
en fase estable por lo que apenas se dispone de evidencia a su favor. No
obstante, se reconoce que pueden ser una alternativa pa ra pacientes que Se em pl ean ¡3-adrenérgicos inhalados de acción corta a demanda. En el
experimentan efectos secundarios sign ificativos co n los ¡3-ad renérg icos. asma por esfuerzo son los fárm acos de elección, pudiendo emplea rse
como alternativa un antileucotrieno o una cromo na 20 minutos previos
Se han utilizado numerosos fármacos en pacientes con asma de difícil al ejercicio. Esta medicación de rescate se usa en todos los escalones aso-
control con el objetivo de disminuir la dosis de corticoides orales. El me- ciada al tratamiento controlador para contener los síntomas agudos.
totrexato reduce la necesidad de corticoides en algunos pacientes, pero
con efectos secundarios no despreciables, y en dos ensayos clínicos no
han mostrado superiorid ad frente a p lacebo, por lo que no se recomien-
dan en el t ratam iento d el asma. Se ha empleado tamb ién ciclosporina, En fase es tabl e, los b-agonistas de acc ión corta se em- - -.;::.,.--
pl ea n a demand a.
sa les de oro, inmunoglobulinas intravenosas y anticuerpos monoclo nales
fre nte al factor d e necrosis tumoral a (TN F-a), todos ellos con efi cacia

Escalones terapéuticos Subir


Escalón 2 Escalón 3 Escalón 4 Escalón 5 Escalón 6
De Glucocorticoide Gl ucocorticoide Glucocortico ide Glucocorticoide Glucocorticoide inhalado
elección inhalado a dosis inhalado a dosis inhalado a inhalado a dosis a dosis altas +agonista 132
bajas bajas+ agonista dosis med ias altas+ agon ista adrenérgico acción larga
b2 adrenérgico +agonista 132 132 adrenérgico + glucocorticoides orales
acción larg a adrenérgico acción larga
acción larga
Otras Anti leucotrieno Glucocorticoide Glucocorticoide Añadi r Añadir
opciones inhalado inhalado
a dosis medias a dosis bajas +
Antileucotrieno Antileucotrieno
antileucotrieno
y/oteofi lina y/o teofilina
Glucocorticoide y/o omalizumab y/o omalizumab
inha lado
a dosis bajas +
antileucotrieno
A demanda Agonista 132 Agon ista 132 Agonista 132 Agonista 132 Agon ista 132 Agonista 132 adrenérgico
adrenérgico ad renérgico adrenérgico ad renérgi co adrenérgi co acción corta
acción corta acción corta acción corta acción corta acción corta
Educación, control ambiental, tratamiento de las comorbilidades
Considerar inmunoterapia con alérgenos

Tabla 1 O. Tratamiento del asma según el ni ve l de cont rol


Manual CTO de Medicina y Cirugía, 2 .a edición

Escalón 2 forma y a dosis correctas. Se empleará la dosis más baja eficaz del corti-
co ide y du rante el menor tiempo posible.
Se utilizan corticoides inhalados en dosis bajas de base y un ~-adrenérgico
de acción corta a demanda si aparecen síntomas. Este es el tratamiento Tratamiento de las exacerbaciones
inicial más adecuado en pacientes con asma persist ente no tratados. Se
puede intentar sustituir el corticoide por un antileucotrieno (útil en pa- El método más eficaz para valorar las exacerbaciones de una crisis as-
cientes con rinitis asociada) y, en niños, por una cromona. mática es la realización del PEF. Este parámetro se utiliza para clasificar la
exacerbación según su gravedad (Tabla 11 ).
Escalón 3
Existen unos signos llamados de extrema gravedad o riesgo vital inmi-
En este nivel el tratam iento de elección consiste en la asociación de nente: la d isminución del nivel de consciencia, la brad icard ia, la respira-
corticoide inhalado a dosis bajas junto a un ~-agonista de acción pro- ción parad ójica y el silencio auscultatorio.
longada. Si aparecen síntomas, también se emplea como fármaco de
rescate un ~-adrenérgico de acción corta (no más de 3-4 veces al día). El tratamiento de las crisis siempre dependerá de su nivel de gravedad.
Como alternativa, se puede añadir al corticoide inhalado un antago- Exacerbación leve. El tratam iento de elección son los ~ -agonistas
nista de leucotrienos o emplea r dosis m ed ias del corticoide inh alado de acción corta por vía inhalada. En caso de mejoría no es necesario
solo. ningún otro tratamiento.
Exacerbación moderada-grave. El tratamiento de elección consiste
Si se elige la combinación de budesonida/formoterol se puede emplear en la administración de ~-agonistas de acción corta por vía inhalada
como tratamiento de mantenimiento y de rescate al mismo tiempo, dado junto a corticoides sistém icos. Para los corticoides la vía oral es tan efec-
que el formoterol posee un inicio de acción rápido que lo hace útil para tiva como la intravenosa. Con independencia de la vía empleada, el
este doble objetivo. Es lo que se conoce como terapia SMART. efecto beneficioso de los corticoides no aparece hasta pasadas cuatro
horas de su administración. Los corticoides han demostrado que ace-
Escalón 4 leran la recuperación de las crisis y están indicados en todos los casos
excepto en las crisis leves.
El tratam iento de elecc ión en este nive l es la comb inación de un cor- El empleo de bromuro de ipratropio inhalado de entrada junto al
ticoide inhalado a dosis medias asoc iado a un ~-agonista de acción ~ -agon i sta, o en pacientes que no responden adecuadamente al tra-
prolongada. La alternativa es asociar un antileucotrieno al corticoide tamiento inicial produce una mayor respuesta que cada fármaco por
inhalado. separado. El beneficio de la teofilina no ha sido demostrado en las crisis
de asma y cuenta con toxicidad significativa, por lo cual no se recomien-
Escalón 5 da su empleo sistemático. No obstante, su uso se puede considerar en
las crisis más graves con mala respuesta al tratamiento convencional.
El tratamiento de elección es la asociación de dosis altas de corticoide Los corticoides inhalados a dosis altas proporcionan una broncodi-
inha lado más un ~- agon i sta de acción prolongada, añadiendo otros fár- latación significativa y una reducción en los ingresos hospitalarios.
macos controladores (anti leucotrienos y/o teofi linas) si es necesario. En Están indicados en las crisis moderadas-graves con ma la respuesta
pacientes con asma alérgica se puede recurri r al omalizumab. al tratamiento inicial.
El su lfato de magnesio intravenoso no está aconsejado en el trata-
Escalón 6 miento estándar, pero ha demostrado beneficio en las crisis más gra-
ves que no mejoran con la terapia inicial.
Consiste en asociar corticoides orales al tratamiento del esca lón anterior. El oxígeno está indicado siempre que exista insuficiencia respirato-
Antes se debe est ar seguro de que el paciente usa todo el tratamiento de ria (crisis graves). Se puede adm inistrar por mascaril la o por cánulas

Leve Moderada Grave Parada inminente


Disnea Al andar, puede recostarse Al hablar De reposo
Habla Párrafos Frases Palabras
Sibilancias Moderadas, teleespiratorias Intensas Intensas Silencio auscultatorio
Uso de musculatura accesoria No Sí Sí Respiración paradójica
Nivel de consciencia Normal Normal/agitado Agitado Dism inuido/confuso
Frecuencia respiratoria Aumentada Aumentada > 30
Frecuencia cardíaca 60-100 100-120 > 120 Bradicardia
FEV,fPEF* >80% 60-80% <60%
Saturación 0 2 > 95% 91-95% <90%
Pa0 2 > 60 mmHg < 60 mmHg
PaC0 2 < 45 mmHg > 45 mmHg
Pulso paradójico Ausente (< 1O) 10-25 mmHg > 25 mmHg Ausente (fatiga muscular)
• PEF medido tras la administración de un broncodilatador
Tabla 11. Clasificación de la exacerbación del asma según su gravedad

05 · Asma
Neumología 1 05
nasales con el objetivo de conseguir saturaciones de oxígeno supe- Criterios de ingreso en UCI

riores a 90%. Necesidad de ventilación mecán ica: deterioro del nivel de conciencia
Se debe considerar la ventilación mecánica en casos de insuficiencia o parada cardiorrespiratoria
respiratoria refractaria a tratamiento, deterioro del nivel de conscien- Insuficiencia respiratoria (Pa0 2 < 60 mmHg y/ o PaC0 2 > 45 mmHg)
cia o signos de fatiga muscu lar respirat oria (Tabla 12). a pesar del tratamiento con oxígeno en altas concentraciones
Tabla 12. Criterios de ing reso en UCI en las crisis asmá ti cas
A los 20-30 minutos se debe reevaluar el PEF y la clínica . El alta es posible
cuando el PEF se mantiene superior al 70% una hora tras el último trata- En la Figura 28 se ha resumido la actuación general ante el manejo de
miento y hay estabilidad clín ica mantenida. las crisis de asma.

Evaluación inicial del nivel de gravedad (ESTATICA)


Anamnesis, exploración física, FEV, o PEF, SatO,

Crisis leve Crisis moderada-grave Parada cardiorrespiratoria


PEF o FEV, > 80% PEF o FEV, < 80% inminente

• Oxigeno < 40% si SatO, < 92% • Oxigeno


• Salbutamol + ipratropio 4 pulsaciones c/1 0-1 S min • Salbutamol + ipratroplo 10-20 pulsaciones x min IP
Salbutamol 2-4 pulsaciones • Hidrocortisona e.v. 200 mg o prednisona • ConslderarVMNI
1Pc/20min 20-40 mg V.O. • Considerar intubación orotraqueal
• Flucasona 2 pulsaciones c/1 Q-15 min IP
o budesonida 400 ¡.tg NEB c/1 S mln
{pacientes con mala respuesta)

¡ Ingreso en UCI

N
Evaluación de la respuesta al tratamiento (DINAMICA) FEV, o PEF c/30 min, SatO,, clínica
e
•O

"'::;¡
¡¡; Buena respuesta (1-3 h) Mala respuesta (1-3 h)
>
LJ.J FEV, o PEF > 60% estable FEV, o PEF < 60% inestable
Asintomático Sintomático

!
Alta
!
Hospitalización
• Prednisona v.o. 4o-60 mg 7-1 Odlas • Oxigeno < 40% si SatO, < 92%
• GCI y agonistas )32 de acción larga y medicación alivio • Salbutamol 2,S + ipratropio O,S mg NEB c/4-6 h
• Plan de acción escrito • Hidrocortisona i.v. 1oo-200 mg c/6 h o.. .
• Concertar cita de control • Prednisona 20-40 mg v.o. c/12 h
--~--------------~ • Considerar Mf e.v.

FEV,: vol umen espiratorio forzado en el primer seg undo; PEF: flujo espiratorio máximo; SatO,: saturación de oxihemog lobina; IP: inhalador presurizado; NEB: nebulizado
v.o.:vía ora l; e.v.: vía en dovenosa; GCI: glucocorticoides inhalados; VMN I: ventilación mecán ica no invasiva; min : minuto; mg: miligramo; ~g : microgramo; e/: cada

Figura 28. Proced imiento de actu ación ante las crisis de asma

" Los corticoides sistémicos están indicados en las crisis modera-

1
Ideas clave ~ das-graves.

" Es una obstrucción reversible (diferencia con EPOC), provocada " El mejor parámetro para valorar la gravedad de una crisis y mo-
por una inflamación crónica de la vía aérea. nitorizar su evolución es el pico de flujo espiratorio (PEF).

" Los fármacos de elección en el tratamiento de mantenimiento " En una crisis, la presencia de normocapnia o hipercapnia es un
son los corticoides inhalados (anti inflamatorios) . dato de gravedad.

" Los fármacos de elección en las crisis son los ago nistas p-adre-
nérgicos por vía inhalada.
Manual CTO de Medicina y Cirugía, 2. a edición

La gasomet ría arterial basal muestra un pH 7,46, PC0 2 44 mmHg,


Casos clínicos P0 2 58 mmHg y bicarbonato 16 mmol/1. ¿Cuál de los siguientes
planteamientos es el más adecuado?
Mujer de 21 años de edad, que ha comenzado hace t res meses
con "pitos" en el pecho y tos, especialmente por la noche. Se ha 1) Es una crisis asmática leve, por lo que hay q ue añadir esteroides
hecho más frecuente, y los últimos días no le deja descansar. La inhalados, rem it iéndola pa ra su contro l méd ico.
exploración muestra una discreta disminución del murmullo 2) Se trata de un abuso de beta miméti cos. Hay que desaconsej ar
vesicular y algunos sibilantes, de forma poco intensa en ambos su uso de esa forma y asociar una xantin a o ipratropio.
hemitórax, con frecuencia cardíaca de 86 pulsaciones por minu- 3) Es un cuadro de ansiedad por abuso de betam iméticos. Hay que
to. ¿Cuál será la primera decisión terapéutica? ajustar la dos is a las recomendada s y añadir una benzod iacepi-
na.
1) Prescribir broncodi latadores ~2 estimulantes de corta acción, 4) Es una crisis asmática severa que puede requerir asistencia ven-
cada cuatro horas. tilatoria de forma inmediata.
2) Prescribir broncodilatadores de larga acción, como formotero l,
asociado a corticoides inhalados. RC:4
3) Indicar antibióticos asociados a broncod ilatadores anticolinér-
gicos. Una enferma con antecedentes de asma en la infancia acude a su
4) Prescribi r N-acetilcisteína, cada 6 horas, asociada a amoxici lina. consulta porque en los últimos meses presenta disnea, tos seca
y ruidos torácicos, sobre todo a primera hora. Hace tiempo que
RC: 2 suspendió toda la medicación antiasmática. Estos síntomas apa-
recen a diario en el último mes. Se le realiza un PF en la consulta
Un paciente de 35 años acude a Urgencias por disnea. Refiere que es del 70% del valor teórico. ¿Qué tratamiento sería el más
una historia de asma bronquial, pero nunca ha requerido asis- indicado para ella?
tencia ventilatoria. En los últimos días, ha presentado disnea si-
bilante, y en las últimas 12 horas, ha utilizado al menos 12 veces 1) Cromog licato sódico de base y sa lbuta mol a demanda.
su inhalador de salbutamol. En la exploración, está bien perfun- 2) Una asociación de fl uticasona/sa lmetero l empleada a demanda.
dido y coloreado, con una FC de 11 O lpm y una FR de 35 rpm, con 3) Una asociación de budeso nida/formoterol pautada de base y
tiraje supraclavicular e intercostal. Su TA es de 140/80 mmHg y terbutali na a demanda.
se objetiva un pulso paradójico de 20 mm Hg. En la auscultación 4) Sa lmeterol y teotilina de li beración reta rd ada.
pulmonar, hay una disminución global del murmullo vesicular y
prácticamente no se escuchan sibilancias. La auscultación car- RC: 3
díaca es rítmica, sin soplos. El resto de la exploración es anodina.

1) Perform arteri al blood gases and a chest x-ray, and nebu lized
Case Study ! sa lbut amol ad mini stered methyl pred nisolone.
2) Perform arterial blood gases, ad min ister oxygen by mask hi gh
An 18-year-old with a history of extrinsic asthma from age 1O flow 35%, nebulized sa lbutamol administered methylpredniso-
goes to the emergency room because after a summer storm lone.
there was an increase in rapidly progressive dyspnea, wheezing 3) Perform arterial blood ga ses, ad minister oxygen by nasa l can-
and coughing, badly sagging despite two clicks salbutamol be- nula to 3 lpm, nebul ized sa lbutamol and iprat rop ium, hydrocor-
ing administered at home and repeated within two hours. Upon t isone and delayed-release t heophylli ne.
arrival at the ER the patient was conscious, oriented, tachypneic 4) Perform arterial blood gases, adm inister oxygen by mask high
with intercostal intense, scattered wheezes heard on both sides fl ow 35% i.v. salbutamol and hig h-dose inhaled flut icasone.
of the thorax. Vital signs were: pF: 200 (40% predicted), Fe: 125,
Fr: 32, 0 2 saturation: 86% while breathing air. Which approach Correct answer: 2
should be adopted in this case?

05 · Asma
_Neumología_

Bronquiectasias

Este es un tema prescindible. Bronquiectasias saculares o quísticas: la dilatación se acentúa dis-


Se debe conocer la fibrosis tal mente, terminando en un fondo de saco.
quística y la aspergilosis
broncopulmonar alérgica
(esta se expone en el Capítulo 6.3. Patogenia
de Eosinofilias).

La teoría más aceptada es la de la inflamación que sigue a la colonización


6.1. Concepto bacteriana. Los microorganismos producen sustancias, como proteasas y
otras toxinas, que ocasionan una lesión t isular que altera el aclaramiento
mucociliar con la consiguiente retención de secreciones y colonización
Son dilataciones anormales e irreversibles de los bronq uios proximales bacteriana secundaria .
de med iano ca li bre (mayores de 2 mm de diámetro) debidas a la destruc-
ción de los componentes elástico y muscular de la pared. En el pasado, La infección persistente ca usa una inflamac ión crónica con m igración de
eran a menudo consecuencia del sa ramp ión y la tos ferina, pero en la neutrófilos que liberan enzimas proteolíticas, como la elastasa y citocinas.
actualidad, los principales causantes son los adenovirus y el virus de la Se produce así un círculo vicioso, ya que estos mediadores producen más
gripe. La prevalencia es desconocida, pero ha disminuido en los últimos alteración en el epitelio, que a su vez lleva a un mayor deterioro del acla-
años en los países desarrollados debido a la eficacia de las campañas de rami ento mucociliar, y este a colonización.
vacunación (sarampión, tos ferina, gripe) y al tratamiento antibiótico pre-
coz de las infecciones respiratorias.
6.4. Etiología
6.2. Anatomía patológica
Bronqu iectasias localizadas
Infecciones. Los virus im pl icados son adenovirus, influenza, saram-
La inflamación de las paredes bronquiales conduce a la destrucción de pión y rubéola. Entre las bacterias, destacan estafilococos, Klebsie/la
los componentes elástico y muscular, que son reemplazados por tejido y gérmenes anaerobios. La tuberculosis también es una causa fre-
fibroso. El tejido pulmonar circunda nte ej erce una tracción que ocasio- cuente de bronquiectasias, en este caso, de predominio en lóbulos
na la distensión bronquial crón ica, provocando una contracción del te- superiores.
jido muscular que los rodea, que se hipertrofia e hiperplasia. Todo ello Obstrucción endobronquial. La causa más frecuente en adultos de
conduce a un deterioro del aclaramiento mucociliar, que predispone a obstrucción endobronquial es el carcinoma pulmonar, aunque no
colonización e infección bacterianas. Además, la inflamación causa una da lugar a bronquiectasias al estar limitada la supervivencia del pa-
hipervascularización de la pared bronquia l, que con lleva la aparición de ciente. Los tumores endobronquiales benignos, como los carcinoi-
anastomosis entre arterias bronquia les y pulmonares. des, debido a su crecimiento lento, sí se asocian a bronq uiectasias.
Otras causas de obstrucción son la compresión extrínseca por ade-
La clasificación anatómica más utilizada es la de Reid, que las d ivide en: nomegalias o la estenosis cicatriza! postinflamatoria. En los niños, la
Bronquiectasias cilíndricas o fusiformes: la d ilatación es uniforme causa más frecuente es la aspiración de cuerpos extraños.
hasta que la luz adquiere el calibre norma l de forma brusca y trans- Otras causas de bronquiectasias localizadas. Pulmón hi per-
versal, como consecuencia del taponamiento por secreciones de los transparente unilateral o síndrome de McLeod-Swyer-James (bron-
bronquiolos y bronquios de menor tamaño. qu iectasias quísticas congénitas y agenesia o hipoplasia de la arteria
Bronquiectasias varicosas: la dilatación es irregular, como un ro- pulmonar), el secuestro pulmonar, las atelectasias posoperatorias,
sario. etcétera .
Manual CTO de Medicina y Cirugía, 2. a edición

Bronquiectasias difusas Las comp licacio nes más frecuentes son la neumonía recurrente, empie-
ma, neumotórax y absceso de pulmón. Si hay enfermedad grave y difusa
De origen pulmonar. Algunas susta ncias t óxicas producen una res- con hipoxemia, se desa rrolla cor pulmonale. La am iloidosis y el absceso
puesta inflamatoria intensa que puede o rig inar bronqu iectasias. Tal cerebral son raros en los países desa rro llados. Se encuentran a menudo
es el caso de la inhalación de amoníaco, la aspiración de contenido crepitantes inspiratorios y espiratorios en bases y puede haber roncus y
gástrico o la heroína (incluso por vía intravenosa). sibilancias. El paciente puede tener sinusitis crónica y póli pos nasales. Si
Infecc io nes bacterianas necrosantes (S. aureus, Klebsiel/a, tuberculo- la afectaci ó n es grave, se pueden observar acropaquia s.
sis) o víri cas, como el adenovirus 7 y 21.
La asociación de asma y bronqu iectasias es conocida y no siempre La radiografía de t órax puede ser no rm al o a veces hay imágenes en "n ido
se encuad ra en la asperg ilosis broncopu lmo nar alérgica. de golondrina': "raíl de tra nvía" o "en an illo de sello'; aunque son poco
De origen extrapulmonar. sensibles y específicas y se observan mejor con la TC de alta resolución
Aspe rgilosis broncopu lmonar alérg ica (ABPA). Se debe pensar (TCAR). Suele haber obstrucción al flujo aéreo como consecuencia de
en ella si aparece n bronquiectasias centrales en un asmático re- bronquiectasias d ifusas o de EPOC asociada. La hiperreactividad bron-
fra ctario al tratamiento. qu ial es frec uente. En ocasiones puede haber restricción pulmonar aso-
lnmunodeficiencias congénit as o adq uiridas. Las más com ún- ciada (Figura 29).
mente asociadas son la inmunodeficiencia va ri able común, el
défi cit selectivo de subclases de lgG y la panhipogammag lo-
bu linem ia.
Infección por VIH . Intervienen múltiples facto res: infecciones
bacteri anas recurrentes (parece lo más impo rta nte), defectos
funcionales de los linfocitos B, alteracio nes de la función ciliar y
lesión direct a por el virus en la vía aérea.
La fibros is quística, enfermedad hereditaria más frecuente y le-
ta l en la raza caucásica, en la que las vías aéreas pueden esta r
co lonizadas de forma persist ente por P. aeruginosa mucoide, S.
aureus, H. influenzae y E. co/i. La co lo nización por Burkho/deria
cepacia implica mal pron óstico. Es frecuente la sinupatía y la es-
teri lidad en el varón (por o bliteración de los conductos deferen-
tes, prin cipalmente por alteraciones bioquímicas).
Discinesia ciliar primaria. Ag ru pa una serie de entidades en las
que se produce una alteración en la estructura y función de los
cilios, por lo que se altera la eliminación bacteriana. Los pacien-
tes suelen tener sinusitis, bronqu iectasias e incluso otitis. Los va-
rones suelen ser estéri les. La mitad de los pacientes presentan Figura 29. Bronquiectasias en ambos lóbulos inferiores pulmonares
situs inversus, denominándose sín drome de Kartagener.
El síndrome de Young cursa con infeccio nes sinopulmonares Para co nfirmar el diagnóstico, en la actualidad se rea liza la TCAR, que
recurrentes, pero sin altera cio nes en la estru ctura ciliar ni en el aporta una sensibilid ad y especificidad pa ra las bronqu iectasias lobares
test del sudor. Los va ron es son estéril es debido a azoospermi a de casi el 1OOo/o (a lgo menor para las segment arías). La TCAR ha despla-
obstructiva con espermatogénesis no rm al. zado a la bro ncografía, pu es esta técn ica implica ri esgos, ya que consiste
El déficit de a 1-antitripsin a se asocia con bronquiectasias, aun- en instilar contraste en el interior de los bro nqu ios. Una vez confi rmado el
que no está aclarado su origen. diagnóstico, hay que po ner en marcha el estud io etiológ ico.
Defectos del ca rtíl ago (síndrome de Wi lliams-Campbell y sín-
d rome de Mounier-Küh n o traqueobroncomegalia). La fibrobroncoscopia está indicada si hay hemoptisis, cuando las bron-
Síndrome de uñas amarillas (uñas ama rill as, li nfedema, derrame qu iect asias están loca lizadas o existe sospecha de obstrucción ende-
pleura l y bronqu iectasias). bronqu ial. También si se requ iere hacer estudios m icrobiológicos en pa-
cientes con comp licaciones infecc iosas de mala evolución.

6.5. Clínica y diagnóstico


6.6. Tratamiento
La caract erística clínica por exce lencia es la tos productiva cró nica o la
broncorrea puru lenta. Algunos pacientes expectoran más de 150 mi/d ía El tratam iento se basa en t res pilares:
(la secreción bronquia l normal es < 100 mi/d ía). La tos es intermitente Eli minar, si exist e, la obstru cc ió n bro nq uial.
durante todo el día y empeora al leva ntarse por la mañana. A veces, la tos Mejorar la eliminación de las secrecio nes, que se consigue con una
no es prod uctiva, generalmente por bronquiectasias "secas" en lóbu los adecuada hidratació n, con fi sioterapia respiratoria y drenaje postura!
superiores. La hemoptisis, que suele ser leve por sangrado de la mucosa mantenidos (las dos última s son especialmente importantes si hay
inflamada, apa rece en más del 50o/o de los pacientes y predomina en los una producción de esputo mayor de 30 mi/día). El empleo de muco-
que tienen bronquiectasias "secas': A veces, es masiva por sang rado de líticos está sujeto a controversia.
las arterias bronq uia les hipertróficas, aunque sea una causa rara de mor- Controlar las infecciones con el uso de antibióticos en las agudizacio-
talidad. nes d urante 10-15 días. Genera lmente se usa un tratamiento de amplio

06 · Bronquiectasias
Neumología 1 06
espectro (ampicilina, amoxicilina, amoxicili na-ácido clavulánico, cefu- intensivo durante un año no es eficaz, y, en el caso de las difusas, sólo
roxima o cefaclor). A veces, el ciclo debe durar cuatro semanas para en caso de hemoptisis con com prom iso vita l no controlable med iante
consegu ir que el esputo deje de ser puru lento. Si se sospechan anae- embolización.
robios, se utilizan amoxicil ina-ácido cl avulánico o clindam icina. En la
infección por Pseudomonas, el único tratamiento efectivo por vía ora l La explo ració n m ás adecuada pa ra planea r la interve nció n qu irú rg ica
es el ciprofloxacino, pero induce a una rápida apari ción de resistencias. es la b roncogra fía. Sin embargo, ya se ha co m entado q ue en la actua li-
dad es muy raro que se real ice, pues ha sido ree m p lazada po r la TCAR.
Además, se establece un t ratam ient o específi co. Por ejemplo, si existe Actua lmente puede n hacerse reconstrucciones tridimens ionales de la
un déficit de inmunog lobuli nas, se realiza t ratam iento sustitut ivo, o en TCAR, que proporcio nan una image n sim ilar a la de la b roncog rafía.
la fibros is quística se admi nistra ADNasa recombinante en aerosol, que
mejora la funció n pu lmona r al reducir la viscosidad del esputo. Si existe Debe realizarse t ambién una broncofibroscopia que desca rte enferme-
obstrucción al fluj o aéreo, se utilizan broncodi latadores. Si cumple crite- dad endobronquial causante o concomitante, y p ruebas de función res-
ríos, se indica oxigenotera pia domici liaria. piratori a que va loren si el paciente tolera rá la resección . En ocasiones, la
hemorrag ia bro nqu ial se puede controla r med iante embolización, sin ne-
Se establece indicación qui rú rgica en el caso de bronquiectasias loca- cesidad de recu rrir a la cirugía. En caso de gran afec tación y destrucción
lizadas en uno o dos lóbulos contig uos, cuando el trata miento médico pu lmonar, puede rec urrirse al t ra splante.

" El síndrome de Young se caracteriza por bronquiectasias y


Ideas clave ~ azoospermia obstructiva con cilios y concentración de cloro en
sudor normales.
" Las infecciones pulmonares son causa frecuente de bronquiec-
tasias. " El síndrome de Mounier-Kühn (traqueobroncomegalia) y el sín-
drome de Williams-Campbell se producen por anomalías carti-
" La fibrosis quística es la alteración hereditaria potencialmente laginosas.
letal más predominante entre las causas de bronquiectasias, y
se caracteriza por tener concentraciones elevadas de cloro en " El diagnóstico se establece por TC de alta resolución (TCAR).
el sudor.

" El síndrome de discinesia ciliar prima ri a se asocia a sinusitis, es-


terilidad (espermatozoides inmóviles) y, en ocasiones, situs in-
versus (síndrome de Kart agener).

3) Pseudomonas aeruginosa mucoid variety.


Case Study . · 4) Mycoplasma.

In the following micro-organisms, indicate which colonizes more Correct answer: 3


than 90% of patients with cystic fibrosis (CF):

1) Staphylococcus aureus.
2) Haemophi/us influenzae.
Enfermedades pulmonares
intersticiales

La patología intersticial es muy amplia. Es obligado conocer la fibrosis pulmonar idiopática


ORIENTACIÓN (neumonía intersticial usual de la nueva clasificación}, la linfangioleiomiomatosis, la histiocitosis
ENARM X y la afectación por nitrofurantoína y amiodarona. Muy importante conocer y saber reconocer
los aspectos radiológicos.

7 .1. Generalidades 7 .2. Neumonías intersticiales


idiopáticas
El término engloba a más de cien enfermedades que afectan al espa-
cio intersticial, localizado entre la membrana basa l del epite lio alveolar La clas ificación actual de las neumonías intersticiales idiopáticas se expo-
y el endotelio capila r, en el que se encuentran en condiciones norma les ne a continuación:
células (macrófagos, fibrob lastos, miofibroblastos) y componentes de la Neumonía intersticial usual (NI U) : enfermedad progresiva, irrever-
matriz (colágeno, fibron ectina, etc.). sible y de muy mal pronóstico (supervivencia menor a tres años).
Neumonía intersticial descamativa: es un proceso patológico
Estas enfermedades tienen ca ra ct erísticas clín icas, radiológ icas y funcio- muy poco frecuente asociado al hábito tabáquico. En general, de
nales comunes y ciertos caracteres que las d iferencian y orientan hacia buen pronóstico.
su etiología. El diagnóstico suele precisar material histológico que puede Bronquiolitis respiratoria asociada a enfermedad pulmonar in-
obtenerse por biopsia transb ronqu ial con el fibrobroncoscopio, a través tersticial difusa. Enfermedad de fumadores que cursa con inflama-
de la biopsia por toracoscop ia o mediante biopsia pulmonar abierta (to- ción del intersticio peribronqu iolar y que provoca afectación intersti-
racotomía). cial detectable clínica y radiológica mente.
Neumonía intersticial no específica: se caracteriza por una infla-
El lavado broncoa lveo lar (LBA) puede orienta r, pero rara vez es diag nósti- mación intersticia l crón ica, de predominio linfocitario, acompaña-
co. Sí lo es en la linfang it is carc ino m atosa, el carcinoma de cé lulas alveo- da de grados va ri ables de fibros is. Algu nos pacien tes tienen como
lares, ellinfoma, la neumonía por P. carinii, la histiocitosis X y la proteinosis antecedente la exposició n a aves o enfermedad sist émica del t ipo
alveo lar. La composición celular norma l del líqu ido del LBA es de un 80% co lagenovascular. Ti ene un pronóstico favorable, y buena respues-
de macrófagos, 1Oo/o de linfoc itos, 1-So/o de cé lulas plasmáticas, 1-3% de ta al t ratam iento con co rticoides sistém icos.
neutrófilos y un 1o/o de eosinófi los. Neumonía intersticial aguda: es una enfermedad aguda de causa
desconocida. Su patrón histológico corresponde a la fase de organi-
La relación entre linfocitos CD4/CD8 es de 1,5. En algunas enfermedades zación de daño alveolar d ifuso. Es un síndrome de insuficiencia res-
aumenta el número de linfocitos (sarcoidosis, neumonitis por hipersensi- piratoria progresiva del adulto. Se conoce tamb ién como síndrome
bilidad, beriliosis, fármacos, etc.) y en otras, el de neutrófilos (fibrosis pul- de Hamman-Rich.
monar id iopática). Neumonía organizada criptogenética o bronquiolitis obli -
terante con neumonía organizada: se prefiere el primer tér-
La clasificación de las enfermedades pu lmonares intersticiales se defi- m ino, pues la vía aé rea distal se afecta de forma secundaria y
ne en funció n de su etio logía: algunas cuentan con la presencia de gra- las alterac iones principa les son parenquimatosas. Tiene buen
n u lomas entre sus caract erísticas histológ icas, como la sarcoidosis, la p ro nóstico.
neumon itis por hipersensibilidad, la vascu litis granulomatosas y otras Neumonía intersticial linfocítica: considerada un trastorno li nfo-
por polvos inorgánicos (silicos is y beriliosis) . pro liferativo, es excepciona l su transformación a linfoma.
Neumología 1 01
Neumonía intersticial usual/fibrosis terior), que aparecen como infiltrados difusos con preferencia en
campos inferio res. En estad ios tempranos, se observa un infiltrado
pulmonar idiopática
granu lar fino que sugiere un p roceso de llenado alveolar (aspecto de
vidrio deslustrado o esmerilado). Con la progresión, aparece el !la-
En la actualidad, únicamente se considera fibros is pu lmona r idiopática mado "pulmón en pana l o en queso suizo", que representa el estadio
(FPI) el cuad ro histológ ico de neumo nía intersticial usual. Es una enferme- te rmi nal de la enfermedad y consiste en múltiples espacios qu ísticos
dad de etiología desconocida, también denom inada alveol itis fi brosante o densidades anulares de 5 a 1O mm de d iámetro. Junto a esto, apa-
criptogenética. La prevalencia aproximada es de 3-5 casos/1 00.000. recen también cambios bronquiectásicos (por tracción). La radiología
muestra pérdida de vo lume n pu lm ona r, sa lvo que se asoc ie a enfer-
lnmunopatogenia medad obstructiva.

Estímu los desconocidos generan, en individuos susceptibles, un da ño ini-


cial que estimu la la formac ión de inmunocomplejos, q ue atraen macró-
fagos y los activan. Los macrófagos act iva dos segrega n citocinas, algunas
de ellas (LT-84, IL-8, TN F-a, etc.) atraen células inflamatorias a los alvéolos
(sobre todo, neutrófil os y eosinófi los, y menos a linfocitos), y otras (PDGF-8,
IFN y, IGF, etc.) estimulan a los fibrobla stos y células musculares lisas para
que proliferen, con lo que la fibrosis va progresando hasta formar el t ípico
patrón en panal con áreas q uísticas llenas de moco y células inflam atorias.

Anatomía patológica

Se afectan las paredes alveolares, tapizadas por los neumocitos tipo 1 y 11,
y el intersticio. Pueden afectarse el endotelio vascu lar y los bronquiolos
respiratorios, pero no las vías de conducción. El hallazgo inicial caracte-
rístico es el foco fibroblástico. Se observa n tanto lesiones inflamatorias
como fibróticas, dando así un aspecto heterogéneo, característico de la
entidad, y que ayuda a distinguirla de otras neumonías intersticiales. El
estadio final es la panal ización, con intensa alteración de la arqu itectura
pulmonar normal. Figura 30. Fibrosis pulmonar idiopática (Rx tórax)

Clínica La TC de alta resoluc ión (TCAR) toráci ca es úti l para el diagnóstico dife-
rencia l con otras enfermedades interstici ales, para determinar la exten-
Aparece generalmente entre los 50 y 70 años, aunque puede hacerlo a sión y, sobre todo, para el diagnósti co en fases precoces en la s que la
cualqu ier edad. Existen casos fam iliares, por lo que ciertos factores gené- rad iog rafía puede se r normal, aunque haya enfermedad histológ ica. La
ticos pueden infl uir en la suscept ibilidad a padecerla; la clínica es similar, asociac ión de imágenes retic ulares, engrosamiento irregular de septos,
pero suele aparecer a edades más tempranas. El paciente típico presenta b ronq ui ectas ias por tracc ió n y zona s de panal basales y subpleurales
disnea al ejercicio y tos no productiva, de inicio insidioso. En ocasiones, el es muy sugestiva de neumo nía intersticial usual (Figura 31), y junto a
debut clínico coincide con una infecc ión víri ca, aunq ue suele haber una un cuad ro clíni co ca ra ct erístico, perm ite establecer el diagnóstico sin
hist oria de más de tres meses de sín tom as antes del diag nóstico. Una his- neces idad de biopsia .
toria clínica deta llada es vita l para desca rtar posibles etio logías conocidas.

Cuad ro subagudo con crepitantes en bases y, con fre-


cuenc ia, acropaquias.

La mayoría de los pacientes presentan, en la auscultación pu lmonar, cre-


pitantes finos al final de la inspiración (teleinspiratori os) en las bases de
"tipo velero". Las acropaqu ias son un signo tardío y frecuente q ue apa rece
en el 40-75% de los pacient es. La osteoartropatía hipertrófica y el neu-
motórax son raros. En estad ios avanzados, puede haber cor pulmona/e en
relación con el desarrollo de hipertensión pulmonar.

Diagnóstico

Estu dios de imagen. La radiografía de tórax (Figura 30) muestra


como patrón más co mún el reticu lar (dens id ades lineales o cu rvi lí-
neas) o reticu lo nodu lar (d ensidades nodula res superpuestas al an- Figura 31. Fibrosis pu lmonar idiopática (TC torácica)
Manual CTO de Medic ina y Cirugía, 2.a edición

Áreas de pana l de predom inio en bases sugieren el t rat am ient o co nsiste en el aba ndo no del tabaco y la adm inistración
diagnóstico. Puede haber zonas con aspecto de vidrio de co rti co ides, con bue n pronóstico (s upervivenc ia a largo plazo
des lustrado, pero si son las lesiones predom inantes, 70%) .
hay que pensar en otra entidad.

Bronquiolitis respiratoria asociada


Función pulmonar. Inicialmente, los volúmenes pueden ser norm ales,
a enfermedad pulmonar intersticial difusa
sobre todo, si hay EPOC asociada. Cuando progresa, disminuyen la CPT,
CV, VR y CFR. También pueden dism inuir la CVF y el FEV 1, pero la re lación
FEV,fCVF se mantiene o aumenta. Existe, por tanto, una alteración venti- La bronqu io liti s respiratoria es una enfermedad provocada por el
latoria restricti va . La dism inución de la DLCO es muy frecuente y puede tabaco, q ue se ca racteriza por el acúmu lo de macrófagos hiperpig-
preceder a la disminución de los volúmenes. Hay hipoxemia, que típica- mentados en los b ronqu iolos. Si estas alterac iones se extienden al
mente empeora con el esfuerzo. pa rénquima peribro nq ui o lar adyacente y ocasionan enfermedad in-
tersticia l (detectab le rad iológ ica mente), entonces se habla de bron-
Fibrobroncoscopia . Con biopsia tran sbronquial, proporciona el diag- qu iol itis respiratoria asociada a enfe rmedad inte rsticial difusa. Afecta
nóstico en el 25 %de los pacientes. Si no lo hace, el siguiente paso es la a fumadores de más de 30 paquetes/año, y m uchos autores piensan
videotoracoscopia o biopsia pu lmonar abierta (actualmente se prefiere que es la fase inicial de la neumonía intersticial descamativa. La clíni-
la primera). Dada la natu raleza heterogénea de la enfermedad, casi siem- ca cons iste en tos y d isnea de esfuerzo leve, de inicio subagudo. La
pre se requiere una técn ica qu irúrg ica para el diag nóstico por el mayor radiografía de tórax y la TC muestran engrosam iento de las paredes
tamaño de muestra. alveo lares e im áge nes en vidrio deslustrado. El tratamiento consiste
en el abandono del tabaco y el pronóstico es bueno. Sólo si persis-
En el LBA de pacientes con FPI se aprecia un aumento de polimor- ten las alteraciones tras la supres ión de l tabaco, están indicados los
fonucleares (~ 20%) y de eos inófilos (hasta un 4%). Los linfocitos no corti coides.
sue len aumentar, y si lo hacen, indi ca mejo r pronóstico (véase la
Tabla 13). Neumonía intersticial no específica

Trat amiento Engloba a las enfermedades intersticiales difusas cuyo patrón histo-
lógico no encaja en ninguno de los cuad ros descritos. Se la conside-
Ningún tratamiento se ha mostrado eficaz en la fibros is pu lmonar idio- ra una ent id ad bien defini da, que puede ser id iopática, asociada a
pática. El tratam iento clásico, basado en la asociación de corticoides e in- en fermedades del colágeno o a fármacos. Se d istinguen dos tipos:
munosupresores (ciclofosfamida o azati oprina) no dispone de evidencia celular (predom inio de inflamación) y fibrót ica (predom in io de la fi-
firme sobre su eficacia. Probablemente todos los pacientes requ ieren tra- brosis). La clín ica consiste en disnea de esfuerzo y tos seca de ini-
tam iento, pero si la afectación es leve o existe n contraind icaciones, pue- cio subagudo. La mitad de los pacientes tienen síntomas generales
den someterse a una pauta de observación ca da tres meses que incluya (fiebre, aste nia o pérdida de peso) . Los crep itantes son frecuentes y
evaluación clínica, radiológica y funcional con difusión y test de esfuerzo. presentan acropaqu ias el 30% de los pacientes. La radiografía mues-
El trasplante estaría indicado en pacientes con enfermedad progresiva a tra infiltrados intersticiales o alveolo intersticiales, de predom ini o en
pesar de tratamiento médico. bases. La TC muestra opacidades en vidrio deslustrado e imágenes
reticulares, sie ndo poco frec uente el panal. El diagnóstico se realiza
El resto de entidades englobadas dentro de las neumonías intersticiales por biopsia quirúrg ica. El pro nóstico es m ejo r que el de la neumonía
id iopáticas t ienen una seri e de características comu nes: intersticia l usual, y depende de l tipo: es mejor en la forma celu lar que
El LBA no es diagnóstico. en la fibrót ica.
El diagnóstico se establece genera lmente por biopsia por toracosco-
pia o toracotomía. Neumonía intersticial aguda
El tratamiento son los corticoides y el pronóstico es bueno.
Es una entidad ca racterizada por la presencia de daño alveolar difuso
A continuación, se expondrán la s características di stintivas de cada en el parénquima pulmonar, lesión anatomopatológica ca racterística
una. del síndrome de distrés resp iratorio agudo. El término de neumonía in-
tersticial aguda debe reservarse pa ra los casos de distrés respiratorio
Neumonía intersticial descamativa idiopático.

Entidad ca racterizada por el acúmulo intraa lveolar de macrófagos. La clínica se ca racte ri za po r la aparició n de disnea de comienzo agu-
Inicialmente co nside rada la forma inflamatoria de la fibrosis pulmo- do o subagudo, en ocas io nes preced id a de sínto mas sim ilares a los
nar idiopática, hoy se sabe qu e son dos entidad es diferentes. Afecta de una virias is. En pocos días, el cuad ro evo luciona a insuficiencia
cas i exclus ivamente a fumadores. Hi stol óg icamente se ca racteriza resp irato ri a grave (d istrés). La radiografía de tó rax mu estra infiltrados
por un acúmulo intraa lveo lar de macrófagos un iforme y d ifuso (con- alveolares bilaterales con broncograma aé reo, hallazgos confirmados
trasta con la natura leza parcheada de la neum on ía intersticial usua l por la TC.
[NI U]). La clínica es de inicio subag udo y consiste en tos seca y disnea
de esfuerzo, pudiendo en algunos casos evolucio nar a in suficiencia El pronóstico es ma lo, con una mortal idad a los dos meses del 50%. En
respiratoria grave. El 50% de los casos presentan acropaq uias. Es rara los pacientes que sobreviven, la evo lución puede ir desde la resolu-
su asociaci ón con otras enfermedades. La rad iog rafía de tórax y la TC ción com pleta hasta la aparición de enferm edad pu lmonar intersticial,
muestran im ágenes en vidrio deslustrado, en ocas iones reticulares. El generalmente del t ipo neumonía intersticial no específica.

07 · Enfermedades pulmonares intersticiales


Neumología 1 01
Neumonía organizada criptogenética en un 20% de casos, sobre todo, en pacientes varones con títulos altos
de factor reumatoide. Hay casos en los que la afectación pulmonar
Anteriormente llamada bronq uiolitis obliterante con neumonía orga- precede a la articu lar. Otras alteraciones pulmonares que aparecen
nizada (BONO), se prefiere el térm ino de neumonía organ izada crip- son nódulos reumatoideos pulmonares, incluyendo el síndrome de
togenética (NOC) por ajustarse mejor a la histología. Ésta consiste en Caplan (nódu los pu lmona res en pacientes afectados de neumoconio-
la presencia en alvéo los, conductos alveolares y bronquiolos respira- sis), la bronquiolitis obliterante (a veces asociada al tratamiento con
torios de un tejido fibrob lastico con grados variab les de inflamación D-penicilamina) o enfermedad intersticial por los fármacos emplea-
(neumonía organizada) que no altera la arqu itectura pulmonar nor- dos (sales de oro, muy raras veces utilizados en el momento actual,
mal (cosa que sí ocurre en la NI U). Es una forma inespecífica de reac- metotrexato, etc.).
cionar el pulmón frente a numerosos estímu los, pudiendo encontrar-
se estas lesiones en neumonías, reacción a fármacos, neumonitis por Espond ilitis anquilosante. En fases avanzadas puede aparecer enfer-
hipersensibilidad, neumonía eosinófila crónica, absceso de pulmón o medad bilateral apical fibrobu llosa. Los pacientes pueden tener tos y dis-
vasculitis. Sólo cuando estos procesos han sido excluidos, se puede nea, pero generalmente se encuentran asintomáticos. Con la evolución
hablar de NOC. se puede producir distorsión completa del pu lmón y retracción hi liar ha-
cia arriba.
El inicio de la enfermedad es subagudo, con tos, d isnea, fiebre, astenia
y pérdida de peso. La radiografía de tórax muestra infiltrados alveo- Escleroderm ia . Es la enfermedad de este grupo que presenta con
lares unilaterales o bilaterales, a veces cambiantes, y en otros casos, más frecuencia afectación intersticial (casi dos tercios de los pacien-
las imágenes son intersticiales. La TC orienta el diagnóstico (zonas de tes), generalmente de predominio en lóbulos inferiores, siendo ésta
consolidación subpleurales o peribronquiolares). La función pulmonar la alteración pulmonar más frecuente. También es común la hiper-
muestra un patrón restrictivo con disminución de la DLCO (diferencia tensión pulmonar, con o sin fibrosis (esto último, en la forma locali-
con las bronquiolitis que dan patrón obstructivo). El diagnóstico se es- zada).
tablece en presencia de un cuadro clin icorradiológico compatible y
demostración de neumonía organizada en una biopsia transbronquial Síndrome de Sj ogren. La afectación se caracteriza por infi ltración
o quirúrgica. linfocítica intersticia l (neumonía intersticia l linfocítica). Es común en
el síndrome primario. Puede comportarse como un linfoma de bajo
Neumonía intersticiallinfocítica grado.

Enfermedad rara caracterizada por infiltrados linfocitarios intersticiales Dermatopolimiositi s. La enfermedad intersticia l aparece en un 5-10%
en el parénquima pulmonar. Se la consideró un trastorno linfoprolifera- de los pacientes, sobre todo, cuando hay anticuerpos anti-Jo-1.
tivo, pero su transformación a linfoma es excepcional. Se asocia con fre-
cuencia a enfermedades inmunológ icas: síndrome de Sjógren, miastenia
grave, anemia perniciosa, tiroiditis de Hash imoto, cirrosis biliar primaria. 7 .4. Histiocitosis X pulmonar
También se da en pacientes con SIDA, sobre todo, en niños. La clínica es
inespecífica, cursa con aparición insid iosa de disnea de esfuerzo, tos seca,
fiebre y pérd ida de peso. La rad iog rafía de t órax y la TC no son diagnósti- La histiocitosis X primaria (granu loma eosinófil o, gra nulomatosis de cé-
cas. El LBA presenta linfocitosis (Tabla 13). lulas de Langerhans) se ca racteriza por una infi ltración de órganos por
células de Langerhans (linfocitos CO l ), que son histiocitos dendrít icos de
la serie monocito-macrófago que presentan, al microscopio electrónico,
AumentoCD4 Sarcoidosis gránu los intracitoplásm icos de Birbeck (cuerpos X o cuerpos raqueto i-
AumentoCD8 NH y BONO des). Estas células se encuentran en la dermis, en el sistema reticu loendo-
telia l, en la pleura y en el pulmón .
Aumento polimorfonucleares BO, NH aguda y FPI
Aumento eosinófi los NEC
La histiocitosis X primaria pu lmona r es infrecuente y produce una enfer-
Células de Langerhans Histiocitosis X medad intersticial que se inicia en los bronq uiolos más pequeños con
Tabla 13. Relación entre LBA y enfermedad intersticial un infi ltrado inflamatorio asociado (linfocitos, neut rófi los y algú n eosi-
nófilo). La enfermedad tiene distribución broncovascular, afectando con
frecuencia a arteriolas y vénulas. Al avanzar el proceso, aparece fibrosis
7 .3. Enfermedad intersticial asociada intersticial y pequeños quistes aéreos. En estadios finales, puede aparecer
patrón en panal.
a las enfermedades del colágeno

Lupus eritematoso sistémico (LES). Aunque aproximadamente la mi-


tad de los pacientes desarrollan enfermedad pulmonar, la enfermedad Proliferación de células de Langerhans es característi- ,.,..._.....,.
ca de histiocitosis X.
intersticial crónica y progresiva es ra ra. La afectación pu lmonar más fre-
cuente es la pleu ritis, con o sin derrame pleura l (suele ser un exudado, a
menudo bilateral).
Más raras aún son las lesiones óseas solitarias. Excepcionalmente se des-
Art ritis reumat oide (AR). La afectación más frecuente también es la cribe enfermedad diseminada. Más del 90% de los pacientes son fuma-
pleural. La evidencia radiológica de enfermedad intersticial aparece dores. Es más frecuente en varones jóvenes (20-40 años).
Manual CTO de Medicina y Cirugía, 2. 8 edición

La presentación clínica es variable, pudiendo tener el paciente sínto- la biopsia por toracotomía o por toracoscopia, pues la transbronqu ial
mas respiratorios o constitucionales, debutar con neumotórax por ro- suele ser ineficaz.
tu ra de qu istes o ser un hallazgo radio lógico. El neumotórax recidiva en
el 25% de los casos. Hasta el 20% de los pacientes presentan lesiones
quísticas óseas, generalmente únicas en huesos planos, que pueden
preceder a la alteración pu lmona r o in clu so ser la única manifestación. La histi ocitosis X (junto co n la proteinosis alveola r) se
A veces, se acompaña de diabetes in sípida, lo que indica peor pronós- puede diagnosticar por el LBA.
tico por afectación del SNC. No son comunes las acropaquias ni los
crepitantes.
El pronóstico es muy variable, con pacientes en los que remite de
En la radiografía es altamente específica la comb inación de peque- modo espontáneo y otros en los que progresa a fibrosis pulmonar.
ños nódu los mal defin idos, infiltrados reticu lo nodulares, qu istes y, Existe mayor incidencia de progresión si el paciente contin úa fuman-
en estadios avanzados, imagen en pana l con volúmenes pul mona- do, y de regres ión si deja de fumar, por lo que sobra recalcar la impar-
res conservados y respeto de ángulos costofrénicos. Las alteraciones tancia de abandonar el tabaco. La penic il ami na se ha empleado con
radio lóg icas predo mi nan en campos superiores y medios. La ma rca resultados variab les. Los corticoides no son útiles. En estadios finales
radiológica de esta enfermedad son los pequeños qu istes y los nódu- se indicaría el trasplante.
los (Figura 32).

Las intersticia les que afecta n a lóbulos superiores:


FITNESS (Fibrosis quística, (h) lstiocitosis X, TBC,
NH, Espondilitis anquil osa nte, Sarco idosis y Sili co-
sis).

Ante un varón j oven, fumador, con patrón intersticial


de predominio en campos superiores y vo lúmenes
altos, pensar en hi stiocitosis X. Frecuente el neumo-
tórax.

7 .5. Linfangioleiomiomatosis

Es una rara enfermedad que se caracteriza por la pro liferac ión del mús-
culo liso alrededor de las estructuras broncovascu lares y en el inters-
ticio pu lmonar, unido a la d ilatación qu ística de los espacios aéreos
term inales. Los vasos linfát icos y venosos son tortuosos y d ilatados, así
como el conducto torácico. Puede haber afectac ión extrapulmonar
de los linfáticos y se asocia al angiomiolipoma renal. No se conoce el
mecanismo patogénico, pero los estrógenos parecen jugar un papel
importante, ya que se presenta casi exclusivamente en mujeres pre-
menopáusicas (20-40 años) .

Patrón reticulonodular y quistes en lóbulos superiores respetando ángulo El síntoma más frecuente es la disnea. El neumotórax espontáneo por
costofrénico ruptura de qu istes aparece en la mitad de los casos y puede ser recurren-
Figura 32. Patrón radiológico de la histiocitosis X te y bilateral, pudiendo requerir pleurodesis. El quilotórax apa rece en un
tercio de los pacientes a lo largo de la evolución, y es muy característico
Puede apa recer cua lquier patrón ventilatorio, aunque a medida que pro- de esta enfermedad, pero es raro al diagnóstico. Es frecuente la hemop-
gresa la enfermedad, predomina la alteración ventilatoria obstructiva. La tisis leve.
CPT se mantiene y el VR puede aumentar. La DLCO está desproporciona-
da mente disminuida, lo que sugiere una afectación vascular. El aumento
de la relación VR/CPT se relaciona con la formación de quistes. En algunos Ante mujer en edad férti l con patrón intersticial y vo-
pacientes hay hiperreactividad de la vía aérea. lúmenes altos, si n predominio por una localización
concreta, pensar en linfangioleiom iomatosis. Son fre-
cuentes el neumotórax y el qui lotórax.
Diagnóstico. La imagen característica en la TCAR en asociación con
una historia compatible es prácticamente diagnóstica. En el LBA. las
célu las de Langerhans suponen más del 3-5% (en otras situaciones, En la radiografía de tórax se aprec ia un patrón reticular o reticulono-
como la FP I, carcinoma bronquioloalveola r o fumadores sanos, tam- du lar (Figura 33). La obstru cción de los li nfáticos con la aparición
bién pueden aparecer, pero no en proporciones tan altas). Si se pre- de líneas B contribuye al patrón reticular. En el 50% de las pacientes
cisa confirmación histológica para el d iagnóstico, suele ser necesaria se encuentra hiperinflación y dilatación quística, aunque con TCAR,

07 · Enfermedades pulmonares intersticiales


Neumología 1 01
los pequeños quistes de pared fina aparecieron en el 100% en un El tratamiento con progesterona, considerado de elección, o tamoxi-
estudio. feno producen algún efecto beneficioso. Si el tratamiento fracasa, se
debe cons iderar el trasplante, aunque hay casos de recidiva en el in-
Es una de las pocas enfermedades intersticiales que se presenta con au- jerto.
mento de volúmenes pulmonares y un patrón funciona l obstructivo o
mixto. A menudo tienen hiperinflación con un aumento de la CPT. Está
aumentado el VR y la relación VR/CPT. Con frecuencia, hay limitación al 7 .6. Proteinosis alveolar
flujo aéreo, con disminución del FEV 1 y de la relación FEV /CVF. Es carac-
terística la disminución de la DLCO.
La proteinosis alveolar realmente no es una enfermedad intersticia l,
ya que no hay proceso inflamatorio. Se produce al acumularse en
los alvéolos un materia l proteináceo PAS+ rico en fosfolípidos (es
una acumulación anormal de los constituyentes no rma les del sur-
factante).

La enfermedad puede se r congén ita, primaria o secundaria. La prima-


ria es rara, con una incidencia aproximada de 1/100.000 habitantes, y
p redomina en varones de 20-50 años.

Datos actuales sugieren que se trata de un proceso autoinmunitario,


en el cua l se producen anticuerpos contra el factor estimulante de las
colonias de los monocito-macrófagos, lo que ca usa disfunción de los
neumocitos tipo 11, que son incapaces de aclarar el surfactante. La se-
cu nda ria consiste en la acumu lac ión del materia l referido en relación
con situaciones diversas, como la inhalación de síl ice, aluminio, otros
polvos inorgá nicos, e incluso con trastornos hematológicos.

La clínica consiste en disnea de curso lentamente progresivo, que sue-


le acompañarse de tos seca. En la auscultación pulmonar se eviden-
cian crepitantes teleinspiratorio s. La rad iografía muestra un patrón de
consol idación de los espacios aéreos b ilateral y perih iliar, que recuer-
da al del edema pu lmonar. Estos pacientes tienen mayor riesgo de
padecer infecciones por Nocardia, M. avium y Pjiroveccii.

El diagnóstico se basa en el análisis del líquido del LBA, que presenta un


Patrón reticu lonodular y quistes en lóbu los inferiores respetando ángulo aspecto lechoso y contiene macrófagos cargados de lípidos y grandes
costofrénico cantidades de un material extracelular eosinófi lo PAS+. A veces se requie-
Figura 33. Patrón radiológico de la linfangioleiomiomatosis re biopsia transbronquial o incluso abierta.

Para el diagnóstico, suele recomendarse estudio histológico, dada la El tratamiento consiste en lavado pu lmonar completo, que sólo se realiza
gravedad de la enfermedad. Se realizan tinciones inmunohistoquímicas en casos de enfermedad progresiva y deterioro funcional importante. Se
específicas para componentes del múscu lo liso, actina o desmina, y re- han descrito casos de proteinosis alveola r primaria con remisión espon-
cientemente HMB-45. tánea.

La historia natural de la enfermedad es, por lo general, progresiva, con En la Tabla 14 se resumen las características más importantes de algunas
una media de supervivencia de unos ocho años desde el diagnóstico. de estas enfermedades intersticiales.

EPI O asociada a enfermedad del colágeno Histiocitosis x Linfangioleiomiomatosis Proteinosis alveolar

LES: pleuritis y/o derrame pleural. Rara EPID Más frecuente en varones Mujeres premenopáusicas. Patrón Rx como edema
AR: lo más frecuente es la afectación pleural. jóvenes Disnea, neumotórax pulmonar
20% EPID (más en varones Neumotórax recidivante espontáneo, que puede Riesgo de infecciones
con altos títulos de FR) ser bilateral y recurrente por Nocardia, M. avium y P.
LBA > 5% de células
EA: enfermedad fibroapical bullosa bilateral de Langerhans Quilotórax carinii. Tto.: LBA completo
en fases avanzadas Aumento de volúmenes Aumento de volúmenes
Esclerodermia: la que más presenta EPI D. y patrón obstructivo o mixto
Frecuente HTP Tto.: prog esterona/
Sjogren: infiltración linfocitaria intersticia l antiestrógenos
DM: máximo riesgo si anti-Jo-1
Tabla 14. Enfermedades intersticiales
Manual CTO de Medicina y Cirugía, 2. a edición

7.7. Granulomatosis broncocéntrica tota l recibida(> 450 unidades), el empleo de radioterapia torácica previa
o concom itante, la exposición a altas concentraciones de oxígeno inspi-
rado en los meses siguientes y el empleo de ciclofosfamida . La toxicidad
Se produce una invasión granulomatosa de las vías respiratorias infe- puede ser reversible si ha habido pocos cambios. En raras ocasiones cursa
riores por linfocitos y células p lasmáticas que produce úlceras y des- con neumonía eosinófi la. El tratam iento consiste en suprimir el fármaco
trucción bronqu ial. La afectación de los vasos es rara, a diferencia de y administrar corticoides.
las vasculitis y los procesos angiocéntricos. Afecta a pacientes asmáti-
cos. En la radiografía de tórax se aprecian infi ltrados, nódu los o áreas Metotrexato. La clínica suele comenzar días o semanas tras el inicio
de atelectasia. No suele producir apenas clínica y no hay manifestacio- del tratamiento y consiste en disnea, tos no productiva y fiebre. En el
nes extra pu lmonares. 50% de los casos hay eosinofi lia, y un tercio de los pacientes presen-
tan granu lomas. La radiografía muestra un infiltrado más homogéneo
Posteriormente a las descripciones originales, se han referido casos de que con otros agentes, con adenopatías hi liares o derrame en algunos
pacientes con esta enfermedad que, además de asma, presentaban eosi- casos. La DLCO no disminuye hasta que aparece la clínica. La toxicidad
nofilia y evidencia serológica de exposición a Aspergillus u otros hongos, no se relaciona con la dosis. El proceso es casi siempre reversible, con
y otros en cuyo tejido pu lmonar se encontraban hifas deg radadas del o sin corticoides.
hongo en los granu lomas necrosantes. Debido a ello, algunos autores
piensan que la granu lomatosis broncocéntrica y la asperg ilosis bronco- Nitrofurantoína. Existe una forma de toxicidad aguda que se produce
pulmonar alérgica (ABPA) podrían estar relac ionadas, si bien es una cues- por un mecanismo desconocido y que es el prototipo de neumonitis
tión aún no aclarada. eosinófila aguda inducida por fármacos, más frecuente en la mujer, con
fiebre, esca lofríos, tos y disnea. Un tercio presentan eosinofi lia sanguínea,
pero no en el LBA. En la histología, hay infiltración linfoplasmocitaria y
7 .8. Afectación pulmonar proliferación fibrob lástica.

en la amiloidosis En la rad iografía se observa un infiltrado alveolointersticia l en bases con


ocasiona l derrame pleura l, generalmente un ilateral. La toxicidad es dosis-
Consiste en el depósito extracelular de ami lo ide, un materia l proteináceo, independ iente. El tratamiento consiste en retirar el fá rmaco, y a veces, en
e insoluble. Puede ser una enfermedad sistémica o loca-
cristalino, fibrilar administrar corticoides.
!izada. Se expone ampliamente en la Sección de Reumatología.
La toxicidad crónica se produce por el efecto oxidante del fármaco. Es
A nivel del parénquima pulmonar, el amiloide se puede depositar de menos común que la aguda. La histología muestra células inflamatorias
forma localizada, produciendo una imagen radio lóg ica de uno o varios y fibrosis, sim ilar a la fib rosis pulmona r idiopática. La clínica (disnea y tos)
nódulos, o de modo difuso (se suele asociar a enfermedad sistém ica), pro- aparece de fo rma insid iosa desde meses a años después de iniciar el tra-
duciendo una enfermedad intersticia l. En este caso, la radiografía muestra tamiento. También es más frecuente en la mujer. La fiebre y la eosinofilia
un patrón reticulonodular o incluso miliar. Genera lmente, la afectación son raras. Radiológ icamente presenta patrón intersticial. En el LBA hay
intersticial no da síntomas. A veces se detecta un patrón restrictivo, con linfocitos. El tratamiento consiste en suspender el fármaco e iniciar corti-
o sin disnea. El depósito afecta a los septos, intersticio y paredes de vasos coides, si no se resue lve.
pequeños.
Amiodarona. Este fármaco presenta muchos efectos colaterales, siendo
El diagnóstico se rea liza con biopsia transbronquia l y tinción de rojo Con- el más serio la neumonitis intersticial, que ocurre en el 6% de los pacien-
go. El depósito de am iloide también se puede produci r a nivel traqueo- tes y puede ser fata l. Su histología es pecu liar, con macrófagos alveolares
bronquial, originando placas difusas o local izadas. espumosos y neumocitos tipo 11, con inclusiones lamelares compuestas
de fosfolípidos. La clínica es insidiosa, con disnea, tos no productiva y fe-
brícula. En un 20%, la presentación es aguda, simulando una neumonía
7 .9. Afectación pulmonar infecciosa.

por fármacos La mayoría de pacientes han tomado el fá rmaco durante un mes, en


una dosis de al menos 400 mg/día, aunque se ha informado casos con
Busulfá n. La incidencia es del 6%. El interva lo entre el in icio del trata- 200 mg/día. La toxicidad pulmonar no se re laciona con los niveles séri-
miento y el establecimiento de los síntomas suele ser de ocho meses cos. La radiografía muestra al principio afectación asimétrica limitada a
a diez años, aunque a veces aparece antes. No está claro si el efecto se lóbulos superiores, pero si progresa, afecta al resto de los pulmones con
relaciona con la dosis. La rad iografía muestra un patrón alveolointers- un infi ltrado intersticia l o alveolar. No hay eosinofi lia. Son características
ticial. El tratamiento consiste en suspender el fármaco y administrar la disminución de la DLCO y la hipoxemia. En el LBA se pueden ver los
corticoides, pero la respuesta es ma la y la mayoría progresan hasta macrófagos espumosos, pero no son específicas de toxicidad pulmonar,
causar la muerte. pues se pueden apreciar también en pacientes expuestos al fármaco sin
datos de toxicidad. La mayoría mejoran al suspender el fármaco y añadir
Bleomicina. Es el qu im ioterápico que produce con mayor frecuencia to- corticoides.
xicidad pulmonar (20%) y el mejor estudiado. Se produce una disminu-
ción de la DLCO antes de que aparezcan los síntomas. Existe relación en- En la Figura 34 se exponen, a modo de resumen, los patrones radiológi-
tre la toxicidad y la edad (más frecuente en mayores de 70 años), la dosis cos de las enfermedades intersticiales.

07 · Enfermedad es pulmonare s intersticiales


Neumología 1 07
Histiocitosis X (tl
Lóbulos superiores

·Silicosis
· Histiocitosis X (tl
· Espondilitis anquilopoyética

Patrón reticulonodular y quistes en lóbulos


superiores respetando ángulo costofrénico

Silicosis simple:
Patrón nodular en
lóbulos superiores
Adenopatías en
cáscara de huevo

Lóbulos inferiores

Fibrosis pulmonar idiopática Fibrosis masiva progresiva:


Dermatopolimiositis Grandes nódulos
Esclerodermia · Distorsión del parénquima
Asbestosis
Nitrofurantoína Fibrosis pulmonar
· Artritis reumatoide idiopática

Imagen en vidrio Patrón reticu lar difuso Imagen en panal


deslustrado en campos inferiores o queso suizo

• :-- -:~~~. _:~,'· r :~'~~


. -- " - _._.' ....,..
- -- .'
'.

Nódulos cavitados Negativo del edema Infiltrados nodulares finos


Infiltrados perihi liares
múltip les pulmonar desde los hilios

Figura 34. Patrones radiológ icos de las enfermedades intersticiales


Manual CTO de Medicina y Cirugía, 2. a edición

" Afecta a varones jóvenes fumadores.


Ideas clave ~
" Son hallazgos radiológicos típicos la combinación de nódulos-
" Clínicamente, la fibrosis pulmonar idiopática se caracteriza por quistes, de predominio en vértices, con volúmenes pulmonares
presentar tos y disnea de esfuerzo de más de seis meses de evolu- aumentados.
ción, crepitantes secos en las bases y, con frecuencia, acropaqu ias.
" En las pruebas de función respiratoria se observa patrón obs-
" Funcionalmente, cursa con restricción con descenso de la DLCO tructivo con descenso de la DLCO.
y caída de la P0 2 con el esfuerzo.
" La linfangioleiomiomatosis es exclusiva de mujeres en edad
" En la Rx de tórax se aprecia patrón intersticial, de predominio ba- fértil. Cursa también con quistes diseminados, patrón inters-
sal, con zonas de panal; en la TC se aprecia engrosamiento de sep- ticial con volúmenes altos, obstrucción con DLCO baja, y son
tos y zonas de panal de predominio en bases, siendo las áreas de frecuentes el neumotórax y el derrame pleural (quilotórax).
aumento de densidad "en vidrio deslustrado" escasas o ausentes.
" La amiodarona produce toxicidad pulmonar en el 6% de los
" En el lavado broncoalveolar predominan los neutrófilos. pacientes que la toman, generalmente con dosis superiores a
400 mg, y predomina en campos superiores.
" La histiocitosis X se caracteriza por una proliferación de células
de Langerhans.

tes resultados de pruebas complementarias, ¿cuál sería el menos


Casos clínicos relacionado con el diagnóstico de este paciente?

Paciente de 65 años, fumador de 25 paquetes/año, de profesión 1) Elevación de la enzima conversora de angiotensina sérica .
ingeniero, sin antecedentes de interés. Presenta disnea de es- 2) Lavado broncoalveolar con una cifra elevada de neutrófilos.
fuerzo progresiva y tos seca desde hace un año. La exploración 3) Estudio funcional respiratorio con CVF 65%, FEV 71%, cociente
1
revela acropaquias y estertores crepitantes, bilaterales y persis- FEVJFVC 0,89 y DLCO 52%.
tentes. La radiografía de tórax muestra imágenes reticulonoduli- 4) Biopsia pulmonar compatible con neumonía intersticial usual.
llares basales y simétricas, con reducción de los campos pulmo-
nares. La exploración funcional pulmonar únicamente indica: RC: 1
capacidad de difusión (DLCO), 43% referencia; Sa0 2, 94%, en
reposo, y 72%, al esfuerzo. ¿Cuál es el diagnóstico más probable?

1) Enfermedad pulmonar obstructiva crónica .


2) Bronquiolitis obliterativa con neumonía organizada.
3) Alveolitis alérgica extrínseca.
4) Fibrosis pulmonar idiopática.

RC: 4

Un paciente de 59 años, sin antecedentes de interés, no fumador,


consulta por tos seca y disnea de 6 meses de evolución. En la ex-
ploración física se auscultan crepitantes en la mitad inferior de
ambos hemitórax y se objetivan acropaquias. Su radiografía de
tórax posteroanterior se muestra a continuación. De los siguien-

1) Prevalence of the di sea se about 50 years.


'Case Study . 2) lnspiratory crackles in the lung bases.
3) Reticular pattern on chest x-ray predominantly in the lower
lt would not be expected to find one of the following in a patient fields beca use of the existen ce of pleural plaques.
with suspected idiopathic pulmonary fibrosis (cryptogenic fibro- 4) Decreased CPT.
sing alveolitis):
Correct answer: 3

07 · Enfermedades pulmonares intersticiales


Neumo

Enfermedades por inhalación


de polvos

En los casos agudos, hay inflamación alveolar e intersticial con pre-


Tema de importancia
domin io de linfocitos y aumento de cé lulas plasmáticas y macrófa-
relativamente escasa. Hay que
gos alveo lares activados. Son típ icos los macrófagos con citoplasma
centrarse en las neumonitis por
hipersensibilidad, la silicosis y espumoso, y puede haber células gigantes. Se suele apreciar daño
la asbestosis, haciendo hincapié en el epite lio alveolar y en el endotelio con un material proteináceo
en los hallazgos radiológicos. intraalveolar. Se ha descrito bronqu iolítis obliterante obstructiva,
que puede ser la causa de la obstrucc ión al flujo aéreo que presen-
tan algunos pacientes. Con frecuencia, la forma aguda se resuelve
sin secuelas.
La inhalación de polvos se asocia a diferentes enfermedades pulmona-
res. Clásicamente, se diferencia entre polvos orgánicos (neumonitis por En la forma subaguda se observan granulomas no caseosos de células
hipersensibilidad, bisinosis, etc.) y polvos inorgánicos (neumoconiosis: epitelioides con células gigantes multinucleadas. Recuerdan a los de la
sil icosis, enfermedad de los mineros del carbón, exposición al asbesto, sarcoidosis, pero están pobremente definidos en la periferia. En la forma
beriliosis, etc.). crónica, los granulomas persisten o desaparecen, según se va desarro-
llando la fibrosis pulmonar.

8.1. Neumonitis por hipersensibilidad Patogenia

El desarrollo de NH depende de una re lación compleja entre ambiente y


La neumonitis por hipersensibil idad (NH) o alveolitis alérgica extrínseca factores de l huésped (incluidos los genéticos). Es necesa ri a cierta suscep-
representa un grupo de enfermedades que afectan a la porción dista l de tibilidad del huésped, ya que sólo el 10% de los expuestos desa rro llan la
la vía aérea, mediadas inmunológicamente, y asociadas a una exposición enfermedad . La reacc ión inm unológ ica más re leva nte es el daño tisular
intensa y/o repetida a polvos orgánicos. Predomina en no fumadores, lo mediado por inmunocomplejos (reacción tipo 111). También parece ínter-
que podría estar en re lación con las alteraciones que produce el tabaco ven ir la reacción tipo IV (aparecen granu lomas en la biopsia pu lmonar).
en la respuesta inmunológica. Algunos estudios sugieren la posibi lidad de que el mecanismo tipo 1tam-
bién participe.
La mayoría de agentes etiológicos derivan de exposiciones ocupaciona-
les (granjeros, cosechadores de cañas de azúcar, trabajadores de granos Clínica
de cereales y productos de madera, empaquetadores de champiñones,
etc.). También puede relac ionarse con aparatos de calefacción central, Se describe una forma aguda, subaguda y crónica, según la intensidad y
humidificadores, aficiones como la cría de palomas, etcétera. frecuencia de la exposición (algunos autores sólo diferencian la aguda y
la crónica).
Los antígenos productores del daño pueden derivar de microorganismos
(actinomicetos termófilos, algunas bacterias ... ), de productos de animales Forma aguda
o plantas, de sustancias químicas de bajo peso molecular y de algunos
productos farmacológicos. Se produce ante exposiciones intermitentes y breves a grandes dosis
de antígeno. Típicamente, a las 4-8 horas de la exposición, el paciente
Anatomía patológica presenta fiebre, esca lofríos, disnea, tos no productiva, mia lgias y ma-
lestar general. Se aprecian crepitantes en ambos pulmones, taquipnea
La mayoría de las NH tienen cambios histológ icos similares que depen- y a veces cianosis. Es fáci l confund ir el cuadro con una neumonía in-
den de la intensidad de la expos ición y de l estadio de la enfermedad en fecciosa. En sang re perifé rica existe leucocitosis con neutrofi lia, linfo-
el momento en que se rea liza la biopsia. penia y no hay eos inofilia. En el LBA aparece neutrofilia . La clínica casi
Manual CTO de Medicina y Cirugía, 2.a edición

siempre desaparece si cesa la expos ici ón, pero si esta conti nú a, puede Diagnóstico
desarrol larse la forma crónica .
Se basa en una historia clínica compatible en un paciente con exposición
a algunas de las etiologías conocidas. Además de la radiografía y los estu-
La forma aguda predomina en campos infer iores y dios de función pu lmonar mencionados, ayudan al diagnóstico:
en el LBA ex iste neutrofi li a; en la crónica, se afec- Analítica sanguínea. Tras la exposición aguda, aparece neutrofilia
tan sobre todo los ca mpos superiores, y en el LBA y linfopenia. Todas las formas de NH pueden tener elevación de la
hay predominio de linfoc itos CD8 (lo más ca racte-
VSG, proteína C reactiva, inmunoglobulinas y factor reumatoide. La
rístico).
eosinofilia no es característica de este proceso.
Estudios inmunológicos. Los pacientes suelen tener anticuer-
Forma subaguda pos precipitantes contra el agente que produce la enfermedad,
pero un número elevado de personas expuestas pueden tener
Se produce ante exposiciones más continuadas, pero también de corta las precipitinas sin estar enfermos. También puede haber falsos
duración. Tiene un com ienzo insid ioso con tos, productiva o no, disnea negativos.
prog resiva al ejercicio, malestar genera l, anorexia y pérdida de peso. Tam- Fibrobroncoscopia. En el LBA, típicamente hay incremento de lin-
bién suele ser reve rsibl e. foc itos T. En la forma crón ica aumentan de modo predominante los
CDS (supresores), con lo que la re lación CD4/CD8 es menor a l. En
Forma crónica la forma aguda, cua ndo el tiempo desde la exposición es menor de
48 horas, el líquido del LBA muestra neutrofilia y, pasados cinco días
Aparece ante expos iciones prolongadas a dosis menores de antígeno. de la última exposición, hay linfocitosis con predominio de CDS,
Presenta, durante meses o años, los síntoma s referid os en la subagu- aunque en algunos casos se observa aumento de CD4; esto parece
da, con predominio de la disnea progresiva, que ll ega a ser grave, y relacionado con los diferentes t ipos de exposiciones. Los eosinófi-
tos (productiva o no). No es raro el cor pulmonale. No existe fiebre. los están ausentes o aparecen en un mínimo porcentaje. La biopsia
La clínica remeda la de la bronquitis crón ica del fumador, por lo que transbronquial raras veces obtiene material suficiente para hacer el
hay que pensar en NH crónica en pacientes no fumadores y con estas diagnóstico, por lo que a veces se recurre a la biopsia por toracosco-
características clínicas, y hacer una buena historia de exposiciones. Es pia o toracotomía.
poco probable que revierta. A veces se desarrolla una forma crónica Pruebas cutáneas específicas. Son útiles, aunque su especifidad
tras exposiciones agudas repetidas. es baja.
Test de provocación. La inhalación de un extracto del antígeno
sospechado se hace raras veces, y sólo en laboratorios especiali-
zados, porque puede producir daño permanente pulmonar en un
La alteración funcional ca racterística es el patrón res-
trictivo co n DLCO baja, pero puede cursar con tras- sujeto sensibilizado, y además los antígenos no están bien estan-
torno obstructivo (por afectación bronqu io lar). darizados.

Tratamiento
Rad iológica mente, tanto la forma aguda como la subaguda se caracteri-
zan por infiltrados nodu lares mal defin idos y a veces con áreas en vidrio A largo plazo, es fundamental el reconocimiento y la eliminación del
deslustrado e incluso infiltrados alveolares. Predomina en lóbu los inferio- agente etiológ ico. En las formas agudas leves sue le ser suficiente para la
res y respeta los vértices. Puede haber imágenes linea les, expresión de resolución en unos días la retirada del paciente del lugar de la exposición.
áreas de fibrosis como secuela de episod ios ag udos previos. Tras el episo- Cua nd o los episod ios son más graves se requieren corticoides. La forma
dio agudo y el cese de la exposición, la radi ografía ti ende a normalizarse. subag uda suele precisar tratamiento con corticoides. En la forma crón ica,
En la forma crónica, hay infi ltrados nodu lares y linea les, de predom inio la retirada del paciente del ambiente produce a veces la resolución de
en lóbulos superiores y que respetan las bases. Incluso puede haber un los síntomas y de las alteraciones fisiológicas en un mes. Sin embargo,
patrón en pana l con pérdida de volumen que predomina en lóbulos su- en muchos pacientes, la fibrosis pu lmonar y las alteraciones fisiológicas
periores. sólo son parcialmente revers ibles al cesa r la exposición. En ocasiones, se
emplean corticoides con disminución posterior de la dosis hasta la dosis
En la forma aguda, hay un patrón ventilatorio restrictivo, con pérdi- mínima que mantenga el estado funcional.
da sobre todo de la CV y disminución de la DLCO y la distensibilidad.
Suele existir hipoxemia con d ism inu ción de la PaC0 2 por hiperven-
tilación. En la forma crónica tamb ién predomina un patrón restric- 8.2. BisinOSÍS
tivo con dism inución de la DLCO, pero lleva asociado con frecuen-
cia un defecto obstructivo leve que se cree debido a la bronquioli-
tis. Cursa con hipoxemia inducida por el esfuerzo y, a veces, en re- Está en relación con la exposición al polvo de algodón, especialmente en
poso. el proceso de cardado. Existen síndromes parecidos, aunque más leves,
en relación con exposición a lino, cáñamo y yute. Muchos la consideran
un asma ocupacional. La clínica es de disnea y opresión hacia el final de
la jornada del primer día de trabajo, cuando se acude tras un periodo de
En la neumoniti s por hipersens ibilidad no ex iste eo- descanso como el fin de semana ("opresión torácica del lunes"). Algunos
sinofi lia. pacientes siguen con síntomas todos los días de la semana, y otros desa-
rro ll an tolerancia y no tienen clínica hasta el sigu iente lunes.
Neumología 1 8
El polvo de algodón asociado al tabaco aumenta la preva lencia de bron-
quitis crónica, pero es controvertido que el algodón produzca por sí mis-
mo obstrucción irreversible. No se conoce el mecanismo exacto por el
que se induce la broncoconstricción. Una teoría lo rela ciona con la exis-
tencia de cantidades significativas de histamina en el algodón.

El dato clínico que más ayuda a separar neumonitis


por hipersensibilidad de neumoconios is es el tiempo
de exposición/edad (joven: neumonitis, edad media:
neumocon iosis).

8.3. Silicosis

Es una enfermedad fibrótica de los pulmones causada por inhalación, Figura 35. Radiografía posteroanterior de tórax de un paciente
retención y reacción pulmonar al polvo de sílice cristalina. Las exposicio- con si licosis clásica
nes ocupaciona les a partículas de sílice (cuarzo) de tamaño "respira ble"
(d iámetro de 0,5-5 micras) ocurren en minería, canteras de granito, perfo- Fibrosis masiva progresiva (silicosis complicada)
raciones y tunelizaciones, cortadores de piedra, industrias abrasivas, fun-
diciones e industrias cerám icas. La fibrosis masiva progresiva es una imagen radiológica que aparece en
el seno de diversas enfermedades (silicosis, neumoconiosis de mineros
El desarrollo y la prog resión de la si licosis suele ocu rrir cuando la exposi- del carbón, etc.). La silicosis complicada ocurre cuando coa/escen Jos
ción ha cesado, por lo que la prevalencia de la enferm edad es desconoci- pequeños nódu los si/ icóticos, formando grandes conglomerados (> 1O
da. En la patogen ia, parece fundamental/a interacción entre los macrófa- mm). La rad iografía de tóra x muestra estos grandes nódulos múltiples
gos alveolares y las partícu las de sílice. bilaterales en lóbulos superiores. Con el tiempo, estas masas tienden a
contraer los lóbulos superiores, dejando zonas hipertransparentes en Jos
Clínica y diagnóstico márgenes. Ocurre incluso cuando la exposición al sílice ha cesado. Hay
compromiso clínico con disnea de esfuerzo y tos (el polvo puede produ-
Se diferencian cuatro formas de silicosis, según la intensidad de la exposi- cir bronquitis, o grandes nódulos linfáticos pueden comprimir la tráquea
ción, el periodo de latencia y la historia natural. o bronquios principales). Se suele objetivar un patrón ventilatorio restric-
tivo con disminución de la DLCO. A veces lleva asociada obstrucción de-
Silicosis crónica, simple O clásica bida a la bronquitis concomitante. Puede haber infecciones bacterianas
recurrentes, y si hay cavitaciones de los grandes nódulos, se debe pensar
Suele apa recer t ra s una o más décadas de exposición al polvo de sílice en tubercu losis u otras m icobacterias. Comp licaciones term inales de la
(Tabla 15). Es común una latencia de 15 o más años desde el comienzo enferm edad son la insuficiencia respiratoria hipoxémica con cor pulmo-
de la exposición. na/e.

Silicosis acelerada
Neumoconiosis Asbestosis Silicosis crónica
Se relaciona con una exposición más corta (5 -1O años) e intensa. Las al-
Carbón Amianto (frenos) Cuarzo (cantera
granito) teraciones patológicas, fisiológicas, clínicas y radiológicas son parecidas
Patrón restrictivo Restrictivo
Obstructivo a la forma crónica, pero la progresión es más rápida . También es más fre-
Campos superiores Bases
Vértices cuente encontrar afecciones del tejido conjuntivo y enfermedades auto-
Ka plan Riesgo neoplasia
inmunitarias como la escleroderm ia, siendo la progresión aún más rápida.
Sin acropaquias Con acropaqu ias Cásca ra de huevo
Algunos autores consideran que esta forma también puede desarrollar la
NoTBC NoTBC Con acropaquias
fibrosis masiva progresiva.
No tratamiento No t ratamiento SiTBC
No tratamiento
Silicosis aguda
Tabla 15. Polvos inorgánicos
Es consecuencia de una exposición intensa a altos niveles de polvo de
Radiológicamente, presenta pequeñas opacidades redondas (menos de sílice, como en trabajadores con chorros de arena o en la fabricación de
1O mm), sobre todo, en lóbulos superiores, y pueden existir adenopatías pastas abrasivas (véase la Tabla 1 S). La enfermedad se desarrolla desde
hil ia res, a veces calcificadas "en cáscara de huevo" (Figura 35). El nódulo meses a cinco años tras la exposición masiva. El paciente sue le tener dis-
si/icótico es la marca patológica de esta forma de silicosis. Pueden apa- nea importante con debilidad y pérdida de peso.
recer en la pleu ra visceral, en ganglios linfáticos regionales y en otros
órganos. Puede ser asintomática o presentar tos y disnea progresiva al La radiografía de tórax muestra un patrón alveolar difuso que predomina
esfuerzo. No hay alteración fun cional significativa. No disminuye la espe- en campos inferiores, con o sin broncograma aéreo (Figura 36). La his-
ranza de vida. tología revela datos similares a los encontrados en la proteinosis alveolar.
Manual CTO de Medicina y Cirugía, 2.a edición

de Ca plan consiste en la asociación de


nódulos pulmonares de 5 a 50 mm,
generalmente bilaterales y periféricos,
con artritis reumatoide. También se
observa en otras neumoconiosis.

En la radiología de la neumoconiosis
simple por carbón destacan peque-
ñas opacidades menores de 1 cm,
que predominan en lóbulos superio-
Silicosis aguda Silicosis simple res, y que se van extend iendo al resto
Patrón miliar 11 con solidación · Patrón reticular en lóbulos superiores
del pulmón. La forma complicada o
· Adenopatía s en "cáscara de huevo"
fibrosis masiva progresiva se caracte-
Figura 36. Patrones radiológicos en la silicos is riza por opacidades mayores de 1 cm
que predomina n también en lóbulos
Progresa hacia insuficiencia respiratoria y m uerte en menos de dos años. superiores. Esta fo rma se asocia con muerte prematura, y es menos fre-
cuente encontra rl a que en la silicosis.
En la si licosis, la espirometría puede ser norm al, y si no lo es, puede mos-
tra r patrón obstru ctivo, restrictivo o mi xto, siendo qu izás el obstructivo
el más frec uente. Genera lmente no es preciso rea liza r biopsia pulmona r 8.5. Exposición al asbesto
para llegar al diagnóstico.

Complicaciones El asbesto es un silicato magnésico hidratado fibroso con una gran varie-
dad de usos comercia les, dada su indestructibilidad y resistenc ia al fuego.
Todas las formas de sil icosis, pero sobre todo la aguda y la acelerada, pre- Desde los años setenta, ha sido sustitu ido en gran medida por fibras mi-
disponen a tuberculosis o infección por micobacterias atípicas. Las per- nera les como la fibra de vidrio, pero aún se emplea en la fabricación de
sonas expuestas a sílice pueden desarrolla r bronquitis crónica y enfisema frenos y como aislante de tuberías y calderas.
y, a veces, disminución de la función pulmonar, aunque no haya altera-
ciones radiológicas que sugieran silicosis. Pueden aparecer neumotórax Generalmente, la asbestosis sólo se hace evidente tras una latencia de
y cor pulmona/e. En la actual idad, continúa en estudio la posible re lación unos d iez años después de una exposición prolongada (1 0-20 años). La
entre silicosis y cáncer de pu lmón. prevalencia es mayor cuanto más intensa y mantenida sea dicha exposi-
ción y cuanto más tiempo haya t ranscurrido desde la misma. Hoy día es
excepcional, debido a los controles laborales existentes.

La sili cosis pred ispone al desarro ll o de tubercu los is. Clínica


Recuerda El síntoma más precoz y común es la d isnea. También puede haber tos y
expectoración. Son característicos los crepitantes basa les y en las zonas
Tratamiento axi lares. Puede haber acropaqu ias. El curso de la enfermedad es va riable
y puede perma necer estable durante años y luego progresar, o bien pro-
Una vez que se establece el proceso fibrót ico de la silicosis cró nica, se cree g resa r rápida ment e.
que es irreve rsible, y el ma nejo se dirige a prevenir las complicaciones. En
los pacientes con Ma ntoux posit ivo, se rea liza profi laxis con isoniacida. Los pacientes afectados de asbestosis t ienen riesgo aumentado de sufrir
cá ncer de pu lmón (el cá ncer más común en estos pacientes) a partir de
los 15 años desde la expos ición, siendo los más frecuentes el epidermoi-
8 .4. Neumoconiosis de los mineros de y el adenocarcinoma, aunq ue puede ser de cua lqu ier t ipo histológ ico
(Figura 37). La exposición al asbesto y al tabaco tiene un efecto sinérg ico
del carbón (NMC) en cuanto a la carcinogénesis pu lmonar.

Es la enfermedad del parénquima pulmonar que se prod uce por la inha- Otra complicación es el mesotelioma mal igno pleural o peritoneal, en
lación y el depósito del polvo del carbón y la reacc ión tisular que provoca el que no parece influir el tabaco, de difícil diagnóstico, dada la presen-
(véase la Tabla 15). Se precisa una exposición de 5- 1O años. Es más fre- cia frecuente de derrame pleura l que enmascara el tumor subyacente. El
cuente en trabajadores de minas de antracita bajo la superficie. Además periodo de latencia es muy largo (20-30 años). No parece haber riesgo
de la NMC la exposición al polvo de carbón aumenta el riesgo de bron- incrementado de tuberculosis.
qu itis crónica y enfisema y acelera la pérd ida de func ión venti latoria.

Algunos pacientes no tienen clín ica. Otros presentan tos crónica con
El asbesto es el pricipa l factor de riesgo pa ra el desa-
expectoración, probablemente debida a cambios de bronquitis en las rro ll o de mesoteli oma, y también au menta la inc iden-
grandes vías. Cuando la neumoconiosis ava nza, puede haber disnea. No cia de cáncer de pu lmón. Recuerda
implican acropaquias ni aumento del ri esgo de tuberculosis. El síndrome
Neumología 1 8
Exposición al asbesto
Diagnóstico

Siempre debe estar basado en una histori a adecuada de exposición . En


su ausencia o si la clínica es confu sa, el LBA y, sobre todo, la biopsia bus-
cando fi bras de asbesto, que pueden estar cubiertas de un materi al pro-
teináceo que les confiere aspecto de halterio (cu erpos de asbesto) o no
Asbestosis Cáncer de pulmón Mesotelioma cubiertas (más frecuentes), pueden se r útiles.

· Enfermedad intersticial · Epidermoide Máx. riesgo 30-35


· Rx: opacidades lineales o adenocarcinoma años de exposición
Tratamiento
en campos inferiores · M ínim o 15-19 años Rx: derrame pleural
· Placas pleurales solas de expos ición Mu erte por extensi ón No se conoce ni ngú n trata miento eficaz. Los corticoides no son úti les. Se
indican exposición Efecto sinérgico local
recom ienda vigi lancia méd ica, debido a las complicaciones mencionadas.
con el tabaco

Figura 37. Patrones de exposición al asbesto


8.6. Beriliosis
La imagen rad iológica típica es un patrón reticu lar que predomi na en
campos inferiores y en las zonas laterales unido a placas pleu rales, que
consisten en lesiones blancas, foca les e irreg ulares, generalmente en La exposición al beril io puede prod uci r una en fermedad aguda que afee-
pleu ra parietal, ra ras en la visceral. Las placas pleurales indican exposi- ta al tracto resp iratorio superior, o, si el nivel de exposición es más alto,
ción al asbesto, y cuando son bilatera les en el seno de una enfermedad puede prod ucir una neu mon it is química.
intersticial de predom in io en ca m pos inferiores, orientan fuertemente al
diagnóstico de asbestosis. La TCAR añade informació n sobre la extensión La enfermedad crónica por berilio es más frec uente que la ag uda y con -
de la enfermedad, sobre todo, en estadios tempranos y sobre la afecta- siste en un trastorno gran ulomatoso pulmonar, aunque tam bién puede
ción pleural. afectar a otros órganos (nód ulos li nfáticos, piel, hígado). La enfer medad
aparece unos años después de la exposición (fábricas de cerá mica y tu -
Suele prod ucirse una alteración ventilatori a restrictiva con DLCO dism i- bos fluorescentes o industri as de alta tecnolog ía).
nuida (este es un signo precoz de enfermedad grave). En estad ios inicia-
les puede haber dism in ución del flujo aéreo en volúmenes bajos, lo que La clínica es sim ilar a la de otras enfermed ad es intersti ci ales pulmonares.
sugiere alteración de pequeña vía. Radiológ ica mente suele haber un patrón reticulonod ular, a veces con
adenopatías hiliares, similar al de sa rcoidosis. La histología muestra gra-
Los pacientes expuestos al asbesto pueden presentar derra me pleura l nulomas idénticos a los de la sa rcoidosis, por lo que, para diferencia rlas, a
benig no y atelectasia redo nda. veces se req uiere determ inar la concentración ti sular de berilio.

Ideas clave z " El LBA ca racterístico muestra linfocitosis con predominio de


CD8.

" Las neumonitis por hipersensibilidad se producen como reac- " El hallazgo de adenopatías calcificadas en "cáscara de huevo"
ción a la inhalación de polvos orgán icos, por lo que el antece- sugiere el diagnóstico de silicosis.
dente de exposición es crucial en la sospecha diagnóstica.
" Las placas pleura les asociadas a patrón intersticial, de predom i-
" En su patogen ia, están involucradas las reacciones inmunoló- nio en campos inferiores, son muy sugestivas de asbestosis.
gicas tipo 111 (antígeno-anticuerpo) y IV (presencia de granulo-
mas), y en ocasiones, la de tipo 1(hipersen sibilidad inmed iata) .

Hombre de 56 años, evaluado porque, en una radiografía de tó-


Casos clínicos rax realizada para un estudio preoperatorio de hernia inguinal,
aparece un patrón intersticial reticular en bases pulmonares y
Un paciente de 30 años, habitante de un pueblo de Cantabria, no fu- placas pleurales calcificadas en diafragma. ¿Cuál considera us-
mador, que trabaja por las mañanas en una cantera de granito y por ted que es el dato a conocer?
las tardes ayuda en su casa al cuidado del ganado vacuno, acude al
médico por disnea de esfuerzo y un patrón intersticial. La TAC toráci- 1) Indaga r antecedentes de t ubercu losis.
ca confirma el patrón intersticial, y además objetiva imágenes de vi- 2) Interrogar sobre la existencia de anima les do mésticos.
drio esmerilado de tipo mosaico. El diagnóstico más probable es de: 3) Buscar la existencia de alteraci o nes articulares en la explora ción
fís ica.
1) Neumonitis por hipersens ib ilidad. 4) Interrogar sobre la historia labo ral.
2) Proteinosis alveo lar.
3) Sa rcoidosis. RC: 4
4) Silicatosis.

RC: 1
Manual CTO de Medicina y Cirugía, 2.8 edición

Which one of the following statements regarding diseases cau-


Case Study sed by inorganic dust is NOT true?

A farmer develops acute attacks of fever, dyspnea, cough, and 1) Pleural plaques, which are lesions located especially in the pa-
leukocytosis whenever he works around wet, harvested hay. Lung rieta l pleura, are specific to exposure.
biopsy would be most likely to reveal which of the following? 2) Rounded atelectasis is a rare complication of exposure to asbes-
tos.
1) Alveoli filled with dense, amorphous, proteinaceous material. 3) Most forms of silicosis that predispose to tuberculosis are acute
2) Ferroginous bodies. and accelerated.
3) lnterstitial pneumonitis with lymphocytes, plasma cells, and 4) lnterstitial disease caused by exposu re to asbestos is predomi-
macrophages and interstitial fibrosis. nantly in the lower lobes.
4) Linear immune deposition along the alveolar basement mem-
brane. Correct answer: 1

Correct answer: 3
Eosinofilias pulmonares

brícula y hemoptisis. En la radiografía de tó rax pueden aparecer infi ltrados


Sí es obligado conocer la
t ransitorios y recurrent es y bron q uiectasias proximales (Figuras 38 y 39).
neumonía eosinófila crónica
y la ABPA, una vez más,
con especial atención a las
características radiológicas.

Son procesos caracterizados por infi lt rados pulmonares con eosinófilos,


pudiendo existir (aunque no siempre) tamb ién eosinofilia periférica .

9.1. Aspergilosis broncopulmonar


alérgica (ABPA)

Se estima que complica ell-2o/o de los casos de asma crón ica, sobre todo
corticodependiente, y un 10-lS o/o de los casos de fibrosis quística. Suele
aparecer en la tercera o cuarta década de la vida. Los pacientes suelen
tener historia de atopia con alerg ia a fármacos, asma, rinitis y conjunt ivitis. Figura 39. Rad iografía posteroanterior de tórax de un paciente
con ABPA: infiltrado y bronquiectasias ce ntrales
El agente responsable suele ser el Aspergillus fumigatus, aunque hay des-
critas reacciones similares a otras especies de Aspergillus u otros hongos.
Aspergillus es un hongo ubicuo y termotolerante que reside en la materia
orgánica en descomposición .

Parece que se produce una reacción in munológ ica a la colonización cró-


nica de la vía aérea por el hongo, tanto co n un mecanismo de hipersen-
sibilidad tipo 1 (pues existen nive les altos de lgE total y específica), t ipo
111 (pues existen precipitinas a Aspergillus e inmunocomplejos circu lantes
en las exacerbaciones) y ta l vez de tipo IV (dada la existencia de reacción
cutánea dua l, inmediata y retardada a Aspergillus en algu nos pacient es).

En la anatom ía patológica hay infi ltrac ión eosinófi la de los pulmones y ta-
pones de moco. Además de eosinófilos, hay un infil t rado de mononuclea-
res, célu las gigantes y granulomas. Los tapones están ocupados por hifas de
Aspergillus, pero el hongo no invade ni la pa red ni el pulmón circundante.

Cl ínica

La presentación t ípica incluye asma de d ifícil control, tos productiva con Figura 39. Rad iog rafía posteroanteri or de tórax de un paciente
tapones mucosos gruesos marronáceos, disnea y sibilancias, malestar, fe- con ABPA: infiltrado y bronquiectasias centrales
Manual CTO de Medicina y Cirugía, 2. 8 edición

Existen unos criterios mayores y otros menores pa ra el d iag nóstico de 9.3. Eosinofilias pulmonares
esta enfermedad (Tabla 16).
de etiología desconocida

Criterios mayores

Asma Neumonía eosinófila aguda


Eosinofilia sangu ínea (generalmente más de l .OOO/mm 3)
Precipitinas lgG contra Aspergillus Es una fo rma g rave de neumonía id iopá t ica que afecta a personas de
cualqu ier edad y sexo previamente sanas.
Test cutáneo de hipersensibilidad inmed iata positivo para Aspergillus
lgE total sérica elevada
Se presenta de forma aguda con fi ebre, tos, d isnea, crepitantes auscul-
Historia de infiltrados pu lmonares tra nsitorios
tatorios, m ialg ias e hipoxemia grave. La radiografía es sim ilar a la encon-
Bronquiectasias proximales o centrales, evidenciadas rad iológicamente
t rada en el distrés respirat orio del adu lto (i nfi ltrados alveolointersticiales
lgE e lgG específicas para Aspergil/us elevadas (clásicamente
considerado criterio menor; actua lmente ha pasado a ser bi laterales), a veces con derrame pleura l. En el LBA hay eosinofi lia, pero
uno de los criterios mayores) en sa ngre periférica no es constante. El cuadro suele d urar menos de una
semana, con un pronóstico excelente y buena respuesta a corticoides.
Criterios menores
No hay reca ídas.
Expectoración de tapones mucosos ma rronáceos
Identificación de Aspergillus en esputo Síndrome de Loeffler
Reacción cutá nea reta rdada al hongo
(eosinofilia pulmonar simple)
Tabla 16. Criterios de diagnóstico de la ABPA

Tratamiento Se t rata de una neumo nía eosinófi la ag uda benig na, id iopática o por
hi persens ibilidad a Ascaris lumbricoides, fármacos u ot ros parásitos. Mu-
Se rea liza con cort icoides por vía sistémica. chos autores reserva n este térmi no para las eosinofil ias idiopáticas. La
sintomato log ía es leve o nu la (hallazgo en radiog rafías de tórax de rut ina)
y aut oli m itada en 1-2 se manas. En la ra d iog rafía se aprecian infiltrados
9.2. Otras eosinofilias pulmonares interst iciales y/o alveolares t ran sitorios (< 4 semanas) y migratorios no
seg mentaríos, genera lmente periféri cos. Si se co nfirma la etiolog ía de As-
de etiología conocida ca ris, el trata miento es el mebendazo l.

Neumonía eosinófila crónica


Eosinofilia pulmonar por parásitos
Predom ina en m uj eres de eda d media. Casi la mitad de los pacien-
Varios pa rásitos p ueden prod ucir infilt rados con eosinofi lia, como Asca- tes t ienen hi st o ri a de at o p ia, rinit is o pó lipos nasa les, y hasta dos ter-
ris, Ancy/ostoma, Toxocara y Strongylo ides stercolaris. Algu nas fil arí as como cios, asm a de reciente co mi enzo. La p resentació n clín ica es subaguda
Wuchereria bancroftiy W malayi (típicas de As ia, Africa y Suda mérica) ta m- y consiste en fi ebre m ode rada, sudores nocturn os, pérdida de peso
bién pueden prod ucirlo, denomi ná ndose entonces eosi nofili a tropica l mode rada e im po rta nte t os con escaso es p uto mucoide durante va-
(su tratam iento es la d ietilca rba mazina). ri os meses antes de l d iagnóstico. Poste ri o rme nte apa rece d isnea (con
sibi lancias, si hay as m a) .
Eosinofilia pulmonar por fármacos

Muchos fármacos se han asociado con infi ltrados y eosinofi lia (penicilina,
M uj er asmáti ca con eos inofilia e inf iltrados pulmo na-
sulfamidas, t iacidas, isoniacida, sales de oro, clorpropa mida ...), pero el más res perifé ricos (negativo de l edema de pulmón): neu-
rep resentativo es la nitrofura ntoína, que puede prod ucir una reacción monía eosi nófila cró ni ca.
aguda (horas o días t ras iniciar el trata miento), subag uda (después de un
mes) o crónica (tras seis meses). La forma ag uda es la más frecuente y la
que impl ica mayor eosinofil ia. Se ca racteriza por fi ebre, esca lofríos, t os y Ra d iológ ica mente son característicos los infi lt rados subsegmentarios o
d isnea. lobares perifé ri cos bil atera les, que afecta n típicamente a los dos tercios
externos pu lmona res y predom inan en campos superiores y medios. Si
Radiológ ica mente hay infilt rados tanto intersticiales como alveolares, de son generalizados, dan una imagen de "negativo del edema pulmonar':
predom inio en bases, y derrame pleura l eosinófi lo en un tercio de los Los infi lt rados no son m ig rat orios, y si recu rren, suelen hacerlo en la mis-
casos. El t ratamiento consiste en reti ra r el fá rm aco, y en ocasiones, en ma loca lización (Figu ra 40).
adm inistra r corticoides.
La func ión pu lmonar reve la un patrón rest rictivo g rave con disminu-
Síndrome eosinofilia-mialgia ció n de la DLCO. Los pacientes con componente as m át ico también
p ueden tene r un defect o obstru ct ivo. Es muy frecuente la eos inofilia
La admi nistració n de sup lementos dietét icos de L-tri pt ófano puede pro- perifé ri ca, y hasta u n t ercio de los pacientes presentan elevación de la
d ucir el síndro me eosinofi lia-mialg ias, q ue a veces se acom pa ña de infil- lgE. Una eosinofil ia en el lavado bro nqu ioa lveolar del 30-50% es típica
t rados pulmo nares. de esta enfer medad. Se suele rea lizar b iopsia transbronqu ial para des-

09 · Eosinofilias pulmonares
Neumología 1 09
cartar otros procesos. El diagnóstico se basa en la clínica, la radiología terísticas las recaídas al disminuir o retirar los corticoides, por lo que hasta
y el LBA. una cuarta parte de los pacientes precisan una dosis de mantenimiento de
corticoides, a largo plazo, para mantenerse libres de enfermedad.

Edema pulmonar cardiogénico Neumonía eosinófila crónica


Síndrome hipereosinófilo

Afecta sobre todo a varones de edad media. Hay eosinofilia periférica e


infiltración difusa de órganos por eosinófi los, con gravedad muy variable,
desde enfermedad limitada a órganos no críticos como la piel, hasta afec-
tación multiorgánica amenazante para la vida. La enfermedad cardíaca es
la mayor causa de morbimortalidad. El aparato respiratorio se afecta en la
mitad de los pacientes.

El diagnóstico se establece por la existencia de eosinofilia sanguínea (>


1.500/¡JI) durante seis meses, disfunción multiorgánica y ausencia de
otras causas de eosinofilia periférica. La lgE puede esta r elevada. Sin trata-
Imagen "en alas de mariposa " "Negativo del edema pulmonar"
miento, la supervi vencia es menor de un año, pero con los corticoides se
Figura 40. Radiología típica de la neumonía eosinófil a crón ica prolonga hasta diez años.

El tratamiento son los corticoides sistémicos, siendo muy característica de Granulomatosis alérgica de Churg-Strauss
esta enfermedad la rápida mejoría de la clínica, rad iología y la reducción de
la eosinofilia sanguínea. Aunque el pronóstico es favorable, son muy carac- Esta enfermedad se estudia ampliamente en la Sección de Reumatología.

" La neumonía eosinófila crónica da una imagen radiológica en


Ideas clave ffiS negativo del edema de pulmón (predominio periférico).

" La aspergilosis broncopulmonar alérgica se caracteriza por


asma, eosinofilia, infiltrados, bronquiectasias centrales e imáge-
nes "en dedo de guante".

El diagnóstico más probable, de entre los siguientes, es:


Casos clínicos .
1) Granulomatosis de Wegener.
Un hombre de 50 años acude al médico con asma, febrícula, aste- 2) Aspergilosis broncopulmonar alérgica.
nia de dos meses de evolución e infiltrados en la radiografía de tó- 3) Neumonía eosinófila crónica.
rax que son bilaterales, de tipo alveolar, y situados periférica men- 4) Síndrome de Loeffler.
te junto a las axilas. Las únicas alteraciones analíticas son una cifra
de 15% de eosinófilos y una VSG de 100 mm a la primera hora. RC: 3

2) Chronic eosinophilic pneumonia can be associated with asthma.


Case Study . 3) Acute toxicity nitrofurantoin has basal infiltrate.
4) Simple pulmonary eosinophilia is sometimes treated with me-
Regarding pulmonary eosinophilia which of the following state- bendazole.
ments is FALSE:
Correct answer:
1) Chronic eosinophilic pneumonia presents migrant infiltrators.
eumoJogia_

Síndromes de hemorragia
alveolar difusa

Tema de importancia marginal res y hay datos de asociación familiar. La patogen ia no está aclarada,
en Neumología, aunque pero parece existir un mecan ismo inmunitario, pues se han encontra-
es recomendable que se revisen do anoma lías similares en el lupus. A veces, se asocia a enfermedad
las Secciones correspondientes celíaca y a hipersensibi lidad a leche de vaca; en un SOo/o de casos hay
de Nefrología y Reumatología
niveles aumentados de lgA, y algunos pacientes responden a inmu-
(vasculitis).
nosupresores.

La característica clín ica de esta enfermedad son los episodios recurrentes


10.1. Generalidades de hemorrag ia alveolar durante años (Figura 4 1), caracterizados por la
tríada hemoptisis, anemia e infiltrados pulmonares. Las secuelas son fi-
brosis pu lmonar, fallo respiratorio progresivo y cor pulmona/e. La hemop-
Son un grupo de enfermedades que clínicamente se caracterizan por tisis puede faltar en los niños muy pequeños por no poder expectorar
tener hemoptisis cuya causa no es focal (como ocurre en la bronquitis la sangre. Otros síntomas son fiebre, tos, dolor subesternal y síndrome
crónica, bronquiectasias o infarto pu lmonar). El sangrado proviene de anémico por el déficit de hierro. A veces hay linfadenopatías y espleno-
los vasos pequeños (arteriolas, capilares y vénulas). Se clasifican según mega lia. Al desarrollarse fibrosis pulmonar, aparecen disnea prog resiva y
la existencia o no de capi larit is. Cursan con capi laritis la enfermedad acropaquias.
de Goodpastu re, las vascul it is (Wegener, PAN microscópica, leucocito-
elásticas, etc.) y enfermedades del tej ido conjuntivo, salvo el lupus; y
sin capilaritis las demás (hemosiderosis pu lmonar idiopática, estenosis
mitral, linfangioleiomiomatosis, LES, penicilamina, etc.).

Existe cierta controversia en cuanto a la terminología. Para algunos


autores, el síndrome de Goodpasture se identifica con el síndrome
pulmón-rena l, que consiste en la asociación de hemorrag ia pulmonar
y nefritis, que puede estar producido por numerosas causas que se
exponen en la Sección de Reumatología . La enfermedad de Goodpas-
ture o enfermedad antimembrana basa l glomerular hace refere ncia a
la asociación de hemorragia alveolar di fusa y g lomeru lonefritis (que se
acompaña de anticuerpos antimembrana basa l en el suero) de causa
desconocida. Algunos textos emplean el térm ino sínd rome de Goo-
dpasture haciendo referencia a esta última . Esta enfermedad se expo-
ne en la Sección de Nefrología.

10.2. Hemosiderosis pulmonar


idiopática

Es una causa de hemorragia alveolar extraordi nariamente ra ra, siendo


un d iagnóstico de exclus ión. Predom ina en niños y adu ltos jóvenes.
En los adu ltos, los varones se ven afectados el doble que las muje- Figura 4 1. Hemorragia alveolar difusa
Neumología 1 10
Durante el episodio agudo, la radiog rafía revela infiltrados alveolares
bi lateral es que desaparecen tras una o dos semanas. Los infiltrados
reticulonodulares se aprecian cuando los ep isodios son recurrentes o
durante la resolución del proceso agudo (Figura 42).

Si hay sangrado activo, se aprecia un aumento de la DLCO. Cuando se


superpone fibrosis pu lmonar, la DLCO disminuye. El tratam iento farma-
cológ ico no está claro, en vista de lo raro de la enfermedad. Los episodios
agudos se tratan con corticoides en dosis altas, pero parece que no alte-
ran el cu rso a largo plazo ni el pronóstico.

Ideas clave Z:
" Se debe pensar en hemorragia alveolar en presencia de la tría -
da diagnóstica: anemia, infiltrados pulmonares y hemoptisis
(esta última puede faltar) .

" Muchas de las causas se acompañan de afectación renal (vas-


culitis, síndrome de Goodpasture) y otras no, destacando la
hemosiderosis pu lmonar idiopática. Figura 42. Radiografía posteroanterior de tórax de un paciente
con hemorragia alveolar difusa
r---------"· eui11Dlogía_

Sarcoido sis

ORIENTACIÓN La sarcidosis es un tema


ENARM significativo.

Es una enfermedad granu lomatosa multisistémica, de etiología descono-


cida, en la que existe una respuesta exagerada de la inmun idad celular.

11.1. Epidemiología

Es relativamente frecuente. Afecta algo más a mujeres, con el pico de inciden-


cia entre los 20-40 años. La prevalencia es 10-40/100.000 habitantes, y parece
que afecta con mayor frecuencia a la raza afroamericana. La mayoría de los
pacientes son adultos jóvenes, y existe un ligero predominio femenino.
Figura 43. Gran u loma sa rcoideo no necrotizante

11 .2. lnmunopatogenia El conjunto de LTh, monocitos y granu lomas representa la enfermedad


activa. Si la enfermedad rem ite, la inflamación mononuclear y el número
de granulomas descienden, quedando una pequeña cicatriz.
En la sa rcoidosis se prod uce una respuesta exagerada de los li nfocitos T
colaboradores (LTh) en respuesta a antígenos desconocidos en sujetos En los casos de enfermedad crónica, la inflamación mononuclear persiste
genética mente suscepti bles. Los LTh está n activados y li beran interleuci- du rante años. La coa lescencia de gran ulomas altera la arq uitectura pu lmo-
na-2, que atrae otros LTh y les hace proliferar. También li beran otras cito- na r, prod uciendo fibrosis, quistes aéreos, bronquiectasias y cambios vascu-
cinas (interferón y, factor de necrosis tumoral [TN F) ...) que recluta n a los lares, constituyendo el estad io fi na l, en el que los granu lomas pueden faltar.
monocitos de la sangre y los activa n, proporcionando el sustrato pa ra la
formac ión del granuloma.

~
El granul oma no necrotiza nte es el dato sugestivo de
sarco idos is, pero se deben descarta r otras enfe rme-
11.3. Anatomía patológica dades gran ul omatosas para ll egar al diagnóstico de ~
certeza. Recuerda
El dato característico de sarcoidosis acti va es el gra nuloma no necrotiza n-
te (Figura 43), aunque no es patognomónica. 11.4. Clínica
Está formado por una zona central ocupada por macrófagos, células epi-
telioides y células gigantes mu lt inucleadas (de t ipo Langerhans). El nú- La sarcoidosis es una enfermedad sistém ica que afecta al pulmón prác-
cleo está rodeado de LTh, células plasmáticas y fi broblastos. El granuloma ticamente siempre. A veces constituye un ha llazgo en la radiografía de
está bien deli mitado, y, aunque es rara, a veces se observa micronecro- tórax de un paciente asintomático (disociación clinicorradiológica).
sis. Las células gigantes pueden tener inclusiones, como son los cuerpos
concoides o de Schaumann, los cuerpos asteroides o los cuerpos residua- En un tercio de los pacientes se presenta de forma aguda (semanas) con
les. El depósito prog resivo de colágeno induce hia li nización. síntomas constituciona les (fiebre, malestar, anorexia y pérdida de peso)
Neumología 1 11
asociados o no a síntomas respiratorios, como tos, d isnea y molestias re- Vías respiratorias superiores. La m ucosa nasal se afecta en el 20%
troesterna les. Se describen dos síndromes en la forma aguda : el síndro- de los casos, con congestión nasa l y pólipos. Aparecen granulomas
me de Lofg ren (fiebre, artra lgias, uveítis, eritema nodoso y adenopatías laríngeos en el 5%, produciendo disfonía, estridor y sibilancias. La afec-
hiliares generalmente bilaterales y simétricas), de buen pronóstico y el tación laríngea se asocia a menudo con lupus pernio. Debido a los gra-
de Heerfordt-Waldenstrom, también denominado fiebre uveoparotídea n u lomas y lesiones cicatriza les, puede encontra rse estenosis traqueal.
(fiebre, uveítis, pa rotid itis y parálisis facia l). Gangl ios linfáticos. Hay adenopatías intratorácicas en el80-90% de
los pacientes. Las más frecuentes son las hiliares, típicamente bilate-
rales y simétricas. También es común la afectación de paratraqueales
La presencia de les iones nod ul ares roj o-vio láceas en cervicales, supraclaviculares, axilares, epitrocleares e inguinales. Las
miembros inferiores (eritema nodoso) o de pará li sis adenopatías son indoloras y móviles (Figura 45).
facia l asoc iadas a adenopatías hili ares bilatera les
obliga a pensar en el diagnóstico de sarco idos is. Recuerda

En el resto de pacientes se presenta de forma insidiosa, a lo largo de me-


ses, con síntomas respiratorios y escasa o nu la clínica constituciona l.

Las principa les manifestaciones clínicas son (Figura 44):


Pulmón. Es la mayor causa de morbimorta lidad. La sarcoidosis pro-
duce una enfermedad intersticial que, en unos pocos pacientes, evo-
luciona a una fib rosis progresi va . Los síntomas típicos son la disnea
al esfuerzo y la tos seca. A veces hay sibi lancias. Puede producirse
afectación endobronq uia l. El 90% de los pacientes presentan ano-
malías en la radiografía de tórax en algún m omento de la evolución .
Un 5% de pacientes experimentan hipertensión pulmonar, bien por
afectación vascular directa o por hipoxemia y fibrosis, la incidencia
aumenta en fases avanzadas.
Figura 45. Adenopatías hiliares simétricas en la sarcoidosis

Piel (35%). La lesión más ca racterística es el lupus pernio y una de las


Uveítis Diabetes insípida
anterior ' más frecuentes el eritema nodoso, sobre todo en las formas agudas.
También pueden apa recer nódulos subcutáneos y erupciones ma-
culopapu losas.
Ojos (25%). Puede afectarse cua lqu ier estructura y producir ceguera .
Lupus pernio
Hipertrofia parotídea
Lo más frec uente es la uveítis anterior y algo menos la coroiditis. A
veces hay agra ndamiento lagrimal y queratoconju ntivitis seca. Pue-
de haber exofta lmos por infiltració n orbitaria.
Adenopatías Hígado. Hay afectación histo lógica, especialmente del territorio pe-
riporta l, en el 60-90% de pacientes, pero su expresión clínica es rara .
Hay hepatomega lia y/o alteraciones enzimáticas en un tercio de los
Afectación intersticial
casos. El síntoma más común de la sarcoidosis hepática es la fiebre
prolongada, y a veces hay dolor en hipocondrio derecho.

enzimáticas

Hipercalciuria Asoc ia sa rco idos is a adenopatías hili ares b il atera les.

testicular
Médula ósea. Hay alteración anatomopatológica en el 20-40% de
casos, pero clín icamente sólo suele producir una leve anem ia, neu-
tropen ia y/o tro m bopenia y, a veces, eosinofi lia.
Bazo. Mediante angiografía, se detecta alteración en el 50-60% de
los casos, pero sólo hay esplenomegalia en el 5-l 0%. Puede asociar
trombopen ia, leucopen ia y anemia.
Sistema nervioso (5%). Lo más frecuente es la parálisis de pares
cranea les, sobre todo del Vil, produciendo parálisis facial un ilatera l,
q ue suele ser brusca y transito ria, au nque puede afectar a cualquier
estructura del sistema nervioso.
Corazón. Suele ser asintomática, pero puede haber arritmias (fre-
Figura 44. Manifestaciones clínicas de la sarcoidosis cuentes), fallo congestivo, bloqueos, angina e incluso muerte súbita.
Manual CTO de Medicina y Cirugía, 2. a edición

Sistema musculoesquelético. Hay artra lgias o artritis en el 25-50% La clas if icac ión radiológica es importante porque
de casos, generalmente de gra ndes articu laciones, usualmente mi- guía el tratam iento, pero debe acompañarse de afee- 1
.,...,...;::::,;
tación clínica o funciona l significativas para que esté
g ratoria. Las lesiones óseas aparecen en el 5% de los pacientes y sue-
indicado inic iar un tratamiento esteroideo.
len ser quistes, lesiones en sacabocados o de aspecto reticulado.
Sistema endocrinometabólico. Lo más afectado es el eje hipotála-
mo-hipofisario, apareciendo diabetes insípida. Puede lesionar la ade- Función pulmonar
nohipófisis, siendo rara la enfermedad en otros órganos endocrinos.
Es típica la hipercalciuria, asociada o no a hipercalcem ia, aunque es En estad io 11 o mayor, aparecen alteraciones en el 40-70% de pacientes.
infrecuente (1-2%). Con datos típicos de un patrón restrictivo: disminución de la CV y CPT.
Aparato reproductor. En el varón, lo más frecuente es el agranda- Hay descenso de la DLCO (menos marcado que en la FPI). A veces existe
m iento testicular asintomático y la epididim itis. En la mujer puede li mitación al flujo aéreo, por afectación de pequeñas vías aéreas, infla-
afectar al útero, pero no suele afectar al cu rso del embarazo (de he- mación endobronqu ial, hi perreactividad bronquial o distorsión de la vía
cho, algunas pacientes mejoran en el emba razo y tras el pa rto re- aérea por fi brosis en fases avanzadas.
caen). La esterilidad es poco habitua l.
Glándulas exocrina s. Es clásico el aumento bilatera l de las paró- La CV es el ind icador más sensible en el cu rso de la enfermedad. La alte-
tidas, pero sólo produce clín ica en el 10% de los pacientes. Puede ración func iona l no se re laciona bien con la actividad de la enfermedad,
haber xerostomía. ni con la afectación rad iológ ica (hay casos con infiltrados francos sin alte-
Riñón. Se ha descrito alteración g lomeru lar, tubular y vascular, pero ración funcional), pero sirve para decid ir el inicio del tratam iento y valorar
es raro encontra r afectación rena l primaria. Si hay fa llo renal, suele ser su respuesta.
por la hiperca lciu ria.
Estudios de laboratorio
Radiografía de tórax
En sang re perifé rica pueden encontrarse linfopenia, a veces eosinofilia ,
La alteración más frecuente es el aumento de los ga nglios intrato rácicos, hipergammag lobulinem ia por aumento de actividad de linfoc itos B, y
que apa rece en el 75-90% de los pacientes. Lo más ca racte rístico es el elevación de la VSG. Ya se ha comentado que la hipercalciuria, con o sin
aumento hiliar bilateral y simétri co, a menudo acompañado de adeno- hipercalcem ia, es rara; es consecuencia de una producción au mentada
patía s paratraquea les, sobre todo derechas. Las adenopatías se pueden de la 1,25 hidroxivitamina O a nivel del granuloma.
ca lcificar, adoptando aspecto de "cáscara de hu evo". La afect ación paren-
q uimatosa suele consistir en infiltrados reticul onodulares bilatera les y Se puede elevar la concentración de la enzima convertid ora de la angio-
simétri cos, de predom inio en campos medios y superiores. Hay una cla- tensina, aunque es poco sensible y específi ca, y no es fiable para va lora r
sificación de la enfermedad pulmonar en cuatro estadios, segú n la radio- actividad, ya que puede no aumentar en la enfermedad activa (por au -
grafía, que está relacionada con la probabilidad de remis ión espontánea mento en su degradación) y puede estar aumentada en pacient es asin-
de la enfermedad (Figura 46). La Rx puede ser norma l. tomáticos.

La elevación de la ECA es característica de sarcoido-


sis, pero no es patognomónica, no tiene valor diag- •"""";;:::..;.,:
nóstico ni tampoco como marcador de actividad.

La gammagrafía pulmonar con galio-67 está alterada, mostrando un pa-


trón de captación del isótopo d ifuso, aunque esto no es específico de la
sa rcoidosis, por lo que no se emplea en la actua lidad. Recientemente se
ha descrito que la PET aporta una información similar a la gammagrafía
con galio, por lo que es preciso recordar que ante una captación positiva
Estadio 1 Estadio 11
Adenopatías hiliares bilaterales Adenopatías hiliares de fluorodesoxig lucosa se debe tener en cuenta esta posibilidad diag-
y patrón reticulonodular
nóstica y no sólo las enfermedad es neoplásicas.

El LBA muestra aumento de linfocitos (sobre todo, LTh, CD4, con lo que
hay un aumento en la relación CD4/CD8) y del número absoluto de ma-
crófagos, au nque el porcentaje esté dism inuido, así como de diversas ci-
t ocinas. Un cociente CD4/CD8 > 3,5 sugiere el diagnóstico, aunque es
menos sensible que el incremento de la cifra de linfocitos. A pesar del
entusiasmo inicial, el LBA no predice el pronósti co de la enfermedad ni la
respuesta al tratamiento.

En la sarcoidosis hay deterioro de las reacciones de hipersensibilidad retarda-


Estadio 111 Estadio IV da cutánea (anergia cutánea). No obstante, históricamente se ha empleado el
Lesión del parénquima sin ade nopatías Fibrosis test de Kveim-Siltzbach, que consiste en la inyección intradérmica de un ex-
Figura 46. Clasificación radiológica de la sarcoidosis tracto de bazo con sarcoidosis para biopsiarlo transcurridas unas 4-6 sema nas.

11 · Sarcoidosis
Neumología 1 11
Es positivo en el 70-80% de pacientes con sarcoidosis. Desde la introduc- nan la evo lución de la enferm edad. Ap roximadamente en un tercio
ción de la biopsia transbronquia l (Figura 47) en el proceder diagnóstico de los casos se produce remisión espontánea en uno o dos años, otro
de la sarcoidosis, no se realiza. tercio muestra progresión clínica y rad iológ ica, y el resto permanece
estab le.

El 90% de pacientes que se presentan como síndrome de Lófgren experi-


menta remisión espontánea. Por ello, y dado que el tratamiento con cor-
ticoides no ha demostrado alterar la evolución a largo plazo de la enfer-
medad y no está exento de efectos colatera les, el problema fundamental
es decidir cuándo se debe iniciar la terapia esteroidea. En general, hay
que rea lizar tratamiento con corticoides en caso de afectación significa-
tiva de órganos críticos.

La sa rcoidosis pulmonar se trata si hay infi ltrados radiológ icos (estadio 11 o


mayo r) acompañados de afectación funcional y/o clínica importantes. Si
hay infiltrados radiológicos, pero la afectación funcional y clínica es leve,
se observa al paciente periódicamente, y si no remite o se evidencia pro-
gresión (clín ica, radio lógica o funcional) durant e la observación, también
se trata.
Célu la gigante Cuerpo
mu lti nucleada asteroide Biopsia transbronquial Además de la sarcoidosis pu lmonar, en las situaciones mencionadas,
Figura 47. Biopsia transbronquial en la sarcoidosis precisa tratamiento la afectación grave cardíaca, ocular (uveítis), neu-
rológica, cutánea difusa y la hipercalcemia o hipercalciuria persisten-
tes (Tabla 17).
11.5. Diagnóstico
Sarcoidosis Estad io radiológico 11 o mayor, junto con síntomas
Se basa en la histología, siendo el pu lmón el órgano biopsiado con mayor torácica o alteración funciona l importantes
frecuencia. Hay que demostrar la existencia de granu lomas no caseifican-
Afectación cardíaca
tes. Como no son patognomónicas, este hallazgo tiene que encontrarse en
Afectación ocular (uveítis)
un contexto clínico, rad iológ ico y fu ncional compatible con la enfermedad.
Afectación del SNC
Afectación cutánea extensa
Es muy rentab le la biopsia transbronquial, especia lm ente si existen le-
Hipercalcemia o hipercalciuria persistente
siones parenquimatosas visibles rad iológ icamente. Si fuera insuficiente,
se toma ría biopsia por mediastinoscopia, en caso de que la afectación Tabla 17. Indicaciones de tratamiento en la sarcoidosis
radiológica se loca lice en el mediastino, o videotoracoscopia, si la afecta-
ción fundamenta l es en el parénquima. Raras veces es necesaria la biop- En caso de fracaso de los corti coides se pueden emplear otros agentes
sia pulmonar a cielo abierto. como metotrexato, hidroxicloroquina (útil sobre todo en afectación cutá-
nea) o azatioprina. Recientemente se ha eva luado el papel de los agentes
anti-TNF.
11.6. Tratamiento
El eta nercept (antagonista de receptor del TNF) no se ha mostrado eficaz,
mientras que el infliximab (a nticuerpo monoclonal anti-TNF) mejora la
A mayor est ad io radiológico, menor frecuenc ia de rem isiones espon- función pu lmonar en pacientes previamente tratados con esteroides e
tá neas. En general, los dos primeros años t ras el d iagnóstico condicio- inmunosupresores.

" Cursa con anerg ia cutánea, hipergammaglobul inemia y eleva-


Ideas clave ¡¿S ción de la enzima conversora de angiotensina (ECA).

" El granuloma no necrotizante es el hallazgo histológico caracte- " El LBA muestra linfocitosis con aumento de linfocitos CD4 (co-
rístico, no patognomónica de la enfermedad. ciente CD4/ CD8 aumentado).

" El pulmón es el órgano afectado con mayor frecuencia. " La prueba diagnóstica de elección es la biopsia t ransbronquial.

" Es más hab itual en mujeres y en sujetos de raza afroamericana. " El tratamiento de elección son los corticoides.

" Las adenopatías hiliares bilaterales y simétricas asociadas o no " Se d ebe tratar la afectación torácica en estad io 11 o superior que
a patrón intersticial de predominio en vértice s sugiere el diag- se acompañe de clínica o afectación func ional significativas, así
nóstico. como la afectación extratorácica relevante.
Manual CTO de Medicina y Cirugía, 2.a edición

1) Tuberculosis pulmonar, por lo que hay que iniciar tratamiento


Casos clínicos ~ ,_ tuberculostático inmediato, a la espera del resultado del cultivo
de Lowenstein .
Mujer de 21 años, no fumadora, que consulta por tos seca, febrí- 2) Alveoliti s alérgica extrínseca, lo que obliga a obtener una mues-
cula, astenia, artralgias y lesiones cutáneas sugerentes de erite- tra de tejido pulmonar para descartarla.
ma nodoso. La radiografía de tórax muestra adenopatías hiliares 3) Sarcoidosis en estadio 1, no siendo necesario iniciar tratamiento
bilaterales y en región paratraqueal derecha. La fibrobroncos- de su enfermedad respiratoria.
copia evidencia inflamación difusa de la mucosa bronquial, y el 4) Sarcoidosis en estadio 1, por lo que es imperativo iniciar trata-
lavado broncoalveolar una linfocitosis del 32% con cociente lin- miento inmediato con glucocorticoides para evitar la progre-
focitos T4 (colaboradores)/T8 (supresores) superior a 3,5. ¿Cuál sión de la enfermedad,
es, entre los siguientes, el diagnóstico más probable y la actitud
a seguir? RC: 3

eaS e St udy ¡~~,' 2) Heerfordt syndrome includes posterior uveitis among its fea-
tures.
3) Sarcoidosis is more common in women, and incidence is highest
Regarding sarcoidosis, which of the following statements is at 20-40 years old.
true? 4) lt is typical of smokers.

1) Lofgren syndrome typically presents unilateral hilar lymphade- Correct answer: 3


nopathy anda nodular interstitial pattern .

11 · Sarcoidosis
_N_e_umo lo_gía •

Hipertensión pulmonar

con siste en fibrosis de la íntima hipertrofia de la media, trombosis intralu-


ORIENTACIÓN Tema poco importante.
Se deben conocer las causas minal y lesiones plexiformes.
ENARM y el tratamiento.

12.2. Hipertensión pulmonar


12.1. Generalidades idiopática

Es una enferm edad caracterizada por la elevación mantenida de la presión


La hipertensión arterial pulmonar (HAP) se define como la elevación de la arteria l pulmonar sin una causa demostrable, se trata por tanto de una en-
presión de la arteria pulmonar media por encima de 25 mmHg en reposo. fermedad cuyo d iagnóstico se establece por exclusión (véase la Tabla 18).

La clasificación actual de la hiperten-


sión pulmonar se puede apreciar en la 1.1 ldiopática
Tabla 18. Los grupos 1, 3, 4 y 5 cursan 1.2 Hereditaria
con hipertensión pulmonar precapila r, 1.3 Fármacos y toxinas
el grupo 2 con hipertensión pu lmonar 1.4 Hi pertensión pu lmonar arterial asociada
poscapilar. Se diferencian porque en la 1.4.1 Enfermedades del tejido conjuntivo
1.4.2 lnfecciónporVIH
fo rma precapilar la presión de encla-
1.4.3 Hipertensión portal
vamiento capilar pulmonar es norma l, 1.4.4 Enfermedad ca rdíaca co ngéntita
mientras que en la poscapilar está ele- 1.4.5 Esquistosomiasis
vada. 1.4.6 Anem ia hemolítica crónica

El cor pulmonale se define como un 1.5 Hi pertensión pu lmonar persistente del recién nacido
aumento de tamaño del ventrículo de- 2.1 Disfunción sistól ica
2. Hipertensión pulmonar
recho secundario a enfermedades pu l- secundaria a cardiopatía 2.2 Disfunción diastólica
monares, del tórax o de la circulación izquierda 2.3 Enfermedad va lvular
pulmonar, que puede ir acompañado
3. Hipertensión 3.1 EPOC
de insuficiencia ventricu lar derecha. La
pulmonar secundaria 3.2 Enfermedad intersticial
gravedad del cor pulmona/e viene dada
a enfermedades 3.3 Hipoventilación alveolar crónica
por el grado de aumento de poscarga
pulmonares e hipoxemia 3.4 Trastornos respiratorios durante el sueño (SAHS)
ventricular derecha, es decir, de la pre-
sión arteri al pulmonar. Más de la mitad 4. Hipertensión pulmonar
de los pacientes con EPOC padecen cor tromboembólica crónica

pulmona/e. 5.1 Trastornos hematológicos: esplenectomía, trastornos


mieloproliferativos
Las alteraciones histológicas que apa- 5.2 Enfermedades sistémicas: sarcoidosis, histiocitosis de células
de Langerhans, linfangioleiomiomatosis, neurofibromatosis, vasculitis
recen en los vasos pulmonares en la
5.3 Alteraciones metabólicas: enfermedad del almacenamiento
HAP guardan relación co n el proceso
del glucógeno, enfermedad de Gaucher, trastornos tiroideos
causante, si bien hay un proceso de
5.4 Otros: obstru cción tumora l, med iastinitis fibrosante
remodelado vascular común a todas
las etiologías y a la HAP primaria, que Tabla 18. Clasificac ión de la hi pertensió n pu lmonar
Manua l CTO de Med ic ina y Ci rugía , 2.8 edición

Es una enfermedad co n una fuerte influencia genética. El 20% de los ca- gasto ca rdíaco, incluso puede ser la p ri mera manifestación. A veces hay
sos sufre n una forma hereditaria de la enfermedad. El gen causante es el hemoptisis. Cuando la enfermedad es leve, puede no haber anomalías
que cod ifica el receptor de la proteína m orfogén ica del hueso t ipo 11 (bone exploratorias, pero al prog resar, son evidentes los signos de hipertensión
morphogenetic protein receptor, BMPR-11), m iembro de la superfam ilia del pu lmonar y d ismin ució n de l gasto ca rd íaco con fracaso ventricular dere-
factor transformante del crec im iento (transforming growth factor, TGF). Los cho (edemas, elevación de presión venosa cent ral).
criterios d iagnósticos utilizados incluyen una elevación de la presión med ia
de la arteria pulmona r en reposo, excluyendo otras causas de hipertensión La rad iografía de tórax reve la protrusión de la arteria pu lmonar principal
pulmona r. La hipertensión portal, infección por el VIH, inha lación de cocaí- y aumento en la anchura de la rama descendente de la arteria pulmonar
na, los fármacos anorexígenos y el aceite de co lza desnatura lizado pueden derecha, oligohem ia periférica y card iomegal ia. En la enfermedad venoo-
producir una enfermedad similar a la hi pertensión pu lmonar primaria. clusiva además se aprecian líneas B de Kerley, en ausencia de otros datos
de insuficiencia del ventrícu lo izquierdo.
Epidemiología
Funciona lmente suele haber un patrón restrictivo leve. La difusión suele
La incidencia es de tres casos por mi llón, predominando en mujeres jó- d ism inuir en un grado leve o moderado. La Pa0 2 suele estar disminuida y
venes. La forma venooc lusiva predom ina en la infancia, y cuando aparece la PaC0 2 puede ser baja. Existe corre lación entre la d istancia recorrida con
en adultos, es algo más frec uente en varones. el test de la marcha de seis m inutos y la gravedad, por lo que es útil para
mon itorizar la respuesta al t ratamiento.
Anatomía patológica
El electrocardiog rama en fases avanzadas muestra hipertrofia de ven-
Hay lesiones en las arterias musculares pequeñas y arteriolas pulmonares. La trícu lo y aurícula derechos. La ecoca rdiografía es útil para estimar la hi-
alteración patológica más temprana es la hipertrofia de la media, lo que indica pertensión pu lmonar y para descartar causas secundarias, así como para
que debe haber un estímulo que produzca vasoconstricción y proliferación del evidenciar la sobreca rga ventricu lar derecha con abombamiento del
músculo liso. La lesión histológica clásicamente asociada con la enfermedad tabique hacia el ventrículo izquierdo, lo que origina su disfunción d ias-
es la arteriopatía pulmonar plexogénica, en la que hay hipertrofia de la media, tólica. La gammagrafía de perfusión es normal o tiene alteraciones de
fibrosis laminar concéntrica de la íntima y lesiones plexiformes, y que puede baj a probabilidad de TEP, lo que la d iferenc ia del TEP crón ico. Algunos
asociar arteriopatía trombótica. La forma plexogénica no es patognomónica, autores sugieren que un patrón de perfusión irregular de modo difuso,
pues se ha encontrado en la hipertensión pulmonar de causa conocida (ci- no segmentaría, puede ind icar enfermedad pu lmonar venooclusiva. La
rrosis hepática, enfermedades colagenovasculares y cardiopatías congénitas). arteriografía pu lmonar se emplea, si la gammagrafía de perfusión no es
concluyente, para desca rtar un TEP crónico.
La enfermedad venooclusiva pu lmonar se describió como otra forma de
hipertensión pulmonar primaria (ell 0% de casos). Se caracteriza por una Es necesario el estud io hemodinám ico con cateterismo cardíaco para
obstrucción de venas y vénulas pulmonares por fibrosis de la íntima y descarta r causas secundarias y para demostrar aumento en la presión de
septos fibrosos intravasculares que sug ieren trombos recanalizados. A ve- la arteria pu lmonar y de la aurícula derecha, así como disminución del
ces, las lesiones tamb ién afectan al lecho arteriolar. gasto cardíaco, que únicamente ocurre en fases avanzadas de la enfer-
medad. Se debe rea lizar el test de vasorreactividad con una sustancia va-
La hemangiomatosis cap ilar pu lmonar es otra forma de hipertensión pul- sodilatadora, que tiene va lor pronóstico y guía la terapéutica (es positivo
monar primaria, que cons iste en la proliferación de vasos dilatados de en el 20% de los pacientes). Los vasodi latadores comúnmente emplea-
paredes muy finas en el intersticio alveo lar con tendencia a la rotura y dos son epoprostenol, adenosina u óxido nítrino inha lado. Los criterios
hemoptisis. Es excepcional. para considerar la prueba positiva son una disminución de la presión ar-
terial pulmonar med ia de, al menos, 10 mmHg con una presión final por
Fisiopatología debajo de 40 mmHg sin d isminución del gasto cardíaco.

El aumento de la resistencia vascular pu lmonar se produce por tres ele - La presión de enclavamiento capi lar pulmonar es normal incluso en la
mentos: la vasoconstricc ión, el remodelado de la pared vascu lar y la enfermedad venooclusiva, dada la naturaleza irregular del proceso (sal-
trombosis in situ. vo que se mida en va rios sit ios), hasta estad ios avanzados, en los que
aumenta por d isfunción diastólica del ve ntrículo izqu ierdo. Si está ele-
La presión de la arteria pulmonar aumenta con un gasto cardíaco que vada inicia lmente, hay que hacer cateterismo izquierdo para desca rtar
inicialmente se mantiene, pero con el tiempo disminuye. Conforme la enfermedades de corazón izquierdo (grupo 11 de la clasificación actual
enfermedad progresa, las resistenc ias pu lmonares se hacen fijas y las ar- que cursan con hipertensión poscapilar).
terias dejan de responder a los vasod ilatadores.
No se requiere biopsia pu lmonar pa ra el d iagnóstico.
Clínica
Tratamiento
Los síntomas iniciales son muy sutiles, por lo que el d iagnóstico se demo-
ra una media de dos años. La d isnea progresiva es el síntoma más común. Es una enfermedad progresiva. La supervivencia media hasta hace unos
Su gravedad no se corre laciona con la elevación de la presión arteria l pul- años era de 2,5 años, pero con los nuevos medios de tratam iento parece
mona r. La fatiga y la debilidad son frecuentes por deterioro del gasto car- mayor (si responde a bloqueantes de los cana les de calcio, tienen una su-
díaco. El dolor torác ico subesterna l es común, y parece relacionado con pervive ncia a los cinco años del 95%, pero esto ocurre en una minoría de
insuficiencia coronaria ante el aumento de las necesidades del ventrículo los pacientes). La muerte suele ser por progresivo fallo cardíaco derecho
derecho y con la hipoxemia. Puede haber síncope por disminución del y, a veces, por muerte súbita (Figura 48).

12 · Hipertensión pulmonar
Neumología 1 12
Test de vasorreactividad Anticoagulantes

El tratamiento con estos fármacos aumenta la supervivencia, probable-


Positivo Negativo mente por prevenir la trombosis in si tu en estos pacientes, favorec ida por
el enlentecimiento circulatorio al pasar la sangre por vasos de calibre re-

/ ducido. De acuerdo con esto, en la actualidad se recomienda el empleo


de anticoagulantes orales en todos los pacientes con hipertensión pul-
monar.

· Bosentán
· Sildenafilo
· Bosentán
· Sildenafilo
· Teprostinil
l
1 Epoprostenol i.v.
Trasplante pulmonar

Se indica en los pacientes que, aú n con tratamiento médico intensivo


·lloprost (vasod ilatadores, prostaciclina ...), sig uen con insuficiencia cardíaca dere-
· Epoprostenol i.v.
cha. No se ha descrito la recidiva .
Figura 48. Test de vasorreactividad
Otras medidas
Calcioantagonistas
Los pacientes no deben realizar actividades que requieran gran esfuerzo
Se emplean a largo plazo si, en el estudio hemodinámico inicia l, se obtie- físico, por aumentar este la presión arterial pulmonar. El embarazo se tole-
ne respuesta positiva a potentes vasodi latadores de acción corta (adeno- ra mal y los anticonceptivos orales pueden exacerbar la enfermedad. Los
sina o prostaciclina intravenosas u óxido nítrico inha lado). Los fármacos diuréticos son útiles si hay ascitis y edemas. La digoxina no ha demostra-
más empleados a largo plazo son nifedipino y diltiazem. Un 50% de los do ningún efecto beneficioso. El oxígeno puede mejorar la clínica en los
pacientes con prueba positiva en el test de vaso rreactividad mejoran o se pacientes con hipoxemia.
estabilizan a largo plazo.
No existe una terapia específica para pacientes con enfermedad venoo-
Análogos de prostaciclina clusiva o con hemangiomatosis capilar pu lmonar.

El epoprostenol se emplea en infusión endovenosa continua en los pa-


cientes con test de vasorreactividad negativo en clase funcionaii ii-IV, así 12.3. Hipertensión pulmonar
como en los que fue positivo, pero la mejoría no se mantiene a largo pla-
zo y están en clase funcional III-IV. Los efectos adversos incluyen diarrea,
tromboembólica crónica
sofocos y dolor facial. También se usa como puente al trasplante. Existen
otros análogos de la prostacici lina, como el teprostinil (vía subcutánea), Se produce en aquel los pacientes en los que tras un TEP agudo la fibri-
iloprost (vía inhalada) y el beraprost (vía oral). nólisis fracasa en restaurar por completo el flujo sanguíneo. No siempre
se reconoce el antecedente de TEP, y muchos tienen alguna trombofilia
Antagonistas de receptores de endotelina (anticoagu lante lúpico o factor V de Leiden).

La endotelina 1 es un potente vasoconstrictor endógeno que estimula la La clínica recuerda a la HAP idiopática.
proliferación de células musculares lisas de la pared vasc ular, implicado
en la patogenia de la hipertensión pulmonar. El bosentán es un antago- El diagnóstico se sospecha al encontrar en una gammagrafía pu lmonar
nista de receptores de endotelina, indicado en pac ientes con hiperten- de perfusión defectos de perfusión segmentarías sugestivos de TEP o
sión pulmonar en clase funcional li-111 con prueba negativa vasodi latado- defectos de rep leción en angiografía-TC, pero el diagnóstico de certeza
ra o que se deterioran bajo tratamiento crónico con calcioantagonistas. requiere la real ización de una arteriografía pulmonar, que además deter-
mina la localización exacta de los trombos. El tratamiento de elección
Sildenafilo es la tromboendarterectomía en pacientes con trombos accesibles a la
cirugía (vasos centra les). Es una técnica con una mortalidad de alrededor
lnhibidor selectivo de la fosfodiesterasa S, enzima responsable de la degra- dell2% incluso en centros experimentados. Los pacientes no candidatos
dación del GMPc, a través del cual el óxido nítrico ejerce su acción vasod i- a cirugía deben ser anticoagulados de por vida. La fibrinólisis no suele ser
latadora. Su uso ha sido aprobado para el tratamiento de la HAP idiopática eficaz, lo que unido a las complicaciones que pueden ocurrir con estos
en pacientes con test de vasorreactividad negativo en clase funciona ll l-111. fármacos hacen que no esté indicada.

" Los pacientes que no responden en clase funcionaiiii/IV deben


Ideas clave .i6 rec ibir análogos de prostaciclina intravenosa.

" Todos los pacientes se benefician de anticoagulación. " Para los pacientes en clase funcionaiiii/IV el tratamiento defini-
tivo es el trasplante pulmonar.
" Los que tienen un test de vasorreactividad positivo (un 25%)
cuentan con mejor pronóstico, y se benefician de tratamiento
con vasodilatadores (calcioantagonistas) a largo plazo.
Manual CTO de Medicina y Cirugía, 2.a edición

3) HIV infection, inhalation of cocaine intake and appetite


Case Study .- suppressant drugs and portal hypertension can cause pulmo-
nary vascular disease with clinical and pathologic features sim-
Which of the following statements is FALSE about primary ilar to the PPH.
pulmonary hypertension (PPH): 4) A call veno-occlusive anatomic shape predominates, however,
from 60.
1) Diagnosis requires a mean pressure of the pulmonary artery
greater than 25 mmHg at rest. Correct answer: 4
2) To diagnose primary pulmonary hypertension, heart or lung
disease that can produce a secondary form and connective tis-
sue disease should be discarded.

12 · Hipertensión pulmonar
__Ne_umología_ _ _ _~

Tromboembolismo pulmonar

Tema importante que obliga leves de folato, y a veces, de vitaminas 86 y 812) y algu nas enfermedades
ORIENTACIÓN
a conocer bien los aspectos crónicas como la policitemia prim aria, HTA o EPOC.
ENARM diagnósticos y terapéuticos.
La resistenc ia a la prot eína C activada (fact or V de Leiden) es el estado de
hipercoagu labilidad hereditario más frecuente en el TEP, seg uido de las
m utaciones del gen de la protrom bin a. Sin em bargo, los factores gené-
13.1. Concepto t icos sólo justifican una q uinta parte de los casos de TEP, y la mayoría de
pacientes con fact ores genéticos nu nca desarrol la rán ETV.

La enfermedad tromboemból ica venosa (ETV) representa un espectro de La trombofili a es un té rmin o que se utiliza para los casos recu rrentes, tan-
enfermedades que eng loba n ta nto la trombosis venosa profunda (TVP) to hereditarios como adqu iridos. Ante TEP sin fa ctores de ri esgo, o idiopá-
como el tromboembolismo pu lmonar (TEP), q ue comparten los mismos ticos, también hay que pensar en cá ncer oculto.
factores pred isponentes. En un 90-95% de los casos, el émbolo que origi-
na el TEP proviene de una TVP de miembros inferiores, a menudo asinto-
mática . Cuando una TVP proxima l no es tratada, ocu rre TEP clín icamente
en un tercio de los pacientes y otro tercio presentan embolismo subclí- La resistencia a la proteín a C activada (factor V de _.,"""';;::...~,
Leiden) es la trombofilia hered itaria más frecuente.
nico. En el 70% de los pacientes con TEP se diagnostica una TVP si se em-
plean métodos sensibles. Otros orígenes de émbolos más infrecuentes
son las venas pélvicas, las ext rem idades superiores y las cavidades cardía-
cas derechas.
13.3. Fisiopatología
La ETV afecta a 4/ 1.000 pacientes ingresados anualmente en los Estados
Un idos, con una morta lidad aguda de 7- 11 o/o, aunque sólo un 60% de los
pacientes se diagnostican en vida. La rec id iva es frecuente, tres veces más Durante el episodio agudo se pueden apreciar las siguie ntes alteraciones
si el evento inicia l fue TEP comparado con TVP fisiopatológicas:
Alterac ión del interca mbio gaseoso: por aumento del espacio muerto
fisiológ ico (aparece una zona que está siendo ventil ada, pero no perfun-
13.2. Factores de riesgo dida), por desequ ili brio V/Q en el pulmón no obstruido (más perfusión
que ventilación) y por shunt derecha a izquierda, que puede ocurrir a ni-
vel intrapulmonar o intracardíaco, en caso de foramen oval permeable.
Aunque la ETV puede apa recer en pacientes sin factores de ri esgo, norma l- Hiperventilación alveolar por estímulo reflejo nervioso.
mente es posible identifica r más de uno. En un registro internacional re- Aumento de la resistenc ia al fluj o aéreo por broncoconstricción de
ciente sólo el 20o/o de los casos se consideró TEP id iopáti co o no provocado. las vías aéreas d ist ales al bronqu io del vaso obstruido.
Dismin ución de la distensibilidad pu lmo na r por edema, hemorragia
Los factores de riesgo más frecuentes en el embol ismo pulmonar son: o pérdida de surfactante.
historia de ETV previa, la inmovilización, anteceden tes de cirugía en los Aument o de la res istencia vascu lar pu lmonar por obstrucción vascu-
últimos tres meses, máxima en las dos primeras sema nas (cirugía abdomi- lar o liberación de age ntes neurohumora les co m o la serotonina por
na l, pélvica y fundamentalmente ortopéd ica mayor), accidente cerebro- las plaquetas.
vascu lar reciente y neoplasias. Otros factores de riesgo serían la obesidad, Disfunción ventricular derec ha. El fracaso ventricular derecho es la
el tabaquismo, los ant iconceptivos o rales o tratamiento hormona l sus- ca usa de muerte más habitual tras un TEP A m ed ida que aumenta
titutivo, el embarazo, los viajes en avión de la rg o recorrido, el sínd rome la resistenc ia vascula r pulmonar au m enta la tens ió n en el vent rícu-
antifosfolípido, la hiperhomocisteinemia (generalmente por deficiencias lo derecho, que lleva a dilatación y mayor disfunción vent ri cular. La
Manual CTO de Medicina y Cirugía, 2. a edición

dilatación del ventrículo derecho, por med io del abombamiento del ca de causas no aclaradas. La disnea, en primer lugar, seguida del do-
septo interventricula r, compromete el llenado ventricular izquierdo lor pleurítico, son los síntomas más frecuentes. Los síntomas menos
con aparición de síntomas de bajo gasto. Además, la tensión sobre habituales son la hemopti sis, la opresión torácica o el broncospasmo.
el ventrículo derecho puede dificu ltar el flujo en la arteri a corona ria La taquicardia y la taquipnea son los signos más constantes. Genera l-
derecha y provoca r isquemia o infarto de este ventrícu lo. mente, la presencia de d isnea grave, síncope, hipotensión mantenida
o cianos is ind ican un TEP masivo, en tanto que el dolor pleurítico, la
tos, o la hemoptisis sugieren un pequeño embolismo periférico que se
acompaña de infarto pu lmonar.
El aumento del espac io muerto fisiológico es el even-
to in icial, que provoca finalmente desequi li brio V/Q.
Según la gravedad de presentación se distinguen tres tipos de TEP con
d iferente riesgo de morta lidad:
TEP masivo (a lto riesgo, mortalidad > 15%): se presenta con hipoten-
sión o shock. Suelen serTEP de gran tamaño o d ifusos.
13.4. Diagnóstico TEP moderado/grande (riesgo intermedio, mortalidad 3-15%): sin
hipotensió n ni shock pero con sig nos ecoca rdi ográ fi cos de sobre-
ca rg a de l ventrícul o derecho o elevación de marcadores de daño
El diagnóstico es difícil debido a lo inespecífi co de la clínica. El síntoma miocárdico.
más frecue nte de TVP es el dolor de pantorril la, y de TEP la d isnea . Lo TEP moderado/peq ueño (riesgo bajo, morta lidad < 1o/o): con tensión
primero que se debe hacer es determin ar la probabilidad clínica, pues arterial y func ión ventricu lar derecha normales.
una probabilidad clínica baja de TVP o moderada/baja de TEP perm iten
descartar la enfermedad si el dímero D es negativo sin rea lizar pruebas La estimación de la probabilidad clínica es el primer eslabón en la apro-
de imagen. En la Tabla 19 se muestra la esca la más utilizada para eva luar ximación d iagnóstica del TEP (véase la Tabla 19). El árbol de decisión
la probabilidad clínica. Dada la gravedad del proceso, un alto g rado de d iagnóstico y la interpretación de diferentes test dependen de ella.
sospecha clín ica basta para indicar tratam iento anticoag ulante.
Radiología simple de tórax

Síntomas o signos de trombosis profunda 3 Una placa normal o con escasas alteraciones aumenta la sospecha de TEP,
Diagnóstico alternativo menos probable 3 aunque lo habitual es que existan anoma lías. De estas, las más frecuentes
Frecuencia cardíaca > 100 latidos por minuto 1,5 son la elevación del hem id iafragma, atelectasias, anomalías parenquima-
Cirugía o inmovilización en las 4 semanas previas 1,5 tosas pulmonares y derrame pleural escaso y serohemático. Otros signos
Episodio de tromboembolismo pulmonar o trombosis 1,5 menos frecuentes son atelectasias laminares, el signo de Westermark (oli-
profunda previa gohemia focal que produce una hipertransparencia pulmonar) y la joro-
Hemoptisis ba de Hampton (condensació n parenquimatosa triangular de base pleu-
Cáncer ra l). Este signo suele asociarse aiTEP con infarto y presenta con frecuencia
derrame pleu ral de pequeña cuantía (Figura 49).
0- 1 puntos: probabilidad baja
2-6 pu ntos: probabilidad intermedia
~ 7 puntos: probabil idad alta Signo Amputación
de Westermark vascular
Tabla 19. Sistema Wells de estimación de probabilidad clínica

Clínica
La TVP produce dolor en la pantorrill a de inicio insidioso que tiende a
empeora r con el tiempo y se acompaña a la exploración de palpación
dolorosa . La TVP masiva es más fácil de reconocer. El paciente presenta
edema de muslo con palpación dolorosa en la zona inguina l y sobre la
vena femo ral común. Si todo el miembro está edematoso, el diagnóstico
de TVP es improbable y se debe sospechar agudización de síndrome pos-
flebítico. La TVP de miembros superiores puede producir empastamiento
de la fosa supraclavicular, aumento de diámetro del miembro o circu la-
ción colateral en la región anterior del tórax.

La disnea, genera lmente súbita, es el sínto ma de pre- _ _;::,....;.,.


sentac ió n más frecuente en el TEP.

El embolismo pu lmonar debe ser cons iderado ante la apa ri ción de de Hampton

una d isnea generalmente súbita, un síncope, o una hipotensión brus- Figura 49. Posibles alteraciones radio lógicas del TEP

13 · Tromboembolismo pulmonar
Neumología 1 13
Electrocardiograma con gammagrafía de venti lación y radiografía de tó rax normales. El 90%
de estas gammagrafías de alta probabilidad presentan TEP, lo que ofrece
Las alteraciones más comu nes son la taquicard ia sinusal y las anomalías suficientes garantías para establecer el d iagnóstico, especia lmente si se
inespecíficas del ST-T de Vl a V4. A veces existen signos de sobrecarga de- combina con una probabi lidad clínica alta. Por desgracia, la mayoría de
recha, como el patrón "SI,QII I,TII I" (S en la derivación 1, Q y T invertida en la 111), los émbolos producen defectos que son interp retados como de proba-
cor pulmona/e, desviación del eje a la derecha, o bloqueo de rama derecha. bi lidad intermed ia o baja, por lo que no son gammagrafías diagnósticas.

Analítica

La elevación de marcadores de daño miocárdico (troponinas) aumenta el


riesgo de complicaciones y de mortalidad en pacientes con TEP heme-
dinámicamente estables. En la gasometría existe hipoxe mia y aumento
del gradiente alveoloa rterial de oxígeno, aunque hay pacientes sin ante-
cedentes de patología pulmonar con P0 2 y O(A-a)0 2 normales. La PaC0 2
suele estar baja (hipoventilación refleja), pero puede ser norma l e incluso
elevarse en el TEP masivo.

El grad iente alveoloarter ial de 0 2 está elevado en re-


lac ión con el desequ ili brio V/Q. No obstante, la nor- _..,...,.;_.
ma lidad de l grad iente no descarta el d iagnóstico de
TEP.
Figura 50. Tromboembolismo pulmonar. Émbolo en la bifurcación
de la arteria principal derecha (flecha)
Dímero-O

Su elevación reve la la presencia de fibrinó lisis endógena, pero no es es-


Una gam magrafía de perfu sión no rm al desca rta TEP;
pecífico de TEP, pues puede verse aumentado en el infarto de miocardio, una ga mmagrafía de ventilac ió n/ perfusió n de alta
neumonía, fallo derecho, carcinomas, cirugía, inmovi lizaciones, etc. La de- probabili dad lo co nfirma .
terminac ión mediante ELISA del dímero-O tiene un alto va lor pred ictivo
negativo, especialmente al combinar esta prueba con la probabilidad clí-
nica, ya que ante pacientes con baja/intermedia probabil idad, se puede Angiorresonancia magnética
descartar TEP cuando está por debajo de 500 ng/m l. No suelen ser muy
útiles en pacientes con cánce r o cirugía rec iente, ya que la mayoría t ienen Utiliza gadoli nino como contraste, que no es nefrotóxico. Aún no muy ex-
cifras por encima de 500 ng/ml. tend ida, aporta unos resu ltados pa recidos a la TC helicol idal para detectar
TEP, y además permite eva luar la func ión ventricula r.

Angiografía pulmonar
En pac ientes co n probabilidad clíni ca med ia/baj a, un
dímero-O negativo (< 500 mg/ml ) exc luye la ETV. Es el patrón de referencia (gold standard) y puede precisa rse para el diag-
Recuerda nóstico cuando hay una alta sospecha clín ica, la gammagrafía o la TC he-
licoidal no son diagnósticos, y la ecografía venosa y la ecocardiografía son
TC espiral con contraste normales, o bien si la sospecha clínica es baja, pero las otras pruebas indican
la posibilidad de embolismo. También se debe rea lizar en pacientes que se
Es actua lmente la t écn ica de elección ante la sospecha de TEP Ti ene una vayan a someter a algún tipo de intervención, como una embolectomía o
sensibi lidad mayor de l 80% y una especificidad mayor del 90%. Alcanza una trombólisis d irigida por catéter. Es conveniente observa r la interrup-
a visual izar arterias de sexto o rden. No se puede rea liza r si existe insu- ción brusca de un vaso, defectos de llenado en dos proyecciones o un vaso
ficienc ia renal o alergia al contraste. Ante TC norma l y sospecha clínica en "cola de rata" (por la organización del coágulo y retracc ión del mismo).
alta, se debe realizar angiog rafía pulmonar, que también visual iza el árbol
vascular distal. Otras ventajas que aporta son la va loración del tamaño
del ventrículo derecho (valor pronóstico) y la posibilidad de obtener un
diagnóstico alternativo (Figura 50). La arteriografía pul mona r es la pru eba m ás sensib le y
específica para el d iagnóstico de TE P.

Gammagrafía de perfusión pulmonar


Recuerda
Es una prueba de seg unda línea, indicada en casos en los que no es po- Ecocardiografía
sible realizar TC. Una gam magrafía de perfusión normal excluye el diag-
nóstico de TEP. Si fuera anormal, habría que combinar el resultado con Tiene una baja sensibilidad para detectar TEP, pero puede ser muy útil
el de la gammagrafía de venti lación, y así determ inar la probabilidad en casos de pacientes con sospecha de TEP y clínicamente graves, pues
gammagráfica de TEP Una gammagrafía de alta probabilidad diagnós- va lora la función ventricular derecha e incluso permite visualizar trombos
tica es la que presenta dos o más defectos de perfusión segmentarios importantes (especialmente, mediante ecocardiografía transesofágica);
Manual CTO de Medicina y Cirugía, 2. a edición

además, ayuda en el diagnóstico diferencial de otros procesos (I AM, ta- No existe un consenso estri cto sobre el procedim iento óptimo. En gene-
ponamiento ca rdía co, disección aórtica, disfunción va lvular ag uda). La ral, se suele combinar la probabilidad clín ica con pruebas no invasivas, so-
detección de una disfunción ventricu lar derecha ayuda a estratificar el bre todo, dímero-O, eco-Doppler o TC espiral y, según su resultado, estará
riesgo y plantear el manejo más adecuado. La hipocinesia de las paredes o no justificado conclu ir con una ang iografía pu lmonar. En los pacientes
libres con movilidad normal del ápex ventricu lar derecho (signo de Mc- hemodinám icamente inestables, la TC espiral o la angiografía pulmonar
Connell) es muy sugestiva de TEP Sin emba rgo, no está considerada una deben realizarse de entrada .
técnica estándar en el diagnóstico del TEP por su baja sensibi lidad, ya que
la mayoría de los pacientes con TEP t ienen una ecoca rdiog rafía normal.
En pacientes co n sospecha de TEP sin d iagnóstico de
Ecografía venosa certeza tras ga mmagrafía y TC, antes de la arteriogra- _.."'"'"~­
fía, MIRAR LAS EXTREM IDADES INFERIORES (ori gen
del 90% de los trombos): hacer eco-Dopp ler.
Es la técnica no invasiva más utilizada en la actu al idad para va lorar TVP y
ha llevado al desuso a la pletismog rafía y la fl ebografía isotópica. Resulta
muy fiable en pacientes sintomáticos ambulatorios con sospecha de TVP;
en asintomáticos y hospitalizados, sin embargo, la tasa de detección de 13.5. Tratamiento
TVP es mucho más baja. Po r ello, una ecoca rdi og rafía Doppler norma l no
debe concluir el estud io de TEP, especia lmente si el paciente tiene una
alta sospecha clín ica. El signo más fi able de TVP es la ausencia de com- El tratamiento prima rio consiste en la d isolución del coágu lo, ya sea me-
presibi lidad de la vena afectada. diante trombólisis o mediante em bolectom ía. La anticoagu lación o el fil-
tro de vena cava constituyen más bien una prevención secundaria de un
Pletismografía de impedancia nuevo episod io.

Detecta el 95% de las TVP de la vena pop lítea o superio res; sin emba rgo, El tratamiento anticoag ulant e const ituye el elemento más im portante de
es muy poco útil para la detección en las venas de la pantorri lla. la terapéutica de la enfermedad tromboembólica venosa (es simi lar en el
caso de la TVP proximal y el TEP; sin emba rgo, el de la TVP de las pantorri-
Fiebografía isotópica con fibrinógeno marcado 1\as es controverti do).

Detecta trombos recientes en venas de las pantorrillas y poplíteas, no Estratificación del riesgo. El tratamiento primario se reserva para pa-
siendo útil para territo ri os superiores. cientes de alto riesgo segú n la clasificación previa (véase el apartado Clí-
nica). Aq uellos pacientes hemod inámicament e est ables, con disfunción
Flebografía con contraste ventricular derecha o con elevación de las troponinas tienen ri esgo inter-
medio y su manejo debe ser individualizado. En pacientes hemod inámi-
Es la técnica más eficaz para detectar TVP, pero al ser invasiva, es incómo- camente estables sin d isfunción card íaca (riesgo bajo), la anticoagulación
da para el paciente, y no está exenta de complicaciones. Su indicación es aislada aporta buenos resu ltados.
obligada cuando debe efectuarse una interrupción de la vena cava.
Heparinas
El manejo genera l de estas pruebas se resume en el algoritmo de la
Figura 51. La heparina no fraccionada (HNF) acelera la acción de la antitrombina
111 e inactiva el factor Xa, por lo que previene la formación de un trombo
ad iciona l y perm ite que la fibrinólisis endógena disuelva algo del ém-

Estimación
bolo. Requiere monitorizar el TIPa, que debe dupl icarse. Por lo general,
de la probabilidad clínica en la enfermedad tromboembólica se admin istra por infusión continua
intravenosa. Su acción puede ser revertida con su lfato de protamina. Se
utilizan fundamentalme nte en el TEP con comprom iso hemodinámico
Probabilidad alta
o pacientes ingresados junto a los fibrinolíticos.

La heparina de bajo peso m o lecular (HBPM) se administra por vía


Dímero O
J subcutánea. Tiene una vida media plasmática más larga, pues interac-

Normal Alta - -- - - ____


.._Técnicas de imagen
..;;....___.
túa menos con las plaquetas y proteínas. Apenas se unen la antit rom-
bina 111 y ejerce su efecto fundamentalmente inactivando el factor Xa.
t
Observación Por esto, y porque tiene una respuesta más previsible a la dosis, la mo-
No IR o alerg ia IR o alergia
nitorizació n del TTPa y el aju ste de dosis no suele ser necesaria, sa lvo en
al contraste al contraste
Tr a t a r - TVP
t t la obesid ad, en el embarazo o en la insu fi ciencia renal grave. De hecho,
en pacientes con in suficiencia renal g rave debe evitarse su uso repeti-
Normal \ Angio-TC Gammagraffa
do. En situaciones hemod inámicamente estables es tan eficaz y segura
o no diar nóstico ' \
como la HNF.

Angiografía Ultrasonidos 1~i~~


de extrem idades - E! El fondaparinux es un inhibidor selectivo del factor Xa que se administra
pulmo nar
por vía subcutánea una sola vez al día y no necesita controles de coag u-
Figura 51. Algoritmo diagnóstico de TEP lació n. Se debe red ucir la dosis en la insuficiencia rena l.

13 · Tromboembolismo pulmonar
Neumología 1 13
Anticoagulantes orales {ACO) riesgo de la misma (Tabla 20), TEP recurrente a pesar de anticoagu-
lación adecuada, (estas son las indicaciones que cuentan con mayor
Inhiben la activación de los factores de coagu lación depend ientes de la grado de aceptación), gran trombo flotante en la vena cava inferi or,
vitam ina K. El más util izado es la warfasina. Generalmente, se administra realización simu ltánea de embolect omía o tromboendarterectomía y
desde el día sigu iente del in icio de la hepa rina y se mantiene la adm inis- profi laxis en los pacientes con ri esgo extremo.
tración simultánea du rante al menos cinco días, hasta conseguir rango
terapéutico al menos dos días consecutivos. De esta forma, también se
evita el efecto procoagulante de los ACO en los primeros dos días por la Absolutas Relativas
caída precoz de los niveles de proteína C y S. El mantenimiento requiere Diátesis y procesos Hipocoagulabilidad congénita
un INR entre 2-3. hemorrágicos o adquirida (por ejemplo,
Hipertensión arterial grave hepatopatías, malabsorción ...)
Hemorragia y aneurisma Alcoholismo
intracraneales Deficiencia mental
Las HBPM no pueden utilizarse en la insufi cienc ia _ _;;::...;.,. Embarazo (para anticoagulantes Historia de úlcera péptica
renal. ora les, no heparina) o hemorragia digestiva
Cirugía retin iana, cerebral Trombopenia
o de la médula espinal Uso de fármacos que
interaccionan con los ACO

Tabla 20. Contraindicaciones del tratamiento anticoagulante


La heparina es el tratamiento de elección en el TEP _ _,........
estable. Los filtros evitan TEP en la fase aguda pero a largo plazo aumentan el
riesgo de TVP. Siempre que sea posible, los pacientes con un filtro deben
permanecer anticoagulados.
Anticoagulación y embarazo. Dada la teratogenia de los dicuma-
rínicos, las heparinas son de elección en pacientes embarazadas. In- Existen actualmente filtros transitorios que pueden ser retirados si la con-
depend ientemente de la heparin a utilizada, las últimas dos semanas tra indicación de anticoagu lación desaparece.
se debe utilizar la heparina no fraccionada y el acenocumarol debe
iniciarse tras el parto. El tratamiento del TEP en embarazadas debe En la Tabla 21 se recoge el tratamiento de elección según la gravedad
mantenerse al menos de 3-6 meses, incluyendo las 4-6 semanas pos- de presentación.
teriores al parto.

Riesgo
Riesgo bajo
intermedio
Los anti coagul antes orales están contraindicados en
el emba razo.
~
~
~ Disfunción VD, TA normal, VD
o daño miocárdico normal, no daño
Recuerda con TA normal miocárdico

3-15% < 1%
Trombo líticos
Fibrinólisis Individualizar Anticoagulación**
Se consideran de elección en el TEP masivo (única indicación aprobada según riesgo
por la FDA) y en la TVP iliofemoral masiva o de cava inferior, siempre de sangrado*
que haya bajo riesgo de sangrado. La trombólisis debe realizarse de En pacientes con bajo riesgo de sangrado: fibrinólisis; en
forma precoz, aunque puede haber respuesta hasta dos semanas tras pacientes con alto riesgo (por ej., > 70 años) anticoagulación con
el TE P. Su obj etivo es la rápida lisis del trombo, acortando la fase de alto seguimiento estrecho, fibrinólisis si empeora o aparece inestabilidad
riesgo y reduciendo la mortalidad, aunque no exist e evidencia firme hemodinámica

que demuestre esto último. Los más emp leados son la urocinasa, es- ** Se puede emplear HBPM, fondaparinux o HNF (cualquiera de ellos)
asociado a anticoagulante oral
treptocinasa y el activador ti su lar del plasminógeno (r-TPA). Se utilizan
en la s fases precoces deiiAM y ACVA trombótico. Es conven ie nte remi- Tabla 21 . Tratamiento del TEP según la gravedad de presentación
tirse a la Sección de Cardiología y cirugía cardiovascular para profundi-
zar en sus características. Duración del tratamiento anticoagulante

Tratamiento invasivo En la actualidad, se recomienda una duración de 3-6 meses cuando


es el primer episodio en paciente con factores de riesgo reversibles
Consiste en la colocación de filtros en la cava inferior, embolectomía (si (cirugía, traumatismo, inmovil ización transitoria). En caso contrario se
hay compromiso hemodinámico grave que no responde a fib ri nolíticos) recom ienda mantenerlo indefinidamente, va lorando periódicamente
o la tromboendarterectomía (en los casos de hipertensión pulmonar cró- si el riesgo de sangrado supera el beneficio de la anticoagulación. En
nica secunda ria a TEP con clínica grave). pacientes con moderado riesgo de sang rado una alternativa vá li da es
mantener la anticoagulación con un INR entre 1,5 a 2. En pacientes
Las indicaciones del filtro de cava inferior son: contraindicaciones o con cáncer los primeros 3-6 meses la anticoagu lación debe hacerse
complicaciones de la anticoagulación en pacientes con ETV o alto con HBPM.
Manual CTO de Medicina y Cirugía, 2.a edición

Los pac ientes con TVP de las ve nas de la s pantorri lla s que no reciben presión mecá nica intermitente, heparin a no fra ccionada, HBPM, fo ndapa-
tratamiento presentan una ta sa mayor de recurrencia, po r lo que se rinu x, anticoa gulantes orales, filtros de la vena cava inferior y la combina-
recomi enda tratar con anti coa gulantes durante tres meses. ción de va rios de estos métodos.

Profilaxis primaria La profil axis farmacológica, con o sin medidas mecánicas, se inicia cuando
acontece una situación de alto riesgo y se continúa por lo menos durante 5-1 O
Existen varias opciones: movilización frecuente y precoz de miembros días. En algunos casos se mantiene más tiempo, como en pacientes de avan-
inferiores en pacientes enca mados, med ias de compresión gradual, com- zada edad o tras cirugía que se sigue de un largo periodo de inmovilización.

" La probabilidad clínica condiciona la actitud diagnóstica.


Ideas clave Ji6
" El dímero D es una prueba sensible pero muy poco específica,
" El origen de los trombos es, en el 90% de los casos, el sistema por tanto con un alto valor predictivo negativo.
venoso profundo de las extremidades inferiores.
" La heparina es el tratamiento de elección. Ante una alta sos-
" El factor de riesgo más frecuente es el antecedente de un episo- pecha clínica se debe iniciar la anticoagulación sin esperar a la
dio tromboembólico. confi rmación diagnóstica.

" La TVP cursa de forma asintomática en la mitad de los casos. " En caso de contraindicación absoluta de anticoagulación, el tra-
tamiento de elección es el filtro de cava .
" La disnea, habitualmente súbita, es el síntoma de presentación
más frecuente. " En caso de TEP masivo con inestabilidad hemod inámica, el tra-
tamiento de elección es la trombólisis.

radiografía de tórax es
Casos clínicos informada como nor-
mal. Se realiza una TC
Mujer de 70 años, hipertensa con insuficiencia renal crónica mo- torácica cuyo resultado
derada, que acude por disnea súbita con signos de trombosis ve- se muestra a continua-
nosa en miembro inferior derecho. En las pruebas complemen- ción. ¿Cuál es el diag-
tarias destaca una hipoxemia de SS mmHg, hipocapnia de 24 nóstico más probable?
mmHg y taquicardia sinusal a 11 S lpm en el ECG. Las plaquetas
y la coagulación están dentro de los límites de referencia. El dí- 1) Neumomediastino con enfisema subcutáneo.
mero-O es de 981 ng/ml y la creatinina de 3,S mg/dl. ¿Cuál de las 2) Em bolia pu lmonar en arteria pulmonar princi pa l izquierda.
siguientes actitudes le parece más adecuada en este momento? 3) Estenosis esofágica.
4) Fractu ra costal patológica.
1) SolicitarTe helicoidal torácica, inici ando perfusió n con heparina
sódica a 1.000 Ul/ h. RC: 2
2) Iniciar anticoagulación con heparina de bajo peso molecular, en
dosis de 1 mg/kg, cada 12 horas. Hombre de 6S años de edad que presenta, de forma aguda, dis-
3) Admi nist ra r 5.000 Ul de heparina sódica y solicitar gammagrafía nea, dolor torácico pleurítico derecho y hemoptisis moderada.
pulmona r de ventil aci ón/ perfusión. Una gammagrafía pulmonar de ventilación-perfusión se interpre-
4) Solicit ar ecog rafía con Doppler de m iembros inferiores para t a como de alta probabilidad de embolismo pulmonar. Una radio-
confi rma r el diagnóst ico y ad mini st ra r 1 mg/kg de heparina de grafía de tórax muestra un derrame pleural derecho que ocupa
bajo peso molecu lar. menos de un tercio del hemotórax derecho. La toracocentesis de-
muestra que se trata de un derrame serohemorrágico. ¿Cuál de los
RC: 3 siguientes sería el tratamiento correcto para este paciente?

Paciente de 3S años de edad, peluquera de profesión, que acude 1) Coloca r un t ubo de drenaj e torácico e iniciar anticoagulación .
a Urgencias por disnea de inicio brusco con dolor torácico. Lleva 2) Iniciar anticoagu lación con heparina sód ica y col ocar un fi lt ro
dos semanas de reposo por un esguince de tobillo derecho y no de vena cava inferi or.
tiene otros antecedentes de interés. Está en tratamiento con an- 3) Tratar con hepa ri na sódica en bomba de infusión en dosis plenas.
ticonceptivos orales por acné. En la exploración física se aprecia 4) Evitar la anticoagul ación y colocar un filtro en vena cava inferior.
TA 11 0/6S, frecuencia cardíaca 104, taquipnea leve y auscultación
cardiopulmonar normal. El ECG muestra taquicardia sinusal y una RC: 3

3) lmpedance plethysmogra phy in patients w ith deep vein t hrom-


Case Study ._- bosis symptoms has a sensitivity of approximately 90% w hen
eva luating calves.
With respect to PTSD, which one of the following statements is 4) Ult rasonography is curre ntly one of th e non-invasive techn i-
NOT correct? ques to assess t he lower extremities.

1) High cl inical suspicion is very valuable fo r making deci sions. Correct answer: 3
2) Chest x-ray may show elevation of a hem id iaphrag m.

13 · Tromboembolismo pulmonar
_N_eumologi_______ •

Trastorn os de la ventilació n

aumento de la PaC0 2 con unos pu lmones normales, debido a que hay


Hay que conocer y saber
ORIENTACIÓN disminución del volumen de ventilació n por minuto, y por tanto, hipo-
identificar las causas
ENARM de hipoventilación
y la hiperventilación psicógena.
ventilación alveolar.

Localización del
Mecanismo Trastorno clínico
defecto

14.1. Regulación de la ventilación Alteración Quimiorreceptores Disfunción del cuerpo


del impulso centrales carotídeo, traumatismo
respiratorio y periféricos Hipoxia prolongada

Los conceptos básicos de la regulación de la ventilación se exponen en el Alcalosis metabólica

Capítulo de Fisiopatología, y se resumen en la Tabla 22. Neuronas Poliomielitis bulbar, encefalitis


respiratorias Infarto, hemorragia
del tronco y traumatismos del tronco
encefálico encefálico
Localización Estímulos
Administración de fármacos

Neuronas Alteración ácido- base (estimu la Sínd rome de hipoventilación


Automático
(metabólico) medulares quim iorreceptores) alveolar primaria

Neuronas Alteración grado de estiramiento Defectos Médula y nervios Traumatismos cervicales altos
del tronco y velocidad del flujo aéreo (estimula del sistema periféricos Poliomielitis
del encéfalo mecanorreceptores) neuromuscular
Enfermedad de la neurona
respiratorio
Voluntario Neuronas Voluntario motora
cortica les Neuropatía periférica
Tabla 22. Sistemas de control de la ventilación Músculos Miastenia gravis
respiratorios Distrofia muscular
Miopatía crónica
14.2. Síndromes de hipoventilación Alteraciones Pared torácica Cifoescoliosis
del aparato Fibrotórax
respiratorio
Toracoplastia
Definición y etiología Espondilitis anquilosante
Obesidad- hipoventilación
La hipoventilación se define por un aumento de la PaC0 2 (presión parcia l
Vías respiratorias Estenosis laríngea y traqueal
del C0 2 en sangre arteria l) por encima de su límite superior de la norma-
y pulmonares Apnea obstructiva del sueño
lidad (45 mmHg).w
Fibrosis quística

La hipoventilación crónica puede producirse básicamente por tres meca- Enfermedad pulmonar
obstructiva crónica (EPOC)
nismos: alteración del impu lso respiratorio, defectos en el sistema neuro-
muscular o alteración del aparato respiratorio (Tabla 23). Tabla 23. Síndromes de hipoventilación crónica

Los procesos que presentan alteración del impulso respiratorio, defec- En el resto de los procesos que afectan a la pared torácica, vía s respi-
tos en el sistema neuromuscular, algunos trastornos de la pared torácica ratorias bajas y pulmones, se puede producir aumento de la PaC0 2 a
(obesidad) y la obstrucción de las vías respiratorias su periores producen pesar de una ventilación por minuto norma l, debido a la existencia de
Manual CTO de Medicina y Cirugía, 2. a edición

discordancia entre ventilación y perfusión. Sin embargo, con frecuen- Diagnóstico


cia, en la hipoventi lación crónica se combinan va rios mecan ismos; así,
en la EPOC, el aumento de la PaC0 es por alteración de la mecáni- Existen distintas pruebas para diferenciar el mecan ismo causante de la
2
ca ventilatori a y por d isminución del impu lso ventilatorio central (que hipoventilación que se exponen a continuación (Tabla 24) .
puede ser in herente o secunda ri o a la alca los is met abólica, por el uso
de d iuréticos o estero ides). Determinación de respuesta a estímulos químicos

Características clínicas y fisio lóg icas (Figura 52) Consiste en hacer que el paciente respire de una bolsa de aire que contie-
ne una concentración elevada de C0 2. Lógicamente, en sujetos sanos, esto
desencadena una respuesta de hiperventilación, pero en las alteraciones del
Centro respiratorio Centro e
Control volunta rio neumotáxico ·O impulso ventilatorio y en algunos de los defectos de sistema neuromuscu lar
(i nspiración y es piración) 'ü

~entro
"'
:; (trastornos cervicales altos, fundamentalmente), esta respuesta no aparece.
0\
Q)
0::
Alteraciones del
impulso ventilatorio ~apnéustico Estímulo inspiratorio del primer 0,1 segundo
tras la oclusión (Po,1)

Enfermedades
neuromusculares El paciente resp ira voluntariamente a través de una boqu il la que inespe-
Nervios y múscu los radamente se ocluye. Con la oclusión, la respuesta no rma l del individuo
intercosta les
es rea lizar una inspiración más brusca. Se mide el aumento del esfuerzo
inspiratorio en la primera décima de segundo, lo que traduce el impulso
venti latorio. En sujetos con alteración del impu lso respiratorio y trastornos
1
Alteraciones
neuromusculares altos está d isminuido, en el primer caso por alteración
en su generación, y en el segundo por alteración en su transmisión.
ventilat orias

EMG diafragmático

Nervio frénico
j Puede ser anorma l en las alteraciones del impul so respirato rio y en tras-
tornos neuromusculares.
Enfermedades
y diafragma restrictivas Ventilación voluntaria máxima (WM)
Figura 52. Fisiopatología de los trastornos de hipoventilación
Consiste en hacer hiperventilar al máximo al paciente durante 30 segun-
En la hipoventilación se produce un aumento de la PAC0 (presión parcial dos. En los trastornos neuromusculares o con patología del aparato res-
2
de C0 2 en el aire alveolar), lo que conduce a la elevación de la PaC0 . Se piratorio, el paciente claud ica y no consigue mantener tanto tiempo la
2
genera, por tanto, acidosis respiratoria, lo q ue produce un incremento en hiperventilación.
la concentración de HC03· en el plasma. Además del incremento de la
PAC0 2, durante la hipoventilación se produce también una d ism inución Presión inspiratoria y espiratoria máximas (PIM/PEM)
de la PA0 2 y, por ta nto, hipoxemia, q ue si es grave puede inducir la eritro-
poyesis y produci r polig lobulia. La hi poxem ia puede producir vasocons- Se rea liza una medición manométrica de la fuerza que genera el sujeto al
tricción pu lmonar e hipertensión pu lmona r. inspirar o espirar contra una boqui lla cerrada. Se altera en los trastornos
neuromusculares.
Las alteraciones de los gases arteria les se man ifiestan típicamente du-
rante el sueño debido a una reducción ad icional del impulso respirato-
rio centra l. La hipercapnia nocturna puede produci r vasod ilatación ce-
Una ventil ac ión vo luntaria max 1ma norma l sugiere
reb ral y cefa lea matutina; puede altera rse también la cal idad del sueño,
alteración del centro respiratorio con integridad de 1•"""'o:::.;:o.oo:.- :·":
produciendo astenia d iurna, somnolencia diurna, confusión mental y los sistemas neuromuscul ar y resp irator io.
deterioro intelectual.

Estímulo Flujos- Gradiente


p 0,1 EMGd VVM PIM/PEM
químico 1
volúmenes (A-a)0 2
Alteración del .J, .J, .J, Normal Normal Normal Normal
impulso respiratorio
Alteración
del sistema .J, .J, .J, .J, .J, Alterado Normal
neu romuscular
Alteración del
.J, Normal Normal .J, Normal Alterado Alterado
aparato ventilatorio
Tabla 24. Pruebas diagnósticas en las alteraciones de la ventilación

14 · Trastornos de la ventilación
Neumología 1 14
Espirometría Tratamiento

Medida de volúmenes pu lmonares, resistencia y d istensibilidad. Se alte- Algunos respond en a los estimulantes de la respiración (almitrina, me-
ran en los defectos del aparato ventilatorio, en los trastornos neuromus- droxiprogesterona) y al suplemento de oxígeno; pero la mayoría requiere
culares (patrón restrictivo extraparenquimatoso inspiratorio y espirato- ventilación mecánica no invasiva nocturna. También se han descrito be-
rio), pero no se alteran en los trastornos del impulso respirator io. neficios ocasional es con el marca pasos d iafragmático.

D(A-a)0 2 Síndrome de obesidad-hipoventilación


(síndrome de Pickwick)
Es normal en las alteraciones del impulso respiratorio y en los trastornos
neuromusculares, pero está elevado en las enfermedades pulmonares.
La obesidad masiva representa una sobrecarga mecánica para el aparato
Tratamiento respiratorio, ya que el peso sobreañad ido a la caja torácica y abdomen redu-
ce la distensibilidad de la pared torácica, y disminuye la capacidad residual
Debe incluir medidas d irig idas a t ratar la enfermedad subyacente, hacien- funcio nal (CRF), sobre todo, en decúbito. La resp iración co n volúmenes pul-
do especial hi ncapié en evitar los fá rmacos q ue depri men el centro respi- monares bajos hace que algunas vías respiratorias de las bases pulmonares
ratorio (barbitúricos, benzod iacepinas, etc.). La mayoría de pacientes con estén cerradas durante la respiración corriente, produciendo hipoventilación
alteración del impulso resp iratorio o enfermedad neuromuscu lar requiere en bases y aumento de la D(A-a)0 2. En la mayoría de los obesos, el impulso
ventilación mecánica no invasiva con presión positiva intermitente. centra l es suficiente pa ra mantener una PaC02 normal, pero algunos presen-
tan hipercapnia e hipoxemia crónica. En muchos de estos pacientes se aso-
En muchos casos es suficiente durante el sueño, produciendo una mejoría clí- cia un síndrome de apnea del sueño. La mayoría de estos individuos tienen
nica espectacular con una disminución de la PaC0 2 diurna; sin embargo, cuan- una reducción del impulso respiratorio central (inherente o adquirida). En
do la hipoventilación es grave, puede ser precisa durante las 24 horas, y enton- cuanto al tratamiento, es útil la pérdida de peso y la estimulación del impul-
ces hay que pasar a la ventilación mecánica invasiva a través de traqueostomía. so respiratorio con fármacos como la progesterona. Si el paciente presenta
hipercapnia, está indicada la ventilación mecánica no invasiva (Figura 53).
En los pacientes con d isminución del impulso respiratorio, pero sin alte-
raciones en las neuronas motoras inferiores, nervios frén icos y músculos
respiratorios, puede ser útil el marcapasos diafragmático mediante un
electrodo frénico. La hipoventilación relacionada con trastornos restricti -
vos de pared (cifoescoliosis) también puede tratarse con ventilación me-
cánica nocturna con presión positiva intermitente.

Hipoventilación alveolar primaria


(síndrome de Ondina)

Concepto

Trastorno de causa desconocida, ca ract erizado por hi percapn ia e hipoxe-


mia crón icas. Parece deberse a un defecto del sistema de contro l meta-
bólico de la respiración. Es poco habitual, y la mayoría de casos ocurren
en varones de 20 a 50 años.

Clínica

Típicamente se desarrol la de fo rma insid iosa, y con frecuenc ia se diag-


nostica, cuando se produce una grave depresió n respiratoria tras la admi-
nistración de dosis usuales de anestésicos o sedantes. Conforme progre-
sa, se produce letargia, fatiga, somnolencia diurna, alteraciones del sueño
y cefalea matutina. Durante el sueño tienen un deterioro adicional de la
ventilación con frecuentes episodios de apnea central.

Diagnóstico

Existe acidosis respiratoria crónica, con aumento de HC0 3-. Durante el día
puede no objetivarse la hipoventilación, por lo que la Pa0 2 y la PaC0 2 en
vigilia pueden ser normales, pe ro se podrá encontrar el HC0 3- elevado,
como consecuencia de la hipoventilación crón ica noctu rn a. En esta situa-
ción hay que demostrar la hipoventilación nocturna mediante la realiza-
ción de una polisomnog rafía. Figura 53. Sind ro me de Pickw ick
Manual CTO de Medicina y Cirugía, 2. a edición

Trastornos neuromusculares respiratorios Asociada con ingesta de fármacos: salicilatos, derivados de metilxan-
tina, agonistas ~-ad renérg i cos, progestero na.
Síndrome pospolio Otras situaciones como sepsis, fiebre, dolor y embarazo.

Es ca da vez más común en este gru po de procesos. Es una fo rma de in su- Clínica
fi ciencia respiratoria crón ica que típicamente se desarrolla pasados 20-30
años desde la recuperación de la poliomielitis. Ade más de la historia pre- El síntoma más frecuente es la d isnea, ya q ue la hiperventilación se asocia
via de polio, los pacientes tienen debilidad de la musculatu ra respiratoria, a un aumento del im pulso respiratorio, del esfuerzo muscu lar y del volu-
y muchos desarrollan cifoesco liosis. La afectación respiratoria, además de men minuto, pero no hay correlación entre el grado de disnea y la PaCOr
ser genera lmente precoz, puede ser la única manifestación de la enfer- Además de hipocapnia se produce un aumento de la PA0 2 y de la Pa0 2.
medad. Debido a la alca losis que se produce, pueden aparecer diversos síntomas
neurológicos como mareo, síncope, convulsiones y trastornos visuales
Enfermedad de la motoneurona que se deben a la vasoconstri cción cerebral.

Es una degeneración id iopáti ca de las neuronas motoras del asta ante- Debido a la disminució n de ca lcio libre en suero, puede haber pareste-
rior, que puede man ifestarse como fa llo resp iratorio crón ico, sobre todo sias, tetania y espasmo carpopeda l. Por la hipofosfatemia, se puede en-
cuando las neuronas motoras frénicas se involucran. La afectación de los contrar debil idad muscular. Cua ndo la alcalosis es grave, se pueden pro-
múscu los respiratorios en esta enferm edad suele ser una manifestación d ucir arritmias e isquemia ca rdíacas.
tardía de una enfermedad más genera lizada (como ocu rre en la miastenia
grave o distrofia muscular). La enfermedad afecta típicamente a va rones La alca losis respiratoria primaria hace que el paciente pueda presentar
entre 50-70 años, y se diferencia de la HAP por la presencia de debi lidad respiración periódica y apnea central del sueño.
muscular y atrofia, a menudo con fasc iculaciones y espasticidad, siendo
los test típicos de afectación neuromuscular. El curso es progresivo, y el Diagnóstico
paciente suele fa llecer a los 2-5 años del com ienzo de la enfermedad por
comp licaciones resp iratorias. Con la historia clínica, la explo ración física y el conocimiento de las en-
fermedades subyacentes o las otra s situaciones que son capaces de pro-
Parálisis unilateral del diafragma ducir la hiperventilación, en la mayoría de los casos se puede hacer el
diagnóstico.
Puede producirse por trauma del nervio frénico, inflamación, infiltración
neoplásica, pero en muchos casos, es idiopática. Los pacientes están a Cuando se determina la gasometría arterial, es importante fijarse en
menudo asintomáticos y no hipoventilan genera lmente. la D(A-a)0 2, ya que su aumento (norma l hasta 15 m mHg, sin conside-
rar las correccio nes para pacientes más mayo res) implica enfermedad
La debilidad o parálisis bi latera l del diafrag ma puede ser por neuropatía primaria pulmonar. Si el bicarbonato está disminu id o, implica que la
frénica o por enfermedad intrínseca del diafragma, incluyendo distrofia situación es crón ica, ya que el riñón está t ratando de compensa r la
muscular, polim iositis y deficiencia de maltasa ácida. Son ca racte rísticas alterac ión de l pH.
la ortopnea, el movimiento parad ójico del abdomen en supino, el dete-
rioro de los volúmenes pu lmonares y de los gases arteriales en supino, el La determinación de la PaC0 2 t ranscut ánea durante el sueño es de im-
deterioro de la ventilación durante las maniobras de ventilación volunta- portancia en los pacientes en los que se sospeche hiperventilación psi-
ria máxima y la reducción de la presión inspiratoria máxi ma. La presión cógena o de ansiedad, ya que, en estas situaciones, la PaC0 durante
2
transdiafragmática está disminuida o ausente, así como también está d is- el sueño no es baja, dado que durante este periodo no se mantiene la
minu ida la respuesta electromiográfica del diafragma. hiperventilación. Las situaciones que con más frecuencia producen hi-
perventi lación inexplicable son las enfermedades vascu lares pulmonares,
El tratamiento de los procesos neuromusculares requ iere la vent ilación sobre todo, el tro mboem bo lismo recurrente o crónico y las situaciones
mecánica no invasiva. En las lesiones de médula cervical alta donde es- de ansiedad.
tán intacta s las neuronas motora s inferiores frén icas y los nervios, puede
indicarse el marca pasos diafragmático. Resulta curioso cómo los pacientes con hiperventilación psicógena refie-
ren la disnea, sobre todo al reposo, pero no durante los ej ercicios suaves,
y además tienen que suspirar con cierta frecuencia . Suelen presentar sín-
14.3. Síndromes de hiperventilación tomas como mareos, sudoración, pa lpitaciones y pa restesias. Se acom-
paña de una D(A-a)0 2 normal y la hiperventilación tiende a desaparecer
durante el ej ercicio. Sin embargo, los pacientes que hiperventilan por
La hiperventilación alveolar es la situación donde la PaC0 2 es inferior al enfermedad vascular pulmonar tienen disnea de esfuerzo, mantienen la
va lor establecido como límite inferior de la norma lidad (35 mmHg). Las hiperventilació n durante el esfu erzo y la D(A-a)0 2 está elevada.
situaciones que se asocian con hiperventilación son las sigu ientes:
Hipoxemia de cualquier origen. Tratamiento
Trastornos metabólicos: acidosis d iabética, acidosis láctica, insufi-
ciencia rena l e insuficiencia hepática. Es el de la enferm edad o causa subyacente. En alg unos casos, cuando
Enfermedades neurológicas y psicógenas: hiperventilación psicó- aparecen síntomas como mareos, parestesias y otros, el paciente se pue-
gena (por ansiedad) y tumores e infecciones del sistema nervioso de beneficiar de la inhalación de una concentración alta de C0 2 (respirar
centra l. en una bolsa cerrada).

14 · Trastornos de la ventilación
Neumología J 14
" En la hipoventilación de causa neuromuscular están reducidas
Ideas clave RS las presiones inspiratoria y espiratoria máximas (PIM y PEM).

" La hipoventilación alveolar de origen central cursa con ventila- " La hiperventilación psicógena cursa con alcalosis, hipocapnia y
ción voluntaria máxima normal. gradiente alveoloarterial de 0 2 normal.

2) P0 2 markedly increased; PC0 2 marked ly increased and pH de-


Case Study creased.
3) P0 2 markedly increased; PC0 2 marked ly decreased and pH de-
A patient with respiratory failure seconda ry to pol iomyelit is is creased.
placed on a respirator accidentally set at too high a rate. Which 4) P0 2 norma l or sli ghtly increased; PC0 2 markedly decreased and
of the following changes in arterial blood gas st udies would you pH increased.
expect to see?
Correct answer: 4
1) P0 2 marked ly decreased; PC0 2 sl ight ly decreased and pH in-
creased.
Neuma logia_

-
Síndrome de la apnea del sueno

Se trata de una enfermedad


15.1. Apnea obstructiva del sueño
ORIENTACIÓN muy preva/ente. Se debe
ENARM conocer el diagnóstico y, sobre
todo, el tratamiento. El síndrome de apneas obstructivas du rante el sueño (SAOS) es una en-
fermedad secundaria a episodios repetidos de obstrucción de la vía aérea
superior du rante el sueño.

Una apnea consiste en la ausencia completa de fluj o aéreo dura nte Patogenia
al menos 1O segundos. En la apnea obstructiva, la más frecuente, el
flujo cesa por una oclusión de la vía aérea superio r a nivel de orofa- El colapso se produce cuando la presión faríngea subatmosférica gene-
ringe, por lo que existen movimientos toracoabdominales (esfuerzo rada durante la inspiración excede a la fuerza generada por los músculos
muscular respiratorio) durante la apnea. En la apnea central el flujo dilatadores y abductores de la vía aérea superior. A esta presión crítica de
aéreo cesa debido a una ausencia transitoria de impulso venti latorio colapso de la vía superior contribuyen tanto factores anatómicos (macro-
centra l y no hay, por tanto, movim ientos toracoabdominales durante glosia, hipertrofia amigdalar, obesidad), como func ionales (dism inución
la apnea . La apnea mixta cons iste en episod ios centra les seguidos de del tono muscu lar durante el sueño profundo).
un componente obstru ctivo, y se considera una variante de las apneas
obstructivas (Figura 54). El suceso definitivo es el movimiento posterior de la lengua y el palada r
en aposición con la pared posterior de la faringe, con oclusión de la naso-
faringe y orofaringe. Durante la apnea se produce un aumento de los
movimientos toracoabdominales, lo que conduce a un microdespertar
Apnea centra l: cese del fluj o aéreo y de los movi- (arousa l) que fina liza la apnea al aumentar de nuevo el tono muscu lar.
mientos tora coabdominal es.
Esto conduce a fragmentación del sueño, con dism inución de las fases 111
Recuerda y IV (sueño profundo o sueño de onda lenta) y del sueño REM (Figura 55).

1 1 - ••• En el sínd rome de apnea del sueño se produce frag-


mentac ión del sueño, con disminu ción del sueño de _.....,.::::..;;.,.
Flujo aéreo
onda lenta o sueño profundo y del sueño REM.
Movimientos
torácicos

.. -. -
Sueño profundo
Flujo aéreo Cesa la apnea

Movimientos
+
Recuperación tono muscular
J. Generalizada
de tono muscular
torácicos
+
.. -. Microdespertar
(arousal)
Flujo aéreo
+
t Movimientos toracoabdominales
Movimientos
torácicos +
t Esfuerzo respiratorio-+----
' Tooo1'"'"'VAS
Figura 54. Tipos de apnea Figura 55. Círculo de la apnea
Neumología 1 15
Clínica Existen pruebas de cribado más simples, como el registro oximétrico
domiciliario o la poligrafía cardiorrespiratoria (estudio sin variables neu-
Afecta a un 2-4% de la población adulta en los países desarrollados, con rofisiológicas), en cuyo caso, al no d isponer de información sobre los mi-
predominio en varones de edad media (obesidad), muj eres posmeno- crodespertares, para el diagnóstico se uti liza el índ ice de apnea-hipopnea
páusicas (causa desconocida, se cree que pueda deberse a alteraciones (IAH), que es el número de apneas e hipopneas por hora de registro. Si es
hormona les) y niños pequeños (hipertrofi a de am ígda las y adenoides). ~ 5 se diagnostica al SAOS.
Las manifestaciones neuropsiquiátricas y de conducta son consecuencia
de los despertares transitorios breves recurrentes que terminan con cada Tratamiento
apnea y fragmentan el sueño, produciéndose una pérdida de sueño re-
parador de ondas lentas. Las manifestaciones cardiovasculares están rela- Medidas generales
cionadas con los episodios recurrentes de desaturación nocturna.
Encam inadas a controlar los facto res pred isponentes: mejorar la respira-
El síntoma más común es la somnolencia diurna, que puede llegar a ser ción nasal (disminuye la presión subatmosférica inspiratoria), reducc ión
muy peligrosa e interferir con la vida (accidentes de auto móvil). También de peso (aumenta el cal ibre de la vía aérea), evita r el alcohol y el uso de
pueden presenta r det erioro intelectual y pérd ida de memoria. El ron- medicamentos hi pnóticos o sedantes (d ism inuyen el tono muscular). Se
qu ido está presente durante años, antes de que se desarrol len los otros deben tomar en todos los pacientes con SAOS.
síntomas, es de carácter temporal, interrumpido periódicamente por los
episodios de apnea. La term inación de cada episodio apneico se anuncia CPAP (presión positiva continua en la vía aérea)
por un fuerte ronquido con movimientos del cuerpo. Incluso, a veces, el
paciente se despierta completamente y se queja de disnea. Es muy fre- Aplicada a través de mascarilla nasal, genera una presión positiva continua
cuente la cefalea matutina, debida a la hipercapnia nocturna. en la vía aérea superior impidiendo su colapso. No modifica los parámetros
espirométri cos. Mejora la respiración durante el sueño, la calidad del sueño,
Las arritmias cardíacas son comunes. En la mayoría, hay bradicardia mo- la somnolencia diurna, el estado de alerta, las alteraciones neuropsicológi-
derada de 30-50 lpm y elevación tempora l de la tensión arterial d urante cas y la ca lidad de vida. Es el tratamiento de elección y está indicado siem-
la apnea, segu ido de taquicardia al reanudarse la resp irac ión . Existen da- pre que el paciente presente un lAR > 15 ju nto con síntomas diu rnos (fun-
tos que relacionan el SAOS con las enfermedades ca rdiovascu lares (sobre damentalmente hipersomnolencia) o fact ores de riesgo card iovascular. En
todo, la hipertensión arteri al) y cerebrovascu lares. Además, en el SAOS, el resto de situaciones la ind icación o no de CPAP debe ser ind ividualizada.
hay un empeoram iento de la función ventricular en los pacientes con
cardiopatías subyacentes. Dispositivos de avance mandibular (DAM)

Durante los episodios de apnea se produce vasoconstricción pulmona r Son d ispositivos orales de reposic ionamiento mandibular que actúan
aguda, pero la presión arterial pulmonar durante el día es típicamente desplazando hacia adelante la mandíbula inferior y la lengua, ensanchan-
norma l (el desarro llo de hipertensión pu lmona r mantenida requiere la do la vía aérea faríngea. Mejora n la respirac ión durante el sueño y la som-
presencia de hipoxem ia e hipercapnia du rante el día, unido a desatura- nolencia d iurna. Se consideran el t ratamiento alternativo a la CPAP en
ciones nocturnas graves). No parece que la hipoxemia nocturna aislada aquellos pacientes que no la tolera n o no es eficaz.
se asocie con fa llo derecho. Un 10-15% de pacientes presentan hipercap-
nia crónica. La debil idad en el impulso respiratorio se atri buye a factores Fármacos
genéticos, a la hipoxemia crónica o a la fragmentac ión crónica del sueño.
Ningún tra tamiento farmacológico ha demostrado ser efi caz para reducir
Diagnóstico las apneas o las hipopneas. En pacientes con SAOS e hipersomnolencia
diurna, a pesar del tratamiento con CPAP, se ha obtenido algún beneficio
El método definitivo para confi rmar el SAOS es la polisomnografía, que marginal con modafin ilo en la reducción de la hipersomnolencia diurna,
consiste en la medición, durante el sueño, de una serie de parámetros pero los datos disponibles no son concluyentes, por lo que no se puede
card iorrespiratorios (flujo aéreo, movimientos toracoabdominales, satu- sentar su indicación.
ración de 0 2, electrocardiograma, ronquido, posición corpora l) y neurofi-
siológ icos (electroencefa lograma, electromiog rama, electroocu lograma). Tratamiento quirúrgico
Con la polisomnografía se podrán detecta r los siguientes eventos:
Apnea: ausencia de flujo aéreo de al menos 1Osegundos de duración. No existe suficiente evidencia que apoye la utilización de técnicas de ci-
Hipopnea: reducción significativa del flujo aéreo de al menos 1O se- rugía faríngea, incluida la uvulopalatofaringoplastia. Hoy en día existen
gundos de duración, acompañada de una desaturación > 3o/o o de cuatro modalidades de tratamiento quirúrgico en el SAOS:
un microdespertar. Cirugía bariátrica: indicada en los pacientes con obesidad mórbi-
ERAM (esfuerzo respiratorio asociado a microdespertar): au- da, en los que puede ser curativa .
mento del trabajo respiratorio, sin apnea ni hipopnea, que se acom- Amigdalectomía: muy eficaz en los niños, pero muy poco en los adultos.
paña de un microdespertar. Traqueotomía: es curativa en todos los casos. No obstante, dada la
morbilidad asociada, raramente se utiliza en la actua lidad, aunque se
Una vez conocido el número total de apneas, hipopneas y ERAM que debe considerar en los casos de SAOS muy g raves que no respondan
aparecen durante todo el estud io, se divide por las horas de sueño, lo que a ningún otro tratam iento.
se denomina índ ice de alteraciones respiratorias (lAR), es decir, número Osteotomía maxilomandibular: técnica qu irúrgica de adelanta-
de apneas, hi popneas y ERAM que presenta el paciente por hora de sue- miento mandibu lar. Es especialmente eficaz en los casos de retrogna-
ño. Se diagnostica SAOS cuando el lAR es ~ 5. tia, y se debe considerar también en los pacientes jóvenes y delgados.
Manual CTO de Medicina y Cirugía, 2. 8 edición

1 5.2. Apnea central del sueño permeable en la fase meng uante de la respiración, puede dar como
res ultado el colapso y desa rro llo de apnea o bstructiva . Por ta nto, la
resp irac ión periód ica puede asociarse con apnea centra l del sueño
Hay un fa llo tran sitorio del estímul o central d irig ido a los músculos de u obstru ctiva, lo q ue explica q ue las apneas obstru ctivas y centra les
la respiración. Esto conlleva una serie de acont ecim ientos sim ilares a los puedan coexistir en el mi smo paciente.
que ocurren en la apnea obstructiva.
Clínica
Patogenia
Cuando el origen es el defecto del sistema de contro l metabólico o el
Exist en distintos meca ni smos q ue originan la apn ea central: neuro mu scu lar, el cuadro ti ene las ca racterísticas de la hi povent ilación
Defecto del sistema de control metabólico o de los m úsculos res- alveo lar crónica (ret ención de C0 2, hi poxemia y poli g lo buli a, hi pertensión
piratorios (que incluye n los sínd romes de hipovent ilació n alveolar pulmonar, etc.), así como cefa lea e hi persomnia d iurna como consecuen-
primarios o secunda rios y la debilidad de los m úscu los resp iratorios). cia de la agravación noctu rna y de la fragmentación del sueño. En con -
Se produce una hipoventilación alveolar crónica con algún grado de traste, los pacientes con inest abilidad tra nsitoria del impulso no tienen
hipercapn ia d urante el día, pero principa lmente d urante el sueño, al hipercapnia y no desa rro llan las complicaciones cardiopulmona res, sino
desa parecer el efecto estimu lante de la vig il ia sobre la respirac ión. ún icamente las alteraciones de l sueño.
Fluctuación transito ri a o inest abilidad del impulso respiratorio. Es la
apnea central más frecuente. Se manifi esta como respiración peri ó- Diagnóstico
dica, que co nsiste en una respiració n regul ar creciente y decreciente,
como resultado de las fl uctuacio nes del im pul so respiratorio centra l. Se precisa polisom nografía. Como se ha expuesto, la med ición de la PaC0 2
Si, durante el periodo decreciente, se incluye un periodo corto de ap- sirve para discern ir si lo que fa lla es el contro l metabólico (elevada y crece
nea centra l completa, se denomina "respiración de Cheyne-Stokes': en el sueño) o si se debe a fluctuación en el impulso (norma l o baja).
Ocurre típicamente du rante el inici o del sueño. Durante la vigi lia o
du rante el sueño de o ndas lentas, no hay alteración del impu lso res- Tratamiento
pi ratorio, por lo q ue la PC0 2 se mantiene en rango de normalidad.
Para explicar est as flu ctu acio nes se han est ablecido va rias teorías, y En los pacientes con defecto en el sist ema de contro l respiratori o me-
la más reco noc ida es la siguiente: durante la vigilia es necesa ri a una tabó lico o enfe rm edad neuro mu scular, depende de la enfe rmedad de
PaC0 2 más baja que d urante el sueño pa ra ma ntener la respiración, base. En algunos pacientes, el oxígeno suprime las apneas, probablemen-
y en la tra nsición de la vig il ia a sueño, esa PaC0 2 puede no ser sufi- te al aliviar la depresión hipóxica. Algunos mejoran con estimu lantes res-
ciente pa ra estimu larla, orig inando apnea, que va seguida de un au- piratorios, como la med roxiprogesterona.
mento de la PaC0 2- hast a el valor que ya prod uce ritmo respiratorio.
Se ha demost rado que, en muchos pacientes con SAOS, hay t am- El manej o de los pacientes con apnea central por inestabilidad del impul-
bién flu ctu ación periód ica en el impu lso resp iratori o a los músculos so (no hipercápnica) no est á cla ramente establecido. Se emplean suple -
de la pared torácica y a los músculos di latado res y abd ucto res de la mentos de oxíge no, acetazo lamida, y últimam ente, la CPAP, pues parece
vía aérea superi o r. Debido a la estrech a luz de la vía aérea superi or, q ue aumenta la PaC0 2 por encima del umbral apneico debido al sobrees-
la pérdida t ransitoria del tono de la musculatura q ue la ma ntiene fuerzo mecá nico q ue hay q ue rea lizar durante la espiración.

" El diagnóstico se rea liza por poli somnografía , cu ando el índice


Ideas clave ~ de alteraciones respiratorias es mayor de S.

" El síndrome de apnea obstructiva del su eño afecta al 2-4% de la " El t ratamiento de elección es la presión positiva continua en la
población adulta. vía aérea (CPAP) .

" El síntoma más frecuente es la hipersomnolencia diurna.

15 · Síndrome de la apnea del sueño


~o~lo~g~ía________~.

Síndrome de distrés
respiratorio agudo

ORIENTACIÓN Se debe tener en cuenta el 16.1. Etiología


concepto de distrés y sus
ENARM criterios diagnósticos.
Se pueden distinguir dos grandes categorías causantes de SDRA:
Lesión pulmonar directa: neumonía, aspiración de contenido gástri-
co, contusión pulmonar o inhalación de tóxicos.
El síndrome de distrés resp iratorio agudo, o síndrome de dificultad res- Lesión pulmonar indirecta: sepsis, traumatismo extratorácico grave,
piratoria del adulto (SDRA), es un síndrome clínico que se caracteriza por quemaduras, transfusiones múltiples, intoxicaciones por fármacos o
el desarrollo de d isnea con una insuficiencia respiratoria aguda grave de pancreatitis.
rápida evolución, junto con un infiltrado pulmonar difuso alveolointers-
ticial. La mayoría de los casos se deben a sepsis (la causa más frecuente), neu-
monía, aspiración de contenido gástrico, traumatismos, politransfusiones
Los criterios diagnósticos del SDRA son: o intoxicaciones med icamentosas.
Hallazgo de patología desencadenante.
Signos de insuficiencia respiratoria: taquipnea, cianosis central, uso
de músculos accesorios, etc. 16.2. Fisiopatología
Infiltrados alveolares difusos bilaterales (Figura 56).
Descarte de edema pulmonar cardiogénico (presión capilar pulmo-
nar o presión de enclavamiento normal ::; 18 mmHg) . Es una forma de edema pulmonar producido por aumento de la permea-
Cociente PaOjFi0 2 menor o igual a 200 mm Hg. bilidad de la membrana alveolocapilar al ser lesionada, bien de modo
Existe otra condición llamada daño pulmonar agudo, menos grave d irecto o indirecto.
pero que puede evolucionar hacia un SDRA, que se caracteriza por
los mismos criterios sa lvo el cociente PaOj Fi0 2 ::; 300 mmHg. El aumento de permeabilidad hace que el líqu ido intraalveolar sea muy
rico en proteínas, con act ivación de complemento de la coagulación y
de la respuesta inflamatoria. Este líquido interfiere con el surfactante, que
además ve alterada su síntesis cuantitativa y cualitativamente por lesión
de los neumocitos t ipo 11, por lo que se produce colapso alveolar.

Pueden formarse membranas hialinas como resultado del depósito de


fibrina y otras sustancias en el alvéolo. Todas estas alteraciones predo-
m inan en zonas decl ives. Debido al edema intersticial y al colapso alveo-
lar, los pulmones se vuelven ríg idos y aumenta el trabajo respiratorio. La
sobrecarga mecánica produce fatiga de los músculos respiratorios, con
disminución de los volúmenes ventilatorios.

Se producen importantes alteraciones de la ventilación/perfusión y, en


estadio avanzado, hay cortocircuito intrapulmonar por los alvéolos colap-
sados y áreas atelectasiadas.

Se describen tres fases en la historia natural de l SDRA:


Fase exudativa. Aparece en la primera semana y se caracteriza por
edema, lesión de neumocitos t ipo 1, aumento de concentración de
Figura 56. Síndrome de distrés respiratorio del adulto mediadores inflamatorios y formación de membranas hialinas.
Manual CTO de Medicina y Cirugía, 2.8 edición

Fase proliferativa . Aba rca desde el día 7 hasta el 21. En esta fase
Pensar en el SDRA cuando se hab le de un paciente
la mayoría de los pacientes se están recuperando. Histológ icamen- grave (seps is, en UCI, etc.) que no responde a oxíge-
te apa recen fenó menos de repa ració n tisular, con orga nizació n del no.
exudado alveo lar, proliferació n de neumocitos tipo 11 y sustitu ción
de neutrófi los por linfocitos. Los neumocitos tipo 11 resta blecen la
integ ridad del epite lio alveolar, reinician la síntesis del surfactante y
finalmente se diferencian hacia neumocitos tipo l. 16.4. Tratamiento
Fase fibrótica. La mayoría de los pacientes se rec uperan de la
fase proliferat iva, pero algu nos entran en una fase fi b rótica, que
supone mantenimiento de la ventilación mecánica y peor pro- Se basa en tratar el problema subyacente (sepsis, hipotensión, etc.) y la
nóstico. Se desarrol la una intensa fibro sis que altera la arqu itectu- insuficiencia resp iratoria. Para ello se dispone, por orden de gravedad, de:
ra pulmonar normal y se acompaña de proliferac ión de la íntima Oxigenoterapia, en concentraciones elevadas, y a ser posible, con
vascular con oclusión secundaria y, finalmente, hi pertensión pul- mascarilla de alto fluj o de efecto Venturi.
monar. Si fuera refractaria al tratami ento, se añadi ría el PEEP (positive end ex-
piratory pressure, presión positiva al fina l de la espiració n), mediante
ventilación mecá nica no invasiva, que aumenta el volumen pulmo-
16.3. Clínica nar y abre los alvéolos colapsa dos, di sm inuyendo el cortocircu ito.
Ventilación mecánica invasiva. La mayoría de los pacientes la requieren.
Se han empleado diferentes estrategias y distintos tratamientos. La única
El primer sig no en aparecer es la taqu ipnea, seg uida por d isnea, que van medida que en ensayos clínicos ha demostrado red ucir la mortalidad es
progresando con la en fermedad. la ventilación controlada por volumen y limitada por presión a volumen
corriente bajo (6 ml/kg). Parece que la reducción de mortalidad con esta
La radiografía de tórax muestra extensos infiltrados alveolointersticiales modalidad ventilatoria se relaciona con la minimización del daño pul-
difusos. Al principio, la hipoxem ia mejora con oxígeno, pero al avanzar monar inducido por ventilador al evitar la sobredistensión alveolar.
y desarrollar el cortocircuito pulmonar, deja de hacerlo y hay q ue inicia r No se dispone de evid encia firm e actualment e para recomendar el
ventilación mecá nica . uso rutinari o de PEEP elevadas, posición en decúbito prono (evita la
mayor perfu sió n a zonas declives donde au menta el shunt), o latera-
En cuanto al estudio hemodinámico, se observa un aumento de la pre- pia con oxigenación con membrana extracorpórea. Se recomienda
sión en la arteri a pulmonar, pero la presión de enclava miento es norma l, la restricció n de fluid os para reducir en lo posible el edema alveolar.
a diferencia de lo q ue ocurre en el edema pu lmonar cardiogénico, princi- Los corti coides no han demostrado beneficio, el surfacta nte no es
pa l entidad con la que se establece el diag nósti co diferencial. eficaz en el ad ulto (sí en el niño), y las prostag landinas y el óxido
nítrico, como fármacos vasod ilatadores, no parece que d isminuyan
La presencia de card iomega lia sugiere o rigen card iogén ico y, por ta nto, la morta lidad del SDRA.
va en contra del d iagnóstico de SDRA.
Las compl icaciones principa les del tratamiento son el barotrauma, la toxi-
cidad por el oxígeno y las neumonías nosocom iales. La morta lidad sigue
siendo alta, pero se observa un descenso en los últimos años. Los pacien-
Para el diagnóstico de SDRA se debe descartar el ede- tes que superan el episod io ag udo suelen quedar libres de secuelas, pero
ma de pulmón de origen ca rdiogéni co.
unos pocos desarrollan como secuela neumonía intersticial no específica .
Recuerda Estos pacient es muestran con mayor frec uencia una DLCO reducida.

" En la Rx de tórax aparece un patrón alveolointersticial difuso


Ideas clave RS ("pulmón blanco").

" Se produce por un aumento de la permeabilidad de la membra- " Existe hipoxemia grave (cociente Pa0/ Fi0 2 < 200 mmHg) con
na alveolocapilar. escasa respuesta a la oxigenoterapia (efecto shunt).

" La sepsis es la causa más frecu ente.

¿Qué diagnóstico, entre los siguientes, es el más probable?


Casos clínicos .
1) Neumonía nosocomial.
Un paciente ingresado por pancreatitis aguda comienza con ta- 2) In suficiencia card ía ca.
quipnea, taquicardia, sudoración y cianosis progresivas. La Pa0 2 3) Distrés respiratorio del adulto.
es de 55 mmHg, la Rx de tórax muestra infiltrados alveolares bi- 4) Embolia grasa.
laterales y la presión de enclavamiento capilar pulmonar es nor-
mal. El aporte de oxígeno suplementario no mejora la situación. RC: 3

16 · Síndrome de distrés respiratorio agudo


__Neumolog__..__ía_________

. ,
Ventilación mecan1ca

ORIENTACIÓN 17 .1. Ventilación mecánica


ENARM Tema poco relevante. .
no mvas1va
.

La ventilación mecánica no invasiva (VMN I), en la actua lidad, se circuns-


Pa ra que haya intercambio entre los gases de la atmósfera y el alvéolo, es cribe casi exclusivamente a la ventilació n con presión positiva aplicada
necesa ri o q ue se generen unos gradientes de presión cíclicos. En cond i- a través de mascarilla nasa l o fa cial (Figu ra 58). Existen varios tipos de
ciones normal es, éstos se prod ucen por la contracción de los músculos ventiladores:
respiratorios. Al contraerse los múscu los inspiratorios, d ism inuye la pre-
sión intraalveolar por debajo de la atmosférica, generándose un flujo de
aire desde la atmósfera al alvéolo.

Durante la espiración, por retracció n elástica pasiva del parénquima pul-


monar y la caja torácica, la presión intraalveolar aumenta por encima de
la atmosférica, lo que genera un flujo aéreo del alvéolo a la atmósfera.

Cuando se altera este flujo de gases entre atmósfera y alvéolo, o cuando


lo está el intercambio gaseoso alveolocapilar (a lteración de la d ifu sión,
de la ventilación/perfusión), se recurre a los respi radores, q ue insuflan
una mezcla gaseosa, enriq uecida o no en oxígeno, en la vía aérea del pa-
ciente, a través de un tubo endotraq uea l o una cá nula de traq ueostomía
(Figura 57) (ventilación mecán ica invasiva) o mediante d ispositivos que
no req uiera n la creación de una vía aérea artific ial (ventilación mecánica
no invasiva). Los gases son espirados de forma pasiva. Figura 58. Tipos de ventiladores y mascarillas

Ventilación ciclada a volumen. Se prefija el volumen que se debe


Cartílago tiroides
Cartílago aritenoides adm inistrar al paciente en cada cic lo ventilatorio, así como la fre-
cuencia mínima de los ciclos. Es de elección en la ventilación do-
miciliaria de pacientes con alteraciones restrictivas (enfermedades
Ligamento neuromusculares, alteracio nes de caja torácica), ya que se asegura la
cricotiroideo ~~­
ventilación en caso de apnea.
medio
Ventilación ciclada a presión. Se genera un nivel de presión cons-
tante durante toda la inspiración tras detectar un esfuerzo inspiratorio
del paciente, y se puede mantener también un cierto nivel de presión
du rante la espiración. Es posible lijar una frecuencia mínima de ciclos
respiratorios. Es de elección en el ám bito hospitalario en pacientes
con insuficiencia respiratoria ag uda, debido a la comodidad para el
paciente, y la limitación de presió n ejercida sobre la vía aérea.
Presión positiva continua en la vía aérea (CPAP). Estrictamente
no es un método de VMNI, ya que la respiració n es completamente
espontánea, pero a un nivel de presión q ue se puede prefijar.

Figura 57. Técnica de traqueostomía La VM NI se puede realizar en el ámbito hospita lario o en el domiciliario.
Manual CTO de Medicina y Cirugía , 2.8 ed ic ión

Ventilación mecánica no invasiva indicación sería el síndrome de apnea del sueño con hipoventilación per-
sistente a pesar del tratamiento con CPAP.
hospitalaria
Los objetivos de la VMN I domiciliaria son: mejorar la calidad de vida, dis-
El objetivo es consegu ir un soporte venti latorio eficaz en los pacientes minu ir el número de ingresos hospitalarios y en lentecer la progresión de
con insuficiencia resp iratoria aguda. la enfermedad crón ica . Sin embargo, no está todavía demostrado que
Ventajas: evitar la intubación, mejor tolerancia, preservación de los aumente la supervivencia .
mecanismos de defensa de la vía aérea y, si se utiliza mascarilla nasal,
se preserva el habla y la deglución.
Limitaciones: es necesaria la colaboración por parte del paciente, 17 .2. Ventilación mecánica invasiva
por lo que es necesario un nivel de alerta adecuado, fugas aéreas
alrededor de la máscara, falta de acceso directo a la vía aérea (difícil,
por tanto, en pacientes con secreciones abundantes), aerofagia, le- Moda lidades de venti lación
siones cutáneas por la mascarilla.
Indicaciones: Depend iendo de si puede o no participar el paciente de forma activa,
EPOC agudizada: es la indicación mejor establecida. Se utiliza existen d istintas modalidad es:
cuando existe acidosis respiratoria moderada (pH > 7,20/7,25). Venti lación mandataria continua (CMV). La insuflación se produce
Mejora el patrón ventilatorio y la disnea . Dism inuye la necesidad con una periodicidad impuesta por el resp irador.
de intubación orotraqueal y la morta lidad. Ventilación mandataria asistida (AMV). La insuflación se produce en
Edema agudo de pulmón cardiogénico grave: aumenta la respuesta a iniciativas inspiratorias del paciente, detectadas por sen-
presión intratorácica, disminuyendo la precarga y la poscar- sores regulables del respirador.
ga, y aumenta la capacidad residual funcional, mejorando la Ambas requieren una buena adaptación a la máquina, y se suelen
oxigenación y disminuyendo el trabajo respiratorio. Tanto la emplear fármacos para sedación y/o relajación. Las complicaciones
CPAP como la VMNI acortan el tiempo de recuperación y d is- típicas de estas modalidades son el barotrauma y la incapacidad para
minuyen la necesidad de intubación orotraquea l y, por tanto, desconectar al paciente del ventilador, pues aportan una asistencia
mejoran la supervivencia. En general la eficacia de la CPAP y completa en cada inspiración .
de la VMNI es simi lar, aunque se eleg irá esta última si existe Ventilación mandataria intermitente sincron izada (SIMV). Permite la
hipercapnia. ventilación espontánea del paciente y la complementa con ciclos
Insuficiencia respiratoria hipoxémica grave: es una indicación prefijados de venti lación con presión positiva intermitentes (IPPV)
más discutida. Algunos estudios han mostrado eficacia de la que no coinciden con los espontáneos. Las presiones obtenidas en
VMNI junto con elevada Fi0 2 en la insuficiencia respiratoria hi- la vía aérea son menores que con la IPPV simple (CMV y AMV). Per-
poxémica grave secundaria a: 1) inmunodeprimidos con fiebre mite real izar la desconexión del ventilador, pero un problema impor-
e infiltrado pulmonar, 2) neumonía, 3) cirugía torácica o abdo- tante es la posibilidad de asincronía entre el paciente y la máquina.
minal alta. Presión positiva espiratoria final (PEEP). Se emplea en ciertas enfer-
Ay uda a la retirada de la venti lación mecánica invasiva: la VMN I medades como el síndrome de distrés respiratorio agudo, que tie-
puede ayudar a la extubación precoz, sobre todo, en pacientes nen tendencia al colapso alveolar, pues aporta una pequeña presión
con EPOC agud izada. al fina l de la espiración para evita r el cierre de la vía aérea. Con eso se
consigue aumentar la CRF y disminuir el cortocircuito intrapulmonar.
Ventilación mecánica no invasiva Presión positiva continua en la vía aérea (CPAP). Es el equivalente al
domiciliaria PEEP. cuando el paciente respira espontáneamente de forma par-
cial o completa. Se emplea para evitar el colapso de la vía aérea, por
ejemplo, en el SAOS.
La VMNI nocturna mejora el intercambio de gases diurno en pacientes
con enfermedades neuromusculares, alteraciones de caja torácica y, en Tipos de respiradores
general, hipoventilación de origen extrapulmona r. No solo mejora la Pa0 2
y la PaC0 2 diurna, sino tambi én los síntomas asociados como cefalea ma- Se dice que un respirador cicla cuando deja de insuflar aire para permitir
tutina e hipersomnolencia diurna, reduciendo, además, el número de in- la espiración pasiva del paciente. Según ello, se pueden distingu ir tres
gresos hospita larios. tipos de respiradores:
Ciclados por presión (manométricos). Insuflan aire hasta alcanzar
Está indicada, por tanto, en pacientes con alteración ventilatoria restricti- una presión prefijada en la vía aérea .
va que curse con hipercapnia y aparezcan síntomas atribuibles a la hipo- Ciclados por volumen (volumétricos). Insuflan un volumen prefijado
ventilación nocturna. Las enfermedades en las que con más frecuencia de aire.
se utiliza son: enfermedades neuromuscu lares crón icas lentamente pro- Ciclados por tiempo. Se comportan como volumétricos, es decir,
gresivas (d istrofias musculares, secuelas pospolio, esclerosis múltiple), al- que insuflan hasta un determinado volumen, pero prefijándose el
teraciones de caja torácica y síndrome de obesidad-h ipoventilación . Otra volumen minuto y la frecuencia de insuflación .

17 · Ventilación mecánica
Neumología •

Recommended reading 1

50-year-old patient, who smokes two packs of cigarettes a day since From t he radio logica l standpoint, w hich is w hat m ost interests us here,
he was young, with a history of frequent respiratory infections, visits it is essenti al fo r you to lea rn t o recog ni se t he characterist ic facts of
the physician beca use he has presented productive cough for the past chronic bronchitis, and, especially, t o disting uish t hem from the X-rays
few years, with not very abundant purulent sputum. Moreover, he of patients with emph ysem a. Below, w e offe r you an example of each
reports mild dyspnoea. Haematocrit 57%, PaC0 2 54 mmHg, Pa0 2 58 case, with t he m ost importa nt rad iolog ical charact eristics, in the form
mmHg anda chest X-ray su eh as the one shown below. The patient will of atable.
most li kely suffer from one of the following [Figure 1a]:
In th is patient's X-ray, we find a certain th ickening of the bronchia l w alls
1. Pulmonary emphysema . and increased bronchovascular markings, w hich is sometimes described
2. Chronic bronchitis. as a "dirty chest". No cardiomegal y is observed.
3. Asthma .
4. lnterstitiallung disease. Fuente: Ga rcía Maca rrón J Casos clínicos en imágenes. Mad rid. CTO Edi-
5. Sarcoidosis. toria l, 2012.

The history of tobaccoism should make


us suspect COPO in the fi rst place.
On the other ha nd, t he predomi nant
symptom is coughi ng w ith expectora-
tion. Solely on t he basis of th ese data,
even without analysing the X-ray, we
cou ld suspect t hat it is probably a case
of chronic bronchitis, and not emp hy-
sema. Remember that, in pati ents w it h
emphysema, dyspnoeic symptom s
predominare; this is w hy it is described
as "PP, pink puffer': On t he contrary, pa-
tients with chronic bronchitis are de-
fined as BB in Anglo-Saxon literature Figure 1b. Ch ronic bronch itis. Fig ure 1c. Emphysema
(blue and bloated). Hyperinsufflation (less than in emphysema) Sign ificant hyperinsufflation. lncreased height
of the right lung, flattening of the diaphragm,
horizontalisation of the rib cage, increased
retrosterna l clear space
Th icken ing of the bronchial walls Oligohaemia (hyperclear lung)
lncreased bronchovascular ma rkings, Sabre sheath" trachea
especially at the bases
Cardiomegaly. Chron ic corpulmona/e more frequen t Reduction of the card iac si lhouette
than in emphysema ("tear-shaped" heart), except when there is chron ic
cor pulmona/e
Evidence of pul monary hypertension Evidence of pu lmonary hypertension
(most frequent) (less frequent)
Figure la
• Neumología

Recommended reading 2

A 35-year-old patient visits the Emergency Room dueto dyspnoea.


She reports a history of bronchial asthma, but she has never requi-
red mechanical ventilation . In the past few days, she has presented
wheezing dyspnoea, and for the past 12 hours she has used her sal-
butamol inhaler about 10-15 times. In the examination, she appears
to have good perfusion and colouring, a heart rate of 130 bpm and
a respiratory rate of 35 rpm . She has supraclavicular and intercos-
tal retraction. Blood pressure 140/80 mmHg. A paradoxical pulse
is observed. Pulmonary auscultation: overall decrea sed vesicular
sounds. The chest X-ray has the appearance shown below. Hardly
any wheezing is heard. The cardiac auscultation is anodyne. Baseline
arterial gasometry: pH 7.46, pC0 2 44 mmHg, p0 2 58 mmHg, bicar-
bonate 16 m mol/ l. Which of the following approaches is the most
adequate? [Figure 2a]:

l. lt is a mild asthma attack. We should add inhaled steroids, and


refer the patient to externa! consultations for medical follow-up.
2. lt is a case of abuse of betamimetics. We should advise against its
abusive use and associate a xanthine or ipratropium. Figure 2a
3. These are symptoms of anxiety
caused by abuse of betamimetics. Mild Moderate lmminent arrest
The dose must be adjusted to the
Dyspnoea When walking, may lie When talking At rest
recommended ones anda benzo-
down
diazepine should be added.
Speech Paragraphs Phrases Words
4. lt is a severe asthma attack that
may require mechanical ventila- Wheezing Moderate, tele-expirato ry Intense Intense Silence upon
tion immediately.
Use of accessory No Yes Yes Paradoxical resp iration
5. lt is a case of somatisation, for muscles
which reason a psychiatric inter-
Levelof Norma l Norma l/agitated Ag itated Reduced/confused
na! consultation should be re-
consciousness
quested.
Respiratory rate lncreased lncreased > 30
From the rad iolog ical standpoint, t he Heart rate 60-100 100-120 > 120 Bradycardia
most frequent find ing in an asthma FEV,fPEF* >80% 60%-80% <60%
attack is a norma l chest X-ray, both du-
0 2 saturation >95% 91%-95% <90%
ring the stable phase and during t he
Pa0 2 > 60 mmHg <60 mmHg
attacks. However, in t he case of a seve-
re attack, we m ay find evidence of tho- PaC0 2 <45 mmHg > 45 mmHg
racic hyperinsufflation. In t his case, we Paradoxical pulse Absent (< 1O) 10-25 mmHg > 25 mmHg Absent (muscle fatigue)
may find horizontalisation of the ri bs,
* PEF mea su red fol lowing the administration of a bronchodilator
widening of the intercosta l spaces and
pu lmonary hypertranspa rency. Figura 2b. Classification of the exacerbation of asthma according to its severity.
ía__________
_N_e_umolo_gf-L--

Recommended reading 2

However, the severity of the symptoms is not defined by the radiolo- chanical ventilation, since she is incapable of maintaining it by herself
gical manifestations, at least not prim ari ly. What shou ld rea lly ca ll our (answer no. 4 correct).
attention are the gasometric alterations. As you know, in an asthma
atta ck, due to the bronchospasm, there is a compensatory response, Other signs of severity that we find in this case are the paradoxical pul-
which is hyperventilation. For this rea so n, we would expect a decrease se and the use of the accessory muscles (supraclavicula r and intercostal
in PaC0 2. However, this patient has the PaC0 2 level at 44 mmHg, i.e. in retraction). In the auscu ltation, the most cha racteristic manifestation of
the upper limit of normal. This means that her respiratory mu scles are asthma is wheezing, normally expiratory wheezi ng. When the obstruc-
beginning to yield. When the respiratory muscles have been hype rve n- tion is severe, it may be barely audible and may even disappear.
ti lating for some time, they end up exhausted and this is reflected in
the gasometry, beca use the PaC0 2 is normalised and, if we do not do Fuente: Ga rcía Maca rrón J. Casos clínicos en imágenes. Mad rid. CTO Edi-
anything, it wil l continue to rise. Consequently, the patient requires me- torial, 2012.
Manual CTO de Medicina y Cirugía, 2.a edición

Bibliografía
Neumología

óiJ Capítulo 02. Malformaciones Hauser SL, Longo DL, Jameson JL, Loscalzo J, (eds.). McGraw-
Verea Hernando H. Malformaciones broncopulmonares. En: Hill, New York. 2008; 1643-1 651.
Farreras-Rozman Medicina Interna, 16.• ed. Ausina Ruiz V, Car- Xaubert Mir A, Morell Brotad F. Enfermedades intersticiales
mena Rodríguez R, Carreres Molas A, et al, (eds.). Elsevier Es- difusas del pulmón. En: Farreras-Rozman Medicina Interna,
paña, Barcelona. 2008; 776-779. 16.• ed. Ausina Ruiz V, Carmena Rodríguez R, Carreres Molas
A, et al, (eds.). Elsevier España, Barcelona. 2008; 810-822.
óiJ Capítulo 03. Fisiología y fisiopatología
Roca Torrent J. Exploración de la función respiratoria. En: óiJ Capítulo 08. Enfermedades por inhalación de polvos
Farreras-Rozman Medicina Interna, 16.• ed. Ausina Ruiz V, Car- Kline JN, Hunninghake GW. Hypersensitivity Pneumonitis
mena Rodríguez R, Carreres Molas A, et al, (eds.). Elsevier Es- and Pulmonary lnfiltrates with Eosinophilia. En: Harrison's
paña, Barcelona. 2008; 716-721. Principies of Interna/ Medicine, 17th ed. Fauci AS, Braunwald
Rodríguez-Roisín R. Insuficiencia respiratoria. En: Farreras- E, Kasper DL, Hauser SL, Longo DL, Jameson JL, Loscalzo J,
Rozman Medicina Interna, 16.• ed. Ausina Ruiz V, Carmen a (eds.). McGraw-Hill, New York. 2008; 1607-161 1.
Rodríguez R, Carreres Molas A, et al, (eds.). Elsevier España, Orriols Martínez R, Costa Solá R. Enfermedades pulmona-
Barcelona. 2008; 725-734. res de origen ocupacional. En: Farreras-Rozman Medici-
Weiberger SE, Rosen IM. Alteraciones en la función respirato- na Interna, 16.• ed. Ausina Ruiz V, Carmen a Rodríguez R,
ria. En: Harrison. Principios de Medicina Interna, 16.• ed. Fauci Carreres Molas A, et al, (eds.). Elsevier España, Barcelona.
SA, Braunwald E, Kasper DL, Hauser SL, Longo DL, Jameson 2008; 822-828.
LJ, Loscalzo J, (eds.). McGraw-Hill/ lnteramericana, México Speizer FE, Salmes JR. Environmental Lung Diseases. En:
DF. 2009; 1586-1 592. Harrison's Principies of Interna/ Medicine, 17th ed. Fauci AS,
West JB, et al. Pulmonary Pathophysiology. The essentials. Braunwald E, Kasper DL, Hauser SL, Longo DL, Jameson JL,
West JB, ed. Lippincott -Williams and Wilkins, Baltimore. 2007. Loscalzo J, (eds.). McGraw-Hill, New York. 2008; 161 1-161 9.
West JB, et al. Respiratory Physiology. The essentials. West JB,
ed. Lippincott -Williams and Wilkins, Baltimore. 2008. óiJ Capítulo 09. Eosinofilias pulmonares
Ancochea Bermúdez A. Eosinofilias pulmonares. En: Farreras-
óiJ Capítulo 04. Enfermedad pulmonar obstructiva crónica (EPOC) Rozman Medicina Interna, 16.• ed. Ausina Ruiz V, Carmen a
Barberá M ir JA, Agustí García-Navarro A, Rodríguez-Roisín R. Rodríguez R, Carreres Molas A, Darnea TeyA, et al, (eds.). Else-
Enfermedad pulmonar obstructiva crónica. En: Farreras-Roz- vier España, Barcelona. 2009; 828-832.
man Medicina Interna, 16.• ed. Ausina Ruiz V, Carmena Rodrí- Kline JN, Hunninghake GW. Hypersensitivity Pneumonitis
guez R, Carreres Molas A, et al, (eds.). Elsevier España, 2008; and Pulmonary lnfiltrates with Eosinophilia. En: Harrison's
Barcelona. 742-753. Principies of Interna/ Medicine, 17th ed. Fauci AS, Braunwald
Global lnitiative for Chronic Obstructive Lung Disease E, Kasper DL, Hauser SL, Longo DL, Jameson JL, Loscalzo J,
(GOLD). Global strategy for the diagnosis, management and (eds.). McGraw-Hill, NewYork. 2008; 1607-1611.
prevention of chronic obstructive pulmonary disease. Upda-
ted 2009. Disponible en: http://www.goldcopd.com óiJ Capítulo 1O. Síndromes de hemorragia alveolar difusa
Reilly JJ, Silverman EK, Shapiro SD. Chronic Obstructive Pul- Barberá Mir JA, Ruiz Manzano F. Hemorragias pulmonares
monary Di sea se. En: Harrison's Principies of Interna/ Medicine, difusas. Enfermedades vasculares del pulmón. En: Farreras-
17th ed. Fauci AS, Braunwald E, Kasper DL, Ha u ser SL, Long o Rozman Medicina Interna, 16.• ed. Ausina Ruiz V, Carmena
DL, Jameson JL, Loscalzo J, (eds.). McGraw-Hill, 2008; New Rodríguez R, Carreres Molas A, et al, (eds.). Elsevier España,
York. 1635-1643. Barcelona. 2008, 842-844.

óiJ Capítulo OS. Asma óiJ Capítulo 11. Sarcoidosis


Barnes P. Asthma. En: Harrison's Principies oflnternal Medicine, Baughman RP, Lower EE. Sarcoidosis. En: Harrison's Principies
17th ed. Fauci SA, Braunwald E, Kasper DL, Hauser SL, Longo of Interna/ Medicine, 17th ed. Fauci AS, Braunwald E, Kasper
DL, Jameson LJ, Loscalzo J, (eds.). McGraw-Hill, 2008; New DL, Hauser SL, Longo DL, Jameson JL, Loscalzo J, (eds.). New
York. 1596-1606. York. McGraw-Hill, 2008; 2135-2142.
http://www.ginasthma.com/Guide lineitem. Mañá Rey J. Sarcoidosis. En: Farreras-Rozman Medicina Interna,
asp??l1 =2&12=1 &intld=1 561 16.• ed. Ausina Ruiz V, Carmena Rodríguez R, Carreres Molas A,
http://www.gemasma.com/descarg as/GEMA%202009.pdf et al, (eds.). Elsevier España, Barcelona. 2008; 1143-1 147.

óiJ Capítulo 06. Bronquiectasias óiJ Capítulo 12. Hipertensión pulmonar


De Gracia Roldán J, Girón Moreno RM. Bronquiectasias y Galie N, Hoeper MM, Humbert M, et al. Guidelines for the
otras enfermedades obstructivas de las vías aéreas. En: Farre- diagnosis and treatment of pulmonary hypertension. Eur
ras-Rozman Medicina Interna, 16.• ed. Ausina Ruiz V, Carmena HeartJ 2009; 30: 2493-2537.
Rodríguez R, Carreres Molas A, et al, (eds.). Elsevier España, Rich S. Pulmonary hipertensión. En: Harrison's Principies of In-
Barcelona. 2008; 753-757. terna/ Medicine, 17th ed. Fauci SA, Braunwald E, Kasper DL,
Tino G, Weimberger SE. Bronchiectasis and Lung Abscess. Ha u ser SL, Longo DL, Jameson LJ, Loscalzo J, (eds.). McGraw-
En: Harrison's Principies oflnternal Medicine, 17th ed. Fauci AS, Hill, New York. 2008; 1576-1 581 .
Braunwald E, Kasper DL, Hauser SL, Longo DL, Jameson JL,
LiJ Capítulo 13. Tromboembolismo pulmonar
Loscalzo J, (eds.). McGraw-Hill, New York. 2008; 1629-1632.
Goldhaber S. Pulmonary thromboembolism. En: Harrison's
óil Capítulo 07. Enfermedades pulmonares intersticiales Principies of Interna/ Medicine, 17th ed. Fauci AS, Braunwald
King TE. lnterstitial Lung Diseases. En: Harrison's Principies of E, Kasper DL, Hauser SL, Longo DL, Jameson JL, Loscalzo J,
Interna/ Medicine, 17th ed. Fauci AS, Braunwald E, Kasper DL, (eds.). McGraw Hill, NewYork. 2008; 1651-1657.

Bibliografía
Neumo1ogí a 1 Bibliografía

Bibliografía
Neumología

Kearson C, Khan SR, Agnelli G, Goldhaber S, Raskob GE, Co- LJi1 Capítulo 16. Síndrome de distrés respiratorio agudo
merota AJ. Antithrombotic Therapy for Venous Thromboem- Levy BD, Shapiro SD. Acute respiratory distress syndrome.
bolic Disease: American College of Chest Physicians Eviden- En: Harrison's Principies of Interna/ Medicine, 17th ed. Fauci AS,
ce-Based Clinical Practice Guidelines (8th edition). Chest Braunwald E, Kasper DL, Hauser SL, Longo DL, Jameson JL,
2008; 133: 454-545. Loscalzo J, (eds.). McGraw Hill, New York. 2008; 1680-1684.
Torbicki A. Perrier A, Konstantinides S, et al. Guidelines on the Torres Martí A. Síndrome del distrés respiratorio agudo. En:
diagnosis and management of acute pulmonary embolism. Farreras-Rozman Medicina Interna, 16.• ed. Ausina Ruiz V, Car-
Eur Heart J 2008; 29: 2276-2315. mena Rodríguez R, Carreres Molas A, et al, (eds.) . Elsevier Es-
paña, Barcelona. 2008; 735-739.
óJ Capítulo 14. Trastornos de la ventilación
Phillipson EA. Disorders of ventilation. En: Harrison's Princi- ¡ji Capítulo 17. Ventilación mecánica
pies of Interna/ Medicine, 17th ed. Fauci AS, Braunwald E, Kas- Ferrer Monreal M, Roca Torrent J. Ventilación artificial. En:
per DL, Hauser SL, Longo DL, Jameson JL, Loscalzo J, (eds.). Farreras-Rozman Medicina Interna, 16.• ed. Ausina Ruiz V, Ca r-
McGraw-Hill, New York. 2008; 1661-1665. mena Rodríguez R, Carreres Molas A, et al, (eds.). Elsevier Es-
paña, Barcelona. 2008; 739-742.
¡ji Capítulo 15. Síndrome de la apnea del sueño lngenito EP. Mechanical Ventilatory Support. En: Harrison's
Douglas NJ. Sleep apnea. En: Harrison's Principies of Interna/ Principies of Interna/ Medicine, 17th ed. Fauci AS, Braunwald
Medicine, 17th ed. Fauci AS, Braunwald E, Kasper DL, Hauser E, Kasper DL, Hauser SL, Longo DL, Jameson JL, Loscalzo J,
SL, Longo DL, Jameson JL, Loscalzo J, (eds.). McGraw-Hill, (eds.). McGraw-Hill, New York. 2008; 1684-1688.
New York. 2008; 1665-1668.
Monserrat Canal JM, Durán Cantolla J. Síndrome de apneas ¡ji Bibliografía general
durante el sueño. En: Farreras-Rozman Medicina Interna, 16.• Grupo CTO. Manual ao de Neumología y cirugía torácica. 9.•
ed. Ausina Ruiz V, Carmen a Rodríguez R, Carreres Molas A, et ed. CTO Editorial, Madrid, 2014.
al, (eds.). Elsevier España, Barcelona. 2008; 779-784.
Manual CTO
de Medicina y Cirugía

2. 8 edición

Cirugía torácica

Revisores
josué Daniel Cadeza Aguilar
jesús lván García Rivera
David Calleja Crespo

Autores
Jorge Castelao Naval
jesús Fernández Francés

Grupo CTO
• • Editorial
NOTA

La medicina es una ciencia sometida a un cambio constante. A medida que la investigación y la experiencia
clínica amplían nuestros conocimientos, son necesarios cambios en los tratamientos y la farmacoterapia.
Los editores de esta obra han contrastado sus resultados con fuentes consideradas de confianza,
en un esfuerzo por proporcionar información completa y general, de acuerdo con los criterios aceptados
en el momento de la publicación. Sin embargo, debido a la posibilidad de que existan errores humanos
o se produzcan cambios en las ciencias médicas, ni los editores ni cualquier otra fuente implicada
en la preparación o la publicación de esta obra garantizan que la información contenida en la misma sea
exacta y completa en todos los aspectos, ni son responsables de los errores u omisiones ni de los resultados
derivados del empleo de dicha información. Por ello, se recomienda a los lectores que contrasten dicha
información con otras fuentes. Por ejemplo, y en particular, se aconseja revisar el prospecto informativo
que acompaña a cada medicamento que deseen administrar, para asegurarse de que la información
contenida en este libro es correcta y de que no se han producido modificaciones en la dosis recomendada
o en las contraindicaciones para la administración. Esta recomendación resulta de particular importancia
en relación con fármacos nuevos o de uso poco frecuente. Los lectores también deben consultar
a su propio laboratorio para conocer los valores normales.

No está permitida la reproducción total o parcial de este libro, su tratamiento informático, la transmisión
de ningún otro formato o por cualquier medio, ya sea electrónico, mecánico, por fotocopia, por registro
y otros medios, sin el permiso previo de los titulares del copyright.

© CTO EDITORIAL, S.L. 2016

Diseño y maquetación: CTO Editorial

C/ Francisco Silvela, 106; 28002 Madrid


Tfno.: (0034) 91 782 43 30 - Fax: (0034) 91 782 43 43
E-mail: ctoeditorial@ctomedicina.com
Página Web: www.grupocto.es

ISBN Cirugfa torácica: 978-84-16527-27-4


ISBN Obra completa: 978-84-16527-11-3
Depósito legal: M-26880-2015
Cirugía torácica

Manual CTO
de Medicina y Cirugía

2.a edición

Grupo CTO
• • Editorial
Q)
u
·-
-e
e
,_
04. Neoplasias pulmonares .
4.1 .
4.2.
4.3 .
Tumores malignos . ............................................................. . ...........
Tumores pulmonares metastásicos .................. .......
Nódulo pulmonar solitario .
13
13
19
19
4.4. Tumores benignos . .......... ............................................ ..................... 20

05. Trasplante de pulmón ... 22


5.1.1ndicaciones y contraindicaciones . .. .. . .. 22
01. Enfermedades de la pleura 1
1.1. Derrame pleuraL.....
1.2. Neoplasias pleurales primarias ..... 4 06. Traumatismos torácicos . . 25
1.3. Neumotórax. .... S 6.1. Reconocimiento primario de lesiones
con compromiso vital inmediato ......................................... 25
6.2. Reconocimiento secundario
02. Enfermedades del mediastino ... 8 de las lesiones torácicas . ....... ........ .. . .. .... .. 26
2.1. Masa mediastínica ..... 8
2.2. Infecciones... ................................................... . .. .. .. ... 10
Bibliografía ... 33
2.3 . Neumomediastino ..... . 10

03. Enfermedades del diafragma .. 12


3.1 . Parálisis del diafragma . . .... 12
3.2. Hernias diafragmáticas ...................................................................................... 12
Cirugía torácica •

Enfermedades de la pleura

DRifNI/\CIDN Tema muy importante. Contiene varias cifras que es necesario saber (criterios de exudado,
derrame pleural metaneumónico complicado, indicación de tubo de drenaje en el neumotórax).
ENARM Se debe conocer bien el derrame pleural tuberculoso.

1.1. Derrame pleural

El derrame pleural (DP) consiste en la acumulación de líquido en el es-


pacio pleura l. Surge cuando la síntesis de líquido pleura l (LP) es mayor
que la reabsorción . En condiciones norma les el líqu ido entra al espacio
pleural desde los capilares de la pleura parietal, y es reabsorbido por los
linfáticos de la pleura parietal. El líquido puede entrar también desde el
espacio intersticial a través de la pleu ra visceral o desde la cavidad peri-
toneal, atravesando pequeñas soluciones de co ntinuidad del diafragma.

Radiológicamente se sospecha si apa rece alguna de las siguie ntes mani-


festaciones: lo más frecuen te es el borramiento u obliteración del ángulo
costofrén ico posterior en la radiografía de tórax latera l (visible con aproxi-
madamente 75 m i); y el borra m iento del ángulo cost ofrén ico lateral en la
proyección posteroanterior (precisa unos 150 m i); o lo más característico,
que es la opacidad de la base pul monar con una línea cóncava superior
que mira hacia el pulmón, denominada menisco de Damoisseau (precisa Figura 1. Derrame pleura l izq uierdo. Cu rva de Damoisseau
mayor cantidad de líqu ido) (Figura 1). Otras manifestaciones típicas son
el derrame subpu lmonar, el derrame encapsulado o incluso el derrame
masivo. Cuando se sospecha, se confi rma si es o no líquido libre con una
proyección en decúbito lateral sobre el lado afectado (Figura 2). Si el
líquido se deposita en la zona más declive, entonces se d ice que es li bre.
Si no se moviliza, se habla de derrame loculado, o encapsu lado, y la mejor
forma de localizarlo es con la realizac ión de ecografía torácica.

Clínicamente, los pacientes con derrame pleural suelen tener dolor pleurí-
tico por irritación de las terminaciones sensitivas que hay en la pleura pa-
rietal, y cuando el acúmulo de líquido es importante, presentan disnea. Si
la etiología es infecciosa, suele haber fiebre. En la auscultación pulmonar se
puede encontrar hipofonesis, reducción en la t ransmisión de las vibraciones
vocales y el típico roce pleura l. En la percusión torácica encontrará matidez.

Una vez confirmado el derrame pleural, el d iagnóstico etiológico co-


mienza con la rea lización de una toracocentesis (Figura 3), mediante la
que se obtiene líquido que se remite a los laboratorios de bioqu ím ica,
microbiolog ía y citología. Figura 2. Derrame pl eural izq uierd o. Radiog rafía en decúbito lateral
Manual CTO de Medicina y Cirugía, 2. 8 edición

Las tres Ten el derrame hemático: Tumor, Tra uma y


TEP.

Cuando el aspecto del líqu ido obtenido en la toracocentes is es sa ngu i-


Vena Zona de seguridad nolento, se mide el hematocrito. Si es menor del 1%, no tiene ningún
Arteria sign ificado orientativo, y si es mayor del 1% (derrame pleural hemático),
Nervio orienta a derrame ma ligno, traumático o porTEP Una entidad específica
es el hemotórax, que se define como un derrame con hematocrito mayor
o igual aISO% del va lor en sangre (la causa más frecuente es el traumatis-
Externo]
Interno Músculos intercostales mo), es indicación siempre de tubo de drenaje torácico y de cirugía si se
Int imo objetiva un sangrado superior a 200 mi/h.

Análisis celular. Cua ndo h ay más de 10.000 leucocitos/m m 3 suele


ser un derrame paraneu mónico o empiema. Es más úti l el porcenta-
Figura 3. Técnica de toracoce ntesis je celular que el número tota l. Los leucocitos son fundamenta lmente
neutrófilos cuando el proceso es agudo (neumonía, embolismo, pan-
Análisis bioquímico. Lo primero es la distinción de los derrames en exuda- creat itis, tubercu losis inicial) y mononucleares cuando es crónico. Si el
dos y trasudados. Los trasudados surgen si no funcionan adecuadamente número de linfocitos es mayor del 50%, orienta a origen tuberculoso
los factores sistém icos que influyen en la síntesis y reabsorción del líquido, o tumoral.
en tanto que los exudados obedecen a alteración de factores loca les. La
causa más común de derrame en los pa íses desa rrollados es la insuficiencia El porcentaje de eosinófi los es útil en el d iagnóstico diferencial, pues la
cardíaca (trasudado). Otras causas de t rasudado son la cirrosis hepática y mayoría de pacientes con eosinófil os > 10% t ienen o aire o sangre en el
el sínd rome nefrótico. Si se consideran ú nica m ente los exudados, la ca usa espacio p leural, o b ien la ca usa es el asbesto, fá rmacos (n it rofurantoína),
más común son los derrames paraneu mónicos, los derrames ma lignos, las parásitos (paragonomiasis) o síndrome de Chu rg-Strauss. En el tubercu-
infecciones víricas y el TEP En algunos países, aún siendo desarrollados, el loso y el tumoral (salvo la enfermedad de Hodgkin), es muy raro encon-
exudado tuberculoso es muy frecuente, sobre todo, en jóvenes. El derrame trarlos. El p rimer estudio citológico es positivo para malignidad en más
se ca lifica de exudado cuando cumple al menos uno de los criterios de del 60% de casos de los derrames malignos, sobre todo, en los adeno-
Light. Si no se cumple ninguno, se cal ifica de trasudado (Tabla 1). carc inomas.

Análisis microbiológico. Es importante la t inción de Gram y la determi-


Proteínas en LP/proteínas en suero> 0,5. nación de BAAR (bacilos ácido-a lcohol resistentes) en el líqu ido, así como
LDH en LP/LDH en suero> 0,6. el cu ltivo en los medios adecuados, aunq ue la sensibilidad del cu ltivo
LDH en LP m ayor de los 2/3 del lím ite su perior de la normalidad para el d iag nóstico de t u bercu losis es baja.
de la LDH p lasmática.
Siempre que se trate de un exudado, cuando con la toracocentesis
Tabla 1. Criterios de Light no se ha llegado al diagnóstico, el siguiente procedimiento a real izar
es la biopsia pleural cerrada o ciega, cuya mayor utilidad es para el
Hasta un 25% de los trasudados pueden ser erróneamente identificados diagnóstico de p leurit is tubercu losa, ya que es positiva para granu-
como exudados por los criterios de Light. Si se cump len uno o más de lomas caseificantes en el 50-80% de casos, y cuando se comb ina con
los criterios de exudado pero el paciente tiene una causa clínicamente el cultivo de la biopsia, el rendim iento asciende al 90%. Si no es d iag-
evidente de trasudado, se debe ca lcular la diferencia entre las proteínas nóstica, y sobre todo cuando la sospecha de derrame ma ligno es alta,
séricas y las proteínas en líqu ido pleura l: si esa d ifere ncia es mayor de 31 el siguiente p roceder es la b iops ia p leu ral por toracoscop ia (o p leu-
g/1(3, 1 g/d i) se d ebe ig nora r la clasificació n co mo exudado, ya que en la roscop ia) y, en últ imo luga r, la to racotom ía. Si no se cons ig ue llega r al
mayoría de los pacientes se t rata rá de un tras udado. d iag nóstico etio lóg ico, co nv iene desca rta r TEP y va lo rar la rea lización
de una broncoscopia .
Otros parámetros que se analizan son: la glucosa (el derrame reumatoideo
suele tener una concentración de glucosa< de 30 mg/dl), el aumento de la Algunos derrames pleurales
amilasa (se relaciona con pancreatitis aguda, crónica, ruptura esofágica y neo-
plasias; si el origen es esofágico o neoplásico, la ami lasa elevada es la isoenzima Insuficiencia cardíaca
salival, y si es por enfermedad del páncreas, es la isoenzima pancreática), el
va lor del pH (tiene importancia en los derrames paraneumónicos), la LDH (es Es la causa más frecuente de derrame p leura l, y es un trasudado. El
muy buen marcador de la inflamación), colesterol y trig licéridos, ADA, etcétera. d iagnóstico se real iza por la h istoria cl ín ica compatible, y no es pre-
cisa la toracocentes is, sa lvo si el derram e no es b il atera l o de tamaño
compa rab le, cuando hay fi ebre o do lor p leu rít ico o si no responde a
un tratamiento correcto de la insuficiencia cardíaca. Un nivel en líqui-
La causa más frecuente de derrame hemático (hema- do pleura l de Pro-BNP (péptido natriurético cerebral) mayor de 1.500
tocrito del LP 1-50%) es la tumo ral.
pg/ml es diagnóstico de derrame pleural secundario a insuficiencia
cardíaca.

01 · Enfermedades de la pleura
Cirugía torá cica 1 01
Cirrosis hepática Derrame pleural por infección vírica

El principal mecanismo fisiopato lógico en la aparición de derrame pleural Un porcentaje de exudados no diagnosti cados pueden producirse por
es el paso de líqu ido ascítico peritoneal al espacio pleural a través del dia- infecciones víricas. Se resuelve n de modo espontáneo.
fragma . La dism inución de la presión oncótica juega un papel secundario.
El tratam iento en principio se dirige al de la cirrosis y la ascit is, ya que es Derrame pleural tuberculoso
una extensión del fluido peritonea l.
Es la cau sa más común de exudado en algunos países en vías de de-
Derrame pleural paraneumónico sarrollo, especialmente en pacientes jóvenes. Suele ser un exudado
unilateral. El estudio celular muestra un porcentaj e de linfocitos mayor
Se denomina así al que se asocia con infección bacteriana pulmonar no del 50% (en casos muy incip ientes, puede haber neutrófilos) y me-
tuberculosa (neumonía bacteriana, bronquiectasias o absceso de pul- nos del 5% de células mesoteliales. La glucosa puede ser menor de
món). El40% de los pacientes con neumonía bacteriana tienen derrame, 60 mg/ dl. Para el diagnóstico de pleuritis tuberculosa puede ser útil la
siendo más frecuente en las neumocócicas (su presencia se asocia con determinación de ADA (valor límite: 43 U/1, pero tiene falsos positivos
una mayor morta lidad). Es la ca usa más frecuente de exudado. Siempre como el empiema, la artri tis reu matoide y el de rrame tumora l), y el
que se eva lúe a un paciente con neumonía, debe descartarse su presen- interferón y (más específico). La reacción en cadena de la polimerasa
cia. Se considera no significativo si, en el decúbito lateral, la cantidad de (PCR) en busca de ADN de M. tuberculosis está dispon ible en muchos
líquido acumulada es menor de 1O mm, en cuyo caso no se realiza tora- centros y confiere una alta rentabil idad diagnóstica. El diagnóstico de
cocentesis. Si excede de 1O mm, sí se realiza. pleurit is tubercu losa se establece generalmente con la biopsia pleural
cerrada (ya que sólo crece en el cu ltivo en el 15-25% de casos). Pue-
Se define como empiema el líquido pleural infectado, para lo cual debe cum- de aceptarse el diagnóstico de tubercu losis pleural sin rea lizar biopsia
plir al menos unos de los siguientes criterios: 1) aspecto purulento (el líquido siempre que se objetive un derrame pleural exudado con predominio
del empiema tiene muchos neutrófilos), 2) tinción de Gram que demuestre de linfocitos y ADA elevado en un paciente menor de 30 años con
gérmenes, 3) cultivo positivo. La mayoría de los empiemas provienen de un Mantoux positivo.
derrame paraneumónico o de procedimientos quirúrgicos torácicos. El em-
pierna necessitatis es el que se abre paso a través de la pared torácica. Derrame neoplásico

Se denom ina derrame pleura l complicado al derrame paraneumón i- Los más frecuentes son secundarios a metástasis de carcinoma de pul-
co que presenta en líquido pleural un pH < 7,20, o un nivel de glucosa món, mama y linfoma (suponen el 75% entre los tres). Es el segundo tipo
< 60 mg/dl. Tiene un alto riesgo de evo lucionar a empiema . La causa de exudado más frecuente. En el 10% de los pacientes no se identifica el
más frecuente de derrame p leural complicado y empiema es, en pacien- tumor primario. El líquido tiene características de exudado generalmente,
tes con neumonía adquirida en la comunidad, Streptococcus milleri (en sobre todo según criterios de LDH. Es la causa más frecuente de derrame
pacientes con enfermedad renal es Klebsiella pneumoniae), seguida de hemático. El diagnóstico se realiza, en más de la mitad de los casos, por
anaerobios, neumococos y estafi lococos. En los pacientes con neumonía la citología del líquido pleural (según la destreza del citólogo y el tipo de
nosocomial, la ca usa más frecuente son los estafi lococos (la mayor parte tumor, siendo más rentable en el adenocarcinoma).
resistentes a meticilina), seguida de los gérmenes gramnegativos.
El derrame pleural maligno indica enfermedad sistémica, y se debe
El empiema y el derame pleural complicado necesitan colocación de un valorar la quim ioterapia si el tumor primario es sensible. Cuando esta
tubo de drenaje torácico. no da resu ltado o no se puede real izar o si es rec idivante, y el derrame
es sintomático, hay que valorar trata miento sint omático. El método
En las primeras fases, el empiema está libre en la cavidad pleural. Poste- más efectivo es la p leurodesis quím ica, p revia evacuac ión del derrame
riormente, se empiezan a formar bridas que lo "tabican': lo que impide (Figura 4).
que el líquido drene al exterior. Si se sospecha la existencia de bridas,
se realiza una ecografía torácica para confirmarlo. Si el empiema está ta-
bicado o loculado, puede intentarse la lisis de los tabiques mediante la
instilación en la cavidad pleural de fibrinolíticos (estreptocinasa), siempre
que no existan contra ind icaciones para su empleo. En caso de fracaso
terapéutico se procederá a la toracotomía y desbridamiento de toda la
cavidad pleural. Algunos casos en los que el empiema está tan evolucio-
nado que existe una coraza pleura l, hacen necesa ri a una decorticación
durante la intervención pa ra conseg ui r la reexpansión del parénquima
pulmonar. En estos casos, es muy importante que la cirugía sea precoz,
porque si la coraza evoluciona hacia la ca lcificación, se constituye un fi-
brotórax y es imposible consegu ir dicha decorticación .

En el DP pa raneumónico hay que colocar un tubo de


drenaje si es un emp iema, o si el LP tiene pH < 7,20
o glucosa < 60. Si con el tubo no se resuelve o hay
tab icación: fibrinó li sis intrapl eural.
Figura 4. Drenaje endotorácico
Manual CTO de Medicina y Cirugía, 2.a edición

Derrame pleural paraneoplásico sintomático y recidi-


En la AR, el derra me es de predominio derecho (right)
vante en ausencia de pulmón atrapado: p leurodesis
y en el LES, de predom in io izqui erdo (left).
química.

Es necesaria la co mprobación radiológica de que am bas pleuras están en Pseudoquilotórax


contacto entre sí. Puede realizarse con diversas sustancias:
Tetraciclina. Es un método doloroso y no se encuentra disponible en Tiene un aspecto macroscópico similar al del quilotórax, aunque es me-
casi ningún centro. Prácticamente ya no se emplea. nos frecuente. Suelen ser derrames de larga evolución, por lo que la ma-
Talco. Es el agente de elección. Se rea liza en forma de suspensión a tra- yoría de los pacientes presentan superficies pleurales engrosadas y, a
vés del tubo de d ren aje torácico, o en forma de nebulización median- veces, ca lcifi cadas. Las ca usas m ás frecuentes son la artritis reumatoide
te toracoscopia o toracotomía. El inconve niente que presenta es el de y la tuberculosis. El líquido presenta co lesterol elevado(> 250 mg/d l) y, a
poder dar luga r a una reacción granulomatosa de cuerpo extraño. veces, crista les de colestero l. El tratam iento es el de la enfermedad subya-
Bleomicina. Es la que presenta mayor índi ce de recidivas. ce nte. Pocas veces requiere drenaje o decorticación (si el paciente tiene
Como altern at iva a la pleurodesis química se puede co nsiderar la uti- síntomas y el pu lmón subyace nte es fun cio nal).
lización de un catéter tun elizado.
SIDA

Exudado linfocitario con glucosa muy baja(< 30 mgldl) El derrame p leura l no es muy habitual. Las causa más común es el sa r-
y pH bajo: artri tis reumatoide. En el empiema, sin em- coma de Kaposi, aunque dada su extrañeza con los tratam ientos an-
bargo, predominan los neutrófilos y en la TBC, pese tirretrovira les, ta l vez en la actual idad sea el derrame paraneumónico.
a tener un exudado linfoc ito, la gl ucosa tiene valores
Otras causas a tener en cuenta son la tuberculosis, la cr iptococosis y
más altos (< 60 mgld l).
los linfomas.

Antes de rea lizar un intento de sínfisis pleura l, se ha de comp roba r que


en la rad io logía no existe desviación del mediastino hacia el lado del de- 1.2. Neoplasias pleurales primarias
rrame, pues eso indicaría que existe una obstrucción bronquial con ate-
lectasia, y en caso de co loca r un drenaje, la imposibilidad de reexpan sión
del pulmón causaría un neumotórax permanente. Mesotelioma maligno

Derrame pleural lúpico Es un raro tumor que deriva de las cé lu las mesoteliales que tapizan la
cavidad pleural (Fig u ra 5). En la mayoría de los casos se encuentra el
Suele ser bilateral de predom inio izquierd o. Es un exudado con un pH y antecedente de exposición al asbesto. El periodo de latencia entre la ex-
glucosa normales o levemente d isminuidos. El mejor test de cribado es la posición y el tumor es de 20-40 años. El derrame pleural es la manifes-
detección de ANA elevados en el líquido pleural. El comp lemento suele tac ió n más frecuente, y aparece prácticamente siem pre. En un terc io de
estar bajo en el líqu ido. Es diagnóstico encontrar cé lulas LE en el líquido. casos se aprecian placas pleura les. Seg ún el tumor avanza, revist e a todo
el pu lmón, el hemitórax se co ntrae y el mediastino se desvía hacia ese
Derram e pl eural reumatoideo lado. La TC muestra la pleu ra engrosada con un margen interno irregular
o nodular. El aná lisis del líquido muestra un exudado, que en la mitad de
Es poco habitual, predom ina en varones con nódu los subcutáneos. Suele los casos es serohemático. Si el tumor es g rande, la g lucosa y el pH dismi-
ser unilateral y de predom inio derecho. Es un exudado con predominio nuye n (por tanto, son signos de peor pronóstico).
de linfocitos, con pH bajo, y LDH alta . La g lucosa es menor de 30 mg/d l en
el80% de los casos. El complemento es bajo y hay un título alto de factor
re umatoide. Puede comporta rse como un pseudoqu ilotórax.

Quilotórax

Se define por la presencia de linfa en el espacio pleural. La causa más fre-


cuente es la rotura del conducto torácico por traumatismo o cirugía, y a con-
tinuación los tumores mediastínicos como los linfomas. Es típico el aspecto
lechoso del líquido. Es diagnóstico el hal lazgo de quilomicrones en el líquido
pleural o un nivel de trigl icéridos superior a 11O mg/d l en líquido pleural. El
tratamiento incluye, independientemente de la causa, drenaje, administra-
ción de octreótido y nutrición parenteral total o nutrición rica en triglicéridos
de cadena media (que pasan desde la luz intestinal a la sangre directamente,
sin pasa r por la linfa). Si es secu ndario a infiltración neoplásica, es subsidiario
de quimioterapia y/o radioterapia según el tumor primario. Si es secunda rio
a traumatismo, quirúrgico o no, se coloca drenaje endotorácico, pues al re-
expandir el pulmón con frecuencia cicatriza, pero si en diez o quince días no
mejora, se procede a la ligadura quirúrgica del conducto torácico. Figura 5. Mesotelioma pleural maligno

01 · Enfermedades de la pleura
Cirugía torácica 1 01
El diag nóstico suele requerir to racoscopia, e incl uso en muchos casos Clínica
toracotomía, ya que la biopsia cerrada no suele discerni r entre adenocar-
cinoma y est a neoplasia. Hasta el momento, no hay ningún t ratamiento Suele existir dolor torácico súbito y disnea, junto con ma nifestac iones vege-
satisfactorio, aunque se ha probado con rad ioterapia, q ui mioterapia y ci- tativas como sudoración, taquica rdia y pal idez. En la exploración física suele
rugía rad ical. Puede precisa rse pleurodesis. haber ti mpanismo y disminució n, o incl uso abol ición, del murmullo vesicular.

Mesotelioma benigno Diagnóstico

Es un raro tumo r que no pa rece relacionarse con la exposición a asbesto. El d iagnóstico se confi rma con la radiografía de tórax en inspiración y es-
Su origen son las cé lulas subse rosas mu lti potencia les. Suele ser asinto- piración máxima, especia lmente úti l en neumotórax de pequeño tamaño
mático. Se asocia a dos síndromes paraneoplásicos, que son la hipoglu- (esta técnica tambié n se puede em plea r en el d iagnósti co de cuerpos
cemia (por secreción de una sustancia insulina-l ike) y la osteoartropatía extra ños bronq uiales). Se recomienda la realización de TC torácica para el
hipertrófica. Presentan derrame el 10% de los casos. El diagnósti co se diagnóstico cuando los ha llazgos de la radiografía de tórax no son claros:
suele hacer con to racotom ía y el t ratamiento q uirúrgico es eficaz (véase enfermedad pu lmonar bu llosa, rad iog rafía artefacta da por enfi sema sub-
la Figura 5) . cutá neo, etcétera.

Cálculo del tamaño del neumotórax


1.3. Neumotórax
El vo lumen de un neumotórax es propo rcional al cubo de la relación en-
t re el diámetro del pulmó n y el diámetro del hemitórax. Los métodos pa ra
Es la entrada de aire en el espacio pleural, que despega ambas superficies ca lcular con exactitud el tamaño de un neu motórax son m uy engorrosos
serosas y provoca un co lapso pulm o nar, por lo que prod uce d ismi nució n y ca recen, por tanto, de utilidad práctica. En la prácti ca clínica se util iza la
de la capacidad ventilatoria. La repercusión clínica depende de la reserva radiografía de tórax. Existen dos métodos descritos: 1) el propuesto por el
ventilatoria del pacient e y del g rado d e co la pso pulm ona r. De este m odo, American College of Chest Physicians, que clasi fi ca a un neumotórax como
un pequeño neumotórax en un paciente de EPOC grave puede induci r pequeño cuando la dista ncia ent re el ápex del pulmón y la cúpula de la
una insufic iencia respiratoria g rave. pa red torácica es< 3 cm, y como gra nde en caso contrario, y 2) el propuesto
por la British Thoracic Society, que lo clasifica como pequeño cuando el neu-
Tipos motórax forma un an illo alrededor del pulmón de menos de 2 cm desde el
borde del pulmón hasta la pared torácica, y com o grande en caso contrario
Traumático {Tabla 2). Ambos métodos pueden ser utilizados, solos o en combinación,
dependiendo del caso clín ico concreto. Cuando es precisa una estimación
Se produce por un traumat ismo abierto {heridas penet rantes) o cerrado más exacta del tamaño de un neumotórax se debe real izar una TC to rácica.
(traumatismos cerrados). Una subcategoría es el iatrogénico, que se prod u-
ce tras man iobras diagnósticas y terapéuticas sobre el pu lmón, siendo las
causas más frecuentes la punción transtorácica y el acceso a vías centrales. Pequeño , Grande

Espontáneo
< 3cm ~3cm

Ocurre sin traumatismo previo. Se subd ivide en:


Primario: se produce sin enfermedad pul monar previa conoc ida .
Aparece entre los 20-40 años, con predom inio por el sexo masculino.
Los pacientes suelen ser altos, delgados y co n frec uencia fumado-
res, aunque el ta baco no interviene en su patogenia. Se produce por <2cm ~2cm

ruptura de bu llas apica les subpleurales (enfise ma pa rasepta l). Reci-


diva en la m itad de los casos.
Secundario: ocurre en pacientes co n enferm edad pu lmona r previ a Tabla 2. Cá lculo del tamaño de un neumotóra x
co nocida, siendo la cau sa m ás frec uente la EPOC, y predom ina en
ancianos. Tratamiento

Hipertensivo Neumotórax espontáneo primario

Es una comp licac ión m uy g rave que ocurre de forma aguda, donde se Neumotórax pequeño co n síntomas m ínimos: observación durante
produce un mecan ismo va lvular que permite la entrada de aire en la cavi- 3-6 horas para desca rta r progresió n del neu m otórax. No requ ieren
dad pleural pero impide su sa lida, produciéndose, po r tanto, un co lapso hospit alización si no se demuestra progresió n. Durante el periodo
pulmonar tota l con desplazam iento contralateral del mediast ino. de observaci ó n se recomie nda la oxige not erapia con alto fl ujo (1 O1/
mi n), ya que una eleva da concentración de oxígeno red uce la pre-
Catamenial sión parcial de nit rógeno, lo q ue d ism inuye la presión parcial en los
cap ila res pleura les y aumenta el g rad ient e de presión ent re los capi-
En relación con la menstru ación. Aparece en mujeres mayores de 25 la res p leura les y la cavidad pl eural, m ult iplicando por cuatro la velo-
años. Es frecuente la recidiva. cidad de reabso rción del neu m otórax.
Manual CTO de Medicina y Cirugía, 2.a edición

Neumotórax grande o sintomático: req uiere hospitalización. Neumotórax espontáneo secundario


Paciente clín icamente estable: aspiración simple o colocación
de un catéter con vá lvula unid ireccional (Heimlich) integ rada. Requ iere hospita lización . Se puede rea liza r observación sólo en el pacien-
Si fracasa, colocación de un tubo de drenaj e t orácico con sello te con neumotórax muy pequeño (estri cta m ente apica l) y asintomático.
de agua. Cuando se coloca un tubo de drenaje to rácico no es En el resto de los pacientes es necesa ria la rea lización de: 1) aspiración
necesa ria la conexión a un sistem a de aspiración continua sa lvo sim ple o catéter con vá lvula de Heimlich, en pacientes menores de 50
que no se consiga rápidam ente la expansión pulmonar. años con neumotórax pequeño y sínt omas mín imos, o 2) colocación de
Paciente clín icamente inestabl e: colocación de un tubo de dre- tubo de drenaje torácico en pacientes mayores de 50 años o con neumo-
naj e torácico. tórax grand e o sintomático. La mayoría de los autores recomiendan, tras
un primer episod io, la rea lización de un proced imiento para preven ir la
Retirada del tubo de drenaje torácico: el tubo torácico se puede re- recu rrencia. El que está aconsejado es la to ra coscopia médica o quirúrgi-
tirar cuando la radiografía de tórax demuestre expansión pulmonar ca (o la minitoracotomía) con rea lización de pleurodesis, sa lvo en pacien-
com pleta y no exista fuga aérea, tra s suprimir la aspiración continua tes con elevado riesgo quirúrgico o que rechacen la ciru gía, en los que se
si se estaba utilizando. Au nque no existe acuerdo genera liza do, al- puede rea lizar pleurodesis química a través del tubo torácico.
gunos autores recomiendan rea liza r una radi ografía para demostrar
ausencia de neumotórax tras mantener clampado el tubo torácico Neumotórax traumático
durante 5-12 hora s.
Fuga aérea persistente: en este caso se recomienda mantener el tubo Sa lvo el neumotórax iatrogénico pequeño y co n mín imos síntomas, que
torácico bajo aspiración continua durante tres día s. Si la fuga aérea se puede manejar con observación o aspiración sim p le, el neumotórax
persiste durante más de tres días se debe considerar la rea lización de t raumático sue le requerir la colocación de un tubo de drenaje torácico,
toracoscopia (o toracotomía) para el cierre quirúrgico de la fístula y rea- que es imprescindible siem pre que, como ocurre con frec uencia, al neu-
lización de pleurodesis. En pacientes con elevado riesgo quirúrgico o motórax se asocie hemotórax (hemoneumotórax).
que rechacen la cirugía se debe considerar la realización de pleu rodesis
química instilando la sustancia esclerosante a través del tubo torácico. Neumotórax catamenial
Prevención de la recurrencia: se debe realizar un procedimiento para pre-
veni r la recurrencia en los casos de segundo episod io o de primer epi- Se trata con anovul atorios, y si recidiva, pleurodesis.
sod io bilateral simultáneo. Se puede considerar, tras un primer episodio
unilateral, en aquellos pacientes que tienen una profesión o actividad de Neumotórax hipertensivo (a tensión)
riesgo (buceadores, pilotos, etcétera). El procedimiento de elección es
la tora coscopia para resección de las bullas subpleurales y pleurodesis. Es una emergencia médica. Se debe insertar de inmed iato un catéter en
En pacientes con elevado riesgo qui rúrgico o que rechacen la cirugía se el espacio pleural a través del segundo espacio intercostal anteri or, línea
puede realizar pleurodesis química a través del tubo torácico. axilar media, hasta que se pueda colocar un tubo de drenaje torácico.

" Exudado linfocitario con fiebre y ADA elevada: sospechar tuber-


Ideas clave /6 culosis, sobre todo en jóvenes.

" La causa más frecuente de derrame pleural es la insuficiencia " El neumotórax espontáneo primario se produce por rotura de
cardíaca. bullas apicales subpleurales, y afecta fundamenta lmente a va-
rones jóvenes fumadores.
" La causa más frecuente de exudado es el derrame pleural me-
ta neumón ico. " Cualquier neumotórax grande (véase texto) o que provoque
síntomas significativos debe ser drenado.
" Si el derrame pleura l metaneumónico tiene un pH < 7,20 o una
gl ucosa <de 60, o es un empiema, se debe colocar un tubo de " El neumotórax a tensión es indicación de drenaje urgente.
drenaje.

1) Cambiar el tratamiento antibiótico empíricamente, a la espera


Casos clínicos de los cultivos del líquido pleural.
2) Colocación de un tubo de drenaje pleural.
Mujer de 64 años, ingresada por neumonía en lóbulo inferior 3) Solicitar una broncoscopia urgente para toma de muestras y
derecho, en tratamiento con antibiótico empírico. Presenta una cultivo microbiológico de las mismas.
evolución tórpida los días posteriores al ingreso, con persisten- 4) Las características del líquido son compatibles con un trasuda-
cia de la fiebre, disnea y dolor de características pleuríticas en do, por lo que debería iniciarse tratamiento con diuréticos ante
hemitórax derecho. La radiografía de tórax presenta veladura de la sospecha de una insuficiencia cardíaca.
la mitad inferior del hemitórax derecho, compatible con derra-
me pleural. Se realiza toracocentesis diagnóstica, obteniéndose RC: 2
un líquido claro de aspecto seroso, con un pH 6,95, LDH 200, glu-
cosa 30, y el Gram del líquido es negativo. ¿Cuál de las siguientes Varón de 40 años de edad que acude a Urgencias por cuadro
sería la actitud correcta a seguir en este momento? clínico de dos semanas de evolución de febrícula, sudoración

01 · Enfermedades de la pleura
Cirugía torácica 1 01
nocturna y dolor torácico de nas LP/suero 0,7; cociente LDH LP/suero 0,9; celularidad con 85%
características pleuríticas en de linfocitos; ADA 76 U/1. Señale la respuesta correcta:
hemitórax izquierdo. En la ex-
ploración física usted objetiva 1) Se trata de un trasudado. Hay que comenzar tratamiento diu-
una disminución del murmullo rético.
vesicular en el tercio inferior 2) Hay que realizar biopsia pleural cerrada.
del hemitórax izquierdo. La ra- 3) Se trata de una tuberculosis pleural. Hay que comenzar trata-
diografía de tórax se muestra miento específico.
en la Imagen. En la toracocen- 4) Hay que repeti r la toracocentesis diagnóstica para vigilar la evo-
tesis diagnóstica se obtiene un lución del pH del líquido pleural, colocando un tubo de drenaje
líquido pleural amarillento con torácico si disminuye por debajo de 7,20
las siguientes determinacio-
nes: pH 7,28; cociente proteí- RC: 2
r - -_ _ _ _ Cirugía torácica

Enfermed ades del mediasti no

ORlENTAClDN Conviene saberse bien la división anatómica del mediastino, las causas más frecuentes en cada
ENARM compartimento, lo síndromes paraneoplásicos asociados y las características radiológicas.

Anatomía Mediastino
Clavícula

Desde el punto de vista clín ico, el mediastino se d ivide en tres co mpar-


timentos (a natómicamente, el anterior se divide en superior y anterior)
(Figura 6):
Anterior (por delante y encima del corazón): co ntiene t imo, cualquier
extensión intratorác ica de tiroides o paratiroides, arco aórtico y sus ra-
mas, venas innominadas, vasos y ga ngl ios linfáticos. Corresponde a lo
que anatómicamente son dos compartimentos: el anterior y el superior.
Medio (inferoposte rior al anterior): contiene co razón y pericardio,
tráquea, hilios pulmonares, algunos gang lios linfáticos y los nervios
Mediastino
frénico y vago). posterior
Posteri or (márgenes vertebrales): contiene esófago, aorta descen-
dente, vena ácigos y hemiácigos, co nducto torácico, ganglios linfát i-
cos, nervio va go y cadena si mpát ica.

2.1. Masa mediastínica


Aorta

Los tumores más frecuentes, cons iderando su frecuencia g lobal (adultos y


niños), son los neurogénicos (Figura 6). Según aparezca en compartimento
anterior, medio o posterior, hay que pensar en distintas etiologías (Tabla 3). Figura 6. División anatómica del med iastino

Es característica de las m asas del mediastino la frecuencia con que se


asocian a síndromes genera les.

Así, por ejemplo, el t imoma se asocia a m iastenia gravis, a aplasia pura de


la serie roja y a hipogammaglobulinemia; alg unos tumores germinales
a ginecomastia; el feocromocitoma y ganglioneu roma a hipertensión; el
adenoma paratiroideo y ellinfoma a hipercalcemia.

El síndrome paraneoplásico más frecuente asociado a


ti moma es la miastenia gravis.
Figura 7. Rad iografía posteroanterior y lateral de un paciente
con un tumor neurogén ico del mediastino posterior
Cirugía torácica 1 02
Compartimento Masa Características

Anterior Timo La más frecuente del compa rtimento


Asociaciones:
Cushing
- Aplasia pura serie roja
- Miastenia gravis
- Agammaglobulinemia
Tiroides
Teratoma y otros tumores de células germinales · Calcificaciones en la masa
· Asociación: ginecomastia, hipoglucemia, tirotoxicos is
Linfoma Asociación: hiperca lcemia
Medio Quistes de desarrollo: Los más frecuentes del compartimento
· Pericárdicos
· Broncógenos
Linfoma
Hernia de Morgagni Localización más frecuente: ángulo cardiofrénico derecho
Posterior De nervios periféricos: neurofibroma Todos ellos son tumores neurógenos
De los gangl ios simpáticos: · Los más frecuentes del compartimento
- Ganglioneuroma
- Neuroblastoma
De los paragang lios:
- Paraganglioma
- Feocromocitoma
Linfoma
Esófago
Quistes entéricos
Otros:
- Pseudoquiste pancreático
- Hern ia de Bochdaleck
Meningocele
- Hematopoyesis extramedular

Tabla 3. Masas m ed iastínicas

Diagnóstico
Las cuatro T de las masas med iastín icas anteriores:
Tiroides, Timoma, Teratoma y Terrible linfoma.
:L~
~
La TC y la RM son las técnicas de imagen más sensibles (Figuras 8 y 9).
Recuerda

Masa en mediastino anterior con ca lcio en su interior: u~


teratoma. ~
Recuerda

El diagnóstico definitivo se suele realizar con técnicas invasivas, pues las mues-
tras obtenidas por PAAF únicamente aportan una aproximación citológica.

Así, los tumores benignos se suelen d iagnosticar en la pieza obtenida de


una cirugía con intención curativa, sin necesidad de diagnóstico histoló-
gico previo a la intervención.

Ante la sospecha de malignidad, se realiza cirugía con intención diag-


nóstica con toma de biopsias. El abordaje diagnóstico de las masas de
mediastino anterior se rea lizará por mediastinotomía; el de las masas de
med iastino med io req uerirá med iastinoscopia, y las masas situadas en el
med iastino posterior se evalua rán por videoto racoscopia. Si la sospecha
es que se trata de un tumor metastásico, puede int entarse la PAAF pa ra
confirmarlo. Un dato radio lóg ico de interés es la tendenc ia del teratoma
a calcificarse. Figura 8 . TC de m asa mediastínica (corte axial)
Manual CTO de Medicina y Cirugía, 2.a edición

2.2. Infecciones

Mediastinitis aguda
Lo más frecuente es que sea una comp licación posquirúrgica o bien se deba
a una rotura esofágica. La rotura esofágica, a su vez, puede ser espontánea
tras un vóm ito (síndrome de Boerhaave) o por traumatismo o instrumenta-
ción (esofagoscopia la más frecuente, y también sondas, balones, dilatado-
res ... ). La mediastinitis hemorrágica es una rara complicación del carbunco.

La clínica tiene un inicio brusco con fiebre, taquicardia, taquipnea, enfise-


ma subcutáneo y sig no de Ham man en la auscu ltación (crujido sincróni-
co con el latido ca rdía co en decúbito latera l izquierdo).

El t ratamiento consiste en el drenaje quirúrgico inmediato, reposo diges-


tivo y antibióticos, que si se hace en las pri meras 24 horas, reduce la mor-
ta lidad del 75% al 25%.

Mediastinitis crónica
Puede ser una med iastinitis granulomatosa que progresa hacia mediastini-
t is fibrosante o un proceso de fibrosis inicial asociada a distintas etiologías
(Tabla4).

Figura 9. TC de masa mediastínica (corte sagital) Tipo Etiología


Mediastinitis Histoplasmosis Asintomática Ningún
Tratamiento granulomatosa Tubercu losis tratamiento,
médico
El tratamiento de elección de las masas mediastínicas es la extirpación Fibrosis Fármacos Síndrome o quirúrgico,
mediastínica (metisergida) de cava se ha
quirúrg ica, a excepción de los linfomas, germinomas y carcinoma me-
Sil icosis superior mostrado
d iastínico (rad ioterapia y/o quimioterapia) .
Sífili s eficaz

En los timomas es de especial interés la cirug ía, ya que el criterio funda- Radiación
menta l de malig nidad de est os tumores viene dado por su capacidad de Tumores ma lignos
invasión local, que se evalúa durante el propio acto quirúrgico. Tabla 4. Mediastinitis crónica

En los timomas, "es mejor el ojo del ciruj ano que el 2.3. Neumomediastino
ojo del patólogo", pues aun en los invasivos, la apa-
ri encia histol ógica es de ben ignidad.
Se produce por rotu ra de alvéolos o bu llas al intersti cio mediastínico o
por lesión de la vía aérea principal o del esófago a su paso por esta es-
En ocasiones, en los pacientes que presentan una miastenia g ravis, se tructura anatómica.
real iza timectomía, pese a no esta r aumentado de tamaño. Siempre
que el paciente presente un ti moma asociado, está indicada su resec- El signo de Hamman (auscultación de un crujido sincrónico con el latido
ción por el riesgo de ag res ión loca l. Cuando no existe t imoma, pero ca rdíaco) es típico, así como el enfisema subcutáneo en yugulum. Suele
existe miastenia gravis, se indi ca la cirugía en las form as de m iaste nia ser asintomático. Se reabsorbe antes si se somete al paciente a respira-
generalizada en pacientes con edades comprend ida s entre la puber- ción con una Fi0 2 elevada. Si produce compresión, se evacúa con aspira-
tad y los 55 años. ción o varias agujas hipodérm icas supraestern ales.

" Los timomas, los más frecuentes en el m ed iastino anterior,


Ideas clave RS se asocian a miastenia gravis, ap las ia de serie roja o hipo-
gammaglobuline mia .
" Los tumores mediastínicos más frecuentes son los neurogéni-
cos, situados en mediastino posterior. " La causa más frecuente de mediastinitis aguda es la perforación
esofágica.
" Sin embargo, el mediastino anterior es el compartimento don-
de con mayor frecuenci a aparecen masas mediastínicas.

02 · Enfermedades del mediastino


Cirugía torácica 1 02
El diagnóstico más probable es:
Casos clínicos
1) Neuroblastoma.
Mujer de 38 años, asintomática, a quien, en la primera radiog ra- 2) Teratoma.
f ía rutinaria laboral que se le practica, se le descubre la existencia 3) Quiste pericárdico.
de una imagen tumoral en mediastino anterior, bien delimitada, 4) Qui ste broncogénico.
de 15 cm de diámetro mayor. En la tomografía se observa que la
masa tumoral tiene distintas densidades, con zonas diseminadas RC: 2
de calcificación.
• Cirugía torácica

Enfermedades del diafragma

ORIENTACIÓN Con el estudio de las Ideas


3.2. Hernias diafragmáticas (Figura 1ol
ENARM Clave puede ser suficiente.

Hernia de Bochdalek

3.1. Parálisis del diafragma Se local iza en la zona posterolateral del tórax, con más frecuenc ia en el
lado izqu ierdo. Ocurre más habitualmente en lactantes. Puede contener
grasa el polo rena l superior o el bazo.
Parálisis unilateral
Hernia de Morgagni
Se sospecha al encontrar un hem idiafragm a elevado en la rad iografía de
tórax posteroanterior. Se localiza en la zona anterior del tórax, generalmente como una masa que
ocupa el ángulo cardiofrénico derecho. Ocurre con mayor frecuencia en adul-
El d iagnóstico se confirma mediante rad ioscopia dinámica con la "pru eba tos obesos. Puede contener grasa epiploica, intestino, estómago o hígado. El
del olfateo'; que objetiva cómo el diafra gma, durante la inspiración, o no diagnóstico de las hernias del diafragma se rea liza con la TC del tórax o con
se mueve o asciende (movimiento paradójico). Genera lmente no produ- radiolog ía con contraste. Cuando son sintomáticas se deben intervenir. La
ce síntomas. eventración del diafragma consiste en una elevación del hemidiafragma
por un desarro llo incompleto del músculo o por atrofia loca lizada que
La causa más frecuente es la infi lt ración del nervio frénico por una neo- predomina en adu ltos obesos y es asintomática.
plasia maligna (la más habitual es el cá ncer de pulmón), pero también
puede ser idiopática o a veces por lesión del nervio frénico durante un
acto quirúrgico.

Vena cava inferior


Parálisis bilateral

Las ca usas más frecuentes so n las lesiones medulares altas y los trauma-
t ismos torácicos.

También la orig inan la escleros is mú ltiple y enfermedades muscu lares.

Tienen intensa disnea, que empeora al adoptar el decúbito. La CV está


d isminu ida, sobre todo en decúbito supino, y hay hipercapnia. El trata- Aorta
miento es la ventilación mecán ica no invasiva, y si el nervio frénico está
intacto, el marca pasos diafragmático. Figura 1O. Tipos de hernias diafragmáti cas

" Una capacidad vital normal, sobre todo en decúbito supino,


Ideas clave Jf!S descarta parálisis frénica clínicamente significativa.

" La causa más frecuente de parálisis unilateral es la infiltración " El diagnóstico se establece por radioscopia.
del frénico por un tumor, por lo común, cáncer de pulmón.

" La causa más común de parálisis bilateral es la lesión de médula


cervica l (por encima de C4).
_Cirugía torácica •

Neoplasias pulmonares

ORIENTACION Tema fundamental. Es imprescindible conocer muy bien la clasificación TNM. Las Ideas Clave
habrán de guiar en los datos clínicos que se deben conocer. No hay que olvidar la valoración
ENARM preoperatoria y el tratamiento.

4.1. Tumores malignos Existe una clara relación entre el consumo de tabaco, sobre todo de cigarrillos,
y el cáncer de pulmón. Esta asociación se observa en el 90% de los pacientes.
Algunos contaminantes ambientales, como el asbesto o el radón, son tam-
Generalidades bién factores de riesgo para desarrollar una neoplasia pulmonar, y además po-
tencian el efecto carcinogénico del tabaco. El riesgo relativo aumenta más de
Aunque las metástasis pulmonares de tumores de otras localizaciones diez veces en los fumadores activos y 1,S veces en los que durante años son
son muy frecuentes, el pulmón es uno de los pocos órganos en que son "fumadores pasivos': Hay clara relación entre el riesgo de aparición y la tasa de
más habitua les los tumores primarios que los metastásicos. Más del 90% mortalidad por carcinoma pulmonar con la cantidad total de cigarrillos fuma-
de las neoplasias pulmonares primarias so n tumores malignos. dos. Al dejar de fumar, disminuye el riesgo, y a los quince años, se aproxima
al de los no fumadores, aunque nunca llega a ser igual al de un no fumador.
Anatomía patológica
Presenta su máxima incide ncia entre los 55 y 65 años, es más frecuente
Existen cuatro va ri edades histológ icas fundamenta les de tumores ma- en hombres, au nque la incidencia en muj eres ha aumentado de manera
lignos, que son el carc inoma epidermoide (escamoso, espinocelu lar); el considerab le. El adenocarcinoma es el más habitual en pacient es jóvenes
carcinoma anaplásico de cé lulas pequeñas (de cé lulas en grano de avena, (< 45 años), en mujeres o en aquellos que nunca han fumado. Este t umor
oat ce!!) o microcítico; el adenocarcinoma (que incluye el bronqu ioloal- puede asentar sobre zonas cicatriza les, por ejemplo, tuberculosas, y tiene
veolar, que se orig ina en los septos alveola res) y el carcinoma anaplásico más tendencia que el resto de los CNCP a diseminarse por vía sanguínea
de células grandes. Es fundamental la diferenciación entre el carcinoma (si bien el que más lo hace es el oat ce//).
de células pequeñas (CCP) y los otros tres subtipos, que componen el
grupo de carcinomas no de células pequeñas (CNCP), ya que implica una No existe una patrón de herencia mendeliano en el cáncer de pulmón,
actitud terapéutica diferente. El epidermoide es el que con más frecuen- aunque se ha observado una incidencia superior en aquellos pacientes
cia se cavita (más del 20%), y el de células grandes tamb ién lo hace con con capacidad para inducir la enzima P-450. Las células tumorales pre-
asiduidad (cercano al 20%). senta n lesiones genéticas adqu iridas que inducen la activación de onco-
genes (myc para el oat ce//, ras para el adenoca rcinoma .. ) e inhibición de
antioncogenes (p53, rb, etcétera). La alteración genética más frecuente
es la mutación del p-53, mi entras q ue el cromosoma que con mayor fre-
El carcinoma microcítico es el más agres ivo, el más cuenc ia se encuentra altera do es el 3p. También parece haber un incre-
quim iosens ib le y el de peor pronóstico. mento de riesgo en pacientes con EPOC y fibrosis pulmonar idiopática.

Clínica
Etiología
El cáncer de pu lmón puede permanecer silente durante meses, por lo
El cáncer de pu lmón es el segundo tumor maligno más frecuente (su- que, en el momento del diagnóstico, sólo la qu inta parte se encuentra en
perado en el hombre por el cáncer de próstata y en la mujer por el de un estadio localizado.
mama), sin emba rgo, es la primera causa de muerte por tumores ma lig-
nos, tanto en el hombre como en la mujer. A pesa r de los grandes avan- Tanto la clín ica como la radiografía de tórax dependen de la localización del
ces en el diagnóstico y tratamient o, la supervivencia a los cinco años sólo tumor, por lo que se dividen en carcinomas centrales (Figura 11) o proxima-
alcanza el 12- 15%. les (visibles mediante broncoscopia) y periféricos. El epidermoide y anaplásico
Manual CTO de Medicina y Cirugía, 2. 8 edición

de célu las pequeñas suelen aparecer como una masa en grandes bronquios tal del brazo. A menudo, coexiste con afectación del simpático cervica l
(centrales). El adenocarcinoma y el anaplásico de células grandes suelen ha- (ganglio estrellado), produciendo el sínd rome de Claude-Berna rd-Horner
cerlo como masa periférica, a veces con afectación pleural (periféricos). (Figura 13), cons istente en miosis, ptosis, enofta lmos y, en ocasiones, an-
hidrosis ipsi latera l de la ca ra.

Síndrome de Horner por afectación


del simpático cervical
(ganglio estrellado)
Miosis
· Ptosi s
· Enoftalmos
· An hidrosis facial
ipsilateral

Erosión de las

Figura 11. Imagen endoscópica de una neoplasia pulmonar central y antebrazo


(afectación
del plexo
El síntoma más frecuente es la tos. Los tumores centra les producen au- braqu ial)
mento de la tos, expectoración hemoptoica (en fumadores mayores de
40 años es indicación de broncoscopia, au nque la rad iografía sea nor-
ma l). Si es de mayor tamaño, puede orig inar atelectasia por la obstru c-
ción comp leta de la luz bronqu ial (el carc inoma pulmonar es la causa más
frecuente de atelectasia) y puede infecta rse el parénquima dista l, produ- Figura 12. Tumor de Pancoast
ciéndose una neumo nitis obstructiva con fi ebre y tos productiva, que no
se resuelve tota lmente co n el tratam iento antib iótico o es recidivante en
la misma loca lización. A veces, hay d isnea, sibilancias y estridor.

Los tumores periféricos generalmente no dan síntomas hasta fases ava nza-
das, en que pueden prod ucir dolor por afectación de la pleura o la pared
costa l y derrame pleura l maligno (el que lo hace con más frecuencia es el
adenoca rcinoma). Con cierta frecuencia suponen un hallazgo en una ra-
diografía, como un nódu lo pulmonar solita ri o. El carcinoma bronq uioloa l-
veolar tiende a diseminarse por vía bronquial, por lo que puede presenta rse
como masa periférica única o mú ltiple o como un infiltrado alveolar d ifuso.
Dada su local ización en la zona en que se produce el intercambio gaseoso Figura 13. Síndrome de Horner (tumor de Pancoast)
origina disnea e hipoxemia, co n producción de esputo abunda nte.
La disem inación linfática produce adenopatías regionales cuya evalua-
ción es fundame ntal para establ ecer el estadio, y a veces origina una li n-
fang itis pulmonar carci nomatosa con disnea y un patrón intersticial en la
En presenc ia de derrame pleural , la hi sto logía más rad iografía de tórax.
prob abl e es adenoca rcinoma (tumor periférico).
Recuerda Las metástasis hematógenas son muy frecuentes y pueden afectar a casi
cualquier órgano, pero sobre todo aparecen en hígado, cerebro, hueso,
Por invasión de estruct uras adyacentes o adenopatías reg ionales, puede méd ula ósea y suprarrenales, produciendo clínica en esas loca lizaciones
produ cir obstrucción traqu ea l, disfag ia por co mpresión del esófago, d is- (déficit neurológ icos, fracturas patológ icas, co lestasis d isociada, reacción
fonía por lesión del nervio laríngeo recurrente, parálisis del frén ico que leucoeritrob lástica, etcétera). Es excepcio nal la d isem inación hematógena
produce elevació n hemid iafragmática, síndrome de la vena cava superior al pu lmón contralateral.
(siendo la ca usa más frecuente el ca rcinoma pu lmonar, especialmente
el oat ce/1), invasió n del pericardio con t aponamiento card íaco, etcétera. Los sín dromes paraneoplásicos son frecuentes. Los tumores que más los
producen son los m icrocíticos (las células tienen gránulos neurosecreto-
El síndrome de Pancoast se produce por crecimiento loca l de un tumor del res, pues derivan de las cé lulas de Kulchitsky del sistema APUD bronquial).
vértice pu lmonar que penetra fácil mente en el cana l neural y destruye las
raíces nerviosas octava cervical y primera y segunda torácicas (Figura 12). Un tercio de los pacientes presentan síndrome constituciona l, con aste-
nia, anorexia, pérd ida de peso e incluso inmunodepresión. Puede haber
Su causa más frecuente es el cáncer de pulmón, especialmente el epider- hiperca lcemia e hipofosfatemia por secreción de un péptido PTH-Iike
moide. Cursa con dolor en el hombro, irrad iado por todo el borde cubi- (ca rcinoma epidermoide), hiponatremia por secreción inadecuada de

04 · Neoplasias pulmonares
Cirugía torácica 1 04
ADH o del péptido natriurético atrial (sobre todo, el oat cell) o hipopota- regionales (campo tolerab le de radioterapia torácica). Comprende la
semia por secreción ectópica de ACTH (sobre todo, el microcítico). enfermedad confinada a un hemitó rax (incluidos ganglios linfáticos
mediastínicos contra latera les), afectación del nervio recurrente y
Aparecen acropaquias (dedos en pali llo de tam bor) en el 30% de los casos obstrucción de la cava superior.
de cua lquier tipo histológico, aunque más en el de no célu las pequeñas. A Enfermedad avanzada. No abarcable por la radioterapia (supera
veces, generalmente en adenoca rcinomas, hay osteoartropatía hipertrófi- los límites anteriores), es la forma más frecuente de presentación.
ca, que consiste en periostitis en huesos largos con dolor y tumefacción;
con frecuencia se asocia a acropaquias. Carcinoma no de células pequeñas (CNCP)

Puede aparecer el sínd rome miasténico de Eaton-Lambert y la ceguera re- La estadificación se real iza según el sistema internaciona l TNM {Tabla 5) .
tiniana, asociados preferentemente al
microcítico. La ginecomastia se asocia
al anaplásico de cé lulas grandes. Menos Tx. Tumor primario que no puede ser evaluado, o tumor probado por la presencia
de células malignas en esputo o aspirado bronquia l, sin que el tumor sea visible
frecuentemente se observan otros sín-
por imagen ni por broncoscopia
dromes como neuropatías periféricas,
TO. Sin evidencia de tumor primario
dermatopolimiositis, degeneración ce- Tis. Carcinoma in situ
rebelosa subaguda o disfunción cortica l Tl. Tumor s; 3 cm de diámetro mayor, rodeado por pu lmón sano o pleura visceral, sin
(más frecuente en el oat cell), la trombo- evidencia broncoscópica de invasión más proximal que el bronquio lobar (es decir, no
flebitis migratoria (síndrome de Trous- hay afectación del bronqu io principa l). Se subdivide en:
seau), endoca rd itis asépti ca trombótica - Tla: tumor s; 2 cm de diámetro mayori'l
(endocarditis ma rántica), CID, anem ia - Tl b: tumor> 2 cm pero s; 3 cm de diámetro mayor
T2. Tu mor> 3 cm pero s; 7 cm de diámetro mayor, o tumor con cualquiera de los siguientes datos:
normocítica normocrómica, eritema
Afectación del bronquio principal a más de 2 cm de la carina traquea l
gyratum repens, acantosis nigricans y
- Invas ión de la pleura visceral
glomerulonefritis membranosa.
Atelectasia o neumonía obstructiva que se extienda hasta la región hiliar, pero no de
todo el pulmón. Se subdivide en:
Diagnóstico > T2a: tumor> 3 cm pero s; S cm de diámetro mayor
> T2b: tumor> S cm peros; 7 cm de diámetro mayor
Requiere co nfirmación histo lóg ica. T3. Cualquiera de las siguientes ca racterísticas:
Generalmente, la muestra se obtiene - Tumor> 7 cm de diámetro mayor
mediante b roncoscopi a y real ización - Invasión directa de cua lquiera de las siguientes estructu ras: pared torácica (incluyendo
de biopsia bronq uial, si es ce ntral, o tumor del ulcus superior), diafragma, nervio frén ico, pleura med iastínica, pericardio parietal
transbronqu ial en los periféricos. - Tumor en bronquio pri ncipal a menos de 2 cm de la carina traq ueal, sin llegar a inva dirla
- Neumonía obstructiva o atelectasia de todo el pulmón
- Presencia de nódulo(s) tumoral(es) separado(s) del tumor primario pero en el mismo
Una vez diagnosticado, es fundamen-
lóbulo que este
tal determinar la localización y el gra-
T4. Tumor de cua lquier tamaño que cumpla alguna de las siguientes características:
do de extensión del tumor (est adifi- - Invasión de cua lquiera de lo sig uiente: mediastino. grandes vasos, corazón, nervio
cación), así como la valo ración de la recurrente, tráquea, carina traqueal, esófago, cue rpo vertebral
operabilidad en los pacientes poten- Presencia de nódulo(s) tumora l(es) separados del t umor primario, en el mismo pulmón
cia lmente reseca bl es. que este pero en diferente lóbulo
Nx. No puede evaluarse la existencia de metástasis gangl ionares regionales
Estadificación NO. Ausencia de metástasis ganglionares regionales
N l. Metástasis en ganglios peribronquia les o hiliares homolaterales (se incluye la afectación
por extensión directa)
La estadificación del cáncer de pul-
N2. Metástasis en ganglios mediastínicos homolaterales y/o subcarinales
món es una forma de clasificación que
N3. Metástasis en ganglios contralaterales (h iliares o mediastínicos), o bien en ganglios
permite conocer el grado de exten- escalenos o supraclavicu lares (homolatera les o contralaterales)
sión tumoral y las opciones terapéuti- Mx. No puede eva luarse la presencia de metástasis a distancia
cas disponibles en cad a estadio. MO. Ausenc ia de metástasis a distancia
Ml. Presencia de metástasis a distancia. Se subd ivide en:
Carcinoma microcítico Mla: cua lqu iera de las siguientes:
> Presencia de nódulo(s) tumoral(es) en el pulmón contra lateral al tumor primario
(CCP)
> Presencia de nódulos pleurales tumorales
> Existencia de derrame pleura l o pericárd ico ma ligno 121
M1 b: metástasis a distancia
La clasificación más utilizada diferencia
[1) Se clasifica también como Tl el infrecuente tumor superficial de cualquier tamaño con el componente
dos estadios tumorales dependiendo
invasivo limitado a la pared bronquial, aunque se extienda proximalmente al bronqu io principal
de si existe o no afectación extratorá-
[2) La mayoría de los derrames pleurales o pericárdicos que aparecen en un paciente con cáncer de pu lmón se
cica (últimamente se recomienda util i- deben a este. Sin embargo, hay algunos pacientes en los que se puede excluir que el derrame esté relacionado
zar tamb ién la clasificac ión TNM) . con el tumor y, por tanto, ser considerados MO: para ello es necesario que el derrame no sea hemático, ni un
Enfermedad localizada. Ind ica exudado y que mú ltiples estudios citopatológicos del líquido pleural (o pericárdico) hayan sido negativos,
además del juicio clínico
que la enferm edad está co nfina-
da a un hemitórax y a sus ganglios Tabla S. Clasificación TNM para el CNCP
Manual CTO de Medicina y Cirugía, 2.a edi ción

Conviene reseñar algunos datos de esta clasificación : Mediastinoscopia. Clásicamente se ha considerado la


La afecta-ción del pericardio parietal se considera T3, pero la del líqui- técnica de elección en la evaluación de las adenopatías
do pericárdico o pericardio visceral es M 1a. mediastínicas. Permite un fácil acceso a la s adenopatías
El derrame pleural maligno se considera M 1a. paratraquea les derechas y traqueobronquiales proxima-
La afectación del nervio frénico y el síndrome de Pancoast se consi- les. Se introduce el med iastinoscopio a través de una
deran T3. incisión en yugulum esternal hasta región pretraqueal,
La afectación del nervio recurrente, síndrome de cava superior, com- explorándose las cadenas ganglionares paratraqueales y
presión de la tráquea o esófago se considera T4, sa lvo que el tumor subcarínica. La s complicaciones más graves son la he-
sea periférico y sin relación con estos síntomas. morragia (que puede requerir esternotomía o toracoto-
Si existen nódulos tumorales en el mismo lóbu lo que el tumor pri- mía inmediata s) y la med iastin itis, pero las más frecuen-
mario, se considera T3, si están en el mismo pulmón pero en distinto tes son la paráli sis del nervio recurrente izquierdo y la
lóbulo se considera T4, y si están en el otro pulmón se considera M1 a. infección de la herida qu irúrgica.
Mediastinotomfa. A través de una incisión paraesternal
Según lo anterior, se habla de diversos estadios tumorales (Tabla 6). con resección del segundo y/o tercer cartílago costal iz-
quierdo, se accede a las cadenas preaórticas y de la venta-
Métodos de estadificación na ao rtopulm onar (inaccesibles al mediastinoscopio). Se
debe tener especial cuidado (al igual que en las pericar-
Eva luación de la extensión tumoral en el carcinoma no microcítico diocentesis) de no lesionar la arteria mamaria interna. El
Evaluación de la extensión tumoral intratorácica. resto de complicaciones son similares a las de la medias-
TC de tórax y abdomen superior: permite evaluar la extensión t inoscopia. Se indica en carcinomas del lóbulo superior
mediastínica, pleural, a la pa red torácica y la existencia de ade- izq uierd o (ya que tiene un d renaj e linfático complemen-
nopatías mediastínicas patológ icas (diámetro> 1 cm), así como tario al hiliar y mediastínico, por lo que a veces da metás-
la afectación abdominal. tasis en la ventana aortopu lmonar) y en los del bronquio
Resonancia magnética nuclear: es superior a la TC en la valo- principa l izqu ierdo.
ración de la afectación vascular mediastínica (vena cava superior, Punción-aspiración transbronquial. Se realiza durante la
arteria s pu lmonares y corazón), y en la evaluación de los tumores broncoscopia, permite obtener muestra de los ganglios
apica les (Pancoast) al permitir una mejor delimitación de la afec- hi liares, paratraqueales derechos, ventana aortopu lmo-
tación de la arteria subclavia, plexo braquia l y canal medular. nar y subcarinales. La sensibilidad es cercana al 90%
Tomografía por emisión de positrones (PET): se fundamen- cuando se realiza g uiada por ultrasonografía endobron-
ta en que la actividad metabólica de las células tum orales es quial, en especial cuando se complementa con la ultra-
superior a la de las células normales, lo que se evidencia con la sonografía por vía digestiva . Su utilización ha provocado
uti lización de un meta bolito marcado con un isótopo radiactivo una dism inución en el número de indicaciones de me-
(18F-fluorodesoxig lucosa). Es más sensible y más específica que diastinoscopia.
la TC convenciona l para la detección de adenopatías tumorales Toracoscopia. Mediante incisión intercostal e introduc-
intratorácicas, sin embargo, los ha llazgos requieren confirma- ción del vid eotoracoscopio, se induce una cámara de
ción citoh istológ ica siempre que sean re levantes para la esta- neumotórax (a no ser que el paciente tenga un derrame
dificación. pleura l, que crea la cámara necesaria para la visualiza-
Técnicas de biopsia: la confirmación histológica de la afecta- ción) . Se visua lizan las áreas ganglionares paratraqueales
ción ganglionar requiere el uso de alguna de las siguientes téc- derechas, aortopulmonar, paraesofágica y del ligamento
nicas: pulmonar, y se observa la relación de la neoplasia con las
distintas estructuras de la cavidad torácica. La s principa-
les compl icaciones son la fuga aérea prolongada a través
Carcinoma oculto Tx NO MO de los drenajes y la hemorragia. Su uso aún no está muy
Estadio O Tis NO MO extend id o, aunque podría ser útil para valorar la exten-
Estadio la T1a-b NO MO sión pleural de la neoplasia.

Estadio lb T2a NO MO
Evaluación de la extensión tumoral extratorácica.
T1a-b,T2a N1 MO En el momento del diagnóstico, la mitad de los pacientes con
Estadio lla
T2b NO MO cáncer de pulmón presentan metástasis. Su búsqueda va a de-
T2b N1 MO pender de los datos encontrados en la anamnesis, exploración
Estadio llb física y exploraciones comp lementaria s. Se deben buscar metás-
T3 NO MO
tasis cerebrales (mediante RM o TC) siempre que el paciente ten-
T1 -2 N2 MO
ga síntomas o signos neurológicos o bien se trate de un carcino-
Estadio lila T3 N1 -2 MO ma microcítico (1 0- 15% de metástasis cerebra les as intomáticas).
T4 N0-1 MO Los cortes abdominales de la TC pe rmiten eva luar la p resencia de
T4 N2 MO metástasis suprarrenales (que suelen ser as intomáticas). Hay que
Estadio lllb descartar metástasis hepáticas (TC, ecografía abdominal) siempre
1
CualquierT N3 MO
que exista hepatomegali a o alteración de las enzimas hepáticas.
Estadio IV CualquierT Cualquier N M1a-b La existencia de dolor óseo, hipercalcemia o elevación de la fos-
Tabla 6. Estadios tumorales del CNCP fatasa alca li na obliga a estudiar la existencia de metástasis óseas

04 · Neoplasias pulmonares
Cirugía torácica 1 04
(gammagrafía ósea). Es posible que la PET reemplace a todas las sión, y luego realizar, mediante broncoscopia, fototerapia en la zona
técnicas que evalúan la existencia de metástasis (excepto para afectada.
el estud io del cerebro), y hay que realizarla también en todo pa-
ciente que va a ser sometido a u n tratamiento potencial m ente Estadios 1y 11
cu rativo. En gene ral, se requ ie re confirmación citohisto lóg ica de
las metástas is siem p re que co nd icionen la estad iti cación y el tra - El tratamiento de elección es la resección qu irúrgica con intención curati-
tamiento. va. La técn ica qui rúrg ica de elección es la lobectomía (si no es posible se
debe plantear la neumonectomía) asociada a linfadenectomía mediastí-
Evaluación de la extensión tumoral en el carcinoma nica. En casos de tumores en lóbulo medio o lóbulo inferior derechos, se
microcítico puede rea lizar resección conjunta de ambos. Excepcionalmente puede
realizarse cirugía muy conservadora (segmentectomía o resección atí-
pica) en pacientes con limitación funcional que no toleren una cirugía
Dada la gran capacidad de diseminación de este tipo histológico, el estu- estándar. Se necesita realizar una linfadenectomía mediastínica que in-
dio de extensión debe incluirTe torácica y abdominal, RM (o TC) cerebral cluya seis gangl ios o más y que no estén afectados para considerar que el
y PET. estadio es NO. En el estadio 11 se debe asociar la qu im ioterapia adyuvante
(posqu irúrg ica) basada en un derivado del platino, ya que ha demostrado
Detección precoz un aumento sign ificat ivo de supervivencia (4-15%), pero no existen datos
concluyentes que permitan recomendarla en el estadio lb. En aquellos
Los estudios realizados en los años setenta empleando radiografías y pacientes que no toleren la cirugía (inoperables), la radioterapia en dosis
citologías de esputo seriadas no consiguieron reducir la mortalidad por curativa s es una buena opción terapéutica .
cáncer, de ahí que no se consideren indicados como método de cribado
poblacional. Trabajos más recientes con TC han demostrado detectar tu- Tumor de Pancoast
mores en estadios precoces, y uno de ellos consigu ió una tasa de cura-
ción del 90o/o en los tumores así diagnosticados; la ausencia de un grupo En la actualidad el tratamiento de elección para el tumor de Pancoast es
control impide, por el momento, considerar la TC como un método de la quimiorradioterapia seguida de cirugía (siempre que sea resecable) de
cribado generalizado. tres a seis semanas después.

Tratamiento Estadio 111-A

Ante todo, hay que resalta r la im portancia de la prevención. Dejar de T3N1: el tratamiento de elección es la resección quirúrgica seguida
fumar y, sobre todo, evitar el inicio del hábito tabáqu ico debe ser la de quimioterapia adyuvante.
actitud prioritaria en la lucha contra esta enfermedad. Una vez instaura- T1-3N2: cuando existe afectación tumora l N2, el tratamiento es con-
do, el tratam iento depende del tipo histológico y el estadio en que se trovertido, y debe ser abordado por un equipo multidisciplinar. En
encuentre. la actua lidad, la tendencia mayoritaria es la quimioterapia neoadyu-
vante, seguida de cirugía si ha existido respuesta de las adenopatías
Carcinoma no microcítico mediastínicas, o de qu imiorradioterapia si no ha existido. En aquellos
pacientes en los que el N2 se demuestra sólo tras la cirugía (N2 míni-
El mejo r tratamiento es la cirugía, pues p resenta menor incidenc ia de mo) se recomienda la quim ioterapia adyuvante.
complicaciones que la ra di ote rap ia, au nque esta tamb ién es eficaz. T4N0-1: en esta situación se debe rea lizar una valoración ind ividua-
La quim ioterapia es sólo m oderada m ente úti l. La cirug ía se rá pos ible lizada de la posibil idad de resección qu irúrgica, genera lmente tras
siempre que el tumor sea resecable y el paciente sea ope rable, por quimiorradioterapia.
tanto, es fundamental realizar un estudio de resecabi lidad y otro de
operabilidad. Estadio 111-B

La resecabilidad hace referencia a la posibilidad de que el t umor sea Es un estadio irresecable. El tratamiento de elección es la asociación de
oncológicamente resecado, es decir, que el cirujano sea capaz de ex- quimioterapia y radioterap ia.
tirpar todo el tejido tumoral. La resecabi lidad depende de la estaditi-
caciónTNM. Estadio IV

La operabi lidad se refiere a la situación fun ciona l y fis iológ ica del paciente El tratamiento de elección es la qu imioterap ia. Se rea liza radioterapia
que le hará tolerar la cirugía, dejando suficiente parénq uima sano para paliativa en caso de hemoptisis, sínd rome de cava superior, disnea, ate-
mantener un adecuado intercambio gaseoso. lectasia, parálisis de cuerda voca l, taponamiento cardíaco, met ástasis ce-
rebrales, metá stasis óseas dolorosas, afectación de plexo braquial o com-
Según el estadio en el que se encuentre el paciente, el tratam iento varia- presión medular. En los derrames pleurales malignos se realiza drenaje
rá según se expone a continuación. para paliar la disnea, y en caso de recidiva, pleurodesis o colocación de
un catéter tunelizado.
Carcioma in sítu
El tratamiento quimioterápico de primera línea se basa en la poliquimio-
Se puede realizar una resección conservadora. Otra posibilidad es ad- terapia, siempre incluyendo un derivado del platino. Recientemente, se
ministrar al paciente hematoportirina int ravenosa, que se lija a la le- ha incluido en el tratamiento:
Manual CTO de Medicina y Cirugía, 2. 8 edición

lnhibidores de la t irosinacinasa (e rloti nib y gefitinib): inhiben el Infección pulmonar o de la herida quirúrgica
EGFR (receptor del factor de crecimiento epidérmico). Están in-
dicados como tratam iento de seg unda línea en el carci noma no Enfisema subcutáneo. Cuando se limita exclusivamente a la zona qui-
microcítico (especia lmente útiles en adenocarcinomas de muje- rúrgica, no tiene demasiada importancia. Conviene comprobar la per-
res no fumadoras) y en primera línea siempre que exista mutación meabilidad de los drenajes y, en caso necesario, instaurar uno o dos su-
del EGFR. plementarios.
Bevacizumab: se trata de un ant icuerpo monoclonal inhibidor
de la angiogénesis, que b loquea el factor de crec imie nto de l En ocasiones, el enfisema se acompaña de un empeoram iento súbito,
endotelio vascu lar (VEGF) . Está indi cado en p rim era línea (aso- que se ca racteriza por disnea y expectoración de líquido serohemáti-
ciado a un p lat ino) en el carcinoma no microcítico y no ep ider- co, que traduce la aparición de una fístu la en el muñón bronqu ial. Esta
moide. situación es urgente, y requiere la colocación inmediata de un drenaje
endotorácico. El paciente debe coloca rse en reposo en decúbito lateral
En algunas ocasiones, pacientes en estad io IV pueden ser ca ndidatos a del lado intervenido (para evitar la aspiración al pulmón contralateral del
cirugía. Los pacientes con metástasis cerebra l, pulmonar o suprarrenal líquido de la cavidad pleural). Si la fístu la apa rece antes del t ercer día de
única pueden ser va lorados para cirugía siempre que el tumor primario posoperatorio, se asume que se trata de una dehiscencia de suturas y se
sea resecable. procede a la reintervención y sutura de la misma. Sin emba rgo, si han
pasado más de tres días, el cierre de la fístula no ofrece buenos resu ltados
Carcinoma microcítico (porque ya se ha formado tejido de granulación y aparecerá una nueva
dehiscencia), siendo de elección la realización de una toracostomía para
La base del tratam iento es la quimioterapia. drenaje del líq uido pleural y cura de la zona de la dehiscencia.

La enfermedad limitada se trata con quim ioterapia y radioterapia torá- En revisiones posteriores, si se comprueba que la fístu la se ha cerrado,
cica. En los raros casos de estad io 1 o 11 sin adenopatías m ediastíni cas también se puede cerra r la toracostomía y/o realizar una toracoplastia.
tumora les puede p lantearse ta m bién el tratamiento quirúrg ico.
El enfisema subcutáneo en un paciente posoperado no es im portante
La enfermedad extendida se trata con qu imioterapia. En caso de metás- por sí mismo, sino por ser la traducción de lo que ocurre bronquial mente.
tasis encefálicas se realiza también rad ioterapia holocraneal. Una vez resuelta la situación causal, el enfisema desaparece progresiva-
mente y si n comp licaciones. Tan sólo en enfisemas muy importantes, que
Tanto en la enfermedad limitada como en la extendida está ind icada la afecten al territorio fac ial, se rea lizará una incisión cutánea cervical para la
radioterapia holocraneal profiláctica si tras el t ratam iento se objetiva res- expresión manual del aire, o bien se colocarán múltiples agujas hipodér-
puesta completa . micas en los tej idos enfisematosos.

Complicaciones posquirúrgicas Estudio de operabilidad

Hemorragia Clásicamente se ha considerado q ue los pacientes con una FEV, mayor


de 2.000 mi pueden tolerar una neumonectomía, y una lobectomía si el
En caso de sangrado por los drenajes torácicos con débito mayor de FEV, es mayor de 1.500 mi.
200 ml/h durante más de tres horas, o de más de un litro en las prim era s
24 horas, se rea lizará revisión quirúrgica para contro lar el punto hemo- Los crite ri os de inoperabilidad aparecen refl ejados en la Tabla 7. Si existe
rrágico. enfermedad pulmonar concom itante se va loran una vez que el paciente
haya sido trat ado correcta mente.
Hipotensión

Genera lmente es secunda ri a a la citada hemorra g ia, a infarto ag ud o Mal estado clínico (Karnofsky < 50%)
de miocard io, al emp leo de sedantes o a t apona mi ento cardíaco en Enfermedades asociadas graves e incontrolables
pacientes en los que se abrió el pericard io durante la intervención. FEV, preoperatorio real < 1 1, irreversible
FEV, posoperatorio pred icho< 0,81 y< 30% del teórico
Trastornos del ritmo cardíaco DLCO < 40%
ve < 40% irreversible
En relación con la ansiedad, el dolor, los sedantes, hipoxemia o la mani-
PaC0 2 > 45 mmHg irreversible
pulación quirúrgica.
Hipertensión pulmonar grave
IAM 3 meses previos
Edema pulmonar
Arritm ia ventricular incontrolable
Generalmente por excesivo aporte de líquidos en el posoperatorio. Tabla 7. Criteri os clásicos de inopera bilidad

Atelectasias Los criterios expuestos en la Tabla 7 son los criterios clásicos, cuyo uso
se ha ido abandonando al esta r basados en va lores absolutos de función
A causa de una mala realización de fisioterapia respirat oria, si el paciente pu lmona r. En la actualidad, para va lorar la operabilidad de un paciente, se
no recibe una ana lgesia adecuada. utiliza el algoritmo que se muestra en la Figura 14.

04 · Neoplasias pulmonares
Cirugía torácica 1 04
4.3. Nódulo pulmonar solitario
--------~~ Ambas~80%

i
alguna < 80%
Se trata de una opacificación radiológ ica de hasta 3 cm de diámetro mayor
(para algunos autores, de hasta 6 cm), rodeada de parénquima pu lmonar

~ sano con bordes bien delimitados, no asociada a atelectasia o adenopatías.

< 35% y
<1Oml/kg/min
~--------
-. -
••:.ea > 75% o
_______... >20 ml/kg/ min Etiología
35-75%y
t Maligno.
10-20 ml/kg/min
Ca rcinoma broncogénico.
t Carcinoide.
Metástasis solitaria.

t~
alguno < 30% ambos > 30%
Granu loma infeccioso.
Granu loma no infeccioso (artritis reumatoide, Wegener, sarcoidosis).

t Factores de riesgo de malignidad

< 35% o
/
<10 ml/kg/min
> 35%y
> 1O ml/kg/min
Edad: la edad inferior a 35 años sugiere benignidad.
Fumador: el tabaqu ismo aumenta el riesgo de malignidad.
Neoplasia previa: aumenta el riesgo de ma lignidad.

/ ~ Tiempo de crecimiento: un NPS sin cambios de tamaño durante más


de dos años es benigno. En general, los nódulos benignos tienen un
Se puede realizar Se puede realizar
Inoperable lobectomfa neumonectomía t iempo de duplicación menor de 30 días o mayor de 400 días.
Calcificación: son patrones de calc ificación benigna las centrales, di-
V0 2 pico: consumo máximo de oxígeno (med ido con test de esfuerzo fusas, concéntricas, laminares o en "palomitas de maíz'~
cardiopulmonar) Márgenes: los bordes regulares y redondeados sugieren lesiones
PP: posoperatorio pred icho (para el cá lculo de la función pulmonar benigna s. El signo de la "corona radiada" es altamente sugestivo de
posoperatoria predicha, véase texto) malignidad.
Figura 14- Criterios actuales de operabi lidad Tamaño: casi todos los nódulos de más de 2 cm de diámetro son
malignos. en contraposición a los de menos de 2 cm (SOo/o de ma-
El cálcu lo de la función pulmonar posoperatoria predicha se puede rea- lignidad).
lizar con la fórmula : Cavitación: las cavidades de paredes gruesas sugieren malignidad.
Nódulos satélite: los nódulos pequeños asociados a uno dominante
FPPP = FPPR - (F PPR x 9 ) f sugieren benignidad con alta probabilidad.

siendo FPPP= función pulmonar posoperatoria predicha, FPPR= función


pu lmonar preoperatori a rea l, a= número de segmentos funcionantes en Sol icitar siempre Rx antiguas del paci ente.
el tejido a resecar. La funcionalidad de los segmentos se va lora con bron-
coscopia y/o técnicas de imagen.

La hipoxemia y la hipercapnia aumentan el riesgo quirúrgico, pero no Diagnóstico


se consideran contra ind icaciones absolutas, debiendo ser va loradas de
forma individual en cada paciente. TC torácica

En todo paciente con NPS de reciente diagnóstico, se debe realizarTe torá-


4.2. Tumores pulmonares cica con contraste. La PET no es sensible para lesiones de menos de 1 cm.
Tiene un alto va lor predictivo negativo en pacientes con nódulos sospe-
metastásicos chosos de benignidad. En algunos centros la utilizan, previa a la cirugía de
un nódulo sospechoso de malignidad, para detectar lesiones metastásicas.
El pulmón es diana frecuente de las metástasis de tumores de casi cua l-
quier localización. Puede dar imagen de "suelta de g lobos" o de linfangitis Broncoscopia
carcinomatosa con patrón intersticial.
Tiene un rendimiento bajo para diag nóstico de NPS, excepto si se realizan
Se realiza extirpación quirúrgica sólo si el tumor primario está contro lado, biopsias tra nsbronqu iales con guía tlu oroscópica. El rend imiento aumen-
no hay extensión a otros ó rg anos y el paciente tolerará la resección pu l- ta si la lesión tiene un tamaño mayo r de 2 cm, o tiene un "signo del bron-
monar. Se ha ensayado con casi cualqu ier tumor, pero los mejores resul- quio" positivo. No está indicada de rutina en el estudio del NPS, pero se
tados los ofrece el osteosarcoma. considera la técnica de elección si se requiere el diagnóstico histológico.
Manual CTO de Medicina y Cirugía, 2 .a edición

Todo NPS en mayores de 35 años es mali gno, mi en-


tras no se demuestre lo contrario.
Recuer4a
Punción transtorácica (PTI)

Genera lmente guiada por TC. Se reali za si la broncoscopia no ha llega-


do al diagnóstico. Puede ser de elección en nód ulos muy periféri cos. La
comp licación más frecuente es el neumotórax.

VATS
Consiste en una videotoracoscopia con resección del nódulo pulmonar.
Es la prueba de elección, ante pacientes de alta sospecha de malign idad,
cuando otras técn icas no han llegado al diag nóstico. Figura 16. Ca rcino ide endobronqu ial

La actitud diagnóstica ante un nódulo pu lmonar solitario (NPS) depende El carcinoide, al ig ual que el carcinoma microcítico, deriva de la célu-
del riesgo de malignidad del mismo (Figura 15). la de Kulch itsky. Esta s cé lulas pertenecen al sistema neu roendocrino
(sistema APUD) y tienen g ránulos neurosecretores que contienen múl-
tiples hormonas. En ocasiones, estos tumores presentan síndromes pa-
raneop lásicos, siendo el m ás típico el sín drome ca rcinoide (rubefacción,
Probabilidad baja Probabilidad media 1 alta
de malignidad de malignidad broncoconstricción, diarrea y lesiones va lvulares ca rdíacas). A diferencia
de los ca rcino id es originados en el tracto gastrointestinal, en esta loca-
lizació n no se req uiere la existe ncia de metá stasis hepáticas para la apa-

1 1 rición de síndrom e carcino ide, aunque es más frecue nte si est as están
presentes. También puede segregar ACTH, ADH o producir un síndrome
Seguimiento radiológico:
• Cada 3 meses el primer año
pe lag ro ide.
• Cada 6 meses el segundo
En ocasiones hay metástasis (ca rcinoi de atípi co o maligno), siendo las
Figura 15. Guía diagnóstica del nódulo pu lmonar solitario más frecuentes en ga nglios linfáticos y en hígado. Suelen ser de loca-
lizació n centra l, en pacientes menores de 40 años, sin relació n con el
tabaquismo y con clínica de tos crónica, hemoptisis (es un tumor muy
4.4. Tumores benignos vascularizado) o atelectasia.

El tratam iento de elección de todos los "adenomas" es la resección qui-


Suponen menos del So/o de los tumores primarios del pulmón. Incluyen rúrgica. Con frecuencia, es suficiente la b roncotomía con escisión local
funda mentalmente los adenomas y los hamartomas (la suma de ambos o la resección atípica. En el caso de los ca rci no ides atípicos, el t rata-
su pone más del90o/o de los tumores ben ignos). miento es simi lar al del ca rc inoma microcíti co de pu lmón con pol iqui-
mioterapia.
Adenomas bronquiales
Hamartoma
Son los tumores "benignos" más frecuentes (representan la mitad de los
casos). De ellos, el 80% son carcinoides (Figura 16), el 10- l So/o cilindro- Es el tumor estrictamente benig no más frecuente en el pulmón. General-
mas (tu mor adenoide-quístico) y un pequeño porcentaje ca rcinomas mente apa rece en ho mbres mayo res de 60 años. Es de localización peri-
mucoepidermoides. La mayoría son lesiones intrabro nquiales de creci- férica y suele ser un hallazgo asintomático en la radiografía. Desde el pun-
miento lento y de loca lizació n central. Suelen presentarse en pacientes to de vista anatomopatológ ico se trata de una malformación en la que se
jóvenes (40 años). pueden ver mezclados tejidos diversos (músculo, grasa, ca rtíl ago, hueso).

Para muchos autores ha variado la clasificación de estos tumores. El tér- La calcificación en "pa lomitas de maíz" se considera patognomónica del
mino "adenoma" ha caído en desuso y el carcinoide, cilind roma y carcino- hamartoma, así como la existencia de densidad grasa en su interior (signo
ma mucoepidermoide han pasado a ser co nsiderados como neoplasias rad io lóg ico que se puede observa r en la TC t orácica). Sa lvo que aparez-
de baja malignidad, ya que algunos de ellos pueden comportarse como can algunos de estos dos sig nos, es frecuente que se haga resección para
neoplasias malignas, incluso originando metástasis. aseg urarse de que no es una neoplasia maligna.

04 · Neoplasias pulmonares
Cirugía torácica 1 04
" La hipercalcemia es un síndrome paraneoplásico producido por
Ideas clave ff!S el carcinoma epidermoide.

" El adenocarcinoma es el tipo histol?gico más frecuente. " El carcinoma microcítico es la variedad que más síndromes pa-
raneoplásicos produce. Destacan el SIADH (hipon atremia) y el
" La causa más habitual de síndrome de Pancoast es el carcinoma síndrome de Cushing.
epidermoide.

" El carcinoma microcítico es la causa más frecuente de síndrome


devenacavasuperio~

3) La clasificación TNM que le corresponde es (T2aNO MO).


Casos clínicos 4) La presencia de una alteración ventilatoria obstructiva contrain-
dica la resección quirúrgica del tumor.
Hombre de 62 años de edad, de profesión taxista, con diag-
nóstico previo de bronquitis crónica sin tratamiento alguno, RC: 3
que consulta por esputo hemoptoico. La exploración física es
normal. La radiografía simple de tórax muestra una masa en Varón de 68 años de edad, fumador activo con IPA acumulado
lóbulo superior derecho que no existía un año antes. La bron- de 50, que consulta por cuadro clínico de aumento de su tos ha-
coscopia objetiva mucosa de aspecto tumoral en bronquio lo- bitual de tres meses de evolución y hemoptisis en la última se-
bar superior derecho que se biopsia y resulta ser un carcinoma mana. En la Imagen se muestra la TC torácica. Tras la realización
epidermoide. El cálculo de FEV 1 posoperatorio predecible es de de broncoscopia se diagnostica un carcinoma broncogénico tipo
900 ce para una lobectomía superior derecha con test de bron- adenocarcinoma en el lóbulo superior izquierdo. La PET demues-
codilatación positivo. La resonancia magnética del cerebro es tra captación patológica de fluorodesoxiglucosa únicamente a la
normal. La tomografía computarizada (TC) muestra una masa altura de la masa pulmonar. La espirometría muestra una fun-
de 5 cm en lóbulo superior derecho, con adenopatías paratra- ción pulmonar con un FEV, del 85% del valor teórico. Señale la
queales derechas de 15 mm de diámetro. ¿Cuál es el siguiente respuesta correcta:
procedimiento a realizar?

1) Lobectomía superior derecha y linfadenectomía mediastínica


radical.
2) Tratamiento quimioterápico combinado definitivo.
3) Mediastinocopia de estadificación.
4) Realización de tomografía por emisión de positrones (PET).

RC: 3

Paciente de 69 años, fumador de 30 cigarrillos/día desde hace 40


años, a quien, a raíz de un episodio de hemoptisis, se le practica
una TC torácica que demuestra la presencia de una masa peri-
hiliar de 5 cm que provoca una atelectasia completa de lóbulo
superior derecho y ausencia de adenopatías mediastínicas y de
otras anomalías torácicas. La broncoscopia confirma la presencia
de una masa de aspecto neoplásico en la entrada del lóbulo su-
perior derecho, situada a más de 2 cm de la carina principal, cuyo
diagnóstico histológico es de carcinoma escamoso. La espirome- 1) Hay que realizar una lobectomía superior izquierda, con linfade-
tría forzada muestra una FVC de 4.200 (88%) y un VEMS de 2.400 nectomía mediastínica sistemática.
(76%) con un cociente VEMS/FVC de 69%. ¿Cuál de las siguientes 2) El tratamiento indicado es la quimiorradioterapia.
afirmaciones es correcta? 3) Hay que realizar un método de estadificación mediastínica (me-
diastinoscopia/mediastinotomía, punción transbronquial).
1) Se trata de un cáncer de pulmón en estadio 111. 4) Es inoperable. El tratamiento es radioterapia.
2) El tratamiento de elección es la quimioterapia neoadyuvante y
posterior cirugía de resección. RC: 3
1
Cirugía torácica_

Trasplan te de pulmón

ORIENTACIÓN Tan sólo hay que centrarse fibros is quística, enfi sema por d éfi cit d e a 1-an titrip sina, hipert ensión

ENARM en las Ideas Clave. pulmonar primaria y sarcoidosis. El TP se debe considerar cuando se
hayan agotado todas las demás opciones terapéuticas y cuando la ex-
pectativa de vida tra s el trasp lante supere a la de la enfermedad. Sin
embargo, en muchos pacientes el obj etivo primario del TP es mejo-
EI trasplante de pulmón (TP) (Figura 17) es una opción terapéutica pa - rar la ca lidad de vida, aunque la mejoría en supervivencia pueda ser
liativa aplicab le a pacientes se leccionados que padezcan cua lquier enfer- marginal. Las ind icac io nes para las enfe rm edades más frecuentes se
medad respirato ria cróni ca no neoplásica, grave e irreversible. El objetivo recogen en la Tabla 8.
fundamenta l del TP es alargar la vida, mejorar la función pulmonar y me-
jorar la ca lidad de vida.
1

EPOC y enfisema FEV 1 posbroncodilatador < 25o/o


por déficit PaC0 2 ~ 55 mmHg
dea 1-
Hipertensión arterial pulmonar
antitripsina
Anastomosis
FEV 1 < 30%
Pa0 2 <50 mmHg
PaC0 2 > 50 mmHg
Hipertensión arterial pulmonar
Ma la situación clínica a pesar de tratamiento
óptimo:
bronquial
- Ingresos hospita larios frecuentes
- Hemoptisis masiva recurrente
- Rápida caída del FEV1

FVC o CPT < 60%


DLCO< 50%
Hipertensión arterial pu lmonar
Corazón Pa0 2 < 60 mmHg en reposo o durante el ejercicio
Venas pulmonares Progresión de la enfermedad a pesar
del tratamiento

Clase funcional NYHA 111 o IV a pesar


de tratamiento óptimo
Figura 17. Trasplante pu lmo nar
Perfil hemodinám ico desfavorable:
- Presión auricular derecha > 15 mmHg

5.1. Indicaciones y contraindicaciones - Presión media en arteria pulmonar > 55 mmHg


- fndice cardíaco < 2 l/min/m 2

Tabla 8. Criterios para la indicac ió n de traspl ante de pulmó n


Las enfe rm edades susceptibl es de ser tratadas co n TP abarcan prácti-
camente toda la patol ogía neumológica. En la actualidad, la ind icac ión La mayoría de los centros sitúan en 65 años el límite de edad. Las contra-
más frecuente es la EPOC, seguida por la fibrosis pulmonar idiopática, indicaciones típicas para el TP se deta llan en la Tabla 9.
Cirugía torácica 1 05
Infección porVIH Complicaciones
Antigenemia para hepatitis B crónica
Infección crónica activa por hepatitis C Disfunción primaria del injerto
Tumor maligno con periodo libre de enfermedad inferior a 5 años
Tabaquismo activo También llamada resp uesta a la implantación y edema de reperfusión. Con-
Abuso o dependencia de alcohol o drogas siste en un daño pulmonar agudo que aparece en las primeras 72 horas
Proceso infeccioso no controlable tras el trasplante, y cursa con infiltrado pulmonar difuso e hipoxemia. Es ne-
fndice de masa corporal < 17 o> 30 cesario realizar diagnóstico diferencia l con neumonía nosocomial, obstruc-
Falta de cumplimentación reiterada del t ratam iento médico prescrito ción venosa pulmonar y rechazo hi peragudo. Su gravedad es variable. Re-
Enfermedad significativa de cua lquier otro órgano vital distinto al pulmón qu iere tratamiento de soporte, incluso óxido nítrico inhalado y oxigenación
Carencia de un entorno familiar y social adecuado que garantice por membrana extracorpórea en los casos graves. La mayoría se recuperan,
el cumpl imiento terapéutico pero los casos graves son una causa frecuente de mortalidad temprana.
Tabla 9. Contraindicaciones para el trasplante de pulmón
Complicaciones de la vía aérea
Elección del tipo de trasplante
En un 4-20% de los casos aparecen complicaciones de la vía aérea como
El TP puede ser unipulmonar, bipulmonar o cardiopulmonar. Aproxima- dehiscencia de suturas, estenosis y broncomalacia. Se suelen resolver
da mente la mitad de los que se realizan son un ipulmonares, y la otra mi- con tratam iento broncoscópico que puede incluir desbridamiento, dila-
tad bipulmonares. El cardiopulmonar supone menos del So/o de total. tación, resección con láser y colocación de prótesis endobronquial.

El tipo de trasplante indicado depende de la enfermedad pulmonar del Rechazo


receptor y de la conveniencia o no de dejar uno de los pulmones. El TP
bipulmonar es necesa ri o cuando el paciente que va a ser trasplantado Agudo. Es una respuesta inmunológica caracterizada por inflamación lin-
tiene pu lmones con focos infecciosos (bronqu iectasias, fib rosis quísti- focitaria arteri olar y bronquiolar. Con los regímenes actuales de inmuno-
ca). El cardiopulmona r es necesario para el síndrome de Eisenmenger supresión el 50% de los pacientes presentarán, al menos, un episodio du-
con alteraciones comp lejas que no pueda n ser reparadas junto con un rante el primer año, aunque es más frecuente en los tres primeros meses.
TP, o en el caso de enfermedad pulmonar y cardíaca concom itante; no Puede ser clínicamente silente, o puede manifestarse con síntomas como
es necesario el trasplante cardiopulmonar cuando existe cor pulmonale, tos, fiebre, disnea, hipoxemia y crepitan tes. En más de la mitad de los casos
porque la función ventricu lar derecha se recuperará cuando el TP nor- aparecen infiltrados pulmonares difusos y deterioro de la función pulmo-
malice la poscarga vascula r pulmonar. En la EPOC y en el enfisema por nar. El diagnóstico debe ser confirmado mediante biopsia transbronquial
déficit de al-antitripsina existe evidencia de una mayor supervivencia (sensibilidad del 70%). El tratamiento habitual consiste en ciclos cortos con
con TP bipulmonar frente al unipulmonar, pero no hay datos conclu- altas dosis de corticoides y ajustar el tratamiento inmunosupresor. La res-
yentes para otras enfermedades. En la actualidad, la causa más frecuen- puesta al tratamiento suele ser favorable en la mayoría de los casos.
te de TP unipulmonar es la EPOC, del bipulmonar, la fibrosis quística, Crónico. Es la principal causa de muerte tardía. Su prevalencia a los cinco
y del ca rd iopulmonar, la hipertensión pul monar p ri ma ria. El trasplan- años del trasplante es del 50%. Los principales factores de riesgo para su
te de un lóbu lo de donante sano tiene un papel lim itado en el TP de desarrollo son los episodios previos de rechazo agudo, neumonitis por
ad ulto; se ha realizado, sobre todo, en adolescentes y adultos jóvenes citomegalovirus, bronquiolitis linfocitaria y fa lta de cumplimentación del
con fibrosis quística, y los resultados son comparab les al TP con órgano tratamiento inmunosupresor. En la mayoría de los casos aparece a partir
procedente de cadáve r. del primer año. Se trata de una bronquiolitis obliterante, y se manifiesta
como una obstrucción progresiva al flujo aéreo. El diagnóstico se basa
Manejo postrasplante en una progresiva disminución del FEV 1 (~ 20%), aunque la mayoría de
los pacientes se detectan en fase asintomática al observar una disminu-
Tras el TP se requiere tratamiento inmunodepresor de por vida, y profi- ción del FEF 25_75,.. Ningún tratamiento ha demostrado eficacia.
laxis contra algunos patógenos (Pneumocystis jiroveci, citomegalovirus,
hongos) durante largos periodos de t iempo. El tratam iento inmunosu- Infecciones
presor consiste en la combinación de tres o cuatro fármacos: co rti co i-
des, inh ibidores de la síntesis de nucleótidos (azatioprina, micofenolato Son la primera causa de morta lidad durante el primer año. Las más fre-
mofetil), inh ibidores de la calcineurina (cic losporina A, tacrolimus) y, en cuentes son:
ocasiones, anticuerpos antirreceptores de la IL-2 (daclizumab, basilixi- Bronquitis aguda y neumonía. Pueden acontecer en cualquier
mab) momento, pero ocurren en prácticamente todos los pacientes du-
rante el periodo posoperatorio. Más tardíamente también son fre-
Resultados cuentes los episodios de bronquitis, sobre todo en pacientes que
desarrollan bronquiolitis obliterante debida a Pseudomona aerugino-
En general, la supervivencia tras el TP se estima en un 70% al primer sa o Staphy/ococcus aureus resistente a metici lina.
año, y un 45-50% a los cinco años. Entre las causas de trasplante, la en- Neumonía por citomegalovirus. Es la infección vírica más frecuente, aun-
fermedad en la que se cons igue mayor su pervivencia a los cinco años que en la actualidad, con las pautas de profilaxis, su incidencia es dellSo/o.
es la fibrosis quística, y las que menor supervivencia presentan son la La mayoría aparece en los seis primeros meses. Desde el punto de vista
fibros is pulmonar idiopática y la EPOC. La primera causa de muerte du- clínico puede ser muy similar al rechazo agudo. Se trata con ganciclovir.
rante el primer año son las infecciones y, tra s el primer año, el rechazo Aspergillus. Es la infección por hongos más frecuente y presenta una
crónico. elevada mortalidad.
Manual CTO de Medicina y Cirugía, 2. 8 edición

" La infección bacteriana es la más frecuente en el posoperatorio


Ideas clave "~ inmediato.

" La indicación más frecuente de trasplante unipulmonar es la " El rechazo crónico se manifiesta como una bronquiolitis oblite-
EPOC, y de trasplante bipulmonar la fibrosis quística. rante.

" La causa más frecuente de mortalidad durante el primer año


son las infecciones y, a partir del primer año, el rechazo crónico.

05 · Trasplante de pulmón
eirugia torácic___a._____ __

Traumatismos torácicos

ORIENTACIÓN Es conveniente conocer al


Neumotórax a tensión (Figura 18)

ENARM menos las Ideas Clave.


Se debe identificar precozmente, incluso prehospita lario: es ca usa de mor-
ta lidad precoz evitable. Ante la sospecha clínica (ausencia de murmullo
vesicu lar, sin movimiento torácico, timpan ismo, desviació n traqueal contra-
Los t raumatismos to rácicos aparecen en un ?So/o de los polit raumatizados lateral e ingurg itación yug ular, con insuficiencia respiratoria) si n necesidad
graves. de co mprobación rad iológica se debe rea liza r descompresió n inmed iata
co n aguja gruesa en 2° espacio intercosta l línea medioclavicular.

6.1. Reconocimiento primario


de lesiones con compromiso vital
inmediato

Obstrucción de la vía aérea por lesión de la vía aérea principal. Para


conserva r la vía aérea libre, puede ser necesa ri a la realización de una
traqueotomía urgente, a la altura del segundo o tercer an illo traqueal
o intubación.
Alterac ión de la ventilación. Neumotórax a tensión, abierto o volet
(tórax inestable).
Alterac iones hemodinámicas: taponam iento cardíaco, hemotórax
masivo o embolismo gaseoso.

Neumotórax abierto
Es consecuencia de una herida penetrante del tórax. El aire entra en el tórax
con más fac ilidad en la inspi ración que cua ndo sa le en la espiración, lo que Fig ura 18. Neumotórax a tensión : colapso del pulmón derecho
origina un colapso progresivo del pulmón. En la inspiración, el mediastino con desviación del mediastino a la izquierda
es "empujado" hacia el lado sano y en la espiración, hacia el lesionado, pro-
duciéndose un "bamboleo" mediastínico que disminuye el retorno venoso Volet costal o tórax inestable (Figura 19)
y, en consecuenc ia, el gasto cardíaco. Suele asociar lesión pulmonar.
Se debe a una doble fractura en tres o más niveles adyacentes. Esto oca-
La primera medida de urgencia consiste en restaurar la integridad de la pared siona una porción central "fl ota nte" en la pared torácica que oscila con la
torácica mediante un apósito fijo en tres puntos (efecto valvu lar) y colocación respiración de un modo inverso o paradójico respecto al resto de la pared.
de drenaje endotorácico lejos de la herida. Posteriormente cierre definitivo.
Lo importante no es el daño de la pared sino las lesiones asociadas: con-
tu sión pulmonar (determ inante de la insuficiencia respiratoria), hemotó-
rax y neumotórax.
El tratamiento ini cial de elección es el taponamiento
parcia l de la heri da, para permitir la sa li da del aire,
pero imped ir su entrada. Para su tratamiento, el primer paso es la analgesia que perm ita fis iotera-
pia respiratoria y, el seg undo, el cont rol de la función respiratoria: en caso
Manual CTO de Medicina y Cirugía, 2.a edición

de evolucionar a insuficiencia respiratori a, se rea liza ventilación mecánica so luciona r el taponam iento card íaco abriendo el peri ca rdi o, masajear
con presión positiva, que es necesario en el 70o/o de los pacientes con d irectamente el corazón, el control del sang rado torácico d irectamente,
volet. El tercer paso es el control de líquidos y el cuarto, sólo en los casos el control del emboli smo gaseoso por clampaje hi liar, con redistribu-
en los que exista un hundimiento de todo el hemitórax con comprom iso ción de flujo a órganos vitales, corazón y cerebro (med iante clampaje
restrictivo se indicará la fij ación quirúrgica. aórtico). Si hay respuesta, se t rasladará a la sala de operaciones para
t ratam iento definitivo.

6.2. Reconocimiento secundario


de las lesiones torácicas

Inspiración
Lesiones de la pared torácica

Fractura costal

Su loca lización más frecuente es el punto de impacto (a menudo lateral).


De la 4• a la 9• costilla. Muy poco habitua les en niños.

Su diag nóstico suele ser clínico (crepitación ósea, signo de la tecla, dolor
a la palpación) y radiológ ico.
Espiración
En cuanto a su manejo, generalmente es ambu latorio sa lvo si es mayor
Figura 19. Voletcostal de tres fracturas, en los ancianos y en pacientes con EPOC que asocien
lesión pu lmonar. Los aspectos fundamentales son tres:
Hemotórax masivo Rea lización de fis ioterapia respirato ria.
Instauración de una ana lgesia adecuada. Se utilizarán analgésicos
Se produce por laceración pulmonar o sang rado de la pa red o lesiones de menores, en caso de menos de tres fracturas. Si son más de tres, ha-
los gra ndes vasos, diafragma u ó rga nos abdom inales. Ante la sospecha se brá que emplea r analgésicos mayores. En ocasio nes se debe real izar
debe proceder a la colocación de un tubo de tórax de g ru eso ca li bre. bloq ueo intercost al o paravertebra l.
Exclusión de lesión de vecindad, que es más frecuente en las fractu-
Dependiendo de la cuantía y velocidad del sa ng rado se clasifican en: ras de 1.• o 2.• costillas (que indican traumatismo intenso y asocian
Hemotórax masivo: cuando se produce la sa lida por el tu bo de tórax lesiones vasculares subclavia y plexo braq uial) o de 9.• a 12.• despla-
de más de 1.500 mi de sa ngre(> 20 ml!kg). zadas (que pueden implica r lesiones de hígado, riño nes o bazo).
Hemotórax continuo: cuando se produce la sa lida por el tubo de tó-
ra x de más de 200 mi de sangre/hora d urante 3-4 horas. Fractura de esternón

Son indicación de toracotomía urgente. Se sospecha por dolor esternal a la pa lpación. El diagnóstico se obtiene
por rad iog rafía lateral. El tra tami ento es ig ual al de las fracturas costa les
(reposo, ana lgesia y fisioterapia) . Puede asociar contusión miocárdica
En general, las hemorragias persistentes son secunda- (siempre hay que realizar un ECG y enzimas ca rdíacas).
rias a la lesión de una arteria intercostal o de la arteri a
mamaria interna, mi entras que la hemorragia proce- Lesiones del parénquima pulmonar
dente del parénquima pulmonar suele interrumpirse
en pocos minutos, dada su baja pres ión.
Neumotórax simple

Taponamiento cardíaco La causa más frecuente es una fractu ra costa l con el extremo de la
costilla fracturada lacerando la pleura visceral. Otras causas pueden
Generalmente se produce por trauma penetrante. En la exploración se ser la iatrogenia (ve ntilación mecánica, colocación de vía centra l) o
presen ta la tríada de Beck: hipotensión, ing urgitación yug ular y tonos una herida penetrante. Sea cual sea el dese ncadenante, se instaura un
apagados cardíacos. El d iagnóstico clín ico de sospecha debe confirmarse mecan ismo valvular por el que el aire pasa al espacio pleura l, pero no
con la realización de una ecocardiografía. Si existe compromiso vital, hay puede salir, con la consecuente inestabilidad hemodinámica si es ma-
que realizar una descompresión por pericardiocentesis subxifoidea pre- sivo (shock no hipovolémico sino por mecanismo compresivo) como
via a la toracotomía. ya se ha visto.

La toracotomía de reanimación se rea liza en la sala de reanimaci ó n En su tratam iento hay que mantener una actitud conservadora si es me-
en un paciente agónico. Está indicada en traumas penetrantes, con le- nor del 15-20o/o y en individuos asintomáticos (excepto traslado, necesi-
siones aisladas a escala cardíaca en pacientes que llegan con signos de dad de ventilación mecánica o cirugía). En el resto de los casos se realizará
vida. Se realiza toracotomía anterolateral izqu ierda. Sus objetivos son: un d renaje torácico conectado a aspi ración.

06 · Traumatismos torácicos
Cirugía torácica 1 06
Hemotórax simple

En el manejo inicial habrá de insertarse un tubo de tórax de calibre grue-


so, debido a que la reexpansión ayuda a la hemostasia y permite la cuan-
tificación de sangre para determinar la necesidad de cirugía .

Genera lmente, la sangre en la cavidad pleural no se coagula, debido a la


presencia de enzimas anticoagulantes, pero un pequeño porcentaje sí
lo hace, evolucionando en la tercera o cuarta semana a fibrotóra x, cuyo
tratamiento será la decorticación lo más temprano posible (ideal en la
primera semana) abierta o por videotoracoscopia. En ocasiones, se pue-
de evitar la evolución a fibrotórax mediante la instilación de urocinasa en
la cavidad pleura l.

Contusión pulmonar

Es una lesión cuya gravedad es muy variable y que puede no ser evidente
en la radiografía de tórax inicial. Es la principal causa de m uerte en trauma
torácico.
Figura 20. Herniació n gástri ca po r defecto diafrag mático izquierdo.
En la clínica se trata de un paciente con antecedentes de un trauma to- Apa ri ción de niveles hid roaéreos en tórax
rácico reciente, que puede presentar d isnea, taquipnea, hemoptisis y, en
ocasiones, febrícu la. Radio lógicamente, aparecen uno o más infiltrados Para su tratamiento, la primera medida es la colocación de una sonda na-
alveolares, más evidentes a las 24 horas. La TC torácica es más sensible sogástrica para evitar la broncoaspiración. A continuación, está indicada
y específica para valorar las zonas co ntundidas. En la gasometría arterial la reparación qu irúrgica más o menos urgente, en función de la clínica.
existe hipoxemia .
Cuando se detecta precozmente se opera por laparotomía, mientras que
En su tratamiento hay que mantener una actitud expectante, con su- si se detecta tardíamente y pueden existir adherencias de vísceras abdo-
plementación de oxígeno, control del dolor y fisioterapia respiratoria. En minales con estructuras torácicas, se prefiere el abordaje por toracotomía
caso de insuficiencia respiratoria, se realizará ventilación mecánica. Habrá (Figura 21 ).
que intentar la restricción hídrica. La complicación más frecuente suele
ser la sobreinfección, con el desarrollo de una neumonía.

Laceración pulmonar

Son lesiones periféricas q ue da n hemotórax o neumotórax (d renaje to-


rácico). Las que progresan centralmente dañan los bronquios y vasos y
requieren lobectomía. Si la pleura está intacta, se produce un hematoma
intraparenquimatoso que puede manejarse conse rvadoramente, aun-
que puede sobreinfectarse.

Lesiones diafragmáticas

Localización más frecuente: hemidiafragma izquierdo.


Mecanismo más frecuente: accidente de automóvil o lesión penetran-
te. Por hiperpresión abdominal en traumatismos de alta intensidad. Precisa la reparación quirúrgica por vía abdominal o, como en este caso,
por vía torácica cuando se descubre tardíamente para eliminar adherencias,
En referencia a la clínica, si la lesión es pequeña, puede pasar desaperci- añadiendo material protésico si el defecto diafragmático es grande
bida, pero si se trata de un desgarro importante, el gradiente de presión Figura 21 . Reparació n diafrag mática
que existe a través del diafragma origina una herniación de las vísceras
abdom inales al interior de la cavidad torácica, presentando el paciente
disnea y cianosis.
Para diagnostica r la parálisis diafragmática se usa la
radioscopia.
En su diagnóstico suele existir elevación diafragmática. A veces, nivel hi-
droaéreo en relación con la víscera herniada (Figura 20).

Puede pasar desapercibida fácilmente en la radiología simple de tórax Lesiones del árbol traqueobronquial
y detectarse por TC o lavado peritonea l. Su sospecha puede ser una in-
dicación de lapa roscopia diagnóstica que a su vez puede repararla en Su localización más frecuente suele ser el bronquio principal derecho. En ge-
algunos casos o de VATS (videotoracoscopia). neral, es más frecuente la localización intratorácica que la tráquea cervical.
Manual CTO de Medicina y Cirugía, 2. a edición

El mecan ismo más común es el m ismo que en las lesiones d iafragmáticas. recho es la parte del corazón que más frecue ntemente se afecta en las
heri das por arma blanca, ya q ue se loca liza anteri ormente. Pueden ser
Presenta una clínica de hemoptisis, enfi sema subcutáneo y mediast ín ico y morta les por hemoperi ca rd io o por hemorrag ia masiva, aunque a veces
neumotórax (que será mayor o menor, en función del grado de comunica- la propia compresión hace hemostasia o el orificio en el pericardio evita
ción con la cavidad pleural). Es característica la no mejoría clínica con el dre- el taponamiento.
naje y fuga importante de aire por el mismo (se sa le el aire de la vía aérea).
Es imprescindible un diagnóstico rápido, mediante sospecha clínica, apo-
En la exploración, en ocasiones aparece signo de Hamman (sonido crujien- yado a veces por el ecocardiografía.
te, sincrónico con el latido ca rdíaco, que se debe a enfisema med iastínico).
En cuanto al tratam iento, el t rauma penetrante ca rdíaco suele suponer
El diagnóstico se rea liza mediante: visualización de la fístu la por fibrosco- una urgencia vital que a menudo obliga a toracotomía de rean imación
pia. Las indicaciones son: fuga aérea masiva, hemoptisis, enfisema cervi- ya explicada.
ca l profundo y mediastínico. La TC valorará las lesiones asociadas.
Se debe realizar una sutura cardíaca, y posteriormente descartar lesiones
Tratamiento: si la fibroscopia indica un desgarro longitudinal y de escasa asociadas va lvulares o de los tabiques mediante estudios hemodinámi-
long itud, y el drenaje endotorácico consigue reexpand ir el pul món, el cos. En el caso de taponamiento card íaco, la pericard iocentesis es de pri-
tratamiento es conservador. En caso contrario, se rea lizará toracotomía y mera elección.
cierre de la fístu la precoz.
Lesiones de los grandes vasos
Lesiones del corazón y grandes vasos
Tienen alta morta lidad inmediata, aunque en ocasiones las va inas periar-
Contusión cardíaca teria les pueden contener la hemorragia y dar lugar a un pseudoaneuris-
ma, que se intervendrá posteriormente.
El mecanismo más frecuente es un traumatismo cerrado en la cara ante-
rior del tórax. Suelen ser secundarios al impacto del volante del automóvi l Deben sospecharse ante un ensancham iento med iastínico (> 8 cm) en la
contra el pecho. La lesió n más frecue nte es una cont usión miocá rd ica radiog rafía de tórax de t odo t raumatismo ju nto con la pérd ida del botón
(20o/o de los traumas cerrados), aunque pueden prod ucirse lesiones va l- aórtico y la desviación de la t ráquea.
vulares (sobre todo la válvu la aórtica segu ida de la mitra!), rotu ra de la
pared de alguna cavidad cardíaca o del t abiq ue interventricular, rotura de Los aneu rismas tra umáticos casi siempre se deben a sección de la aorta
un vaso pericárd ico o coronario con hemopericardio, y otras. torácica a causa de un traumatismo torácico no penetrante. Casi todos
ellos se deben a lesiones por desaceleración, como suele suceder en ac-
Traumatismos menores (pelotazo) en un momento crítico del ciclo car- cidentes automovilísticos y en caídas desde una altura . La mayor pa rte de
díaco pueden ser causa de arritmia leta l. los aneurismas se producen en el istmo aórtico, inmed iatamente distal
a la arteria subclavia izquierda, a la altura de la inserción del ligamento
En la clín ica puede existir dolor de ca racterísticas ang inosas o síncope. A arterioso, ya q ue es donde se une la porción móvil de la aorta ascendente
veces apa rece un derrame pericárd ico sem anas o incl uso meses después con la fija de la descendente y se ejerce la mayor fuerza tangencial en las
del traumatismo. desaceleraciones bruscas (Figura 22).

En el d iagnóstico se evidencia eleva-


ción del segmento ST en el ECG de Arteria subclavia izquierda Hematoma contenido
por adventicia
forma difusa, aumento de la CPK. La
contusión miocárd ica puede prod ucir
alteracio nes electrocardiográficas si-
mila res a las del infarto, así como t ras-
tornos del ritmo y de la conducción.
Además, esta lesión puede producir
alteraciones de la contracti lidad mio-
cárdica y defectos en la gammagrafía
isotópica .

En lo referente al t ratam iento: por


reg la general, evolucio na favo rable-
mente. Se rea lizará ecocard iog rafía
(para descartar lesión asociada), seria-
ción enzimática y electrocard iográfica.

Herida cardíaca
Ligamento arterioso ISTMO AÓRTICO
Suelen prod ucirse por armas blancas
o armas de fuego. El ventrículo de- Figura 22. Rotura traumática de aorta

06 · Traumatismos torácicos
Cirugía torácica 1 06
Cuando la ruptu ra se produce en la aorta ascendente y dentro del pe- Otras
ricardio, el taponam iento cardíaco y la muerte son la reg la. Si la ruptura
se produce en la aorta descendente y se conserva la adventicia intacta, Enfisema subcutáneo
la hemorragia puede contenerse por estructuras vecinas y formarse un
pseudoaneurisma. Puede deberse a heridas penetrantes, rotura del árbol bronquial o esófa-
go o man iobra de Va lsa lva.
Estos aneurismas traumáticos también pueden ocurrir en otros segmen-
tos de la aorta, como el abdomi nal. Una masa localizada suele ser el único En cuanto al tratamiento la reso lución es espontánea en pocos días, que
signo evidente. Conform e esta aumenta, hay dolor o pará lisis por com- se puede acelerar respirando 0 2 al 1OOo/o. En caso de neumomediastino,
presión de los nervios. Si la masa es pulsátil, el diagnóstico es práctica- si existe colapso venoso mediastínico, se rea lizará una incisión cutánea
mente segu ro en la exploración física. cervical para la expresión manual del aire.

El tratamiento es quirú rgico, urgente si está inestable o diferido al estudio Asfixia traumática
angiográfico si está estable.
Se produce a pa rtir de una gran fuerza que comprime el tórax y la parte
Lesiones del esófago superior del abdomen (aplastam iento, buzos). El paciente presenta colo-
ración violácea en cabeza y parte superior del tórax, petequias y hemorra-
Son poco frecuentes incluso en penetrantes. Causan en fi sema med iastí- gias subconjuntivales. Un tercio de los pacientes sufren pérdida de cono-
nico y subcutáneo, hidroneumotórax, distrés y shock. cimiento. En ocasiones, hay una pérdida de visión, que será permanente
si se debe a hemorrag ia retiniana, o transitoria, si tan sólo existe edema
El diagnóstico se realiza mediante esofagograma y esofagoscopia. en la retina. No existe tratamiento específico. De los pacientes que sobre-
Su trata miento es quirúrgico. viven a las primeras horas, el 90o/o se recuperan por completo.

minuye el retorno venoso y el gasto cardíaco. Se debe realizar


Ideas clave Jii5 urgentemente un taponamiento parcial de la herida.

" El tratamiento del volet costal es el mismo que el de las frac- " En el neumotórax a tensión se instaura un mecanismo valvular
turas costales simples. En caso de evolucionar a insuficiencia por el que el aire pasa al espacio pleural, pero no puede salir,
respiratoria, se rea liza ventilación mecánica con presión po- con la consecuente inestabilidad hemodinámica. El tratamiento
sitiva. es el drenaje torácico urgente.

" El neumotórax abierto es consecuencia de una herida pene- " Si un hemotórax drena más de 1.500 mi al poner el drenaje torá-
trante en tórax. Produce un "bamboleo" mediastínico que dis- cico o más de 100 ml/h, tiene indicación de toracotomía.

Un joven sufre un traumatismo torácico por colisión en acciden-


Casos clínicos te de tráfico. Ingresa en el hospital con un Glasgow de 13, do-
lorido en costado e hipocondrio izquierdo. Se palpa crepitación
Varón de 24 años que ha ingresado en urgencias, 3 horas antes, en hemitórax izquierdo. La radiografía simple muestra fracturas
por haber sufrido un grave traumatismo en el hemitórax dere- de 6.a a 1o.a costillas izquierdas unifocales, así como neumotórax
cho. En las radiografías se aprecian fracturas costales múltiples mayor del 30% y derrame pleural. El paciente permanece hemo-
en este hemitórax, y es evidente el movimiento paradójico de la dinámicamente estable, TA sistólica alrededor de 11 O mm Hg. La
pared costal derecha a la inspiración; la situación hemodinámica gasometría arterial con oxígeno es: P0 2 75 mmHg, C0 2 25 mmHg
es aceptable. En la gasometría arterial hay hipoxia e hipercapnia y pH 7,45. Se coloca tubo de tórax, obteniendo aire y 500 mi de
acusadas, que han empeorado desde el ingreso, a pesar de estar- líquido hemorrágico. ¿Cuál de las siguientes respuestas estima-
se aplicando oxigenoterapia con mascarilla, con una Fi0 2 de 0,5 . ría la más correcta?
¿Cuál de estas medidas terapéuticas indicaría inmediatamente?
1) Estaría indicada toracotomía urgente, sin más pruebas.
1) Toracotomía exploradora, para determinar si hay lesiones pul- 2) Muy probablemente no req uiera toracotomía en ningún caso.
monares asociadas. 3) Antes de realizar una tora cotomía urgente, es indispensable
2) Colocar 2 tubos de aspiración en el hemitórax derecho. rea lizar una TC de tórax con contraste i.v. en embolada .
3) Intubación endotraqueal y ventilación mecánica con presión 4) Es preciso realizar arteriografía antes de tomar decisiones, ya
positiva. que es posible un sangrado de origen arterial cuya única solu-
4) Vendaje compresivo, que inmovilice el hemitórax derecho. ción es quirúrgica, aún en caso de estabilidad hemodinámica.

RC: 2 RC: 2
. - - - - -----·rugí aJDrácic_a_

Recom mende d readin g 1

lndicate the most probable etiological diagnosis for a patient with


normocytic normochromic anaemia, low reticulocytes and a deficit of
haematopoietic precursors in the medullary aspirate. The white-cell
and platelet series are completely normal. A chest X-ray is performed,
w hich m ay be seen below [Figure 1a):

1. Lymphoma.
2. Embryonal carcinoma.
3. Choriocarcinoma.
4. Thymoma.
S. Carcinoid tumour.

This is a question that may be approached in two ways. One way is to


ignore the chest X-ray. lt is enough to know that we are presented with
a selective aplasia of the red-ce ll seri es, w hich is one of the possible hae-
matologica l man ifestations of thymoma. In an actua l exam, this would be Figure 1a
enough to sol ve the question, since it is not suggestive of any of the other
diag noses (correct answer no. 4).

However, the purpose of th is book is not precisely to ignore the images.


This chest X-ray shows a mediastinal mass.Th e lateral projection is of great
va lue to place it in the exact position, wh ich in th is case corresponds to
the anterior part of the mediastinum. Felson, in his renowned treaty on
chest radiology, always insists on t he fourT's ofthe anterior mediastinum:
Thyroid, Thymoma, Teratoma and t he "Terribl e lymphoma': In th is case,
given the haematolog ica l alterations, we should considera thymoma. On
the other hand, remember that thymoma is the most frequ ent tumour in
the anteri or mediastinu m.

Be careful with answer no. S. Ca rcinoid tu mou rs do not usually appear


as med iastinal masses. Th ey have endobronchial growth; for thi s reason,
they may produce a collapse of the lobe or segment corresponding to
that bronchus, and even pneumonia. Other times, rather than by endo-
bron chial growth, they man ifest t hemselves by the effect s of th e seroto-
nin that they release into the blood: diarrhoea, w heezing, hypot ension,
cutaneous ru befaction, etcetera . Figura 1 b. Thymoma in the anterior mediastinum. Detail of th e case.
Cirugia torácica •

Recommended reading 1

Compartment Masa Características


Anterior Thymoma The most frequent in this compartment
Associatons:
Cushing
- Pure red-ce ll series aplasia
- Myasthenia gravis
- Agammaglobulinaemia
Thyroid
Teratoma and other germ-cell tumours Calcification in the mass
Associations: gynecomasty, hypog lycaemia, thyrotoxicosis

Linfoma Association: hypercalcaemia

Middle Developmental cysts: The most frequent in this compartment


Pericardial
· Bronchogenic
Lymphoma
Morgagni hern ia Most frequent location: right ca rdiophrenic angle

Posterior Of peripheral nerves: neurofibroma All these are neurogenic tumours


Ofthe sympathetic ganglia: The most frequent in this compartment
- Ganglioneuroma
- Neuroblastoma

Of the paraganglia:
- Paraganglioma
- Pheochromocytoma
Lymphoma
Oesophagus
Enteric cysts
Others
- Pseudoqu iste pancreático
- Hernia de Bochdaleck
- Men ingocele
- Extramedullary haematopoiesis
Figura 1c. Mediastina l masses

Fuente: García Maca rró n J. Casos clínicos en imágenes. Madrid. CTO Ed i-


torial, 20 12.
---·_ _ _____:C__._ir~ugiatoráci_ea_

Recommended reading 2

A 36-year old male, non-smoker, asymptomatic. A chest X-ray is per-


fo rmed wherein an image compatible with a pulmonary nodule is
suspected. For greater precision, a CT is performed, which is shown
below. lndicate, amongst the following options, which would be the
most probable diagnosis [Figure 2a]:

l. Squamous-cell carcinoma.
2. Hamartoma.
3. Wegener's disease.
4. Metastatic adenocarcinoma.
S. Carcinoid tumour

Amongst benign pulmonary tumours, it is worth mentioning bronchial Figure 2a


adenomas, since they are the most frequent, and hamartoma, which
has a very characteristic radiological pattern and is the one shown in
t hi s image: the "popcorn" ca lcifi cations, wh ich are considered to be
pathognomo nic for hamartoma. We may also find fat dens ity in sorne
reg ions in sid e the tumour, alt hough th is is less observable and, since
the calcitications are so typical, frequently it goes unnoticed (correct
answer no. 2).

Histologically, hamartoma involves a ma lformation, w herein we find di-


fferent types of tissues mixed together: muscle, fat, ca rti lage, bone, etc.
Frequently, a su rgi ca l resection is performed in ord er to discard malig-
nant neoplasia. However, w hen one of the radiological data descri bed
above appear (fat density or popcorn ca lcifications), it is not necessary
todo so.

Remember t hat bronch ial adenomas are more frequent than ha mar-
toma. About 80% of adenomas are carcinoid tumours; 1Oo/o-15% are
cylindromas (adeno id-cystic tumours) and a small percentage of ca- Figure 2b. Endobronchial ca rcinoid.
ses are mucoepidermoid carcinomas. Most are intrabronchia l lesions
with a slow growth and a centra l location, as shown in the attached Fuente: García Macarrón J. Casos clínicos en imágenes. Madrid. CTO Edi-
image. torial, 20 12.
Cirugía torácica 1 Bibliograf ía

Bibliografía
Cirugía torácica

¡]il Capítulo 01. Enfermedades de la pleura 16.• ed. Ausina Ruiz V, Carmena Rodríguez R, Carreres Molas
A, et al, (eds.). Elsevier España, Barcelona. 2008; 785-797.
Baumann MH, Strange C, Heffner JE, et al. Management of
Spontaneous Pneumothorax: an American College of Chest American College of Chest Physicians. Diagnosis and Mana-
Physicians Del phi Consensus Statement. Chest 2001; 119: gement of Lung Cancer: ACCP Guidelines. Chest 2007, 132
590-602. (suppl. 3); 1-422.

Ferrer Sancho J, Rodríguez Panadero F. Enfermedades de la Brunelli A, Charloux A, Bollinger CT, et al. ERS/ESTS clinical
pleura. En: Farreras-Rozman Medicina Interna, 16.• ed. Ausina guidelines on fitness for ra dical therapy in lung cancer pa-
Ruiz V, Carmena Rodríguez R, Carreres Molas A, et al, (eds.). tients (surgery and chemo-radiotherapy). Eur Respir J 2009;
Elsevier España, Barcelona. 2008; 844-853. 34:17-41 .

Henry M, Arnold T, Harvey J. BTS guidelines for the manage- Grupo CTO. Manual CTO de Neumología y cirugía torácica. 9.•
ment of spontaneous pneumothorax. Thorax 2003; 58, 1139- ed. CTO Editorial, Madrid, 2014.
1152.
lnternational Association for the Study of Lung Cancer. Sta-
Light RW. Disorders of the pleura and mediastinum. En: ging Manual in Thoracic Oncology. Orange Park. Rx Press,
Harrison's Principies of Interna/ Medicine, 17th ed. Fauci AS, 2009.
Braunwald E, Kasper DL, Hauser SL, Longo DL, Jameson JL,
Loscalzo J, (eds.). McGraw-Hill, New York. 2008; 1658-1 661. Minna JD, Schiller JH . Neoplasms of the lung. En: Harrison's
Principies of Interna/ Medicine, 17th ed. Fauci AS, Braunwald
¡]il Capítulo 02. Enfermedades del mediastino E, Kasper DL, Hauser SL, Longo DL, Jameson JL, Loscalzo J,
(eds.). McGraw-Hill, NewYork. 2008; 551 -562.
Light RW. Disorders of the Pleura and Mediastinum. En:
Harrison's Principies of Interna/ Medicine, 17th ed. Fauci AS, ¡]il Capítulo OS. Trasplante de pulmón
Braunwald E, Kasper DL, Hauser SL, Longo DL, Jameson JL,
Loscalzo J, (eds.). McGraw-Hill, New York. 2008; 1658-1661. Román Broto A, Morell Brotad E. Trasplante pulmonar. En:
Farreras-Rozman Medicina Interna, 16.• ed. Ausina Ruiz V, Car-
Varela de Ugarte A. Enfermedades del mediastino. En: Farre- mena Rodríguez R, Carreres Molas A, et al, (eds.) . Elsevier Es-
ras-Rozman Medicina Interna, 16.• ed. Ausina Ruiz V, Carmen a paña, Barcelona. 2008; 861-863.
Rodríguez R. Carreres Molas A, et al, (eds.). Elsevier España,
Trulock EP. Lung transplantation. En: Harrison's Principies of
Barcelona. 2008; 854-8S7.
Interna/ Medicine, 17th ed. Fauci AS, Braunwald E, Kasper DL,
¡]il Capítulo 03. Enfermedades del diafragma Hauser SL, Long o DL, Jameson JL, Loscalzo J, (eds.). McGraw-
Hil l, NewYork. 2008; 1668-1671.
Montserrat Canal JM, Gea Guiral J. Enfermedades del dia-
fragma y de los músculos respiratorios. En: Farreras-Rozman Varela A, Alvarez Kindelán A, Román A, Usetti P, Zurbano F
Medicina Interna, 16.• ed. Ausina Ruiz V, Carmen a Rodríguez (Grupo de Trabajo de la SEPAR). Normativas SEPAR. Trasplan-
R, Carreres Molas A, et al, (eds.). Elsevier España, Barcelona. te pulmonar. Arch Bronconeumo/2001; 37: 307-31 S.
2008; 857-861.

¡]il Capítulo 04. Neoplasias pulmonares

Agustí García-Navarro C, Rosell Costa R, Felip Font E. Tumores


broncopulmonares. En: Farreras-Rozman Medicina Interna,
• •
• • • • •••
• • • • •••
• • ••
• • •
• • • • • • • • • •
• • ••••
• • •
• • • • • •••
• • • • •
• • • •
• • • • • • • •••
• • • •
• • • • • •
• • • • • • • • • • • • • • •••

• • • • • • • • • • • • • • • •••
• • • • • • • • • • • • • • • • • • • ••••
• • • • • • • • • • • • • • • • • • • ••••
• • • •

• • • • • • • • • • • • • • • • • • • ••••
• • • • • • •
• • • • • • • •
• • • • • • • • • • • • • • • • • • ••• ••
........
• • • • • • • • •
••
. . . . .. .............
• • • • • • • • • • • • • • • • • • •••
• •
• • • • • • • • • • •

. .. .....
. ..
• • • • • • • • • • • • • • • • • • • • • • • • • • • • • • • • • ••• •• ,
••••••••••••••••••••• ••••••••••••••••••••
• • • • • • • • • • • • • • • • • • • • • • • • •• • • •• • •••••••
• • • • • • • • • • • • • • • • • • • • • • • • • • • • • • • • • • • • • • • ••
• •

• • • •
• • • •••••••••••••••••••••
• •••••••••••••
••••••• ••••••••••••••••••••• •••••••••••••••
••••••• ••••••••••••••••••••• •••••••••••••••
••••••••••••••••••
••••• ••••••••••••••••••••••••••••••••••••••••
•••••••••••••••••••••••••••••••••••••••••
•••
••• •••••••••••••••••••••
••••••••••••••••••••
••••••••••••••••••••••••••••••••••••••••••
••••••••••••••••••••••••
•••

••••••••••••••••••••• •••••••••••••••••••••
•••
•••
•••••••••••••••••••••
••••••••••••••••••• ••••••••••••••••••••• •
••••••••••••••••••••• ••••••••••••••••••••• ••
••
•• •••••••••••••••••••••
••••••••••••••••••••• ••••••••••••••••••••• •
••
•• ••••••••••••••••••••• ••••••••••••••••••••
••••••• ••••••••••••••
•••••••••••••••••••••••••••••••••••••••••• •
• •
• ••••••••••••••••••••• ••
••••• •••••••••••••• ••••••••••••••••••••• •
••
• ••••••••••••••••••••• •••••••••••••••••••••
••••••••••••••••••• ••••••••••••••••••••• ••
•••
••
•• ••••••••••••••••••••• •••••••••••••••••••••
••••••••••••••••••••• ••••••••••••••••••••• •
••
••• ••••••••••••••••••••• •••••••••••••••••••••
••••••••••••••••••••• •••••••••••••••• •••
•••
• • • ••••••••••••••••••••• •••••••
••••••••••••••••••••• •••••••••••••
•••••••••••••••••••••
••
••• •••••••••••••••••••••
• • •
••••••••••••••••••••
• • • • • • • • • • • • • •• ••••••••• ••• • • • • • • • • • •
• • • • • • • • • • • • • •••••••••••••••••• •• • • • • • • •

. . . . . . . . . . . . . .• .• .• .• illlll=-~~~~~~~~~~~~~~~~~~~~~~~~

• • • • • •
• • • •
•• •
• • • •••••••••• •
• • • • • • • • • • •• • •••• •••••••••••••• • • • •
• • • • • • • • • • • •

• • ••••••••••••• •• • •
• • • • • • • • • • • • • • • • •• • • • • • • • • • • • • •
• • • •
• • • • • • • • • • • • • • • •
• • • • • • • • • • • • • •

ISBN: 978-84-16527-26-7 ISBN: 978-84-16527-27-4

1
9 788416 527267
1
9 788416 527274

Potrebbero piacerti anche